Vous êtes sur la page 1sur 360

Mohamad Z.

Koubeissi
Nabil J. Azar Editors

Epilepsy
Board Review

A Comprehensive Guide

123
Epilepsy Board Review
Mohamad Z. Koubeissi
Nabil J. Azar
Editors

Epilepsy Board Review


A Comprehensive Guide

123
Editors
Mohamad Z. Koubeissi Nabil J. Azar
Epilepsy Center Associate Professor of Neurology
George Washington University Epilepsy Division
Washington, DC Vanderbilt University Medical Center
USA Nashville, TN
USA

ISBN 978-1-4939-6772-8 ISBN 978-1-4939-6774-2 (eBook)


DOI 10.1007/978-1-4939-6774-2

Library of Congress Control Number: 2016956189

© Springer Science+Business Media LLC 2017


This work is subject to copyright. All rights are reserved by the Publisher, whether the whole or
part of the material is concerned, specifically the rights of translation, reprinting, reuse of
illustrations, recitation, broadcasting, reproduction on microfilms or in any other physical way,
and transmission or information storage and retrieval, electronic adaptation, computer software,
or by similar or dissimilar methodology now known or hereafter developed.
The use of general descriptive names, registered names, trademarks, service marks, etc. in this
publication does not imply, even in the absence of a specific statement, that such names are
exempt from the relevant protective laws and regulations and therefore free for general use.
The publisher, the authors and the editors are safe to assume that the advice and information in
this book are believed to be true and accurate at the date of publication. Neither the publisher nor
the authors or the editors give a warranty, express or implied, with respect to the material
contained herein or for any errors or omissions that may have been made.

Printed on acid-free paper

This Springer imprint is published by Springer Nature


The registered company is Springer Science+Business Media LLC
The registered company address is: 233 Spring Street, New York, NY 10013, U.S.A.
This book is dedicated to my mother, Haja Jammoul, whose love
has been my nest, my wings, and my sky.
Mohamad Z. Koubeissi

I dedicate this book to my wife Joelle, my sons Michel and John,


for their endless love, support, and encouragement.
Nabil J. Azar
Contents

Part I The Normal EEG


1 Physiologic Basis of EEG and Epilepsy . . . . . . . . . . . . . . . . . . 3
Susanta Bandyopadhyay, Mohamad Z. Koubeissi
and Nabil J. Azar
2 EEG Instrumentation, Montage, Polarity,
and Localization . . . . . . . . . . . . . . . . . . . . . . . . . . . . . . . . . . . . 15
Krikor Tufenkjian
3 Normal EEG Variants and Artifacts . . . . . . . . . . . . . . . . . . . . 33
Raed Azzam, Mohamad Z. Koubeissi
and Nabil J. Azar
4 Neonatal EEG and Neonatal Seizures . . . . . . . . . . . . . . . . . . . 55
Kohilavani Velayudam and Ahsan N.V. Moosa

Multiple Choice Questions for Part I 71

Part II The Abnormal EEG


5 Ictal and Interictal EEG . . . . . . . . . . . . . . . . . . . . . . . . . . . . . . 83
Mohamad Z. Koubeissi and Nabil J. Azar
6 EEG and Semiology in Generalized Epilepsies . . . . . . . . . . . . 93
Nandakumar Bangalore-Vittal
7 EEG and Semiology in Focal Epilepsy . . . . . . . . . . . . . . . . . . . 109
Amir M. Arain
8 Status Epilepticus . . . . . . . . . . . . . . . . . . . . . . . . . . . . . . . . . . . 115
Pavel Klein
9 EEG in Encephalopathy and Coma . . . . . . . . . . . . . . . . . . . . . 127
Mohamad Z. Koubeissi, Nabil J. Azar
and Peter W. Kaplan

Multiple Choice Questions for Part II 139

vii
viii Contents

Part III Specific Epilepsy Syndromes


10 Electro-Clinical Syndromes by Age of Onset—Childhood,
Adolescence, and Adult . . . . . . . . . . . . . . . . . . . . . . . . . . . . . . . 155
Tesfaye Zelleke
11 Seizures and Epilepsy in Early Life . . . . . . . . . . . . . . . . . . . . . 161
Adam L. Hartman
12 Imitators of Epilepsy . . . . . . . . . . . . . . . . . . . . . . . . . . . . . . . . . 167
Dewi Frances T. Depositario-Cabacar and Nabil J. Azar
13 Genetic Analysis of Epilepsies . . . . . . . . . . . . . . . . . . . . . . . . . . 171
Sara K. Inati
14 Epilepsy Secondary to Specific Mechanisms . . . . . . . . . . . . . . 177
Amar Bhatt

Multiple Choice Questions for Part III 187

Part IV Management of Epilepsy


15 Principles of Epilepsy Diagnosis and Management . . . . . . . . . 203
Hasan H. Sonmezturk, Mohamad Z. Koubeissi
and Nabil J. Azar
16 Old Generation Antiepileptic Drugs . . . . . . . . . . . . . . . . . . . . . 213
Bassel Abou-Khalil
17 New Generation Antiepileptic Drugs . . . . . . . . . . . . . . . . . . . . 225
Bassel Abou-Khalil
18 Dietary Therapies . . . . . . . . . . . . . . . . . . . . . . . . . . . . . . . . . . . 235
Amy Kao
19 Other Pharmacological Therapies: Investigational
Antiepileptic Drugs, Animal Models of Epilepsy,
Hormonal Therapy, Immunotherapy . . . . . . . . . . . . . . . . . . . . 241
Bassel Abou-Khalil
20 Epilepsy Management in Special Populations . . . . . . . . . . . . . 247
Amir M. Arain

Multiple Choice Questions for Part IV 253


Contents ix

Part V Presurgical Evaluation and Epilepsy Surgery


21 Neuroimaging in Epilepsy . . . . . . . . . . . . . . . . . . . . . . . . . . . . . 273
Anuradha Singh, Priyanka Sabharwal
and Timothy Shephard
22 Neuropsychological Evaluation in Epilepsy . . . . . . . . . . . . . . . 293
Madison M. Berl and Leigh Sepeta
23 Magnetoencephalography and Magnetic Source
Modeling . . . . . . . . . . . . . . . . . . . . . . . . . . . . . . . . . . . . . . . . . . 301
Manoj Raghavan
24 Vagus Nerve Stimulation and Other Neuromodulation . . . . . 309
Gholam K. Motamedi
25 Epilepsy Surgery Assessment and Testing . . . . . . . . . . . . . . . . 317
Gholam K. Motamedi
26 Procedures and Outcomes in Epilepsy Surgery . . . . . . . . . . . . 337
Ronald P. Lesser
27 Quality of Life in Epilepsy . . . . . . . . . . . . . . . . . . . . . . . . . . . . 347
Patsy J. Ramey, Mohamad Z. Koubeissi and Nabil J. Azar

Multiple Choice Questions for Part V 353

Index . . . . . . . . . . . . . . . . . . . . . . . . . . . . . . . . . . . . . . . . . . . . . . . . . 367
Contributors

Bassel Abou-Khalil Department of Neurology, Vanderbilt University


Medical Center, Nashville, TN, USA
Amir M. Arain Department of Neurology, Vanderbilt University Medical
Center, Nashville, TN, USA
Nabil J. Azar Department of Neurology, Vanderbilt University Medical
Center, Nashville, TN, USA
Raed Azzam Department of Neurology, Vanderbilt University Medical
Center, Nashville, TN, USA
Susanta Bandyopadhyay Department of Neurology, Vanderbilt University
Medical Center, Nashville, TN, USA
Nandakumar Bangalore-Vittal Department of Neurology, Nashville Gen-
eral Hospital at Meharry, Vanderbilt University Medical Center, Nashville,
TN, USA
Madison M. Berl Neuropsychology, Children’s National Health System,
Washington, DC, USA
Amar Bhatt Department of Neurology, Vanderbilt University Medical
Center, Nashville, TN, USA
Dewi Frances T. Depositario-Cabacar Department of Neurology and
Pediatrics, Children’s National Medical Center, George Washington
University, Washington, DC, USA
Adam L. Hartman Johns Hopkins Medicine, Baltimore, MD, USA
Sara K. Inati EEG Section, Office of the Clinical Director, NINDS, NIH
Clinical Center, Bethesda, MD, USA
Amy Kao Center for Neurosciences and Behavioral Medicine, Children’s
National Health System, Washington, DC, USA
Peter W. Kaplan Department of Neurology and Pediatrics, Johns Hopkins
Medicine, Baltimore, MD, USA
Pavel Klein Department of Neurology, Mid-Atlantic Epilepsy and Sleep
Center, Bethesda, MD, USA

xi
xii Contributors

Mohamad Z. Koubeissi Department of Neurology, Epilepsy Center, George


Washington University, Washington, DC, USA
Ronald P. Lesser Department of Neurology, Johns Hopkins University
School of Medicine, Baltimore, MD, USA
Ahsan N.V. Moosa Section of Pediatric Epilepsy, Epilepsy Center, Cleve-
land Clinic, Cleveland, OH, USA
Gholam K. Motamedi Department of Neurology, Georgetown University
Hospital, Washington, DC, USA
Manoj Raghavan Department of Neurology, Medical College of Wisconsin,
Milwaukee, WI, USA
Patsy J. Ramey Department of Neurology, Vanderbilt University Medical
Center, Nashville, TN, USA
Priyanka Sabharwal Department of Neurology, NYU Langone Medical
Center, New York, NY, USA
Leigh Sepeta Neuropsychology, Children’s National Health System,
Washington, DC, USA
Timothy Shephard Department of Radiology, New York University Lan-
gone Medical Center, New York, NY, USA
Anuradha Singh Department of Neurology, Bellevue Hospital Center, New
York, NY, USA
Hasan H. Sonmezturk Vanderbilt Department of Neurology, Vanderbilt
University, Nashville, TN, USA
Krikor Tufenkjian Department of Neurology, Creighton University,
Omaha, NE, USA
Kohilavani Velayudam Department of Pediatric Neurology, Emory
University, Atlanta, GA, USA
Tesfaye Zelleke Department of Neurology, Children’s National Medical
Center, Washington, DC, USA
Part I
The Normal EEG
Physiologic Basis of EEG
and Epilepsy 1
Susanta Bandyopadhyay, Mohamad Z. Koubeissi
and Nabil J. Azar

majority of the cortical synapses. Thalamocortical


Organization of the Cerebral Cortex
projections have an important role in modulating
inhibition via thalamic fibers from the reticular
The cerebral cortex is organized into six hori-
nucleus of the thalamus.
zontal layers with layer I being the most superfi-
The electrical activity recorded on electroen-
cial underneath the pial surface and layer VI being
cephalogram (EEG) arises from the extracellular
the deepest overlying subcortical white matter:
field potential generated by changes in mem-
Layer I Molecular layer containing dendrites brane potential of neurons for the most part with
and axons from other layers some contribution from glial cells.
Layer II External granular layer containing
cortico-cortical connections
Layer III External pyramidal layer containing Neuronal Resting Membrane
cortico-cortical connections Potential, Postsynaptic Potentials,
Layer IV Internal granular layer receiving and Action Potential
input from thalamus
Layer V Internal pyramidal layer sending A typical neuron has a cell body (soma) and
output to subcortical structures processes (axons and dendrites), which may be
Layer VI Multiform layer sending output to considered as extensions of the soma. Cell mem-
thalamus branes of neurons have several different ion
channels including sodium and potassium leak
Extensive horizontal cortico-cortical connec- channels as well as voltage-gated sodium and
tions in layers I, II, and III make up the vast potassium channels that permit the passage of ions
depending on their electrochemical gradients.
Equilibrium potential for an ion is the membrane
potential at which there is no net movement of that
S. Bandyopadhyay  M.Z. Koubeissi  N.J. Azar (&) ion across the cell membrane. Equilibrium
Department of Neurology, Vanderbilt University
Medical Center, 1161 21st Ave. South, potential can be calculated using the Nernst
A-0118 Medical Center North, equation. The resting membrane potential of a
Nashville, TN 37232-2551, USA neuron is typically −70 mV, the inside of the
e-mail: nabil.azar@vanderbilt.edu neuron being negative in relation to the outside.
S. Bandyopadhyay The resting membrane potential is determined by
e-mail: susanta.bandyopadhyay@vanderbilt.edu the movement of potassium, sodium, and chloride
M.Z. Koubeissi ions along their electrochemical gradient across
e-mail: koubeissi@gmail.com

© Springer Science+Business Media LLC 2017 3


M.Z. Koubeissi and N.J. Azar (eds.), Epilepsy Board Review,
DOI 10.1007/978-1-4939-6774-2_1
4 S. Bandyopadhyay et al.

the cell membrane. Table 1.1 shows the intracel- This triggers a cascade of events leading to the
lular and extracellular concentrations of these ions fusion of presynaptic vesicles with the membrane
and their equilibrium potentials in a typical of the presynaptic terminal, thereby releasing
mammalian neuron. The membrane potential at neurotransmitter molecules into the synaptic
which there is no net flow of ions across the cell cleft. Binding of neurotransmitters to the recep-
membrane is the resting membrane potential tors on the postsynaptic terminal activates ion
which can be calculated using the Goldman– channels associated with them, allowing the
Hodgkin–Katz equation. The major contributor of passage of ions leading to local changes in
resting membrane potential is the potassium leak membrane potential (Fig. 1.1). Such local chan-
channels with a net outward flow of potassium ges in membrane potential are known as post-
ions (K+) under resting conditions. An action synaptic potentials (PSPs) which are
potential is generated when the negativity in the nonpropagated small-amplitude potentials lasting
interior of the neuron, i.e., the resting membrane 10–100 ms. PSPs can be excitatory (excitatory
potential, decreases to a critical point (typically postsynaptic potential (EPSP)) or inhibitory (in-
around −40 mV). The voltage-gated sodium hibitory postsynaptic potential (IPSP)) depending
channels play a major role in the generation and on the type of ion channel activated and the
propagation of action potential by allowing electrochemical gradient for ions that can pass
sodium to enter into the soma. Once generated, the through the channel. EPSPs generally result from
action potential—a short duration (usually less an inward flow of positive ions such as sodium or
than 2 ms) high-amplitude wave of depolarization calcium and cause depolarization (excitation),
—travels through the neuronal processes and thus decreasing the threshold for triggering an
reaches synapse, a specialized contact between action potential in the postsynaptic terminal. On
neurons usually between axons and dendrites. the other hand, IPSPs result from an inward flow
Synaptic transmission is the major mode of of negative ions (e.g., chloride) or outward flow
transmission of information from one neuron to of positive ions (e.g., potassium) and cause
another. Each synapse has a presynaptic terminal hyperpolarization (inhibition), thus increasing the
containing neurotransmitters in vesicles and a threshold for triggering an action potential in the
postsynaptic terminal containing receptors for the postsynaptic terminal.
neurotransmitters. Neurotransmitters are released A single EPSP or IPSP is not sufficient
from the presynaptic terminal when an action enough to move the membrane potential of the
potential causes sufficient change in the voltage postsynaptic terminal to or away from the
(depolarization) at the presynaptic terminal to threshold for triggering of action potential.
activate voltage-gated calcium channels allowing Summation of several PSPs is necessary for that
calcium to enter into the presynaptic terminal. purpose. Such summation can be spatial

Table 1.1 Intracellular and extracellular concentrations of ions that play a major role in determining the resting
membrane potential in a mammalian neuron
Ion Intracellular (mM) Extracellular (mM) Equilibrium potential (mV)
Potassium (K*) 140–150 5 –90
Sodium (Na*) 5–15 145 +60
Chloride (Cl) 4–30 110–125 –70
1 Physiologic Basis of EEG and Epilepsy 5

Fig. 1.1 Membrane


potential (MP) changes and
current flows during
synaptic activation. a The
MP of the postsynaptic
neuron and the MP of the
presynaptic fibers are
recorded by means of
intracellular
microelectrodes. Action
potentials in the excitatory
and inhibitory presynaptic
fibers lead to excitatory
postsynaptic potential
(EPSP) and inhibitory
postsynaptic potential
(IPSP), respectively, in the
postsynaptic neuron. Two
EPSPs sum up to a
superthreshold potential,
triggering an action
potential in the
postsynaptic
neuron. b During EPSP
and IPSP, ionic current
flows occur through as well
as along the neuronal
membrane, as shown by
arrows. The density
of + and – signs indicates
the polarization of the
subsynaptic (dark area) as
well as that of the
postsynaptic membrane
during synaptic activation.
Adapted from [1]

(summation of several PSPs in the vicinity) or


temporal (summation of several PSPs occurring EEG Generator
in quick succession). Neurons generally have
extensive dendritic arborization, and hundreds to A large number of EPSPs and IPSPs generated in
thousands of axons can form synapses on a a complex network of neurons alter the overall
neuron’s dendrites. Each neuron receives excitability of the neurons in the network. Such
synaptic inputs from multiple sources at the same PSPs generate an extracellular field potential
time—spatial and temporal summation of these (Fig. 1.2) that changes over time which is
inputs determines whether or not the neuron fires believed to be the basis of potentials recorded on
an action potential. EEG. The extracellular field potential is a
6 S. Bandyopadhyay et al.

secondary phenomenon resulting from the electrode at a distance would record negativity
development of potential gradients between areas because of an outflow of negative ions (or an
of localized membrane potential change and the inflow of positive ions) associated with the cur-
remaining areas of the neuronal membrane. rent flowing through the extracellular space.
A ‘sink’ is generated at the site of an EPSP Therefore, polarity of extracellular field
because of an inflow of positive ions into the potentials recorded by surface electrodes on EEG
localized area of the neuron, and there is a cor- depends on the direction of current flow as well
responding ‘source’ at a distance where positive as on the position of the electrode relative to the
ions come out of the neuron; current flows from location of the generator. This translates to the
the ‘source’ to the ‘sink’ in the extracellular fact that superficial EPSPs and deep IPSPs will
space giving rise to the extracellular field show the same polarity (negative) on a surface
potential. Thus, a recording electrode close to the recording electrode. Likewise, superficial IPSPs
synapse receiving an excitatory input (EPSP) and deep EPSPs will show the same polarity
would record a negative potential because of an (positive) on a surface recording electrode
inward flow of positive ions causing negativity in (Fig. 1.3). Thus, orientation of neurons and their
the extracellular space nearby, whereas a deep processes, as well as location of synaptic contacts
recording electrode at a distance would record with respect to the cortical surface, are important
positivity because of an outflow of positive ions determinants of extracellular field potentials
associated with the current flowing through the recorded by EEG electrodes.
extracellular space. The reverse is true for an Pyramidal neurons in the cerebral cortex are
inhibitory input (IPSP): A recording electrode arranged in vertical columns with their cell
close to the synapse receiving an inhibitory input bodies typically in the layer III or V and their
(IPSP) would record a positive potential because processes (dendrites and axons) spanning the
of an inward flow of negative ions (or an outward entire column and receiving thousands of
flow of positive ions), whereas a recording synaptic contacts. This allows for summated

Fig. 1.2 Basic mechanisms underlying the generation description is based on the assumption that an extended
of potentials (electroencephalogram (EEG)) and of mag- neuronal process, e.g., a dendrite, is locally depolarized
netic fields (magnetoencephalogram (MEG)) in the by the activation of an excitatory synapse. Adapted
extracellular space of central nervous system. The from [1]
1 Physiologic Basis of EEG and Epilepsy 7

(a) (c)

EPSP IPSP

Superficial -ve +ve

Deep +ve -ve

(b)

Fig. 1.3 Membrane potential (MP) changes and field deeper recording (E2). a(2) The activation of a deep
potentials (FPs) elicited by the activation of excitatory excitatory synapse elicits a current flow with inverse
and inhibitory synapses in the central nervous system. direction as compared with a(1). Therefore, the extracel-
The elementary processes are explained by means of a lular FP consists in a positive deflection at the surface and
neuronal element (hatched area), the one end of which in a negative one at the depth. b(1) The outward current
contracts the surface of a structure in the central nervous at S generates an IPSP in the region of ME2, as well as in
system. The MP of the neuron element is recorded at both that of ME1. Due to the direction of the extracellular
ends by the microelectrodes ME1 and ME2. The extra- current flow, the FP generated consists in a positive
cellular field is picked up at the surface of the neuronal fluctuation in the depth (E2) and in a negative one in the
structure by the electrode E1, as well as in the vicinity of surface recording (E1). b(2) The current flow during the
ME2 by the electrode E2. Active excitatory and inhibitory activation of a superficial inhibitory synapse is inverse as
synapses are marked by open triangles and black trian- compared with b(1). Therefore, the FP recorded from the
gles (S), respectively. a(1) The inward current at S surface consists of a positive fluctuation. Differences in
generates an EPSP that appears in the region of ME1, the time course of the various potentials are caused by the
as well as in that of ME2. Because S is located electrical properties of the tissue. c Schematic summary
superficially, the FP generated, due to the direction of of the polarity of the potential recorded on the scalp with
the extracellular current flow (arrows), is of negative an EEG recording electrode based on the occurrence of
polarity at the surface (E1) and of positive polarity in the (superficial vs. deep) EPSP or IPSP. Adapted from [1]

potentials with a vertical dipole or a dipole ori- the electrode. Most of the cortical activity
ented at an angle to the recording electrodes recorded from subdural or depth electrodes is not
(discussed further below) which can be recorded evident in the scalp EEG due to the attenuation
on EEG. On the other hand, summated potentials by the intervening scalp and skull.
resulting in horizontal dipoles (oriented parallel The EEG pattern is thought to depend on
to the recording electrodes) cannot be generally numerous areas of the cerebral cortex with
recorded on EEG. Subcortical structures have an opposite electrical poles (dipoles) that constantly
indirect influence on scalp EEG. Of note, to fluctuate. Thus, the EEG signal at a scalp elec-
produce a scalp EEG signal, 6 cm2 or more of trode can be viewed as a result of currents gen-
synchronously active area of cortex is required. erated by a negative and a positive dipole layer in
Scalp electrodes record volume-conducted the electrode ‘view.’ The potential recorded at the
potentials. Signal decreases proportionally to electrode is proportional to the solid angle sub-
the square of the distance between the source and tended by the dipole layer as shown in Fig. 1.4.
8 S. Bandyopadhyay et al.

Fig. 1.4 Use of the solid angle rule to ascertain the electrode A is actually lower than that measured by the
signal measured on the scalp surface relative to the more distant electrode B because of the arrangement of
orientation of the dipole. Top Surface electrode B sees a the dipoles in the discharging region. The smaller solid
large electrical potential because of the orientation and angle, ΩA, is proportional to the voltage measured on the
proximity of the dipole layer, as borne out by the solid scalp. Adapted from [2]
angle ΩB. Bottom In this case, the potential seen by the
1 Physiologic Basis of EEG and Epilepsy 9

Fig. 1.5 There are unusual sources wherein both the middle row of illustrations, the positions of the electrodes
negative and the positive poles are recorded on the on the scalp, relative to the discharging cortex, are shown.
surface. The bottom row of figures shows a patch of cortex The top row illustrates the voltage that would be recorded
containing gyri and sulci. The darker areas represent the on the EEG as a function of the distance along the scalp
cortical mantle that is activated by an epileptic discharge, right below it. Adapted from [2]
with negative and positive poles highlighted. In the

If adjacent active regions of the cortex have the bursts giving rise to fluctuating field poten-
voltage fields with different dipole orientations, tials. When recording from surface is done with
they summate in relation to their representative an amplifier with a finite time constant (as in
field vectors. For example, if two cortical areas conventional EEG), such fluctuations in field
have an opposite dipole orientation such as the potential are recorded as waveforms (Fig. 1.6).
two sides of a sulcus, cancelation occurs, and no Sustained firing of the afferent fiber leads to
voltage field is evident at the scalp. When the sustained depolarization of the apical dendrites
dipole is vertical and the electrode is directly causing a depolarization shift which is not
above it, the electrode records the field maximum reflected on the surface in conventional EEG
(Fig. 1.5). As the orientation of the dipole recorded with an amplifier with a finite time
becomes progressively less radial and more tan- constant. Sustained changes in field potential
gential to a recording electrode, the electrode (baseline shifts) can be recorded using a direct
records a voltage field of lesser amplitude. If the current or DC amplifier that has an infinite time
dipole is directly below the electrode but it is constant (Fig. 1.7). Such recordings are not done
perfectly tangential, the electrode records no from the scalp because of technical difficulties
potential because of its location on the zero and are usually performed in experiments in
isopotential line of the source scalp field. animal models. Of note, not only baseline shifts
are generated by neurons; but glial cells also
contribute to its generation. Increased firing of a
Wave Generation and Baseline Shifts deep-seated neuron causes rise in extracellular
potassium concentration which in turn leads to
When an afferent fiber forming an excitatory depolarization of glial cell superficial to it. Such
synapse on an apical dendrite near the surface depolarization of the glial cell electrotonically
produces bursts of action potentials interrupted spreads to the glial cell network coupled by gap
by periods of quiescence, EPSPs sum up during junctions.
10 S. Bandyopadhyay et al.

Fig. 1.6 Wave generation in the electroencephalogram action potentials in the afferent fiber generate wavelike
(EEG) at the surface of the cerebral cortex. A perpendic- excitatory postsynaptic potentials (EPSPs) in the dendritic
ular pyramidal neuron is shown. An afferent fiber formed region and corresponding waves in the EEG recording.
an excitatory synaptic contact at the superficial part of the Tonic activity in the afferent fiber results in long-lasting
apical dendrite. Simultaneous recordings of the membrane EPSP with only small fluctuations. The long-lasting
potentials (MPs) of the afferent fiber and the dendritic depolarization is not reflected on the conventional EEG
element, as well as of the EEG, are displayed. Groups of recording. Adapted from [2]

Fig. 1.7 Sustained shifts in the electroencephalogram of the apical dendrite. The membrane potentials (MPs) of
(EEG) at the surface of the cerebral cortex resulting from the afferent fiber and the dendritic element were recorded
sustained neuronal activities. If recordings are performed simultaneously, as was the EEG/DC. Increased and
with a direct current (DC) amplifier (EEG/DC), sustained decreased sustained activity in the afferent fiber generated
potentials can also be recorded at the surface. In the sustained depolarizations and hyperpolarizations of the
perpendicular pyramidal neuron depicted, an afferent fiber dendritic region and corresponding negative and positive
formed an excitatory synaptic contact at the superficial part shifts of the EEG/DC recording. Adapted from [2]
1 Physiologic Basis of EEG and Epilepsy 11

Focal Activity for focal epileptiform activity. Intracortical


potential distribution that determines the
Field potentials generated during epileptic occurrence of associated descending activity to
activity are of higher amplitude than those gen- the spinal cord may be different for the same
erated by nonepileptic activity because epileptic epileptiform activity recorded from the surface;
field potentials are the result of highly synchro- negative field potential in the layer V only was
nized neuronal activity. During focal epileptic associated with the corresponding spinal field
activity, negative potentials of high amplitude, potentials.
which repeat themselves in stereotyped form and
periodicity, can be recorded from the area. Such
oscillations of field potential occur in parallel Generalized Tonic–Clonic Activity
with the fluctuation of membrane potential
which is the characteristic of the epileptiform When tonic–clonic activity is triggered in
activity of individual neurons (Fig. 1.8). This is experimental animals by repeated injections of
known as paroxysmal depolarization shift pentylenetetrazol and membrane potential of a
(PDS) which starts with a steep depolarization pyramidal tract in the layer V of the cortex is
that triggers a series of action potentials fol- recorded during a convulsive seizure, typical
lowed by a plateau of continued depolarization; PDSs can be seen which correlate with the
this is followed, after 80–100 ms, by a steep potential fluctuations noted in the DC recording
repolarization with or without an after-going —with superficial negative potential fluctuations
hyperpolarization. (corresponding to synchronized depolarization of
It is important to note that field potentials pyramidal neurons) in the beginning and with
recorded from the surface may differ from those superficial positive potential fluctuations (corre-
recorded from different layers of the cortex. sponding to postictal hyperpolarization of pyra-
This has been shown in animal experiments midal neurons) at the end of the convulsive
using local application of penicillin as trigger seizure (Fig. 1.9).

Fig. 1.8 EEG (a) and membrane potential (MP) changes of penicillin to the cortical surface (hatched area in a).
of a pyramidal tract neuron and extracellular field potential Drawings of original tracings from experiments in the rat.
(FP) recorded in the vicinity of the impaled neuron The sweep speed in b is five times that in a. The recording
(b) during focal interictal activity elicited by the application sites are shown in the schematic drawings. Adapted from [1]
12 S. Bandyopadhyay et al.

Fig. 1.9 Simultaneous recordings of EEG and (Drawings after original tracings from experiments in
DC/EEG (a) and of DC/EEG and membrane potential the cat’s motor cortex. The sweep speed in b is 10 times
(MP) of a pyramidal tract neuron (b) during generalized that in a.) Adapted from [1]
tonic–clonic seizures elicited by pentylenetetrazol.

basal forebrain are thought to be responsible for


Spreading Depression the generation, synchronization, and desynchro-
nization of brain rhythms (regularly recurring
Spreading depression (SD) is a strong and rapid waveforms of similar shape and duration) that
depolarization of neurons that slowly propagates (3– can be recorded on EEG.
5 mm/min) in nervous tissue. It has a prominent Slow delta rhythms: Delta activity represents
high-amplitude negative component followed by a 0–4 Hz frequency range. Thalamus and cortex
smaller amplitude but longer duration positive wave; are the two sources of delta activity. Thalamo-
an initial small positive component may sometimes cortical neurons display rhythmic bursts of
be seen. SD can have maximal amplitude of 5– high-frequency spikes with an interburst fre-
30 mV and can last for 30–90 s. Initial brief period quency of 1–2 Hz which results from interplay
of excitation followed by prolonged depression between a transient calcium current (It) and
which is then replaced by sustained increase in the hyperpolarization-activated cation current (Ih).
neuronal activity is the key feature of SD. The initial Delta activity has been noted in cats with thala-
and late increase in excitability seen in SD correlates mic lesion suggesting a cortical source for delta
with the burst of action potentials and intense oscillations as well.
synaptic noise associated with synchronized neu- The slow oscillation and the K-complex: The
ronal activity. SD has been implicated in the patho- K-complex is a result of a sequence of depolariz-
physiology of several disorders including migraine, ing–hyperpolarizing episodes within a slow cor-
epilepsy, transient global amnesia, cerebrovascular tical oscillatory cycle. Such slow cortical
disease, head injury, and spinal cord disorders. oscillations are seen during sleep. Slow oscillation
becomes more regular and faster with deepening
of sleep. Firing rate of the midbrain reticular for-
Substrates of Brain Rhythms mation and mesopontine cholinergic neurons
decrease at sleep onset removing steady excitatory
The cerebral cortex, thalamus, and several gen- drive to thalamocortical neurons. This leads to
eralized modulatory systems arising from the progressive hyperpolarization of these neurons
brain stem core, posterior hypothalamus, and which corresponds to deepening of sleep.
1 Physiologic Basis of EEG and Epilepsy 13

Theta rhythms: Theta rhythms are in the upon stimulation of brain stem structures. Epi-
4–7 Hz frequency range which is conspicuous in sodes of cortical oscillations faster (100–600 Hz)
limbic regions in various animal species and in than beta–gamma frequency called ripples (100
humans. It is thought to represent a dynamic state to 200 Hz), or fast ripples (>200 Hz) have been
arising from neuronal networks in the described under both normal conditions and
hippocampus associated with spatial navigation epileptic seizures. Ripples probably reflect syn-
and memory processes. chronized IPSPs, whereas fast ripples appear to
Alpha rhythms: Alpha rhythm represents the represent bursts of population spikes. While
frequency range of 8–13 Hz. Aside from occip- high-frequency oscillations like ripples and fast
ital cortex, alpha rhythm can be recorded from ripples may be normal, recent studies indicate
the somatosensory cortex (also called mu that they may be the marker of epileptogenic
rhythm) and temporal cortex (also called tau region.
rhythm). Alpha rhythms are mainly generated
from the cortex with only moderate dependence
on the thalamus. Further Reading
Spindle (sigma) rhythms: Spindles (7–14 Hz)
originate from the thalamus and are considered to be 1. Schomer DL, Lopes da Silva FH, editors. Neider-
the first signs of EEG synchronization during early meyer’s electroencephalography: basic principles,
stages of sleep. The reticular nucleus of the thala- clinical applications, and related fields. Philadelphia,
PA: Lippincott Williams & Wilkins; 2011.
mus is regarded as the pacemaker of the spindles. 2. Wyllie E, Cascino DC, Gidal BE, Goodkin HP,
Faster rhythms: Beta and gamma rhythms are editors. Wyllie’s treatment of epilepsy: principles and
faster rhythms associated with wakeful state or practice. Philadelphia, PA: Lippincott Williams &
REM sleep. They arise when spindle and slower Wilkins; 2011.
3. Duabe JR, Rubin DI, editors. Clinical neurophysiol-
EEG rhythms are suppressed (probably mediated ogy. New York, NY: Oxford University Press; 2009.
by acetylcholine, serotonin, and norepinephrine)
EEG Instrumentation, Montage,
Polarity, and Localization 2
Krikor Tufenkjian

is indeed necessary to detect an individual


The Source of EEG
epileptic spike on scalp electrodes [1].
The source of the EEG potentials recorded from
the scalp is the excitatory and inhibitory postsy-
naptic potentials of pyramidal neurons. Each
Recording the EEG
pyramidal neuron has an apical dendrite and
Commonly used electrodes for scalp EEG have a
multiple basal dendrites (Fig. 2.1). Excitation of
contact surface made of non-depolarizing
the postsynaptic membrane at the apical dendrite
chloride-treated silver. International standards
leads to depolarization with an intracellular shift
specify that electrode resistance should be
of positive ions (Na+). Subsequently the extra-
between 100 and 5000 X. Properly applied
cellular space nearby becomes relatively nega-
electrodes show a resistance of a few hundred
tively charged. This is coupled by an inhibitory
ohms.
potential at the basal dendrites with a relatively
A minimum of 21 electrodes are recom-
positive charge nearby. At cortical layers III, V,
mended for scalp EEG. The international 10–20
and VI, neurons are aligned in a perpendicular
system is commonly used for the placement of
fashion with the cortex. This allows for sum-
these electrodes (Fig. 2.4). With this system,
mation of the small potentials generated by each
inter-electrode distances average from 4 to 6 cm,
neuron when they fire in a synchronous fashion
as the “10” and “20” mean that the distances
(Fig. 2.2).
between adjacent electrodes are either 10% or
Cortical neuronal alignment effectively cre-
20% of the total nasion-inion or right ear–left ear
ates an electrical dipole. Whether a positive or
distance of the skull. In addition, only the supe-
negative potential is recorded on the scalp elec-
rior lateral temporal region is covered. The 10–
trode depends on the location of the recording
10 system is more extensive and includes sub-
electrode with respect to these dipoles (Fig. 2.3).
temporal electrodes.
Epileptiform discharges (spikes or sharp waves)
The EEG potentials are displayed in channels;
are commonly surface negative. Simultaneous
each channel represents the difference in poten-
intracranial and scalp recordings confirmed that
tial between two electrodes. By convention, if the
at least 6 cm2 of synchronous cortical activation
difference between two electrodes is negative,
then it is represented by an upward deflection,
while a downward deflection represents a posi-
K. Tufenkjian (&) tive difference.
Department of Neurology, Creighton University,
601 N. 30th Street, Suite 5300, Omaha, NE 68131,
USA
e-mail: KrikorTufenkjian@Creighton.edu

© Springer Science+Business Media LLC 2017 15


M.Z. Koubeissi and N.J. Azar (eds.), Epilepsy Board Review,
DOI 10.1007/978-1-4939-6774-2_2
16 K. Tufenkjian

Fig. 2.2 Parallel arrangement of the pyramidal neurons


allows for summation of the individual potentials

Fig. 2.1 Pyramidal neuron

Montages

The 10–20 system employs 21 electrodes. Dif-


ferences in potentials between these electrodes
constitute channels. Combinations of different
channels are called montages. The two main
montage types are the bipolar and the referential.
In a bipolar montage, channels are arranged in
chains that follow an anterior-to-posterior or a
Fig. 2.3 Orientation of the sulci and therefore the
transverse arrangement (Figs. 2.5 and 2.6). The dipoles determine what potential is recorded from the
chains imply that the second lead in the first scalp
2 EEG Instrumentation, Montage, Polarity, and Localization 17

the difference in potential between contiguous


electrodes, hence amplifying local potentials.
Visual detection of differences in local potentials
is easier on a bipolar montage particularly when
“phase reversal” is seen, signifying a negative
event taking place in the region of the electrode
that is common to the two channels where
polarity changes (Fig. 2.8).
A referential montage on the other hand
would be highly susceptible to external noise but
it would be able to detect both local (near field)
and distant (far field) potentials. The amplitude
of the deflection on a referential montage would
be a closer representation of the absolute poten-
tial at an electrode.

Fig. 2.4 International 10–20 system for electrode


placement Acquiring, Filtering, and Displaying
the EEG Signal
channel is the first lead in the second channel,
and so forth until the end of the chain. In a ref- Electrocerebral potentials are in the microvolt
erential montage, each channel represents the range and contaminated by significant ambient
difference of the potential of any given electrode electrical noise. In order to record, isolate, and
with a single chosen electrode (Fig. 2.7). represent an interpretable tracing, certain pro-
Each configuration has its advantages and cessing of the signals is required.
disadvantages. In a bipolar montage, external Differential amplifiers and common mode
noise can easily be canceled out as it measures rejection: Each electrode records potentials

Fig. 2.5 Bipolar montage with anterior-to-posterior chains (longitudinal bipolar or double-banana montage)
18 K. Tufenkjian

Fig. 2.6 Bipolar montage with transverse chains

Fig. 2.7 Referential montage using Cz as the common reference

generated by both the brain and the environment. Filtering the EEG signal: Conventional EEG
Filtering out the surrounding noise is done with a interpretation requires the exclusion of very low
differential amplifier, which excludes the signals frequencies using a high-pass (or low frequency)
recorded by both electrodes in a channel and filter, very high frequencies using a low-pass (or
amplifies the differences in between. This func- high-frequency filter) filter, or a specific band of
tion is also known as common mode rejection. frequencies using a high-pass filter.
2 EEG Instrumentation, Montage, Polarity, and Localization 19

Fig. 2.8 To the left is a representation of a negative adjacent electrodes in the same chain. The highest
potential and its field as recorded from the scalp. Right negative potential is recorded from contact C; this would
upper is a representation of that potential as represented lead to B–C to have a positive value (down-going tracing
on a referential montage. Note that the amplitude of the on EEG), while C–D will have a negative value
spike corresponds to the proximity of the recording (up-going). This would result in the so-called phase
electrode to the negative filed maximum. Right lower is reversal on a bipolar montage, where the common
the same potential as recorded from the same electrodes electrode is closest to the maximum negative potential
but arranged in a longitudinal bipolar montage. Each as recorded from the scalp and within that chain
channel represents the difference in potential between two

A signal-filtering device is made from a cir- an alternating current (AC) source, then once the
cuit containing a capacitor and a resistor. A ca- polarity of the source is reversed a new current
pacitor contains two conducting surfaces may pass in the circuit until the plates of the
separated by non-conducting material. When capacitor are once more saturated, though with
placed as a part of a circuit, opposing charges opposite polarity. Increasing the frequency of the
will accumulate on each plate until each plate is AC current above the limit of the saturation of a
“crowded” and the current stops (Fig. 2.9). If this capacitor will allow for a current to pass con-
is a part of a circuit with a direct current (DC), tinuously through the circuit.
then no further current may pass once the In the past, EEGs were obtained using analog
capacitor is saturated. If, however, the circuit has recorders. Frequency filtering in these machines
was done with devices that utilize resistor/
capacitor circuits. Such filters are characterized
by their time constant, which determines what
frequencies will pass through.
The time constant is determined by the
amount of resistance and capacitance in the cir-
cuit. It is defined as the time needed to discharge
the capacitor in the circuit to 36.8% of its initial
full charge. Its value is inversely related to the
frequency that will pass through the filter. For
example, using a filter with a higher time con-
stant will allow the lower frequencies to pass
Fig. 2.9 Resistor–capacitor circuit through. With the more recent digital machines,
20 K. Tufenkjian

the EEG signal from each electrode is digitized than a certain value to pass. The low-pass filter
first and frequency filtering is done using soft- is set to 70 Hz in the usual scalp EEG reading
ware processing. settings. Changing this to 35 Hz will allow only
A low-pass filter (also known as frequencies lower than 35 Hz to pass through.
high-frequency filter) allows frequencies lower This will filter out a lot of the faster myogenic

Fig. 2.10 a High-frequency filter set at 70 Hz. Note the b The high-frequency filter is changed to 15 Hz. Most of
abundant myogenic artifact in the first three seconds of the myogenic artifacts are removed. There is a concurrent
the recording. A high-amplitude spike is also noted. reduction in the amplitude of the spike
2 EEG Instrumentation, Montage, Polarity, and Localization 21

Fig. 2.11 a EEG with the low-frequency filter set to activity. c Low-frequency filter changed to 5 Hz. Only
1 Hz. b Same EEG with the low-frequency filter changed frequencies above 5 Hz pass. Note that there has been no
to 2 Hz. Note the reduction of the amplitude of the slow effect on the fast frequency myogenic artifact

artifact and will also slightly reduce the ampli- A high-pass filter (also known as low-
tude of signals with a steep rise time, such as frequency filter) allows higher frequencies to
epileptiform spikes and sharp waves pass, usually set to about 1 Hz (corresponds to a
(Fig. 2.10). time constant of about 0.16 s) for routine scalp
22 K. Tufenkjian

Fig. 2.11 (continued)

EEG reading. Raising this to 2 Hz, for example,


will filter out some of the lower frequencies giving Digital EEG Acquisition, Processing,
a more flat look to the EEG. This will also reduce and Display
the amplitude of the slower waveforms (Fig. 2.11).
A band pass or a band stop filter is also used. In digital EEG machines, an additional electrode
Commonly used such filters are notch filters, used as the machine reference is also needed. The
which stop a very narrow band around the 50 or signal from each electrode is recorded as the
60 Hz noise generated from alternating current difference in potential between that electrode and
sources such as city power lines. the machine reference, which is then stored as
Filtering in intracranial recording: Subdural digital data (bits). The signal from each channel
and depth electrodes allow the recording of fre- is recorded and stored at regular intervals. This is
quencies that fall outside the usual range of the reflected in the sampling rate of the EEG
scalp EEG. These include high frequencies machine. Most current commercially available
ranging from 80 to 500 Hz known as machines have a sampling rate that ranges
high-frequency oscillations (HFOs) and very between 256 and 1024 Hz. Higher sampling
slow frequencies appearing as slow baseline rates allow more accurate recording of brain
shifts. Specialized systems are needed to acquire signals and smoother appearance of the wave-
these activities. High-pass and low-pass filters forms, but require higher data storage capacity.
are manipulated to facilitate viewing the required Sampling rates determine what EEG fre-
range (Fig. 2.12). quencies can accurately be represented. If the
2 EEG Instrumentation, Montage, Polarity, and Localization 23

Fig. 2.12 a High-frequency oscillations at 250 Hz—the onset represents magnification of the boxed area. b Slow
baseline shift at the beginning of a seizure recorded with grid electrodes
24 K. Tufenkjian

Fig. 2.13 High-frequency signal is sampled at a rate below the Nyquist rate. The resulting waveform remarkably
misrepresents the original waveform, resulting in aliasing

sampling rate falls below a certain point, then the outlet. The third prong ensures shunting of
resulting waveform would no longer represent excess current from the EEG machine to the
the original one. This erroneous representation is earth ground. All electrical devices in the EEG
called aliasing. The Nyquist sampling theorem room should be connected to a common earth
determines that the sampling rate should be at ground.
least twice the frequency of the original signal to A single ground electrode is placed any-
avoid aliasing, or distortion of waveforms. where on the patient and connects to the
The ACNS guidelines recommend a sampling appropriate jack in the input jackbox of the
rate at 3 times or more the frequency of the EEG machine. The patient should not be
original signal (Fig. 2.13). A filter is used on the connected to the earth ground. In ICU setting,
signal prior to digitizing to exclude all frequen- a patient may be connected to another elec-
cies above a certain frequency determined by the trical device with a ground connection. Double
Nyquist theorem (anti-aliasing filter). grounding should be avoided in these situa-
Another important aspect is the monitor dis- tions [2].
play. Most LCD monitors can display 1920 dots
(pixels) horizontally. The sampling rate on the
EEG machine may actually exceed the capacity EEG Artifacts
of the monitor display. Low-definition monitors
will give a “grainy” tracing, and this could be a Artifacts will be discussed in Chap. 3, but in the
particular concern with high-frequency activity remaining part of this chapter we will provide
(Fig. 2.14). some EEG examples with the purpose of further
training of the reader’s artifact pattern recogni-
tion. Artifacts may arise from the electrical
EEG and Patient Safety environment as well as bioelectrical sources
originating from the patient (Figs. 2.15, 2.16,
Proper grounding during EEG is an important 2.17, 2.18, 2.19, 2.20, 2.21, 2.22, 2.23 and
patient safety issue. The EEG machine should 2.24).
be connected to a three-pronged hospital grade
2 EEG Instrumentation, Montage, Polarity, and Localization 25

Fig. 2.14 Effect of reducing the display resolution from 1920  by 1080 pixels (top record) to 1280  by 1024
pixels (bottom record)
26 K. Tufenkjian

Fig. 2.15 a Electrode pop artifact. Poor contact at the P3 contact with the scalp. The difference in impedance
and C4 electrodes resulted in an isolated potential at these compared with the other electrodes interferes with the
two contacts (60-Hz notch filter on). b The notch filter is ability of the differential amplifier to reject the 60 cycle
removed and the 60-Hz artifact is now seen at the P3 and noise which actually gets amplified [3]
C4 electrodes, which have higher impedance due to poor
2 EEG Instrumentation, Montage, Polarity, and Localization 27

Fig. 2.16 Movement artifact. The disorganized EEG potentials do not have the typical field seen in brain-generated
waveforms

Fig. 2.17 Phone ringing artifact


28 K. Tufenkjian

Fig. 2.18 Sweat artifact. Slow undulation (less than 1 Hz) of the EEG tracing is seen in a diaphoretic patient

Fig. 2.19 Myogenic artifact. These high-frequency activities are generated by the frontalis and temporalis muscles;
therefore, these are seen maximally in the anterior midline and temporal chains
2 EEG Instrumentation, Montage, Polarity, and Localization 29

Fig. 2.20 Rhythmic myogenic artifact is seen during chewing

Fig. 2.21 Blink artifact. The cornea has a slightly electrodes. This is represented as a down-going wave-
positive potential compared to the retina. During a blink, form, which falls in amplitude exponentially from the
the eyelid makes contact with cornea allowing for that front to the back
positive potential to be recorded from the anterior frontal
30 K. Tufenkjian

Fig. 2.22 Lateral eye movement artifact and lateral (arrows). This is followed by a slow rightward movement
rectus spike. In this example, the patient is reading a of the eyes with the potentials slowly shifting to the
book, the rapid saccade to the left brings the positive opposite direction (ovals). A small spike preceding the
potential of the cornea closer to the left frontal channels saccade is noted which is generated from the left lateral
producing a positive phase reversal at F7 and away from rectus muscle (arrowhead)
the right with a resulting negative phase reversal at F8
2 EEG Instrumentation, Montage, Polarity, and Localization 31

Fig. 2.23 Glossokinetic artifact. The difference in potential between the tip and the base of the tongue produces
diffuse, slow waves with a frontal maximum
32 K. Tufenkjian

Fig. 2.24 ECG artifact. Small sharp transient can be seen time locked to the ECG QRS potentials. The lower tracing
shows this ECG contamination during an EEG performed for the evaluation of electrocerebral inactivity

2. American Clinical Neurophysiology Society. Mini-


References mum technical requirements for performing clinical
EEG. 2006.
1. Ebersole JS. Current practice of clinical encephalog- 3. Aminoff MJ. Aminoff’s electrodiagnosis in clinical
raphy. 4th ed. Philadelphia: Wolters Kluwer Health; neurology. 6th ed. Elsevier Saunders; 2012.
2014.
Normal EEG Variants and Artifacts
3
Raed Azzam, Mohamad Z. Koubeissi and Nabil J. Azar

Accurate EEG interpretation greatly relies on • Reactive to eye opening and closure.
pattern recognition of normal EEG variants and • Fast alpha variant is similar to beta rhythms
artifacts. Several of these normal patterns may except that it is located in occipital rather than
mimic pathologic EEG findings, leading to in frontal, central, and parietal regions.
potential misinterpretation of normal EEG trac- • Slow alpha variant is more difficult to discern
ings. Some of these normal patterns may even without clear reactivity to eye closure and
mimic ictal discharges, leading to overtreatment. opening.
This chapter covers the electrographic features of
common normal EEG variants and artifacts [1–3]. Alpha Squeak (Fig. 3.3):
Normal EEG variants often present with dis- • Transient increase in frequency immediately
tinctive electrographic features that include after eye closure.
morphology, distribution, and occurrence in • Assessment of the frequency of the posterior
specific stages (wakefulness, drowsiness, or background rhythm should not include the
sleep). Some benign EEG variants are better first 0.5–1 s after eye closure in order to avoid
visualized when using a preferred EEG montage. overestimation.
Fast Alpha (Fig. 3.1) and Slow Alpha Vari-
ants (Fig. 3.2): Rhythmic Mid-Temporal Theta Bursts of
Drowsiness (RMTTBD) (Fig. 3.4):
• Harmonics of the posterior background
rhythm: twice as fast (fast alpha variant) or • Also known as Rhythmic Mid-Temporal
half as fast (slow alpha variant). Discharges (RMTD) and psychomotor
• Notched appearance can resemble Rhythmic variant.
Mid-Temporal Theta Bursts of Drowsiness • Composed of rhythmic bursts or trains of
(RMTTBD) except that it occurs over the theta waves (5–7 Hz) usually with a notched
posterior head regions. appearance that is maximal in mid-temporal
regions.
• Occurs bilaterally with a shifting emphasis
from side to side.
R. Azzam  M.Z. Koubeissi  N.J. Azar (&)
• It is monomorphic and monorhythmic and
Department of Neurology, Vanderbilt University does not evolve into other waveforms or
Medical Center, 1161 21st Ave. South, a-0118 frequencies.
Medical Center North, Nashville, • Occurs during relaxed wakefulness and
TN 37232-2551, USA
e-mail: nabil.azar@vanderbilt.edu
drowsiness.
M.Z. Koubeissi
e-mail: koubeissi@gmail.com
Midline Theta Rhythm (Fig. 3.5):

© Springer Science+Business Media LLC 2017 33


M.Z. Koubeissi and N.J. Azar (eds.), Epilepsy Board Review,
DOI 10.1007/978-1-4939-6774-2_3
34 R. Azzam et al.

Fig. 3.1 Fast alpha variant (arrow); sensitivity 7 lV/mm, low frequency filter (LFF) 1 Hz, high-frequency filter (HFF)
70 Hz

Fig. 3.2 Slow alpha variant (arrows); sensitivity 7 lV/mm, LFF 1 Hz, HFF 70 Hz
3 Normal EEG Variants and Artifacts 35

Fig. 3.3 Alpha squeak (arrows); sensitivity 7 lV/mm, LFF 1 Hz, HFF 70 Hz

Fig. 3.4 Rhythmic Mid-Temporal Theta Bursts of Drowsiness (RMTTBD) (arrows); sensitivity 7 lV/mm, LFF 1 Hz,
HFF 70 Hz
36 R. Azzam et al.

Fig. 3.5 Midline theta rhythm (arrow); sensitivity 7 lV/mm, LFF 1 Hz, HFF 70 Hz

• Also known as Ciganek rhythm. • Widespread distribution with maximal


• Most prominent in the central vertex lead. amplitude over parietal-posterior temporal
• Consists of a rhythmic train of 5–7 Hz head regions.
smooth, sinusoidal, arciform, spiky, or • Usually bilateral but may be asymmetric.
mu-like activity. • May resemble a subclinical EEG seizure
• Occurs during wakefulness and drowsiness. discharge but typically does not correlate with
• Variable reactivity to eye opening and alerting. clinical seizures (this is however
controversial).
Subclinical Rhythmic Electrographic Dis-
charge in Adults (SREDA) (Fig. 3.6a–d): 14- and 6-Hz Positive Bursts (Fig. 3.7):

• Uncommon pattern. • Also known as ctenoids.


• Seen in people older than 50 years. • Occur during drowsiness and light sleep.
• Occurs at rest or drowsiness or during • Consist of short trains of arch-shaped wave-
hyperventilation. forms with alternating positive spiky com-
• Abrupt onset of mixed frequencies in the ponents and a negative, smooth, rounded
delta and theta ranges that evolve into a waveform that resembles a sleep spindle with
rhythmic pattern consisting of a sharp positive phase.
sharp-contoured components 5–7 Hz lasting • Mostly asynchronous and occurs bilaterally
from 20 s to a few minutes. with shifting predominance.
3 Normal EEG Variants and Artifacts 37

Fig. 3.6 Consecutive EEGs showing subclinical rhythmic electrographic discharge in adults (SREDA)
38 R. Azzam et al.

Fig. 3.6 (continued)


3 Normal EEG Variants and Artifacts 39

Fig. 3.7 14- and 6-Hz positive bursts (arrow); sensitivity 7 lV/mm, LFF 1 Hz, HFF 70 Hz

Fig. 3.8 6-Hz spike-and-wave bursts (arrow); sensitivity 7 lV/mm, LFF 1 Hz, HFF 70 Hz

• Predominantly 14 Hz and the 6 Hz can occur • May be enhanced in Reye’s syndrome.


either independently or in association with 14 Hz.
• Maximal amplitude over the posterior tem- 6-Hz spike-and-wave bursts (Fig. 3.8):
poral region.
• Better seen in a referential montage (ear • Also known as phantom spike-and-wave.
references). • Consist of 5–7 Hz brief bursts of a subtle
• Peak at the age of 13–14 and decrease in low-amplitude spikes followed by a more
incidence with increasing age. prominent slow wave.
40 R. Azzam et al.

Fig. 3.9 Benign sporadic sleep spikes (BSSS) (arrows); sensitivity 7 lV/mm, LFF 1 Hz, HFF 70 Hz

• Occurs during relaxed wakefulness and • Usually do not have a slow-wave component
drowsiness disappearing with deep sleep and do not occur in repetitive trains.
(unlike spike-and-wave discharges which • Commonly occur unilaterally but can inde-
persist during sleep). pendently involve the opposite hemisphere.
• Usually occurs bilaterally and synchronously.
• Two types have been described: FOLD Wickets (Fig. 3.10):
(Female Occipital Low-amplitude and
Drowsiness) and WHAM (Wake • Intermittent trains of monophasic arciform
High-amplitude Anterior and Male). waveforms or single spike-like waveforms.
• FOLD is considered to be benign, whereas • Occur exclusively on one side (left > right) or
WHAM is more likely to be associated with bilaterally with shifting predominance.
seizures. • Frequency of 6–11 Hz and possibly represent
fragments of temporal alpha activity or the
Benign Sporadic Sleep Spikes or (BSSS) third rhythm.
(Fig. 3.9): • Seen during wakefulness, drowsiness, and
light sleep, and disappear in deeper sleep.
• Also known as small sharp spikes (SSS) or • Should not be mistaken for a temporal seizure
benign epileptiform transients of sleep (BETS). discharge or spikes; if a single spike is found,
• Seen in adults during drowsiness and light it should be compared with a train of wicket
sleep and disappear with deeper sleep. spikes on other pages.
• Low-voltage (<50 µV) and short-duration • Not associated with a slow wave and do not
(<50 ms) monophasic or diphasic spike with distort the background.
abrupt ascending limb and a steep descending
limb. The Third Rhythm (Fig. 3.11):
3 Normal EEG Variants and Artifacts 41

Fig. 3.10 Left wicket rhythm (arrows); sensitivity 7 lV/mm, LFF 0.5 Hz, HFF 70 Hz

Fig. 3.11 The third rhythm (arrows); sensitivity 7 lV/mm, LFF 1 Hz, HFF 70 Hz

• Also called temporal alphoid rhythm. defect or recorded from epidural


• Rhythmic activity in the alpha and upper electrodes.
theta range over the mid-temporal region. • The origin and function remain debatable,
• Rarely detected in the scalp EEG and more with some related it to cortical auditory
commonly seen when there is a local bone function.
42 R. Azzam et al.

Fig. 3.12 Frontal arousal rhythm (FAR) (arrows); sensitivity 7 lV/mm, LFF 1 Hz, HFF 70 Hz

Frontal Arousal Rhythm (FAR) (Fig. 3.12): • Resemble the Greek letter k with monophasic
or diphasic waveforms with prominent
• Trains of 7–20 Hz waveforms that occur surface-positive waveform.
predominantly over the frontal regions lasting • Occurs over the occipital regions when sub-
up to 20 s. ject is visually scanning.
• May be notched in appearance with varying • Bilateral and synchronous but may be
harmonics. asymmetric.
• Seen mainly in children following arousal • Possibly represent an evoked cerebral
from sleep and disappears with full response to visual stimuli.
wakefulness.
Positive Occipital Sharp Transients (POSTs)
Mu-Rhythm (Fig. 3.13): (Fig. 3.15):

• Archiform 7–11 Hz waveforms occurring • Sharp-contoured, positive transients over the


independently over the central head regions. occipital regions.
• Functionally related to the sensorimotor cor- • Bilateral synchronous but may be asymmetric.
tex and is attenuated by touch, active, or • Occur during light sleep.
passive movement of the extremities, or
thought of such movement. Mitten Pattern (Fig. 3.16):

Lambda Waves (Fig. 3.14): • Seen during sleep.


3 Normal EEG Variants and Artifacts 43

Fig. 3.13 Mu-rhythm (arrows); sensitivity 7 lV/mm, LFF 1 Hz, HFF 70 Hz

Fig. 3.14 Lambda waves (arrows); sensitivity 7 lV/mm, LFF 1 Hz, HFF 70 Hz

• Consists of fast-wave and slow-wave com- of a spindle and the hand portion by the
ponents and resembles a mitten with the slower wave component.
thumb of the mitten formed by the last wave • Variant of a vertex wave or K-complex.
44 R. Azzam et al.

Fig. 3.15 Positive occipital sharp transients (POSTs) (arrows); sensitivity 7 lV/mm, LFF 1 Hz, HFF 70 Hz

Fig. 3.16 Frontal mitten pattern (arrows); sensitivity 7 lV/mm, LFF 1 Hz, HFF 70 Hz
3 Normal EEG Variants and Artifacts 45

Fig. 3.17 Breach rhythm in the left centro-parietal region (arrows); sensitivity 7 lV/mm, LFF 1 Hz, HFF 70 Hz

Breach Rhythm (Fig. 3.17): • All eye movements are generated by corneal
and retinal potentials.
• High-voltage activity over a skull defect. • It is a direct current represented by a dipole
• Consists of a spiky appearance and sharply whose positive pole localizes to the cornea
contoured arciform 6–11 Hz waveforms. and negative pole localizes to the retina.
• Most prominent in temporal and central • The electrodes involved are closest to the
regions and can usually represent wickets and eyeball: Fp1, Fp2, F7, and F8.
mu-rhythms depending on the location of the • The electrodes surrounding the eyeball detect
skull defect. a positive potential which voltage is usually
greater than the cerebral potential.
EEG artifacts are activity recorded by the • For example, when the eyes are closed, the
EEG that is usually not cerebral in origin. Several eyeballs move upward to their natural
sources of artifacts exist and can be divided into position (Bell’s phenomenon) and this
physiologic and non-physiologic. upward movement is detected by a positive
potential recorded at Fp1 and Fp2
(Fig. 3.21). This activity is then followed
Physiologic EEG Artifacts by a falloff recorded at the next electrodes:
F3 and F4. When the eyes are open again,
Eye movements—vertical (Fig. 3.18), horizontal the inverse occurs. If these movements
(Fig. 3.19), oblique (Fig. 3.20), flutter, and happen repetitively, they will result in a
nystagmus: blink artifact.
46 R. Azzam et al.

Fig. 3.18 Vertical eye movements: (a) up gaze, (b) down gaze; sensitivity 7 lV, LFF 1 Hz, HFF 70 Hz

Fig. 3.19 Horizontal eye movements: (a) left gaze, (b) right gaze; sensitivity 7 lV, LFF 1 Hz, HFF 70 Hz

• Another example is when eyes move to the electrode F7 and it becomes more positive
left, the activity at Fp1 and Fp2 remains than other electrodes. Because the eyes move
steady, with no change in potential. However, conjugately, the cornea is moving away from
the positive potential is detected by the F8 and it becomes less positive, or more
3 Normal EEG Variants and Artifacts 47

Fig. 3.20 Oblique eye movement: up and to the left (arrow); sensitivity 7 lV, LFF 1 Hz, HFF 70 Hz

negative, because the retina is now closer to generate an equal positive potential recorded
this electrode. This horizontal movement to in a bipolar montage recording from elec-
the left produces a positive phase reversal trodes Fp1–F7 and a large upward deflection
with a maximal positive potential at F7 and a on the channel recording Fp2–F8. This
negative phase reversal with a maximal neg- occurs because the positive potential
ative potential recorded at F8. involves both Fp1 and F7 relatively equally,
• Oblique eye movements are more difficult to and the potential difference recorded with
interpret and constitute a combination of the differential amplifier approximates to
both vertical and horizontal movements. An zero. The potential difference recorded from
eye movement upward and to the left would Fp2–F8 is negative at Fp2 and positive at
48 R. Azzam et al.

Fig. 3.21 Upward movement of the eyeball brings the positively charged cornea closer to Fp1 and the negatively
charged retina farther from Fp1, resulting in a positive or downward deflection

F8, creating an upward deflection in that • Single motor units appear as repetitive or
channel. This is due to the rules of local- single negative or positive deflections that
ization (if input 1 is more negative than have a comb-like appearance.
input 2, an upward deflection will be • The frontalis electromyogram (EMG) is seen
recorded). in frontal electrodes, as when tightly closing
• Eyelid flutter produces low-voltage slow eyes. This can also be seen during photic
activity and is often limited to Fp1 and Fp2 stimulation, a photomyoclonic response. The
electrodes. temporalis EMG is recorded from F7, F8, T7,
• Horizontal nystagmus is usually detected T8, P7, and P8. This is typically seen with
unilaterally, and the movement is recorded by jaw clenching or chewing.
the electrode on the side of the fast phase of • High-frequency filters should not be used to
the nystagmus because of the larger positive eliminate the EMG artifact because they alter
voltage of the cornea near that electrode. its appearance from a sharp wave to a more
Vertical nystagmus is rarely detected because sinusoidal frequency that resembles cerebral
of the low voltage and the distance of these beta activity.
electrodes from the eyeball.
Electrocardiographic—QRS complex
Electromyographic—lateral rectus, single (Fig. 3.23), pulse, cardioballistic:
motor units, frontalis, temporalis, swallowing,
and chewing: • The QRS complex is easily monitored by
applying electrodes to the chest. The gener-
• The lateral rectus artifact (Fig. 3.22) is a ated signal is high voltage generated by the
low-voltage motor unit potential recorded heart. This activity when recorded at the scalp
from the F7 and F8 electrodes. It appears as a constitutes a far-field potential. It is often
sharp positive deflection of very short dura- picked up by montages using ear electrodes
tion with a slow falloff as the muscle relaxes. as a reference. It is prevalent in obese
3 Normal EEG Variants and Artifacts 49

Fig. 3.22 Lateral rectus spike (arrow); sensitivity 7 lV, LFF 1 Hz, HFF 70 Hz

Fig. 3.23 Electrocardiographic artifact (arrows); sensitivity 7 lV, LFF 1 Hz, HFF 70 Hz
50 R. Azzam et al.

Fig. 3.24 Glossokinetic and chewing artifacts; sensitivity 7 lV, LFF 1 Hz, HFF 35 Hz

patients, and patients with short necks and tongue movements, such as la-la-la or
babies. ta-ta-ta. This artifact may resemble gener-
• Pulse artifact is usually confined to a single alized spike-and-wave discharges when
electrode and appears as a slow-wave poten- filtered.
tial. It occurs when an electrode is placed
over a surface artery. The electrocardiogram Galvanic (Fig. 3.25):
signal will be time locked to the slow wave
and always occurs at the same location. • This artifact is secondary to perspiration and
• A cardioballistic artifact is rhythmic delta results in high-amplitude slow-wave
activity, usually widespread in distribution, potentials.
which represents head movement with each • Standard low-frequency filters reduce this
pulse. The relationship between the cardiac artifact.
signal and these pulsations is not always • A salt bridge may be formed shorting two
time locked to any particular phase of the electrodes contacting the perspiration.
signal.
Physiologic movements—tremor (Fig. 3.26)
Glossokinetic (Fig. 3.24): and jerks:

• Movement of the tongue creates a direct • Tremor is often between 4 and 6 Hz and
current potential where the tip of the ton- localized to the body region involved. It is
gue is negative with respect to its base. often seen in the head and upper limbs.
They are frequently recorded as slow • Jerks produce enough body movement to
activity from the temporal electrodes. This move the electrodes or the head creating a
can be reproduced by having the patient potential in the recording.
repeat words or phrases that produce active
3 Normal EEG Variants and Artifacts 51

Fig. 3.25 Diffuse sweat artifact; sensitivity 7 lV, LFF 1 Hz, HFF 70 Hz

interface). They are usually restricted to one


Non-physiologic Artifacts electrode (Fig. 3.28). These may resemble
repetitive discharges in a longitudinal
Instrumental artifacts: montage.
• An electrode pop is a high-voltage deflection
• Sixty-cycle interference results from poor that exceeds the limits of the individual
electrode application. It is produced by channel sensitivity and blocks or squares off
high-impedance electrodes that affect the at the top.
input circuitry of the amplifier and also • Repetitive and rhythmic electrode artifact
common-mode rejection when impedances can be produced by tapping the electrodes
are not equal (Fig. 3.27). when the mother of the patient pats the
• Capacitative and electrostatic artifacts are baby’s back.
related to movement of wires. An example is
when someone steps on the input cable. The Environmental artifacts:
cable acts as a capacitor because of multiple
insulated wires enclosed in the cable. Moving • Radiofrequency waves are high-frequency
or stepping on the cable causes the capacitor signals that may be continuous or intermit-
to discharge and results in high-voltage tran- tent and affect some or all recording channels.
sient recorded on the EEG. This often results from being in the vicinity of
machines such as microwaves or the operat-
Electrode artifacts: ing room.

• These are mostly related to poorly attached Digitization:


electrodes, high resistance, a broken wire,
or changes in the lead–scalp interface • This often results from failure in the compo-
(change in the gel used to complete this nents used to acquire the EEG data.
52 R. Azzam et al.

Fig. 3.26 Tremor artifact; sensitivity 7 lV, LFF 1 Hz, HFF 70 Hz

Fig. 3.27 60-Hz artifact; sensitivity 7 lV, LFF 1 Hz, HFF 70 Hz


3 Normal EEG Variants and Artifacts 53

Fig. 3.28 Electrode artifact; sensitivity 7 lV, LFF 1 Hz, HFF 70 Hz

• One example is related to aliasing, which is


sampling at a rate that is less than twice the References
frequency of the high-frequency filter. This is
uncommon since most EEG instruments 1. Niedermeyer E, Lopes da Silva FH. Electroen-
sample at a rate of at least 200 samples per cephalography: basic principles, clinical applications,
channel per second. and related fields. 4th ed. Baltimore: Williams &
Wilkins; 1999 (xi, 1258 p., [8] p. of plates).
• With the ability to make changes in sensi- 2. Daube JR, Rubin DI. Clinical neurophysiology, in
tivity, filtering, and montages, many artifacts contemporary neurology series; 75. New York:
can be created where the EEG may appear Oxford University Press; 2009. p. 1 online resource
very different from the true signal. This often (xxvii, 886 p. [6] p. of plates).
3. Ebersole JS, Pedley TA. Current practice of clinical
results in misinterpretation of the EEG as electroencephalography. 3rd ed. Philadelphia: Lippin-
possibly ictal in nature. cott Williams & Wilkins; 2003 (xvi, 974, 3 p).
Neonatal EEG and Neonatal Seizures
4
Kohilavani Velayudam and Ahsan N.V. Moosa

Neonatal EEG • Quick clinical behavioral state change hap-


• Neonatal EEG assists in studying the func- pens and parallel EEG changes also happen.
tional integrity of immature neonatal cortex
and its connections. The exact generators are
unknown for these activities. Neonatal EEG Technical Information that is Unique
helps to assess the prognosis for the neonates in Neonatal EEG and that May Affect
who are at risk for neurological sequelae Interpretation
following a CNS insult.
• Rapid rate of cerebral development happens • First step is identifying the postconceptional
during the neonatal period. Recognizing and age.
identifying age-dependent findings from • Behavioral state of the infant: Whether the
conceptional age less than 28 weeks through infant is awake or asleep.
44 weeks is very critical. • Medications like benzodiazepines, neuro-
• EEG findings in a newborn rapidly change muscular blocking, sedatives, hypnotics,
with brain maturity and hence need to be anxiolytics, general anesthesia, and
carefully correlated with the age of the baby. antiepileptic drugs.
Postconceptional age is frequently used as the • Temperature and depth of hypothermia in
reference. babies being cooled.
– Postconceptional age (PCA) = gestational • Topographic—caput or cephalohematoma,
age (prenatal) + chronological age (postnatal) scalp IV placement, VP shunt.
– Postmenstrual age is also used (usually
postmenstrual age = PCA + 2 weeks)

Neonatal Montage

• Newborn typical montages use both longitudinal


K. Velayudam (&) and transverse chains to record term and preterm
Department of Pediatric Neurology, Emory neonates with head circumference <35 cm.
University, 1605 Chantilly Drive NE, Suite 300,
Atlanta 30324, GA, USA • It is modified 10–20 system with a minimum
e-mail: velayuk@gmail.com of 9 electrodes. Cz electrode plays an
A.N.V. Moosa important role because abnormal positive
Section of Pediatric Epilepsy, Epilepsy Center, sharp waves (and sometimes seizures) can be
Cleveland Clinic, Cleveland, OH, USA

© Springer Science+Business Media LLC 2017 55


M.Z. Koubeissi and N.J. Azar (eds.), Epilepsy Board Review,
DOI 10.1007/978-1-4939-6774-2_4
56 K. Velayudam and A.N.V. Moosa

Fig. 4.1 Neonatal


montage. Special montage
—preterm or newborn with
head
circumference <35 cm

confined to this single electrode without


involving or spreading to nearby regions Timing of the Neonatal EEG
(Fig. 4.1).
• Usually, 16 channels recording are done, • EEG is ideally done within 24 h following
which should include 2 or more non-cerebral the insult.
channels. • EEG changes are very dynamic, and signifi-
• Non-cerebral electrodes include EKG, EMG, cant changes in background activity may
ocular, and respiratory belt. It helps in staging happen rapidly (in hours). Serial EEGs pro-
the sleep cycle, abnormal body movements, vide valuable information for therapy and
and also to identify the artifacts. prognostication.
• Duration of the recording: 60-min recording
is mandatory in an attempt to capture EEG in
various behavioral states. EEG abnormalities
are usually the most prominent in quiet Normal Developmental Landmarks
sleep. In a term neonate, the total sleep cycle
lasts anywhere between 45 and 60 min (ac- Normal developmental landmarks on a new-
tive sleep: *25 min; quiet sleep: *20 min; born’s EEG provide information regarding the
transitional sleep: *15 min). functional maturity and reflect the PCA of the
baby. EEG findings in acute encephalopathy in
4 Neonatal EEG and Neonatal Seizures 57

Fig. 4.2 Trace discontinue. Trace discontinue pattern seen in 26–28 weeks of PCA (Picture source Atlas of neonatal
encephalography—Third Edition Lippincott Williams and Wilkins A Wolters Kluwer Company)

newborns frequently show the patterns that may PCA <30 weeks. It is characterized by
be normal for a baby of a lesser PCA. Such high-amplitude (50–300 µV) bursts of mixed
patterns are described as dysmature patterns and frequency (theta and alpha riding over delta)
indicate encephalopathy in the appropriate set- which are seen simultaneously on both the
ting. This may improve rapidly with the hemispheres followed by low-amplitude
improvement in encephalopathy. Developmental periods of quiescence (<25 µV) which can
landmarks are best categorized in 3 phases: <30 last anywhere between few seconds and
weeks, 30–37 weeks, and >37 weeks. 1 min (inter-burst interval—IBI) (Fig. 4.2).

PCA <30 Weeks of Gestation


Inter-burst interval (IBI)
EEG may be indistinguishable from awake and • The key factor that determines the IBI in
sleep in extremely premature infants of healthy babies is the postconceptional age. As
PCA <30 weeks. A discontinuous EEG pattern the PCA increases, the IBI decreases.
termed trace discontinue is noted in all behav- • Approximately 30 weeks or older, median
ioral states. IBI of 8 s or less has higher survival than
those with longer median IBI.
• Approximately IBI lasts anywhere between
Trace Discontinue 20 and 35 for neonates with PMA <30–
33 weeks with <25 µV. After 34 weeks, it
• Tracé discontinue pattern is a predominant significantly reduces, and closer to the term, it
pattern seen in preterm neonates with
58 K. Velayudam and A.N.V. Moosa

Fig. 4.3 Trace alternant. Trace alternant pattern seen in quiet sleep in 38–30 weeks of PCA (Picture source Atlas of
neonatal encephalography—Third Edition Lippincott Williams and Wilkins A Wolters Kluwer Company)

ranges anywhere between 6 and 10 s and has For these reasons, the abnormalities are most
higher voltage >25 µV. frequently seen in the quiet sleep phase.
• Prolonged IBI is seen in acute encephalopa- Trace alternant: These are characterized by
thy secondary to any type of brain injury, but high-amplitude (50–150 µV) synchronous burst
is also commonly seen in moderate-to-severe of mixed frequencies predominantly delta activ-
hypoxic ischemic encephalopathy. ity for 3–10 s followed by 3–5 s of
low-amplitude burst (25–50 µV) of predominant
theta activity. This patterns replaces the trace
discontinue with advanced PCA. This pattern is
PCA Between 30 and 37 Weeks seen only in quiet sleep in neonates between
PCA of 36 and 38 weeks (Fig. 4.3).
EEG starts showing variation in different
behavioral states around 35 weeks of PCA. • TA usually starts appearing in neonates >35
Identifying different sleep stages in a newborn’s weeks and can be seen until 42 weeks but no
EEG is critical for optimal interpretation. EEG longer than 44 weeks of PCA.
maturational changes occur first in active sleep; • In active sleep (and later in wakefulness), TA
after a lag of about 2 weeks, the awake periods is replaced by more continuous pattern char-
start showing more mature patterns. Quiet sleep acterized by uninterrupted, low-to-medium
is the last stage to show more mature changes. amplitude, mixed EEG activity with <2 s of
4 Neonatal EEG and Neonatal Seizures 59

Fig. 4.4 a Development


of continuous activity
(Picture source Levin and
Luders Comprehensive
Clinical
Neurophysiology W.B.
Saunders Company).
b Evolution of the
background activity

voltage attenuation (<25 µV), and seen in In certain conditions like non-ketotic hyper-
mostly neonates with PCA >30 weeks. glycinemia, the EEG bursts may be accompanied
• TA pattern is replaced by medium-to-high by myoclonic jerks (Fig. 4.5).
voltage delta slow wave sleep after PCA of Synchrony: Burst of morphological similar
44–48 weeks (Fig. 4.4a, b). activity in the homologous head regions is sep-
arated by <1.5 s.

• <28 weeks: no asynchrony


• 28–32 weeks: 70% synchrony
Abnormalities in Voltage • >37 weeks: mostly synchronous
• Abnormal finding: Asynchrony of >1.5 s
Excessive discontinuity: Abnormal discontinu- may be significant if in excess for the PCA
ous tracing separated by prolonged IBI for PCA. (Fig. 4.6).
This is usually seen in neonates with severe
hypoxic ischemic encephalopathy, meningitis,
encephalitis, and severe intraventricular
hemorrhage. Normal Graphoelements
Burst suppression: Burst suppression pattern
is a pattern with high-amplitude burst (50– Delta brushes: Delta brush is characterized by
300 µV) of delta and theta frequency with sharp 0.3–1.5 Hz delta activity superimposed with beta
waves and spikes followed by severe background or fast activity (18–22 Hz).
suppression (<5 µV). Usually, the background
shows no reactivity to stimuli. • Other names are beta-delta complexes, ripples
of prematurity, spindles/delta bursts.
60 K. Velayudam and A.N.V. Moosa

Fig. 4.5 Amplitude


review

Fig. 4.6 Synchrony


(Picture source Levin and
Luders Comprehensive
Clinical
Neurophysiology W.B.
Saunders Company)

• They usually appear in the central region first Frontal sharps (Encoches Frontales): These
followed by temporal and occipital and also are blunt isolated biphasic broad sharp transients
disappear in the same order. (0.5–0.75 s) seen in the frontal region with initial
• Most commonly seen during 24–34 weeks smaller negativity followed by prominent posi-
and starts to disappear first in the active sleep tivity phase.
after 30 weeks. They can be present in the
quiet sleep up to 38 weeks. • They are usually symmetric and synchronous;
• Abnormal findings: Delta brushes are often • Frequently seen in the transitional stage of
asynchronous, but when consistently absent sleep;
on one side, a concern for some cerebral • Usually seen between 33 and 46 weeks of
dysfunction is raised; if it is persistently pre- PCA with peak around 35 weeks;
sent beyond 44 weeks then it is consistent • Mostly associated with rhythmic bi-frontal
with dysmaturity (Fig. 4.7). delta; and
4 Neonatal EEG and Neonatal Seizures 61

Fig. 4.7 Delta brushes in 29–30 weeks of PCA: Delta encephalography—Third Edition Lippincott Williams and
brushes seen in the bilateral central regions in a discon- Wilkins—A Wolters Kluwer Company)
tinuous background (Picture source Atlas of neonatal

Fig. 4.8 Frontal sharps in 34–35 weeks of PCA (Picture source Atlas of neonatal encephalography—Third Edition
Lippincott Williams and Wilkins—A Wolters Kluwer Company)
62 K. Velayudam and A.N.V. Moosa

• Abnormal findings: Consistently absent on Other Abnormal EEG Findings


one side or any asymmetry raises concern
for structural lesion on the other side Abnormalities in a newborn EEG can be in
(Fig. 4.8). various aspects of the EEG such as continuity,
symmetry, synchrony, and amplitude, excess of
sharp waves, reactivity, seizures, and abnormal
representation of normal graphoelements. Some
Temporal Theta of the abnormalities related to graphoelements
and dysmature patterns were already discussed in
• This graphoelement is a developmental mar- earlier sections.
ker that appears as a 2-s burst of 25–120 µV Asymmetry: Asymmetry is defined as a per-
theta burst over the temporal region; sistent difference in amplitude/voltage of >50%
• Typically present between 25 and 32 weeks between homologous regions of right and left
and disappears by 34 weeks hemispheres. It should be interpreted with cau-
tions as extracranial factors such a caput suc-
cedaneum, cephalhematoma, and subgaleal
hemorrhage may result in similar findings.
PCA between 38 and 44 weeks. Positive sharp waves: These are abnormal
positive polarity sharp waves seen mostly in the
• Awake and sleep contains continuous, rolandic and central vertex regions around the
low-to-medium voltage, mixed frequency 5th–8th postnatal day in preterm neonates with
predominantly theta and delta with overriding severe intraventricular hemorrhages.
beta activity (activite moyenne);
• Rudimentary spindles starts appearing in • Examples seen in neonates with interventric-
sleep state; and ular hemorrhage, hydrocephalus, and
• Sleep–wake cycling is more distinctive in periventricular leukomalacia.
terms of neonates after 37 weeks. Fig. 4.9— • It is an electrographic marker often linked to
Developmental Landmarks. parenchymal white matter injury and usually
disappears in 3–4 weeks.

Fig. 4.9 Developmental


landmarks (Picture source
Levin and Luders
Comprehensive Clinical
Neurophysiology W.B.
Saunders Company)
4 Neonatal EEG and Neonatal Seizures 63

Fig. 4.10 Sharp electrographic transients (Picture source Levin and Luders Comprehensive Clinical Neurophysiol-
ogy W.B. Saunders Company)

Table 4.1 Normal and abnormal sharp electrographic transients


Normal sharp transients Abnormal sharp transients
Bitemporal, central One persistent location
Monophasic or diphasic Variable and polyphasic
Polarity: negative Polarity: negative or positive
Amplitude usually <75 lV Amplitude usually >150 lV
Duration <100 ms Duration >150 ms
Sharp waves seen synchronously and asynchronously Spikes, occurs in long runs
Normal background Abnormal background
Quiet sleep Awake and quiet sleep

Sharp transients: These are frequently seen Dyschronism/dysmature: Dyschronism is


in temporal, central, and frontal regions, and less defined as any discrepancy between clinically
frequently in vertex and occipital regions. determined conceptional age and EEG-derived
Abnormal sharp transients are differentiated conceptional age. Discrepancy of 2 weeks or less
from the Encoches Frontales by the presence of indicates transient CNS dysfunction, and of
associated frontal slowing, usually asymmetric 3 weeks or more indicates higher likelihood of
and seen mostly in active sleep and awake state persistent impairment of CNS function.
(Fig. 4.10 and Table 4.1).
64 K. Velayudam and A.N.V. Moosa

Neonatal Seizures Conceptional age:


• <30 weeks—higher incidence *3.9%
• Incidence is *1.5–5.5/1000 live births; • 30 weeks—1.5%
• It is the sign of acute brain injury and mostly Birth weight:
happens within the first week of life; and • <1500 gm ! 57/1000 live birth
• Early recognition and initiation of treatment is • 1500–2499 gm ! 5/1000 live birth
very important to prevent brain damage. • 2500 gm ! 3/1000 live birth.

Risk factors: There are two main specific risk


factors:

Table 4.2 Etiology—neonatal seizures. Modified from Chapter Neonatal seizures in Volpe J Neurology of Newborn.
5th ed
Etiology Key features % Patients
with normal
development
Hypoxic Prenatal: toxemia, fetal distress, abruptio – Common cause both in term and 50%
ischemic placentae, cord compression preterm infants
encephalopathy Perinatal: iatrogenic, maternal – Usually within 4 days of life
(*32%) hemorrhage, fetal distress – Focal clonic or multifocal clonic
Postnatal: hyaline membrane disease, seizures
congenital heart disease, Pulmonary – Severity of seizures parallel with
hypertension grade of the encephalopathy.
Intracranial Intraventricular, intraparenchymal Tonic seizures are common IVH: 10%
hemorrhage subarachnoid subdural hemorrhage SAH: 90%
(*17%)
Stroke (*7%) Arterial stroke, venous infarction due to Focal clonic seizures in the setting of –
venous sinus thrombosis hemiparesis
Trauma Subdural and subarachnoid hemorrhage – SAH: 90%
Infections Beta-hemolytic streptococci, E. coli, HSV infections—PLEDS seen in *50%
(*14%) Herpes simplex, HIV, coxsackievirus B, temporal regions
torch mycoplasma infection HSV DNA PCR—more sensitive test
Cerebral Migration disorders, neurocutaneous Usually occur within 1 week 0%
malformations disorders Peroxisomal disorders—
polymicrogyria.
Early myoclonic epileptic
Encephalopathy—burst suppression
pattern
Poor prognosis
Metabolic Transient: Usually occur within first 2 days Transient:
disorders hypoglycemia, hypocalcemia Seizures common with late onset 50–100%
(*9%) hyponatremia, hypernatremia hypocalcemia, hypomagnesemia with
Persistent: hypocalcemia in premature infants
inborn errors of metabolism Good prognosis
IEM—Infantile spasms, tonic spasms,
myoclonic seizures
Poor prognosis
Unknown – – –
(*10%)
4 Neonatal EEG and Neonatal Seizures 65

Clinical Manifestations Treatable Causes of Inborn Errors


of Metabolism Presenting
Semiology: with Seizures and Encephalopathy
• Focal and multifocal clonic (25%)
• Focal and generalized tonic (5%) Pyridoxine-Dependent and Folinic Acid
• Myoclonic (20%) Responsive Seizures
• Subtle seizure pattern (50%)
– Apnea Age of onset:
– Tonic deviation of the eyes • Neonatal type—presents soon after birth;
– Eyelid fluttering • Atypical type: Late onset between 1 and
– Drooling, sucking, and chewing 3 years in pyridoxine-dependent seizures.
– Swimming movements of the arm • Seizure features: intractable seizures not
– Pedaling movements of the legs controlled with antiepileptic medications
– Paroxysmal laughing. which respond both clinically and electro-
graphically to daily pyridoxine or folinic acid.
Etiology: • Variable seizure semiology seen.
• Prolonged seizures and status epilepticus
See Table 4.2. common.
Testing:
• Elevated—a-aminoadipic semialdehyde
(a-AASA) in urine and plasma.

Table 4.3 Benign neonatal seizure syndromes


Benign familial Benign idiopathic neonatal seizures
neonatal seizures
Age of onset Usually present within first few weeks of life Within first week
“Fifth-day fits”
Genetics AD inheritance potassium channelopathy; None
KCNQ2 and KCNQ3; chromosome 20, 8
History Multiple family members positive for No family history
neonatal seizures
Clinical features Tonic seizures with apnea Focal clonic, multifocal clonic seizures and
Unilateral focal, multifocal manifestation— usually happens every few hours to days
later part
Multiple times/day
Status epilepticus None Very common
Neuro-examination Normal Normal
MRI brain and Normal Normal
metabolic
evaluation
Interictal EEG Normal Normal (“Theta point alternant”—seen for
2 weeks)
Treatment Phenobarbital levetiracetam Phenobarbital, levetiracetam
Remission – *6 weeks of age Good
Prognosis Usually good except 8–16% has tendency to
develop epilepsy as adult
66 K. Velayudam and A.N.V. Moosa

Fig. 4.11 Various neonatal ictal patterns


4 Neonatal EEG and Neonatal Seizures 67

Fig. 4.11 (continued)

• Elevated—pipecolic acid in plasma and both seizures and development is generally


cerebrospinal fluid. good. In this, they resemble the spectrum of
• Mutations in ALDH7A1 cause benign focal epilepsies of childhood. However,
pyridoxine-dependent epilepsy. interictal EEGs are not helpful. These disorders
Management: are often a diagnosis of exclusion in the acute
• Response seen with IV pyridoxine 100– setting. The two well-characterized benign
500 mg or daily PO pyridoxine 30 mg/kg/d neonatal seizure syndromes are described in
for at least 2 weeks Table 4.3.
• Prior to the administration of folinic acid,
CSF neurotransmitters studies are obtained
• Response seen within 24 h with folinic acid Neonatal Non-epileptic Events
started at 4 mg/k/D divided BID.
• Benign nocturnal myoclonus
• Jitteriness
• Opisthotonus
Benign Neonatal Seizure Syndromes • Pathological myoclonus
• Apneic spells
Benign neonatal seizures represent two rare
genetically mediated syndromes with seizures in
the newborn period. The long-term outcome for
68 K. Velayudam and A.N.V. Moosa

Fig. 4.12 Alpha seizures (Picture source Atlas of neonatal encephalography—Third Edition Lippincott Williams and
Wilkins—A Wolters Kluwer Company)

Special Features—Neonatal Ictal EEG Neonatal Seizures Management


Pattern
• Neonatal seizures always require urgent
– Ictal patterns—vary significantly within a treatment.
same neonate EEG recording. Within a same • First-line phenobarbital—82% ! lorazepam
recording, focal or multifocal ictal discharges —9% ! phenytoin—2%
can be seen simultaneously and indepen- • Second-line therapy: lorazepam 50% ! pheny-
dently from different locations of brain. toin (39%) ! phenobarbital (20%)
– Electrical seizure activity—rare before • *58% continued to have EEG seizures after
34 weeks; the administration of AED and cessation of
– Generalized EEG activity—infantile spasm clinical seizures.
and myoclonic jerks;
– BIRDS—brief ictal rhythmic discharges of
unclear significance;
– Special ictal patterns—burst suppression is Long-term Outcome
seen in metabolic diseases and has poor
prognosis; and • *20% neonatal seizures survivors develop
– Alpha seizures—seen in severe HIE and has postnatal epilepsy.
poor prognosis (Figs. 4.11a–c and 4.12). • Neonates with seizures due to perinatal
asphyxia and cerebral dysgenesis develop
postnatal epilepsy about *30 and 80%,
respectively.
4 Neonatal EEG and Neonatal Seizures 69

• Neonatal seizures due to acute encephalopa- 4. Riviello J, et al. Pharmacology review: drug therapy
thy usually improve in 7–14 days. for neonatal seizures: part 1. Neoreviews. 2004;5:
e262.
5. Slaughter L, Patel A, Slaughter J, et al. Pharmaco-
logical treatment of neonatal seizures: a systematic
review. J Child Neurol. 2013;28:351.
References 6. Mizrahi E, Hrachovy R, Kellaway P. Atlas of
neonatal electroencephalography. 3rd ed. Houston,
TX: Lippincott Williams &Wilkins; 2004.
1. Tsuchida T, Wusthoff C, Shellhaas R, et al. Amer- 7. Clancy R, Bergqvist C, Dlugos, D. Neonatal
ican clinical neurophysiology society standardized encephalography. In Ebersole J, Pedley T, editors.
EEG terminology and categorization for the descrip- Current practice of clinical electroencephalography.
tion of continuous EEG monitoring in neonates: 3rd ed. Lippincott Williams &Wilkins; 2003.
report of the American clinical neurophysiology 8. Hrachovy R. Development of the normal electroen-
society critical care monitoring committee. J Clin cephalogram. In: Levin K, Luders H, editors. Com-
Neurophysiol. 2013;30:161–73. prehensive clinical neurophysiology. 5th ed.
2. Shellhaas R, Chang T, Tsuchida T, et al. The Cleveland, OH: W. B. Saunders Company; 2000.
American clinical neurophysiology society’s guide- 9. Volpe J. Neurology of the newborn. 5th ed. Boston,
line on continuous electroencephalography monitor- Massachusetts: Saunders Elsevier; 2008.
ing in neonates. J Clin Neurophysiol. 2011;28:611–7. 10. Fenichel G. Neonatal neurology. 4th ed. Nashville,
3. Riviello J, et al. Pharmacology review: drug therapy TN: Churchill Livingstone Elsevier; 2007.
for neonatal seizures: part 1. Neoreviews. 2004;5:
e215.
Multiple Choice Questions for Part I

1. Neurons in the cerebral cortex are organized 4. Resting membrane potential of a neuron is
in: around:
A. Three horizontal layers A. +90 mV
B. Four horizontal layers B. –70 mV
C. Six horizontal layers with layer IV C. +70 mV
receiving inputs from thalamus D. –70 µV
D. Six horizontal layers with layer VI E. –20 µV
being the most superficial
E. Six vertical layers 5. Choose the one incorrect statement from the
following:
2. What is the predominant pattern seen in a A. At least 6 cm2 of synchronous cortical
<30 weeks PCA preterm infants? activation is necessary to detect an
A. Trace discontinue individual epileptic spike on scalp
B. Trace alternant electrodes.
C. Trace continue B. Epileptic spikes are exclusively sur-
D. Electrocerebral silence face negative.
E. Burst-suppression pattern C. EEG potentials recorded from the
scalp are produced by the summation
3. Which of the following is true about of the excitatory and inhibitory post-
GLUT1 deficiency? synaptic potentials of pyramidal
neurons.
A. High CSF glucose levels D. At the cortical layers III, V, and
B. Ketogenic acid is the treatment of VI, the pyramidal neurons are aligned
choice in a perpendicular fashion to the
C. Macrocephaly is common cortex.
D. Responds fairly well to sodium chan- E. EEG potentials are not mere represen-
nel blockers tations of neuronal action potentials.
E. All of the above
72 Multiple Choice Questions for Part I

6. The EEG of 2-year-old male with Canavan 10. A 2-week-old boy is having daily seizures
disease will likely show: characterized by brief multifocal jerks,
at times with apnea. The boy is alert
A. Multifocal epileptiform discharges and acting normal in between seizures. His
B. LPDs father and paternal grandfather had similar
C. Burst-suppression pattern seizures that resolved after several months
D. Amplitude attenuation from birth. The likely mutation is:
E. Polymorphic delta activity
A. SCN1A, encoding the sodium channel
7. Which of the following is false about a1 subunit
myoclonic epilepsy of infancy? B. CACNA1G, encoding the T-type
voltage-gated calcium channel
A. Males are more affected than females C. CHRNA4, encoding the a4 subunit of
B. Valproate is the treatment of choice the neuronal nicotinic acetylcholine
C. Positive family history of febrile sei- receptor
zures or epilepsy D. GABRG2, encoding the c2 subunit of
D. Developmental delay is seen in the c-aminobutyric acid A receptor
majority of patients E. KCNQ2, encoding a voltage-gated
E. EEG can show generalized polyspike- potassium channel
and-wave
11. The recommended length of a neonatal
8. Which of the following is incorrect about EEG is:
Dravet syndrome?
A. 20 min
A. Progressive neurological regression B. 30 min
B. Overall poor prognosis C. 60 min
C. SCN1A gene mutation D. 90 min
D. Respond to lamotrigine E. 120 min
E. All of the above is true
12. In the following image which of the fol-
9. Resting membrane potential of a cell can be lowing represents the artifact generated
calculated by the following equation: from the left lateral rectus muscle:

A. Nernst equation A. Long arrow


B. Goldman–Hodgkin–Katz equation B. Short arrowhead
C. Henderson–Hasselbalch equation C. Circled area
D. Both A and B D. None of the above
E. None of the above
Multiple Choice Questions for Part I 73

13 In full-term infants, sleep spindles are seen C. If the difference in potential between
at the age of: two electrodes is negative this is rep-
resented by a downward deflection.
A. Birth D. Each channel in an EEG recording
B. 1–3 months represents the difference in potential
C. 6–12 months between two electrodes.
D. 12–24 months E. None of the above
E. Not until adolescence
15. Which of the following is the correct order
14. Choose the one incorrect statement from the in which the maturational changes in
following: neonatal EEG develop?

A. In the international 10–20 system of A. Awake ! Quiet sleep ! Active sleep


electrode arrangement the average B. Active sleep ! Awake ! Quiet sleep
inter-electrode distance is 4–6 cm. C. Quiet sleep ! Active sleep ! Awake
B. The commonly used electrodes D. Active sleep ! Quiet sleep ! Awake
for scalp EEG have a contact E. Quiet sleep ! Awake ! Active sleep
made of chloride-treated silver,
which if applied properly would 16. Which of the following statements is true
show a resistance of a few hundred regarding distribution of ions across the cell
ohms. membrane of a neuron?
74 Multiple Choice Questions for Part I

A. Potassium has a higher intracellular C. Chloride channels


concentration D. Potassium leak channels
B. Sodium has a higher extracellular E. A, B, and C
concentration
C. Chloride has a higher extracellular 19. The major contribution to EEG potentials
concentration recorded from scalp comes from:
D. All of the above
E. None of the above A. Action potentials
B. Excitatory postsynaptic potentials
17. What artifacts can be seen in the following C. Inhibitory postsynaptic potentials
image? D. B and C
E. A and B

A. Blink artifact 20. Which one of the following feature is NOT


B. Chewing artifact true about Frontal sharps (also known as
C. Myogenic artifact Encoches Frontales)?
D. Lateral eye movement
E. All of the above A. Frequently seen during transitional
stage of sleep
18. The channels that play a major role in B. Initial Negative (200 ms)  Positive
generation and propagation of action phase (longer)
potential in neurons are: C. Seen between 33 and 46 weeks
D. Usually Asymmetric and Asynchronous
A. Voltage-gated sodium channels E. Sometimes seen with mixed rhythmic
B. Voltage-gated potassium channels bi-frontal delta
Multiple Choice Questions for Part I 75

21. Infantile spasms are electrographically B. Midbrain


associated with: C. Thalamus
D. Any of the above
A. Burst-suppression pattern
E. None of the above
B. Ripples
C. Electrodecremental pattern
25. Cortical oscillations faster than beta-gamma
D. Electrocerebral silence
frequency (ripples or fast ripples):
E. RTTBD
22. What is true regarding the polarity of A. Have been both described under nor-
potentials recorded on a scalp electrode? mal conditions
B. Have been both described during
A. Superficial EPSPs and deep IPSPs will
epileptic seizures
show the same polarity (negative) on a
C. Are always pathologic
surface recording electrode
D. Are never associated with epileptic
B. Superficial EPSPs and deep IPSPs will
seizures
show the opposite polarity (negative
E. A and B
and positive, respectively) on a surface
recording electrode
26. The intracellular pathophysiological mech-
C. Superficial IPSPs and deep EPSPs will
anism of an epileptiform discharge is:
show the same polarity (negative) on a
surface recording electrode
A. Action potential
D. Superficial IPSPs and deep EPSPs will
B. Depolarization
show the opposite polarity (positive
C. Hyperpolarization
and negative, respectively) on a sur-
D. Paroxysmal depolarizing shift
face recording electrode
E. Presynaptic excitation
E. None of the above
23. Which of the following statements is true 27. Focal slow waves are due to:
about neonatal seizures?
A. Thalamic synchronization
A. Most common etiology—Hypoxic B. Cortical partial deafferentation from
ischemic encephalopathy subcortical structures
B. Decoupling—Neonates treated with C. Cortical hyperexcitability
AED continued to have electro- D. Paroxysmal depolarizing shift (PDS)
graphic seizures and stopped clinical E. Midbrain suppression
seizures
C. Tonic seizures mostly seen in preterm 28. All of the following reduce the of electro-
with intraventricular hemorrhage cution except:
D. Focal seizures with hemiparesis is
seen with ischemic infarcts A. Equipments with small leakage current
E. All of the above B. Connecting people using appliances to
the ground
24. Sleep spindles originate from: C. Using short connecting cords
D. Using one ground per patient
A. Cortex E. All of the above
76 Multiple Choice Questions for Part I

29. Choose the incorrect statement regarding A. They require the use of an additional
“common mode rejection”: electrode used as the machine
reference.
A. Helps to filter out the environmental B. The signal from each channel is sam-
electrical noise. pled and stored at regular intervals.
B. Requires the use of a differential This sampling rate in most machines
amplifier. ranges from 256 to 1024 Hz.
C. Involves exclusion of the signals C. The Nyquist sampling theorem deter-
recorded by both electrodes and mines that the sampling rate should at
amplifying the differences in between. least match double the frequency of
D. Is unable to reject the 60 Hz artifact the original signal to avoid aliasing.
since the latter is also recorded by the D. The ACNS guidelines recommend a
ground electrode. sampling rate at 3 times or more the
E. None of the above. frequency of the original signal.
E. All of the above are correct.
30. The minimal surface area of the postsy-
naptic action potential required for record- 33. Which of the following statements con-
ing of a spike on scalp EEG is: cerning EEG grounding is correct:

A. 0.6 mm2 A. The EEG machine should be con-


B. 6 mm2 nected to a two-pronged hospital grade
C. 0.6 cm2 outlet.
D. 6 cm2 B. A single ground electrode is placed
E. None of the above anywhere on the patient and connects
to the appropriate jack in the input
31. Which of the following statements con- jackbox of the EEG machine.
cerning filter use in EEG is incorrect: C. Connecting a grounding wire from the
EEG machine to the patient’s bed is
A. A low-pass filter allows lower fre- required for patient safety.
quencies to pass. In usual scalp EEG D. In an ICU patient, each electric device
reading settings, it is set to 70 Hz. connected to the patient should have
B. A high-pass filter allows higher fre- separate grounding.
quencies to pass. In usual scalp EEG
reading settings, it is set to 1 Hz. 34. A premature infant with apneic spells has
C. A notch filter can be used to filter out an EEG showing frequent multifocal sharp
the 50 or 60 Hz noise generated from waves. These findings are most consistent
the city power lines. with:
D. Filters in analog EEG devices consist
of a resistor–capacitor circuit. A. Normal finding in this age
E. The time constant of a filter determi- B. Multifocal potential epileptogenicity
nes the half-life of its resistor. C. Non-specific generalized cerebral
dysfunction
32. Which of the following statements con- D. Hypsarrythmia
cerning digital EEG machines is incorrect: E. Status epilepticus
Multiple Choice Questions for Part I 77

35. Changing the filter settings on the EEG B. Ictal discharges are often generalized,
allows for all of the following, except: due to immaturity of the brain
C. Absence seizures are most common
A. Passing of signals above a certain D. Urgent surgery is the treatment of
frequency. choice
B. Exclusion of signals below a certain E. VNS is usually helpful early-onset
frequency. infantile spams
C. Allowing viewing high-frequency
oscillations in intracranial EEG.
D. Stopping a narrow band of Answers
frequencies.
E. Preferential amplification of epilepti- 1. (C). The cerebral cortex is organized in six
form discharges. horizontal layers with layer I being the most
superficial underneath the pial surface, and
36. If 60 Hz sine wave is sampled at 100 Hz, layer VI being the deepest overlying the
the result is: subcortical white matter. Layer IV (Internal
Granular Layer) receives input from
A. Electromagnetic gain
thalamus.
B. Digital filtering
2. (A). At a gestational age below 30 weeks,
C. Aliasing
the EEG activity consists of burst of mixed
D. Sub-harmonic wave
frequency (mostly delta) in a discontinuous
E. Amplification delay
fashion interspaced with periods of EEG
attenuation lasting for few seconds to 1–
37. West syndrome is characterized by:
2 min. The higher amplitude of the bursts
A. Myoclonic seizures occurs in the posterior region This EEG
B. Infantile spasms pattern is referred as to trace discontinue. At
C. Absence seizures this age, there is no distinction between
D. Dialeptic seizures sleep and awake states.
E. All of the above 3. (B). GLUT1 deficiency syndrome is caused
by impaired glucose transport across the
38. Which of these options represents an blood–brain barrier and is linked to low
advantage of a bipolar montage over a ref- CSF glucose levels. Clinically, patients
erential montage: present with acquired microcephaly, and
early-onset epilepsy that is refractory to
A. Ability to detect both local (near field) standard AEDs. Most patients carry muta-
and distant (far field) potentials tions of the SLC2A1 gene. The ketogenic
B. Provides a closer representation of the diet is the treatment of choice as conven-
absolute potential at an electrode tional AEDs are not effective.
C. Makes the visual detection of differ- 4. (B). The resting membrane potential of a
ences in local potential easier neuron is typically −70 mV, the inside of
D. Requires placement of fewer electrodes the neuron being negative in relation to the
E. All of the above outside. The resting membrane potential is
determined by movement of potassium,
39. Which of the following is true about sodium, and chloride ions along their elec-
neonatal seizures: trochemical gradient across the cell
membrane.
A. Majority of ictal discharges originate 5. (B). Epileptic spikes are most commonly
in the central regions surface negative. However, on occasions
78 Multiple Choice Questions for Part I

where the spike is generated in a sulcus seizures persisting beyond 5 years of age to
perpendicular to the scalp, the dipole will be myoclonic-astatic epilepsy, some cases of
parallel to the scalp and surface positivity temporal lobe epilepsy to Dravet Syndrome
can be recorded. Positive epileptiform dis- at the severe end of this spectrum. Seizures
charges are also seen after brain surgery and include absence, myoclonic, atonic, and
in infants with germinal matrix hemorrhage. generalized tonic–clonic and tend to lessen
6. (E). Canavan disease like all leukodystro- by late childhood and adolescence. 10% of
phies affects the white matter. Since Canavan familial GEFS+ cases have mutations in the
disease affects diffusely the white matter, it SCN1A gene; a few have GABA channel
typically produces diffuse EEG slow activity mutations in the GABRG2.
such as polymorphic delta activity. 9. (C). Equilibrium potential for an ion is the
7. (D). Benign myoclonic epilepsy of infancy is membrane potential at which there is no net
a rare disorder occurring in children between movement of that ion across the cell mem-
the age of 5 months to 5 years. Males are brane. Equilibrium potential for an indi-
more affected than females, and a family vidual ion can be calculated using the
history of febrile seizures or epilepsy is pre- Nernst equation. The membrane potential at
sent in 30–40% of cases. Myoclonus is the which there is no net flow of ions across the
most common seizure type followed by cell membrane is the resting membrane
absence and generalized tonic–clonic sei- potential which can be calculated using the
zures. EEG can be normal but most com- Goldman–Hodgkin–Katz equation which
monly show generalized polyspike-and-wave takes multiple ions into account. The Hen-
discharges and valproate is the treatment of derson–Hasselbalch equation describes the
choice. Overall development is usually nor- derivation of pH as a measure of acidity.
mal except for some learning disability. 10. (E). The clinical picture is typical of benign
8. (D). Dravet syndrome is a form of severe familial neonatal seizures. This condition
infantile-onset epilepsy that usually pre- has been linked to mutations of the KCNQ2
sents between 5–15 months of age with or KCNQ3 gene, encoding for a voltage-
generalized, hemiclonic convulsions, or gated potassium channel that has a major
status epilepticus. Common triggers include role in regulating neuronal excitability.
acute illness, fever, or vaccination. Devel- 11. (C). The recommended length of a neonatal
opment is initially normal but then regresses EEG recording is 60 min. In a full-term
slowly afterward. Majority of patients neonate, this will allow the sampling of all
(>70%) have de novo mutations in the the neonatal sleep stages that approximately
voltage-gated sodium channel gene, include 25 min of active sleep, 20 min of
SCN1A. Seizures are worsened by sodium quiet sleep, and 15 min of intermediate
channel modulating drugs such lamotrigine. sleep. Neonatal EEG recording during
The ketogenic diet is a common treatment wakefulness has a low yield due to exces-
modality but the overall outcome is typi- sive artifacts. Unlike neonates, adults have a
cally poor. The term GEFS+ originally typical sleep cycle of 80–120 min.
called generalized epilepsy with febrile 12. (B). The short spike at F7 from the lateral
seizures plus but now has been revised to rectus muscle is immediately followed by a
genetic epilepsy with febrile seizures plus positive phase reversal resulting from the
(as some patients also have partial seizures). positive charge of the cornea moving closer
The GEFS+ spectrum includes several to the electrodes on the left.
entities ranging from typical febrile seizures 13. (B). Sleep spindles are formed at the age of
at the mild end, patients with fever related 1–3 months in term infants, maximal in the
Multiple Choice Questions for Part I 79

central regions. They are initially bilateral interictal correlate of infantile spasm is the
but asynchronous until the age of 2 years chaotic hypsarrhythmia with seizures con-
when they become synchronous. sisting of electrodecremental pattern.
14. (C). By convention, if the difference in 22. (A). The polarity of extracellular field
potential between two electrodes is nega- potentials recorded by surface electrodes on
tive, then it is represented by an upward EEG depends on the direction of the current
deflection. flow as well as on the position of the elec-
15. (B). EEG maturational changes occur first in trode relative to the location of the genera-
active sleep, EEG maturational changes tor. Thus, superficial EPSPs and deep IPSPs
occur first in active sleep; after a lag of about will show the same polarity (negative) on a
2 weeks the awake periods start showing surface recording electrode. Likewise,
more mature patterns. Quiet sleep is the last superficial IPSPs and deep EPSPs will show
stage to show more mature changes. For this the same polarity (positive) on a surface
reasons, the abnormalities are most fre- recording electrode. Therefore, orientation
quently seen in the quiet sleep phase. of neurons and their processes as well as
16. (D). Sodium and chloride ions have a location of synaptic contacts with respect to
higher extracellular concentration while the cortical surface are important determi-
potassium concentration is higher in the nants of extracellular field potentials recor-
intracellular compartment. ded by EEG electrodes.
17. (E). The rhythmic myogenic artifact in the 23. (E). Most common etiology of neonatal
first four and last one-second represent seizures is hypoxic ischemic encephalopa-
chewing artifact. In addition, isolated thy. Tonic seizures are typically seen in
myogenic artifact can be seen in seconds 7– preterm neonates with intraventricular
8–9, blink artifact can be seen in second 5, hemorrhage and focal seizures with hemi-
and lateral eye movement to the left on paresis are typically seen in neonates with
second 7 of this epoch. ischemic infarcts.
18. (A). The voltage-gated sodium channels 24. (C). Sleep spindles (7–14 Hz) originate
play a major role in generation and propa- from the thalamus and are considered to be
gation of action potential by allowing the first signs of EEG synchronization
sodium to enter into the soma. during early stages of sleep. The reticular
19. (D) A large number of EPSPs and IPSPs nucleus of the thalamus is regarded as the
generated in a complex network of neurons pacemaker of the spindles.
generate an extracellular field potential that 25. (E). Episodes of cortical oscillations (100–
changes over time which is believed to be 600 Hz) called ripples (100–200 Hz) or fast
the basis of potentials recorded on EEG. ripples (>200 Hz) have been described both
20. (D). Frontal sharps (or Encoches Frontales) under normal conditions and epileptic sei-
are blunt isolated biphasic broad sharp zures. Ripples probably reflect synchro-
transients (0.5–0.75 s) seen in the frontal nized IPSPs whereas fast ripples appear to
region. They are usually symmetric and represent bursts of population spikes.
synchronous, frequently seen in transitional 26. (D). The paroxysmal depolarizing shift
stage of sleep, between 33 weeks–46 weeks (PDS) is an intracellular mechanism that is
PCA with peak around 35 weeks. If con- not recorded by scalp EEG. It represents the
sistently absent or asymmetrical on one intracellular electrophysiological correlate
side, structural lesion on the that side will of focal epileptiform discharges such as
be highly suspected. sharp waves and spikes. It consists of
21. (C). Infantile spasms are brief epileptic abnormal synchronous activation of multi-
tonic contractions affecting infants and ple neurons at the cellular level causing a
children, seen in West syndrome. The wave of depolarization. It is primarily due
80 Multiple Choice Questions for Part I

to the activation of high-frequency fast 34. (C). In neonatal EEGs, multifocal epilepti-
sodium channel potentials. form discharges (spikes or sharp waves) do
27. (B). Slow activity (slowing) is a function of not necessarily imply potential epilepto-
white matter disturbance while voltage or genicity like in adults. In prematures, mul-
amplitude reduction (attenuation) is a func- tifocal sharp transients can be normal when
tion of cortical disturbance. The mechanism they are rare and random. When multifocal
of focal slowing is likely due to partial cor- discharges are frequent as in this case, they
tical deafferentation from subcortical struc- are often indicative of non-specific
tures. This could be due to functional or encephalopathy.
anatomic deafferentation of the cortex as 35. (E). It is not possibly to preferentially “filter
may be seen in toxic-metabolic encephalo- in” epileptiform discharges. At best, filters
pathies or structural brain lesions. allow to remove some artifacts to allow for
28. (B). Small leakage currents, short cord easier visual detection of epileptiform
connections, people not connected to the discharges.
ground, high-resistant contacts (dry skin), 36. (C). The Sampling Theorem states that if a
and one common ground are all measures signal contains component frequencies
that minimize the risk of electrocution. ranging from 0 to fN, then the minimum
29. (D). The differential amplifier can reject the sampling frequency that can be used for a
60 Hz artifact as long as it is recorded digitized data to adequately represent the
equally by both electrodes in each channel. frequency content of the original signal is
If there is significant difference in the 2 fN called the Nyquist frequency (equal to
impedance between two electrodes, then the two times of the original frequency sam-
60 cycle signal will be recorded unequally pled). Aliasing refers to the distortion of a
and the artifact will appear on the EEG. signal caused by sampling frequency lower
30. (D). The postsynaptic action potential than the Nyquist frequency. In this case, to
should involve at least 6 cm2 of cortex to be avoid aliasing, a minimal sampling rate of
detected as an epileptiform discharge on 120 Hz is needed.
scalp EEG. 37. (B). West syndrome consists of a triad of
31. (E). The time constant equals the time infantile spasms, developmental delay, and a
needed to discharge the capacitor in the grossly abnormal EEG pattern termed hyp-
circuit to 36.8% of its initial full charge. Its sarrythmia. It usually affects infants between
value is inversely related to the frequency the ages of 4–8 month. In the majority of
that will pass through the filter. cases, West syndrome is associated with
32. (C). If the sampling rate falls below a cer- serious neurological abnormalities being
tain point, the resulting waveform would no genetic, structural, metabolic, or infectious.
longer represent the original one (aliasing). 38. (C). In a bipolar montage, external noise
Per the Nyquist sampling theorem the can easily be canceled out as it measures the
sampling rate should be at least twice the difference in potential between adjacent
frequency of the original signal to avoid electrodes and hence amplifies local
aliasing. potentials.
33. (B). A three pronged hospital grade outlet 39. (A). Neonatal seizures often arise from the
should be used. There is no need to connect central region followed by the temporal
the EEG machine to the bed. When a region. In the exception of myoclonic sei-
patient is connected to more than one zures, all neonatal seizures are unifocal or
electrical device a common ground should multifocal at onset, frequently associated
be utilized. with an abnormal EEG background.
Part II
The Abnormal EEG
Ictal and Interictal EEG
5
Mohamad Z. Koubeissi and Nabil J. Azar

The main features of focal spikes/sharp waves


Interictal Epileptiform Discharges
are the following:
Interictal Epileptiform Discharges (IEDs) refer to
(1) They are distinct from the background. That
electrographic patterns seen commonly in indi-
means that they are not part of a preceding
viduals with high predisposition for epileptic
rhythm, like wicket spikes. Rather, they have
seizures. Since such waveforms may be present
an amplitude large enough to stand out from
in individuals without seizure disorders [1], the
the background, and appear to abruptly arise
significance of IEDs varies in light of the
from a morphologically different background.
patient’s history and other diagnostic tests. In
(2) They are often followed by a slow wave.
patients with epilepsy, an initial routine EEG
(3) They tend to disrupt the background. The
detects epileptiform discharges in about 50% of
sharply contoured component is often fol-
times. After four routine EEGs, this yield
lowed by an irregular, slow EEG that is
increases to over 90%. Temporal lobe epilepsy
different from the preceding EEG. Thus,
tends to be commonly associated with IEDs
even if the sharply contoured component is
while frontal lobe epilepsy is least associated
hidden, the reader may still be able to tell
with IEDs on scalp EEG.
that a disruptive event has just taken place
Focal spikes on scalp EEG are sharply con-
based on the aftergoing EEG.
toured waveforms with durations between 20 and
(4) On scalp EEG, IEDs are often surface neg-
70 ms. Sharp waves, on the other hand, are
ative. Exceptions may occur in individuals
similar morphologically except that their dura-
who have undergone craniotomy; in whom,
tion is longer than 70 ms [2]. There is no clinical
spikes may occasionally be positive
significance to this distinction, and it is a mere
(Fig. 5.1). Also, infants with intraventricular
morphological descriptor. The physiologic basis
hemorrhage or periventricular disease may
of a focal epileptiform discharge is the paroxys-
have positive spikes in the central region, the
mal depolarizing shift (PDS).
significance of which is encephalopathy
rather than propensity for epilepsy [3].
(5) The slopes of The IED are often asymmet-
M.Z. Koubeissi rical. The initial, negative component is
Department of Neurology, Epilepsy Center, George typically the steepest, followed by a slower
Washington University, 2150 Pennsylvania Ave., positive component with larger amplitude.
NW. Suite 9-405, Washington, DC 20037, USA
(6) IEDs have a field that often extends over a
N.J. Azar (&) few electrodes. If a relatively high-voltage
Department of Neurology, Vanderbilt University
Medical Center, Nashville, TN 37212, USA sharply contoured waveform is seen only on
e-mail: dr.nabil.azar@gmail.com

© Springer Science+Business Media LLC 2017 83


M.Z. Koubeissi and N.J. Azar (eds.), Epilepsy Board Review,
DOI 10.1007/978-1-4939-6774-2_5
84 M.Z. Koubeissi and N.J. Azar

Fig. 5.1 Two spikes


(arrowheads) from the
same patient who had
undergone right
frontotemporal resection
preserving the
hippocampus for
intractable epilepsy without
resolution of seizures. The
first spike is positive over
the right mid-temporal
region, while the other is
negative

one contact, but not on neighboring ones, it Frequent comorbidities of individuals with mul-
is often more suggestive of an artifact. tifocal IEDs include cognitive and motor deficits.

The clinical significance of IEDs of different


locations is not the same [4]. For instance, sei- Periodic Lateralized Epileptiform
zures occur in 90% of children with anterior Discharges (PLEDs)
temporal spikes, but in only 40% of those with
rolandic spikes or occipital spikes. Occipital As the name implies, PLEDs (also known as
spikes can be seen in migraine [5] or in children Lateralized Periodic Discharges or LPDs) are
with congenital blindness [6]. IEDs that occur on one side on the brain at
In Benign Epilepsy with Centrotemporal regular intervals of 0.3–4 s (Fig. 5.2). They are
Spikes (BECTS), or Benign Rolandic Epilepsy, commonly seen in acute brain injury such as
spikes are equally negative over the central and herpes encephalitis and stroke, among others.
temporal derivations with the positive end of the They can also occur for prolonged periods of
dipole appearing typically in the frontal regions. time after focal status epilepticus. In addition,
Multifocal IEDs referred to spikes or sharp they can be seen in toxic encephalopathies,
waves are seen independently on both sides. including aminophylline or alcohol intoxication.
These are often associated with background PLEDs can occur in individuals with marked
slowing and the vast majority of patients have encephalopathy as well as in ones who at their
seizures, with generalized seizures being very baseline mental status. Half of all patients with
common. In addition, seizure frequency is often PLEDs will have seizures. When PLEDs are
very high and medical intractability common. associated with low amplitude, high-frequency
5 Ictal and Interictal EEG 85

Fig. 5.2 Periodic lateralized epileptiform discharges (PLEDs) over the left frontal region in an adult patient after
resolution of prolonged focal status epilepticus over the same region

rhythmic discharges, often appearing superim-


posed on or after the sharply contoured wave- Generalized IEDs
form, they are termed PLEDs plus and have
increased significance for predicting seizures. The 3-Hz spike-and-wave discharges are the
BiPLEDs are PLEDs that occur independently EEG signature of absence epilepsy, often pre-
on either side of the brain. They occur in indi- senting in bursts lasting 1–3 s, and typically
viduals with severe brain disease and are asso- activated by hyperventilation. They are often
ciated with a poor prognosis. Multifocal PLEDS bilaterally synchronous and have a generalized
refer to 3 or more foci of PLEDs involving both field, typically appearing maximum over the
sides of the brain. They are associated with frontal and midline derivations. However, varia-
multifocal lesions or severe diffuse brain disease. tions of the field of generalized IEDs are not
The majority of patients with multifocal PLEDS uncommon. Occasionally, some asynchrony or
have seizures. asymmetry may be noted, but often such asym-
metries (referred to as fragments of generalized
epileptiform discharges) shift in the same record.
Temporal Intermittent Rhythmic Phase reversals of the spike components may be
Delta Activity (TIRDA) seen over F3 and F4 contacts. Although brief
runs of 3-Hz spike-and-wave discharges may
TIRDA refers to intermittent rhythmic activity of appear asymptomatic, detail assessments
1–3 Hz frequency occurring over the revealed that even brief runs may interfere with
anterior-to-mid temporal derivations on one side. continuous motor tasks [7].
The duration of the train varies, often lasting for In other idiopathic generalized epilepsy syn-
approximately 5 s. The presence of TIRDA is as dromes, such as JME, the spike or
significant for temporal lobe epilepsy as temporal polyspike-and-slow wave complexes often pre-
IEDs are. Indeed, concomitant depth and scalp sent in runs of faster frequencies, typically 4–
electrode recordings have shown that TIRDA 6 Hz, and also occur singly (Fig. 5.3). Atypical
correlates with intracranially recorded mesial generalized spikes may occur as part of other
temporal spikes. generalized epilepsies. These are medium to high
86 M.Z. Koubeissi and N.J. Azar

photic stimulation drain or outlast it. When they


outlast photic stimulation and are self-sustaining,
they may have a higher association with epi-
lepsy, although this is debatable. Up to three out
of four patients with photo epileptiform dis-
charges have seizure disorders. Individuals with
bioccipital discharges have the least association
with epilepsy. These discharges are often part of
primary generalized epilepsy and rarely focal
epilepsy. Interestingly, occipital spike-and-slow
wave discharges may be scotosensitive, i.e., eli-
cited by darkness rather than light and may occur
as part of benign epilepsies as well as such
mitochondrial disorders as myoclonic epilepsy
with ragged red fibers (MERRF) [8].

Ictal EEG
Fig. 5.3 Spike-and-slow wave complex in a patient with
JME. Note the phase reversals over F3 and F4 Recording the ictal EEG is an essential part of
the surgical evaluation of patients with intract-
able epilepsy [9]. In such patients, it is important
voltage without a prominent after going that the recorded seizures are semiologically
slow-wave component and may occur singly. typical of the patient’s habitual episodes before
They are best seen with a referential ear montage. surgical decisions are made. In addition, record-
Slow spike-and-wave complexes present with ing the patient’s habitual episodes is essential for
a frequency that is slower than the 3-Hz pattern characterization of paroxysmal events in indi-
of absence epilepsy. They are a typical electro- viduals with questionable nonepileptic episodes.
graphic feature of Lennox-Gastaut syndrome. Ictal EEG represents a clear deviation from
Their typical frequency is around 1.0–2.5 Hz, the baseline of a pattern that shows some evo-
with wider (less spiky) sharp component than in lution. By evolution, it is meant that the pattern
absence epilepsy. Sleep activates trains of such changes in terms of its frequency, amplitude,
slow complexes in the extent that they may field, or morphology as the seizure occurs. This
appear continuous as in electrical status epilep- applies most typically to focal seizures, espe-
ticus during sleep (ESES). cially temporal lobe seizures, but may start with a
semi-rhythmic delta activity over one temporal
region and soon evolves into a theta range spike
PhotoEpileptiform Discharges discharge over the same distribution that is typ-
(Photoparoxysmal Response) ical of mesial temporal generators. However,
even in generalized epilepsies, such as absence
Photoepileptiform discharges are IEDs that are epilepsy, an evolution pattern can be noted
elicited by photic stimulation. The elicited dis- whereby the initial frequency of the
charges can be generalized (most common), spike-and-slow-wave discharge is higher than 3,
bilateral posterior, or unilateral predominant often 3.5 Hz, whereas toward the end of the
(least common). They may occur within the burst, the frequency slows down to 2.5 Hz.
5 Ictal and Interictal EEG 87

Ictal EEG in Focal Epilepsy whereas the opposite scenario suggests a lateral
neocortical origin.
Only 22% of all focal seizures that are not In extratemporal lobe seizures, a
associated with alteration of consciousness (for- fast-frequency ictal discharge may be more
merly named simple partial seizures) have an common than in temporal lobe epilepsy, but, in
EEG correlate. In the subset of such seizures general, extratemporal seizures are not associated
where a motor component is present, the elec- with a clear ictal discharge as often as seizures of
trographic yield increases to 33% versus only temporal origin [11]. For example, only half of
15% of those that have no motor manifestations frontal lobe seizures have a localizing EEG pat-
[10]. On the other hand, seizures that are asso- tern [13]. Similarly, parietal lobe seizures often
ciated with alteration of awareness (dyscognitive have no clearly localizing EEG [14]. Occipital
seizures, formerly termed complex partial sei- lobe seizures commonly show an ictal discharge
zures) are almost always associated with EEG over the occipital region. Both occipital and
changes. Rare exceptions may apply to seizures parietal lobe seizures tend to propagate to the
originating from the parietal or frontal lobe [11]. temporal lobes, where seizures become semio-
When ictal discharges are present in seizures that logically and electrographically indistinguishable
do not cause alteration of awareness, they are from temporal lobe seizures. Occipital lobe sei-
morphologically indistinguishable from focal zures can also propagate to frontal and insular
ictal discharge in dyscognitive seizures, mani- regions.
festing as focal repetitive spike discharge,
low-voltage fast activity, or focal rhythmic
slowing, among others. In general, when the ictal Ictal EEG in Generalized Epilepsy
discharge consists of fast frequencies, it indicates
proximity of the recording electrode to the sei- In generalized seizures, the earliest clinical and
zure focus. On the other hand, slow discharges, EEG changes typically are not lateralizing and
for example in the delta range, typically represent indicate diffuse brain involvement [15]. A more
propagated activity from distant sites. recent definition of generalized seizures is “sei-
In temporal lobe epilepsy, simultaneous scalp zures originating at some point within, and
and depth electrode recordings show that no scalp rapidly engaging, bilaterally distributed net-
EEG changes are seen when seizure discharges works. These networks can include cortical and
are limited to the hippocampus (Fig. 5.4). As the subcortical structures, but do not necessarily
seizure propagates outside of the mesial temporal involve the entire cortex,” and “they can be
structures into neocortical regions, it is then asymmetric” [16]. In idiopathic generalized
detected by scalp EEG. A 5–9-Hz temporal ictal epilepsies, the baseline EEG is within normal
discharge is highly associated with seizures of limits, and the interictal epileptiform and ictal
hippocampal onset (Fig. 5.5) [12], while neo- discharges are typically bilateral and maximal
cortical seizures often are associated with poly- over the frontal head regions. In contrast, the
morphic, 2–5-Hz On the other hand, seizures EEG background is slow in symptomatic gener-
originating at the temporal neocortex are often alized epilepsy.
associated with irregular, polymorphic, 2–5-Hz
ictal discharge (Figs. 5.6 and 5.7). The main use
of sphenoidal electrodes is not only to increase Idiopathic Generalized Epilepsy
the overall yield of detecting epileptiform EEG
abnormalities, but also to further localize inter- The ictal discharge has an abrupt onset and ter-
ictal and ictal discharges. For example if a dis- mination, and, although widespread, it is often
charge is of higher voltage over the sphenoidal maximum over the frontal regions with phase
electrodes than mid-temporal electrodes (T3 or reversals seen over F3 and F4. In absence sei-
T4), it signifies more inferior and mesial origin, zures, rhythmic spike-and-slow-wave runs that
88 M.Z. Koubeissi and N.J. Azar

Fig. 5.4 Right hippocampal ictal discharge from the same d Further evolution 40 s later. Note the amplitude discrep-
patient in Fig. 5.1 recorded with depth electrodes. a Note the ancy between MH2 and MH3, 4, and 5, despite the proximity
initial high-voltage spike that marks the seizure onset, of these electrodes to one another (5-mm inter-electrode
followed by high-frequency low-voltage activity. b Evolu- distance), which reflects the closed-field nature of the
tion of the seizure with slower rhythms now seen in the hippocampus and the importance of recording with depth
anterior hippocampus. c Further evolution 25 s later, with electrodes in order to detect its activity. AH anterior
highly organized ictal discharge in the middle hippocampus. hippocampus; MH middle hippocampus

Fig. 5.5 Theta-range left temporal ictal discharge


5 Ictal and Interictal EEG 89

Fig. 5.6 Left frontal seizure from the same patient with PLEDs in Fig. 5.2

Fig. 5.7 Right hemispheric seizure in a patient who presented with acute stroke and altered mental state that prompted
an EEG to rule out subclinical seizures

last longer than 3 s often have clinical correlates epilepsy is that of 3/s spike-and-slow-wave
and may be termed “ictal” as opposed to shorter complexes, usually starting at 3.5 Hz and end-
runs, often termed: “interictal”. However, due to ing at 2.5 Hz. Occasionally polyspikes may be
the difficulty of assessing subtle, brief alteration seen.
of awareness, and the distinction between inter- In juvenile myoclonic epilepsy (JME), bursts
ictal and ictal is not straightforward. If absence of diffuse, bi-frontal maximum
seizures are associated with automatisms, they polyspike-and-slow-wave discharges are seen
are called complex absence seizures [17]. The interictally or with myoclonic jerks. These are
classic ictal pattern of childhood absence typically faster than in absence epilepsies,
90 M.Z. Koubeissi and N.J. Azar

Fig. 5.8 Baseline EEG in LGS syndrome

commonly around 5–6 Hz. One third of indi- corresponds to the brief muscle relaxation. Longer
viduals with JME show a photoparoxysmal postictal phases can be expected after longer sei-
response. An ictal discharge of 10–16 Hz fre- zures and in younger individuals.
quency can be seen in association with some
myoclonic seizures. When absence seizures
occur in individuals with JME, they manifest Symptomatic Generalized Epilepsies
electrographically as 3-Hz spike-and-slow
waves, like in other absence epilepsies. The ictal discharge in symptomatic tonic–clonic
Tonic seizures are often associated with voltage seizures is similar to that of idiopathic general-
attenuation with superimposed high-frequency ized seizures. Tonic seizures are typically asso-
activity of 20–40 Hz. In tonic–clonic seizures, ciated with an electrodecremental pattern or
this pattern evolves to patterns with higher paroxysmal fast activity of 10–25 Hz frequency.
amplitude with slower frequencies, followed by This is often followed within 5 s by sharp-and-
yet a slower pattern with intermittent slow waves. slow wave complexes. The vast majority of such
Another ictal pattern in tonic–clonic seizures is discharges are bilateral and frontal maximum.
that of initial fast activity of about 10 Hz that Atypical absence seizures often occur in
gradually increases in voltage before it starts patients with developmental and cognitive delay
mixing up with rhythmic slow waves yielding and are one seizure type in Lennox-Gastaut
polyspike-and-slow wave complexes [17]. What syndrome (LGS), which also includes tonic sei-
marks the switch from the tonic to the clonic phase zures, atonic seizures, and myoclonic seizures.
of the seizure is when the slow-rhythm frequency The EEG in LGS is marked generalized, frontal
reaches 4 Hz. Clonic jerks correspond to bursts of maximum slow spike-and-slow waves (1.5–
multiple spikes separated by a slow wave that 2.5 Hz) (see Fig. 5.8). Tonic, as well as atonic,
5 Ictal and Interictal EEG 91

seizures are associated with a low-voltage fast electroencephalogrpahy: a study of 374 seizures in
pattern. As mentioned previously, the baseline 48 patients. Brain. 1975;98:427–40.
8. Koubeissi MZ, et al. Scotosensitive myoclonic
EEG is slow and disorganized. seizures in MERRF. Neurology. 2009;72(9):858.
9. Koubeissi MZ, et al. Medically intractable seizures
originating from the primary somatosensory hand
References area. Epileptic Disord. 2008;10(4):339–48.
10. Devinsky O, et al. Clinical and electroencephalo-
graphic features of simple partial seizures. Neurol-
1. Sam M, So E. Significance of epileptiform discharges ogy. 1988;38(9):1347–52.
in nonepileptic patients in the community. Epilepsia. 11. Williamson PD, et al. Complex partial seizures of
2001;42:1273–7. frontal lobe origin. Ann Neurol. 1985;18(4):497–
2. Chatrian G, et al. A glossary of terms most 504.
commonly used by clinical electroencephalogra- 12. Ebersole JS, Pacia SV. Localization of temporal lobe
phers. In: International Federation of Societies for foci by ictal EEG patterns. Epilepsia. 1996;37
Electroencephalographers and Clinical Neurophysi- (4):386–99.
ology: recommendations for the practice of clinical 13. Laskowitz DT, et al. The syndrome of frontal lobe
neurophysiology. Elsevier Science Publishers: Ams- epilepsy: characteristics and surgical management.
terdam; 1983. p. 11–27. Neurology. 1995;45(4):780–7.
3. Marret S, et al. Positive rolandic sharp wave and 14. Williamson PD, et al. Parietal lobe epilepsy: diag-
periventricular leukomalacia in the newborn. Neuro- nostic considerations and results of surgery. Ann
pediatrics. 1986;17:199–202. Neurol. 1992;31(2):193–201.
4. Kellaway P. The incidence, significance, and natural 15. Proposal for revised classification of epilepsies
history of spike foci in children. In: Henry C, editor. and epileptic syndromes. Commission on Classifica-
Current clinical neurophysiology: update on EEG tion and Terminology of the International
and evoked potentials. Amsterdam: Elsevier; 1980. League Against Epilepsy. Epilepsia. 1989;30(4):
p. 171–5. 389–99.
5. Slatter K. Some clinical and EEG findings in patients 16. Berg AT, et al. Revised terminology and concepts for
with migraine. Brain. 1968;91:85–91. organization of seizures and epilepsies: report of the
6. Kellaway P, Bloxom A, McGregor M. Occipital ILAE commission on classification and terminology,
spike foci associated with retrolental fibroplasia and 2005–2009. Epilepsia. 2010;51(4):676–85.
other forms of retinal loss in children. EEG Clin 17. Panayiotopoulos CP, Obeid T, Waheed G. Differen-
Neurophysiol. 1955;7:469–78. tiation of typical absence seizures in epileptic
7. Penry J, Porter R, Dreifuss F. Simultaneous record- syndromes. A video EEG study of 224 seizures in
ing of absence seizures with video-tape and 20 patients. Brain. 1989;112(Pt 4):1039–56.
EEG and Semiology in Generalized
Epilepsies 6
Nandakumar Bangalore-Vittal

Classification of generalized epilepsies is based D. Clonic


on seizure semiology, combined with EEG and E. Tonic
appropriate personal and family history. This can F. Atonic
help categorize patients into various generalized
epileptic syndromes, enhance pathophysiologic
understanding, optimize patient care, and provide Generalized Tonic–Clonic Seizures
prognostic implications. [1–4]

1. For the purpose of discussion in this chapter, Primary generalized tonic–clonic seizures are
the proposed ABPN content outline for sei- seen in juvenile absence epilepsy, juvenile myo-
zure classification (based on semiology) will clonic epilepsy, epilepsy with generalized tonic–
be followed [1]. clonic seizures only, as well as in provoked sei-
Generalized seizures are divided as follows: zures, including alcohol withdrawal seizures. The
semiology discussed below is mainly based on
A. Tonic–clonic observations in the video-EEG setting.
B. Absence
(i) The entire tonic–clonic event typically lasts
a. Typical 1–2 min and patients have no recollection of
b. Atypical the event.
c. Absence with special features
Prodrome: This is a state where some
i. Myoclonic absences patients might feel a sense of uneasiness, irri-
ii. With eyelid myoclonia tability, or difficulty concentration and occur
hours to a day before the actual seizure.
C. Myoclonic
Tonic phase:
a. Myoclonic • Brief flexion spasm of axial and arm muscles is
b. Myoclonic–atonic associated with the loss of consciousness.
Involvement in respiratory muscles leads to
vocalization (the ictal cry), and patients
become apneic. During the initial closing of the
N. Bangalore-Vittal (&)
Department of Neurology, Nashville General
jaw, tongue biting can happen, more com-
Hospital at Meharry, Vanderbilt University Medical monly on the side of the tongue or the inner
Center, 1161 21st Avenue S, Nashville, TN 37232, cheek. The normal position of the tongue does
USA not include protrusion for biting to affect the
e-mail: nandakumar.bangalore.vittal@vanderbilt.edu

© Springer Science+Business Media LLC 2017 93


M.Z. Koubeissi and N.J. Azar (eds.), Epilepsy Board Review,
DOI 10.1007/978-1-4939-6774-2_6
94 N. Bangalore-Vittal

tip. Falls can occur if the patient is standing. body, or muscle aches are also commonly
The eyes move up and the pupils dilate. reported. The entire postictal state can last
Autonomic signs consist of increase in heart minutes to hours and is proportional to the
rate and blood pressure, sweating, and tra- duration of the seizure and the age of the
cheobronchial secretions. Foaming at the patient, lasting longer in children. Focal face or
mouth can occur due to involvement of the limb weakness (Todd’s paresis) is less com-
glottic muscles and prolonged seizures can mon than in secondary generalized seizures.
cause cyanosis.
Trauma from falls on hard objects, tongue bit-
Clonic phase: ing, vertebral compression fractures, aspiration
• Starts as a tremor, progressing to 4 Hz pneumonia, and neurogenic pulmonary edema can
activity (which denotes the onset of the clonic occur. The last complication in addition to central
phase) and slower. This phase involves cycles apnea and cardiac arrest can play a role in sudden
of inhibition interspersed by brief muscle unexplained death in epilepsy or SUDEP [5].
activity. Each spasm is associated with
pupillary contraction and dilation. (ii) EEG correlation: (Figs. 6.1, 6.2, 6.3, 6.4,
6.5, 6.6, 6.7, and 6.8)
Post-ictal state: • Interictally waking EEG is often normal
• Respiratory activity resumes with slower and and the yield of interictal epileptiform
deeper phases at times mimicking deep sleep discharges is increased by hyperventila-
with snoring. The muscles are relaxed, tion and sleep. Photosensitivity
including the sphincters, which can lead to (photic-stimulus-induced generalized
bowel or bladder incontinence. Variable epileptiform discharges) can be seen in up
degrees of confusion typically follow the per- to 25 % of cases where there is also a
iod of stupor or sleep. Headache, generalized family history of epilepsy.

Fig. 6.1 A generalized tonic-clonic seizure each EEG clip represents 20 s


6 EEG and Semiology in Generalized Epilepsies 95

Fig. 6.2 Representation of ictal discharge during generalized tonic–clonic seizure which lasted for 1 min and 45 s.
Figure 6.1 shows generalized spike-and-wave discharges that gain rhythmicity as shown in Fig. 6.2

Fig. 6.3 Generalized myogenic artifact obscuring most cerebral activity


96 N. Bangalore-Vittal

Fig. 6.4 Generalized clonic activity

Fig. 6.5 Generalized clonic activity, continued

• Epileptiform discharges, when present, predominant and sometimes can only be


are generalized, high-voltage rhythmic seen anteriorly. In children, however,
spike-and-wave activity, anteriorly there can be posterior predominance.
6 EEG and Semiology in Generalized Epilepsies 97

Fig. 6.6 End of clonic activity followed by generalized voltage attenuation

Fig. 6.7 Representation of post-ictal state at 1 min showing generalized voltage attenuation and generalized slow
activity

• Subtle asymmetry in the generalized • In NREM sleep, briefer, slower, and


discharges is commonly noted. fragmented discharges can be seen.
98 N. Bangalore-Vittal

Fig. 6.8 Representation of post-ictal state at 5 min showing continued generalized slow activity

• Nonepileptiform activities such as gen- and slow waves), then corresponding EEG ictal
eralized excessive low-voltage rhythms can be seen.
beta-activity or mild-generalized slow The tonic phase onset is predominated by
activity can be seen secondary to anti- muscle artifact obscuring the EEG. If neuromus-
seizure medications. Chronic therapy cular blockers are used, the EEG shows higher
with valproate and benzodiazepines can amplitude and decreasing frequency discharge in
abolish epileptiform discharges. Frontal the range of 9–10 Hz called the “epileptic
intermittent rhythmic delta activity recruiting rhythm.” Small side-to-side delay can
(FIRDA) has also been reported. be noted with computer analysis, but this delay is
inconsistent. Slower mixed frequency discharges
Generalized spike-and-wave discharges can with increasing amplitude rhythmic spikes are
be seen in patients with no history of seizures, seen bilaterally followed by repetitive complexes
indicating a familial epilepsy trait. Similar dis- of high-amplitude spike-and-slow-wave activity
charges can also be seen in metabolic encepha- in association with the tremor. Slowing of repe-
lopathies or drug-withdrawal states where they titions occurs with the start of violent jerks of the
present as generalized tonic–clonic seizures. clonic phase, as cortical inhibition progresses.
During the postictal state, an isoelectric EEG
Ictal EEG in Generalized Tonic–Clonic Seizures: followed by diffuse slow activity can be seen,
which corresponds to neuronal hyperpolarization.
If generalized tonic–clonic seizures are pre- Focal abnormalities during postictal states are not
ceded by absence (3 Hz spike-and-slow-wave) or expected and might suggest focal epilepsy with
myoclonic jerks (theta-range spikes or polyspikes secondary generalization.
6 EEG and Semiology in Generalized Epilepsies 99

Absence Seizures [1, 6] the middle of a sentence will continue the


sentence exactly where it was left off, while
(i) Typical Absence: This is the hallmark of the observers can appreciate the pause.
childhood absence epilepsy (CAE). Girls are • In the physician’s office, hyperventilation (in
affected more often than boys and present children, performed by blowing on a pin-
between 4 and 8 years of age with very fre- wheel) can elicit a seizure in approximately
quent staring episodes (up to hundreds per 90 % of patients with childhood absence
day). Prognosis is excellent with treatment, epilepsy.
and hence, a low threshold should be main-
tained to obtain EEG for diagnosis. EEG correlation (Fig. 6.9): EEG record is
ideally obtained between 8 and 10 am when the
Semiology: event frequency can be maximal.
• Staring episodes associated with cognitive
impairment and change in facial expression • “Interictal” and Ictal EEG—The interictal
lasting for approximately 10 s. Rhythmic lid nature of these discharges is debatable.
or eye clonic–tonic activity, ocular retropul- Detailed neuropsychological testing con-
sive movements that might extend to involve ducted in patients with the absence showed
head or trunk, and sometimes oral automa- deficits even when discharges were shorter
tisms can be seen. than 3 s. In 30 % of cases, a rhythmic slow
• If the events last longer, there can be nonoral posterior activity or occipital intermittent
gestural motor automatisms. rhythmic delta activity (OIRDA) can be seen.
• Temporal semiological analysis in relation to • Generalized bilaterally synchronous high-
the EEG discharge can show variability in the voltage 2.5–3.5 Hz rhythmic spike-and-wave
same patient. discharges with bifrontal predominance are
• Posture is usually maintained and falls are not classically seen during absence seizures. The
often reported despite frequent episodes. The initial negative deflection is of lower voltage
patients themselves are not aware of having compared to the after-wave, which is of very
these episodes. A child who has an event in higher amplitude.

Fig. 6.9 Absence seizure. This 10 s EEG clip shows ictal discharge—typical 3 Hz generalized high-voltage rhythmic
spike-and-wave discharge train during which the patient demonstrated delayed responsiveness
100 N. Bangalore-Vittal

Fig. 6.10 Absence seizure in a patient with juvenile myoclonic epilepsy. Note the polyspike-and-wave discharge at the
onset of ictal discharge (center of the EEG clip) that lasts *3 s

• At the onset the discharge can be of higher (3–4 Hz) than in CAE. About 80 % of
frequency *3.5 Hz and slows down over patients also have generalized tonic–clonic
time. Occasionally, generalized polyspike- and myoclonic seizures.
and-wave discharges, even bilateral indepen-
dent focal frontal spikes, can occur in typical (iii) Atypical Absence: This is an important
absence. These discharge trains often last seizure type in patients with Lennox–
5–15 s but can last up to 30 s. Hyperventi- Gastaut syndrome. Seizures occur lifelong
lation often precipitates longer trains of and are more resistant to treatment, and
discharges. photosensitivity is not a feature.
• During non-REM sleep, single or multiple
Semiology:
spike-and-wave discharges are frequent.
Onset and end of seizures is more gradual and
eyelid myoclonus is not rhythmic. Forward head
(ii) Juvenile Absence Epilepsy (JAE,
movement, perioral myoclonus, and drooling are
Fig. 6.10): JAE has a typical age of onset
notable. The seizure is usually less than 10 s in
between 9 and 13 years. Absence seizures
duration and the child can continue simple
typically occur in clusters upon awakening,
activities during an event.
and ocular retropulsive movements are less
often seen. Seizures are less frequent than in EEG in atypical absence:
CAE. EEG during absences shows gener- The ictal EEG shows 2–2.5 Hz slow
alized high-voltage spike-and-wave spike-and-wave discharge, which can be irregu-
discharge, which may be slightly faster lar. Interictal EEG shows an abnormal
6 EEG and Semiology in Generalized Epilepsies 101

background with diffuse slowing and multifocal 10–60 s. Falls from seizures are uncommon, and
interictal spikes. postictal state is not seen. Respiratory arrest and
urinary incontinence can occur.
(iv) Absence with special features [6]: EEG in myoclonic absence (Fig. 6.11): Ictal
EEG consists of generalized rhythmic 3 Hz
Myoclonic absence: The mean age of onset is spike-and-wave discharges coinciding with the
seven years with boys more often affected than rhythmic jerking of arms and can last 10–60 s.
girls. Cognition and development are abnormal Occasionally, the classic discharges are inter-
in two-thirds of patients who can have other mixed with polyspike-and-wave discharges.
seizure types such as generalized tonic–clonic
seizures. These seizures are very frequent, can
occur in sleep, and tend to be resistant to therapy. Myoclonic Seizures [7]

Semiology: (i) Myoclonic seizures are seen in many epi-


Events can be precipitated by hyperventilation or lepsy syndromes, including juvenile myo-
upon awakening. Consciousness is impaired to a clonic epilepsy (JME), juvenile absence
variable extent. Eyelid movements are rare, but epilepsy (JAE), and progressive myoclonic
perioral myoclonia is frequent. The arms tend to epilepsies, among others.
stay high due to coexistent tonic contraction, and
rhythmic jerks are typically clonic (rather than Semiology: Brief, symmetric, bilaterally syn-
myoclonic) in nature and involve the arms more chronous jerks affect mainly the shoulders and
commonly than the legs. Rarely, the clonic arms occurring singly or in clusters. Jerks tend to
activity can be unilateral. Events can last for cluster upon awakening, more often in morning,

Fig. 6.11 A 20 s EEG clip of a 9-year-old boy with spike-and-wave discharges. Also note the delayed respon-
staring and rhythmic symmetric shoulder abduction siveness—command given to touch the nose was fol-
movements synchronous with generalized 3 Hz lowed 7 s later when the discharge had ended
102 N. Bangalore-Vittal

Fig. 6.12 A 10 s EEG clip showing generalized polyspike-and-wave discharge in a patient with juvenile myoclonic
epilepsy with myoclonic seizures

and sometimes during nocturnal awakenings or remember these jerks and are sometimes only
when tired in the evenings. At times, ideation of perceived internally as “mild electrical shock.”
the jerks can induce myoclonic jerks. Patients Objects in the hands tend to be thrown off and

Fig. 6.13 Generalized tonic seizure in a patient with attenuation. Tonic arm posturing was accompanied by
LGS. The EEG clip initially shows background slow generalized low-voltage fast activity (middle 1/3 of EEG
activity with intermixed periods of relative voltage clip) lasting 4–5 s
6 EEG and Semiology in Generalized Epilepsies 103

Semiology: Falls preceded by variable jerky


movement of the face/trunk/arms.
EEG: Interictally, generalized slow
spike-and-wave discharges are seen. Parietal
theta or occipital delta activity can be seen as
well. Ictal discharges consist of generalized slow
spike-and-wave or polyspike-and-wave dis-
charges of 2–3 Hz. The spike component corre-
sponds to the myoclonic jerk, and slow wave
corresponds to atonia.
The EEG can help differentiate from negative
myoclonus seen in frontal lobe epilepsy with
rapid bilateral synchrony. Negative myoclonus
interrupts the tonic activity and is <500 ms in
duration without the evidence of antecedent
myoclonia causing the drops.

Clonic Seizures [8]


Fig. 6.14 Electrographic ictal discharge of tonic seizure
in an 11-year-old boy with Lennox–Gastaut syndrome.
Fast activity which is frontally predominant increases in Generalized clonic seizures can be seen in
amplitude and becomes slower in frequency with progres- Angelman’s syndrome, progressive myoclonic
sion of the seizure. Muscle tone is shown by the deltoid epilepsies, and epilepsy with myoclonic-astatic
and abdominal muscles. EKG and respiratory channels
show tachycardia and changes in respiratory rhythm.
seizures (also known as Doose syndrome). The
Generalized tonic seizure: High-amplitude generalized generalized clonic or myoclonic activity seen
spiking associated with tonic activity seen in rectus during a syncopal episode (convulsive syncope)
abdominis muscle, tachycardia, and pause in respiratory may mimic such seizures.
activity (Picture source Gastaut and Tassinari [11])
Semiology: Clonic seizures are predominantly
seen in infants and children. They can be later-
falls can occur with leg muscle involvement
alized at first and then generalize or vice versa.
while standing.
Sometimes, the tonic phase is short and a tonic–
EEG correlation (Fig. 6.12): The back-
clonic seizure appears like generalized clonic
ground is typically normal. Interictal epileptiform
seizure. Autonomic changes are seen in pro-
and ictal discharges consist of generalized
longed seizures consisting of retention of bron-
high-voltage polyspike-and-wave or
chial secretions leading to respiratory distress.
spike-and-wave discharges that are bilaterally
EEG correlation: The interictal EEG back-
symmetric with bifrontocentral predominance.
ground depends on the underlying etiology of
They can be induced by photic stimulation. In
epilepsy. Generalized epileptiform discharges
sleep, fragments of these discharges with shifting
can be seen. The ictal discharge is usually a
right-left predominance can be seen. Limb elec-
complex discharge of 10 Hz recruiting rhythm
trodes can demonstrate the myogenic activity that
mixed with activity of different frequency over
is time locked with EEG discharges.
different scalp regions fluctuating over time.
Peaking of muscle activity does not coincide
(ii) Myoclonic–atonic seizures: These are the
with EEG activity. In the postictal state, gener-
main seizure type in myoclonic-astatic epi-
alized slow activity can be seen.
lepsy of early childhood—Doose syndrome.
104 N. Bangalore-Vittal

Fig. 6.15 A 10 s EEG clip of atonic seizure in a patient with head drop in sitting posture. Head drop coincides with the
high-amplitude slow wave during the third second of the EEG clip below

syndrome and inverted duplication of chromo-


some 15.
The semiology of tonic seizures consists of
rapid flexor contraction of muscles, starting in
body axis associated with loss of awareness,
followed by tonic muscle contraction in the arms
and legs. In a pure tonic seizure, there is no
clonic event following the tonic activity. Tonic
seizures usually last 5–20 s but can last up to a
minute. They can be occasional or very frequent.
Falls usually occur if the patient is standing.
EEG correlation (Figs. 6.13 and 6.14): The
interictal EEG and Ictal EEG are variable
depending on the epilepsy syndrome. Typically,
Fig. 6.16 Polygraphic record shows loss of muscle tone
recorded in quadriceps and lumbar muscles during atonic the ictal EEG consists of the following:
seizure. The EEG shows ictal discharge—short burst of
multiple spikes and slow waves (Picture source Gastaut • The most common ictal pattern consists of
and Tassinari [12])
rhythmic spiking at 10–25 Hz of
low-amplitude desynchronized rhythm of
Tonic Seizures [9] 100 μV.
• Another pattern is that of diffuse
Tonic seizures are predominantly seen in high-amplitude slow-wave discharges at theta
epilepsies associated with abnormal neurocogni- and delta frequency over the vertex region,
tive examination and abnormal EEG background, lasting 0.5–1 s. This is followed by rapid 10–
classically defined as symptomatic generalized 15 Hz rhythm increasing in amplitude over
epilepsies. Examples include Lennox–Gastaut the next 5–10 s.
6
Table 6.1 Generalized Seizures
Seizure Associated Epileptic syndrome Semiology—all are EEG characteristics—generalized discharges
type seizure associated with the loss of (S&W = spike-and-wave
types awareness/consciousness PS&W = Polyspike-and-wave)
Interictal Ictal
GTCSz Generalized GMA—grand mal Loss of consciousness at 4–5 Hz high-voltage rhythmic Tonic phase = epileptic recruiting
tonic–clonic seizures upon onset of tonic phase followed S&W or PS&W particularly upon rhythm (9–10 Hz)
seizures awakening by clonic and postictal phase eye closure Clonic phase = inhibitory cycles
alone Prodrome—not to be interspersed with tonic muscle activity
mistaken for aura Post-ictal phase = electrocerebral
silence
With Juvenile absence Cluster of absences upon 3–5 Hz high-voltage S&W Runs of 3–5 Hz S&W discharges
absence epilepsy awakening can precede discharges precede the above-described ictal
seizures GTCSz discharge
With Juvenile myoclonic Cluster of myoclonic seizures EEG in wakefulness can show Clusters of PS&W discharges prior to
EEG and Semiology in Generalized Epilepsies

myoclonic epilepsy—higher (upon awakening) precede generalized 4–5 Hz S&W or the above-described
seizures or incidence of GTCSz PS&W discharges discharge for tonic–clonic seizure
all three photosensitivity
together
Absence None Childhood absence Very frequent staring High-voltage 2.5–3.5 Hz rhythmic Runs of described Interictal discharges
epilepsy—more than episodes lasting *10 s, can S&W discharges for longer than 3 s more likely cause
90 % undergo be associated with ocular clinical events
remission with age myoclonic movements
Atypical Tonic Lennox–Gastaut Gradual onset and end of Slow background with Interictal 2–2.5 Hz slow irregular S&W
absence seizures syndrome = seizures head movements with spikes discharges
Atonic are lifelong and perioral myoclonia and
photosensitivity is not drooling last <10 s
a feature Can perform simple activities
during seizure
Myoclonic Absence Absence with special Frequent arm abduction, with High-voltage 3–4 Hz S&W High-voltage 3–4 Hz S&W discharges
absence features—myoclonic perioral myoclonia lasting discharges time locked with myoclonic jerks and
absence 10–60 s without postictal delayed responsiveness when tested
state
(continued)
105
Table 6.1 (continued)
106

Seizure Associated Epileptic syndrome Semiology—all are EEG characteristics—generalized discharges


type seizure associated with the loss of (S&W = spike-and-wave
types awareness/consciousness PS&W = Polyspike-and-wave)
Interictal Ictal
Myoclonic Absence JME, JAE, Brief symmetric bilaterally High-voltage PS&W discharges of High-voltage PS&W discharges of 4–
seizures GTCSz progressive synchronous jerks affecting 4–5 Hz (can lead to GTCSz) 5 Hz in trains—can lead to GTCSz
myoclonic epilepsy shoulder and arms—cluster
upon awakening or when
tired
Clonic Myoclonic, Angelman’s Clonic activity lateralized or Interictal EEG depends on etiology Ictal = 10 Hz epileptic recruiting
atonic— syndrome, epilepsy generalized with associated —generalized epileptiform rhythm with mixed frequency activity
infants and with autonomic changes— discharges can be seen varies in different scalp regions
children myoclonic-astatic retention of bronchial (predominance in central regions Postictally—slow activity
Differential seizures secretions —Angelman’s)
diagnosis—
syncope—
adults more
often
Tonic Atonic Lennox–Gastaut Rigid violent muscular 2–2.5 Hz slow S&W activity or Rhythmic 10–25 Hz spikes of low
Atypical syndrome = frequent contractions with limbs in multifocal spikes amplitude
Absence seizures, associated stiff state, with lateralized eye or
falls can lead to deviation Vertex central slow activity followed
trauma by 10–25 Hz fast activity
Atonic Lennox–Gastaut Brief atonia of head or all Slow S&W activity Burst of PS&W followed by
syndrome postural muscles high-voltage slow wave which
Can stand up immediately corresponds to atonia or fast activity or
unless prolonged atonia electrodecrement EMG shows
*400 ms of silence
N. Bangalore-Vittal
6 EEG and Semiology in Generalized Epilepsies 107

The ictal patterns tend to occur in series and definition have alteration in consciousness or
last for minutes with seizures occurring every alteration in awareness.
20–30 s.

References
Atonic Seizures [10]
1. Engel J Jr. A proposed diagnostic scheme for people
These are also predominantly seen in symp- with epileptic seizures and with epilepsy: report of
tomatic generalized epilepsies. Such patients tend the ILAE Task Force on Classification and Termi-
nology. Epilepsia. 2001;42(6):796–803.2.
to have severe developmental delay and cogni- 2. Casaubon L, Pohlmann-Eden B, Khosravani H,
tive impairment as in Lennox–Gastaut syndrome. Carlen PL, Wennberg R. Video-EEG evidence of
Semiology: Brief atonia can be limited to the lateralized clinical features in primary generalized
head or can involve all postural muscles. Loss of epilepsy with tonic-clonic seizures. Epileptic Disord.
2003;5(3):149–56.
consciousness is extremely brief and patients can 3. Usui N, Kotagal P, Matsumoto R, Kellinghaus C,
stand up immediately after a fall. When atonia is Lüders HO. Focal semiologic and electroencephalo-
prolonged, patients will lay motionless on the graphic features in patients with juvenile myoclonic
floor. Falls occur on the body axis. The arms are epilepsy. Epilepsia. 2005;46(10):1668–76.
4. Epilepsy: a comprehensive textbook, vol 1. 2nd edn.
not involved by tonic or myoclonic phenomenon. Generalized tonic clonic seizures (Chap. 47). Wolters
In mild cases, only upper body is involved, and Kluwer, Lippincott & Williams and Wilkin.
in severe cases, the lower body can be involved. 5. Langan Y, Nashef L, Sander JW. Case-control study
EEG correlation (Figs. 6.15 and 6.16): The of SUDEP. Neurology. 2005;64(7):1131–3.
6. Epilepsy: a comprehensive textbook, vol 1. 2nd ed.
interictal EEG is not specific and can show slow Typical and atypical absences (Chap. 49). Wolters
spike-and-wave activity. EMG when recorded Kluwer, Lippincott & Williams and Wilkin.
shows silence for *400 ms or 7. Epilepsy: a comprehensive textbook, vol 1. 2nd edn.
polyspike-and-wave activity. The ictal EEG Generalized myoclonic seizures (Chap. 50). Wolters
Kluwer, Lippincott & Williams and Wilkin.
shows generalized polyspike-and-wave activity 8. Epilepsy: a comprehensive textbook, vol 1. 2nd ed.
with atonia coinciding with slow activity (this Generalized clonic seizures (Chap. 48). Wolters
pattern is most often seen). Other ictal patterns Kluwer, Lippincott & Williams and Wilkin.
can be seen, including low- or high-voltage fast 9. Epilepsy: a comprehensive textbook, vol 1. 2nd ed.
Tonic seizures (Chap. 52). Wolters Kluwer, Lippin-
activity, and flattening or burst of cott & Williams and Wilkin.
polyspike-and-waves followed by generalized 10. Epilepsy: a comprehensive textbook, vol 1. 2nd ed.
slow wave activity. Atonic and myoclonic-atonic seizures (Chap. 51).
Wolters Kluwer, Lippincott & Williams and Wilkin.
11. Gastaut H, Tassinari CA. Handbook of electroen-
Summary: Table 6.1 summarizes the gener- cephalography and clinical neurophysiology, Part A:
alized seizure types, other seizures that can occur Epilepsies (vol 13). Ictal discharges. Amsterdam:
in the same patient and recognized epileptic Elsevier Scientific Publishing Company; 1975. P. 29.
syndromes. Emphasis has been placed on clinical 12. Gastaut H, Tassinari CA. Handbook of electroen-
cephalography and clinical neurophysiology, Part A:
seizure semiology along with interictal and ictal Epilepsies (vol 13). Ictal discharges. Amsterdam:
EEG findings. All generalized seizures by Elsevier Scientific Publishing Company; 1975. P. 34.
EEG and Semiology in Focal Epilepsy
7
Amir M. Arain

The majority of seizures could be divided into ciated observable motor or autonomic compo-
generalized or focal (also known as partial) sei- nents or involving subjective sensory or psychic
zures based on ictal EEG onset. Generalized phenomenon only. These seizures are also
seizures, as the name implies, appear to have a known as simple partial seizures or auras;
bilateral onset on the EEG since the seizure (2) focal seizures with impairment of awareness,
discharge is believed to rapidly involve bilateral also called dyscognitive, implying a larger cere-
networks. In contrast, focal seizures arise within bral involvement. These seizures are also known
networks limited to one hemisphere. As electro- as complex partial seizures; and (3) secondary
graphic seizure onsets may be discretely local- generalized seizures, which are focal seizures
ized or widely distributed, the clinical semiology that evolve to a bilateral convulsive seizure.
complements the electrographic findings in con-
cluding seizure localization. While historical
description of seizures may help in semiology, Temporal Lobe Seizures
video-EEG monitoring of habitual seizures is key
in analyzing details of seizure semiology. Temporal lobe seizures may arise from mesial and
Scalp EEG fails to detect seizure onset in many lateral temporal regions. This distinction is
patients. Scalp EEG fails to lateralize seizures in important in the surgical evaluation of refractory
about 25% of seizures in patients with unilateral epilepsy. Mesial temporal lobe seizures will often
mesial temporal lobe epilepsy and as high as 33– be abolished after standard temporal lobectomy or
50% of seizures in patients with extratemporal selective amygdalohippocampectomy, while lat-
epilepsy [1–3]. In these cases, seizure semiology eral temporal lobe seizures will often require
is often very helpful. invasive video-EEG monitoring in order to map
Some semiological features are useful for eloquent cortex and tailor a surgical resection. The
lateralizing seizure onset zone to a hemisphere clinical features favoring mesial temporal lobe
(Table 7.1), and others can help in further lobar epilepsy include early age of onset, history of
or sublobar localization (Table 7.2). Focal sei- complex febrile seizures, congenital brain mal-
zures can be divided into (1) seizures without formations, CNS infections, tumors, head trauma,
impairment of awareness with or without asso- perinatal injury, stuttering course of seizure con-
trol, and typically with well-controlled seizures in
early childhood but re-emergence of refractory
epilepsy in adolescence or early adulthood and
A.M. Arain (&) infrequent or rare secondarily generalized sei-
Department of Neurology, Vanderbilt University zures. In contrast, lateral temporal lobe epilepsy is
Medical Center, A-0118 Medical Center North,
Nashville, TN 37232, USA typically characterized by later age of onset of
e-mail: amir.arain@vanderbilt.edu seizures, absence of early risk factors, absence of

© Springer Science+Business Media LLC 2017 109


M.Z. Koubeissi and N.J. Azar (eds.), Epilepsy Board Review,
DOI 10.1007/978-1-4939-6774-2_7
110 A.M. Arain

Table 7.1 Semiologic Sign Lateralization of hemisphere


signs that help in
lateralization of the Dystonic limb posturing Contralateral
epileptogenic focus Head turning (early) Ipsilateral
Head turning (late and versive, in Contralateral
transition to generalization)
Figure of 4 sign Contralateral
Todd’s paralysis Contralateral
Focal clonic activity Contralateral
Unilateral eye blinking Ipsilateral
Unilateral limb motor automatisms Ipsilateral
Postictal nose wiping Ipsilateral

Table 7.2 Semiologic Sign Localization


signs that help in
localization of the Preservation of ictal speech Nondominant temporal lobe
epileptogenic focus Ictal speech arrest Dominant temporal lobe
Postictal aphasia Dominant hemisphere
Ictal vomiting Right temporal lobe
Hypermotor Frontal lobe, less commonly insular or even temporal
Ictal urinary urge Right temporal lobe
Limb paresthesia Contralateral parietal lobe
Simple visual hallucination Contralateral occipital lobe
Complex visual Contralateral temporo–occipital lobe
hallucination

hippocampal atrophy, and more common negative followed within 30 s by 5–7 Hz sphenoidal
structural or functional brain imaging. maximum theta activity [5]. At times, sudden
Mesial temporal lobe seizures often have an generalized or lateralized suppression or attenu-
aura—with rising epigastric discomfort or inap- ation is also seen [6]. Interictal EEG abnormali-
propriate fear or olfactory feeling and or auto- ties consist of frequent spikes or sharp waves
nomic signs like; pallor, flushing, mydriasis, predominantly in the inferomesial (sphenoidal
irregular respiration or respiratory arrest, electrodes) and anterior temporal regions.
abdominal borborygmi, and eructation. These Lateral temporal lobe seizures are relatively
seizures often exhibit contralateral dystonic pos- less common than mesial temporal lobe seizures.
turing of the hand (and ipsilateral hand automa- However, these seizures may be associated with
tisms), preserved ictal language if the focus is in an aura of vertigo, or with auditory or visual
nondominant temporal lobe, ictal speech arrest if hallucinations. These seizures often evolve early
the focus is in dominant temporal lobe, postictal to a unilateral clonic activity and early head
nose wiping with ipsilateral hand, ictal vomiting turning. On scalp EEG, lateral temporal lobe
or retching behavior, and head version in transi- seizures have a high incidence of repetitive
tion to secondary generalization. epileptiform discharges at ictal onset [6]. Also if
Mesial temporal lobe seizures are often asso- present, a transitional sharp wave at ictal onset
ciated with a rhythmic theta-range ictal discharge favors a neocortical rather than hippocampal
on scalp EEG [4]. Typically, an initial focal seizure onset [7]. Neocortical seizures often start
temporal rhythmic activity of <5 Hz frequency is with higher frequency activity (alpha or beta
7 EEG and Semiology in Focal Epilepsy 111

range) on scalp EEG, but may also be associated They often occur in clusters and may present as
with irregular, polymorphic, 2–5 Hz lateralized bizarre attacks that appear hysterical with fencing
activity [4]. Interictal EEG abnormalities in and posturing [9], prominent motor automatisms
neocortical temporal epilepsy may be absent or (hypermotor), usually complex, aggressive sex-
consist of occasional spikes or sharp waves pre- ual automatisms, and vocalizations with variable
dominantly in the anterior or mid-temporal complexity [10]. Frontal lobe seizures can be
regions [8]. grouped into four based on their origin in the
While certain semiological features can help frontal lobe.
lateralize the seizure onset zone and, further, Supplementary motor seizures may occur
localize it to the temporal lobe, they cannot dis- without alteration of consciousness, representing
criminate between mesial and lateral temporal simple partial seizures. Typical characteristics
onsets. These include ictal emeticus, ictal urinary include prominent tonic posturing, usually of the
urge, and ictal spitting, which often localize to contralateral upper extremity, contraversive head
the right temporal lobe, and piloerection, which and eye deviation, preservation of consciousness
localizes to the left temporal lobe. in some patients, and postictal Todd’s paresis.
Anterior cingulate seizures are characterized
by sudden changes in mood and elaborate
Extratemporal Lobe Seizures gestural frequent tonic/dystonic posturing, rare
changes of facial expression of fear, vocaliza-
Extratemporal lobe seizures can mimic temporal tion, complex motor automatisms that are often
lobe seizures semiologically and electrographi- hypermotor, and autonomic features.
cally (1). These are often referred to as temporal Orbitofrontal seizures are characterized by
plus epilepsies (TPE) (2). They are often difficult sudden complex gestural automatism, hypermo-
to differentiate simply by using general clinical tor activity, olfactory fear and prominant facial
features. However, early ictal signs and symp- expression of fear hallucinations, illusions, and
toms that suggest involvement of the perisylvian prominent autonomic features.
region, the orbitofrontal cortex, or the Dorsolateral frontal seizures are often char-
temporo-parieto-occipital junction should acterized by unilateral clonic activity involving
heighten the suspicion of temporal plus epilep- face and spreading to the arm and leg. These
sies (2). This is of clinical importance since seizures often occur without the alteration of
misdiagnosis as temporal lobe epilepsy and rec- consciousness [11]. They often involve forced
ommending temporal lobectomy will result in head turning to the contralateral side with lateral
surgical failure and persistence of seizures. deviation of the eyes indicating activation of
contralateral frontal eye field [12, 13]. In domi-
nant frontal lobe convexity seizures, aphasia is
Frontal Lobe Seizures often seen.
In frontal lobe seizures, the ictal EEG often
Although they constitute the second most com- shows excessive generalized muscle artifact at
mon focal seizures after temporal lobe seizures, the onset, and the ictal discharge is typically brief
frontal lobe seizures can pose a diagnostic chal- and delayed. The ictal EEG can also be falsely
lenge. They do not always produce loss of localizing. The Interictal EEG is commonly
awareness, and when they do, they usually have normal, though multifocal epileptiform dis-
brief or no postictal confusion. Thus, frontal lobe charges may be seen in mesial frontal lobe sei-
seizures can be mistaken for psychogenic zures. In dorsolateral frontal epilepsy, the
nonepileptic seizures, movement disorders, or interictal EEG may show focal interictal epilep-
parasomnias. Frontal lobe seizures are typically tiform discharges localizing to the epileptogenic
characterized by stereotypical pattern, frequent focus. Frontal lobe seizures may also be char-
nocturnal occurrence, and brief duration [9]. acterized by bilateral synchronous interictal
112 A.M. Arain

epileptiform discharges representing secondary when the seizure focus is in the mesial or basal
bilateral synchrony. Alternatively, focal epilep- occipital regions. False localization can also be
tiform discharges are seen in the ipsilateral or seen in occipital lobe seizures [6]. Ictal EEG may
contralateral temporal or frontal lobes. become more evident as a low-voltage fast
Parietal and occipital lobe seizures can often activity progressively followed by a rhythmic
present with temporal or frontal symptomatology epileptiform discharge. Ictal EEG in occipital
because of spread of their locus of origin. They seizures may vary with the pathway of propa-
are often the cause for pseudotemporal and gation, typically spreading to the temporal or
pseudofrontal seizures. frontal region or bilaterally. At times, the initial
occipital onset is missed while the ictal discharge
rapidly becomes predominant in the temporal
Parietal Lobe Seizures region giving a false localization.
The interictal EEG in occipital lobe epilepsy
The symptomatogenic zone of parietal lobe sei- is typically abnormal. Posterior temporal
zures may be distant from the seizure onset zone, epileptiform discharges are the most common
with the seizure semiology representing ictal patterns. Centrotemporal spikes are frequently
propagation beyond the parietal association cor- seen with occipital paroxysms. Other patterns
tex. Propagation pathways can be from the include random unilateral or bilateral occipital
parietal lobe to the sensorimotor cortex, premotor spikes, often with fixation-off sensitivity.
eye field, supplementary motor, or temporolim- Fixation-off sensitivity is characterized by pos-
bic region. These seizures can be characterized terior or generalized epileptiform discharges that
by focal motor clonic activity contralateral to the consistently occur after eye closure and last as
epileptogenic zone, tonic posturing of extremi- long as the eyes are closed. It may represent an
ties, oral–gestural automatisms, complex ictal or interictal phenomenon. Some patients
automatisms, painful or thermal or sexual/groin may also have occipital spikes exclusively dur-
paresthesia, head deviation, Todd’s paralysis, or ing sleep, while others may consistently have
postictal aphasia. normal EEG.
The ictal EEG may be poorly informative and
falsely localizing [6]. It may show diffuse voltage
suppression followed by sharp waves spreading Insular Lobe Seizures
either anteriorly or posteriorly to the frontal or
parietal operculum. Interictal epileptiform dis- The insula is an island of cerebral cortex folded
charges, if present, are typically seen in the deep within the lateral sulcus. It has extensive
temporal region. connections with temporal, occipital, opercular,
and orbitofrontal regions, and with the triangular
and opercular parts of the inferior frontal gyrus.
Occipital Lobe Seizures Seizures restricted to the insular lobe do not
result in impairment of consciousness and may
Occipital lobe seizures are often characterized by manifest as laryngeal discomfort with thora-
elementary visual hallucinations consisting of coabdominal constriction or dyspnea, vomiting,
flashing or steady spots or simple geometric hypersalivation, laryngeal constriction, followed
forms. Other manifestations include repeated eye by unpleasant paresthesias or warmth in the
blinking and tonic eye and head deviation, either perioral region or involving larger somatic areas,
ipsilateral or contralateral to the ictal discharge. dysarthria or dysphonia, ending with focal motor
Occipital seizures are often associated with such manifestations.
vegetative phenomena as vomiting. The ictal The distance of the insular cortex from the
EEG in occipital lobe epilepsy may be normal surface makes scalp recordings imprecise.
7 EEG and Semiology in Focal Epilepsy 113

Ictal EEG often is marked by muscle artifacts and 6. Foldvary N, Klem G, Hammel J, Bingaman W,
can show ictal discharges in the temporal Najm I, Luders H. The localizing value of ictal EEG
in focal epilepsy. Neurology. 2001;57:2022–8.
regions. Interictal epileptiform discharges on 7. Azar NJ, Lagrange AH, Abou-Khalil BW. Transi-
scalp EEG are usually absent. tional sharp waves at ictal onset—a neocortical ictal
pattern. Clin Neurophysiol. 2009;120:665–72.
8. Pfander M, Arnold S, Henkel A, Weil S, Wer-
hahn KJ, Eisensehr I, Winkler PA, Noachtar S.
References Clinical features and EEG findings differentiating
mesial from neocortical temporal lobe epilepsy.
1. Pataraia E, Lurger S, Serles W, Lindinger G, Aull S, Epileptic Disord. 2002;4:189–95.
Leutmezer F, Bacher J, Olbrich A, Czech T, 9. O’Brien TJ, Mosewich RK, Britton JW, Cascino GD,
Novak K, Deecke L, Baumgartner C. Ictal scalp So EL. History and seizure semiology in distinguish-
EEG in unilateral mesial temporal lobe epilepsy. ing frontal lobe seizures and temporal lobe seizures.
Epilepsia. 1998;39:608–14. Epilepsy Res. 2008;82:177–82.
2. Walczak TS, Radtke RA, Lewis DV. Accuracy and 10. Williamson PD, Spencer DD, Spencer SS,
interobserver reliability of scalp ictal EEG. Neurol- Novelly RA, Mattson RH. Complex partial seizures
ogy. 1992;42:2279–85. of frontal lobe origin. Ann Neurol. 1985;18:497–504.
3. Mosewich RK, So EL, O’Brien TJ, Cascino GD, 11. Salanova V, Morris HH, Van Ness P, Kotagal P,
Sharbrough FW, Marsh WR, Meyer FB, Jack CR, Wyllie E, Luders H. Frontal lobe seizures: electro-
O’Brien PC. Factors predictive of the outcome of clinical syndromes. Epilepsia. 1995;36:16–24.
frontal lobe epilepsy surgery. Epilepsia. 12. Janszky J, Fogarasi A, Jokeit H, Ebner A. Lateral-
2000;41:843–9. izing value of unilateral motor and somatosensory
4. Ebersole JS, Pacia SV. Localization of temporal lobe manifestations in frontal lobe seizures. Epilepsy Res.
foci by ictal EEG patterns. Epilepsia. 1996;37:386– 2001;43:125–33.
99. 13. Wyllie E, Luders H, Morris HH, Lesser RP, Din-
5. Risinger MW, Engel J Jr, Van Ness PC, Henry TR, ner DS. The lateralizing significance of versive head
Crandall PH. Ictal localization of temporal lobe and eye movements during epileptic seizures. Neu-
seizures with scalp/sphenoidal recordings. Neurol- rology. 1986;36:606–11.
ogy. 1989;39:1288–93.
Status Epilepticus
8
Pavel Klein

Definition Classification

Status epilepticus (SE) is defined as either A simple classification of SE is shown in


(i)  30 min of continuous seizure activity or Table 8.1.
(ii)  2 sequential seizures spanning this period The task force on SE of the ILAE Commis-
without full recovery between seizures. sion on European Affairs has suggested a more
exhaustive classification, as shown in Table 8.2
[2].
Epidemiology Non-convulsive status epilepticus (NCSE) is
common and continues to be underdiagnosed.
Incidence has been estimated between 18 Continuous video-EEG monitoring is essential
patients per 100,000 population in a retrospective for its diagnosis. NCSE may present with altered
epidemiological study in Rochester, MN, to mental status or psychosis, as well as with focal
41/100,000 population (with 50 SE non-convulsive neurological symptoms or defi-
episodes/year/100,000) in a prospective epi- cits. Seizures may not be picked up on a routine
demiological study in Richmond, VA. Both EEG, and the diagnosis may be missed unless
partial SE and generalized SE occur with a high 24-h-continuous video-EEG (CEEG) monitoring
frequency. Incidence of generalized SE in that is performed.
study was 6.2/100,000. Generalized SE is more In a study of 570 adults with unexplained
common in children at 7.5/100,000 than in the decrease in level of consciousness who under-
elderly 22/100,000. There are estimated went CEEG monitoring, seizures were detected
126,000–195,000 SE events with 22,200–42,000 in 19% of the patients. Seizures were exclusively
deaths per year in the USA [1]. non-convulsive in 92% of these patients. Coma,

P. Klein (&)
Department of Neurology, Mid-Atlantic Epilepsy
and Sleep Center, 6410 Rockledge Drive, Suite 410,
Bethesda, MD 20817, USA
e-mail: kleinp@epilepsydc.com

© Springer Science+Business Media LLC 2017 115


M.Z. Koubeissi and N.J. Azar (eds.), Epilepsy Board Review,
DOI 10.1007/978-1-4939-6774-2_8
116 P. Klein

Table 8.1 Classification Generalized Convulsive


of status epilepticus (SE)
Non-convulsive
Focal Convulsive (epilepsia partialis continua)
Non-convulsive
Non-epileptic (“Pseudo-SE”)

Table 8.2 Classification of SE, European alternative [2]


Classification of SE
1. NCSE occurring in the neonatal and infantile epilepsy syndromes
a. Ohtahara syndrome
b. West syndrome
c. Sever myoclonic encephalopathy of infancy (SMEI; Dravet syndrome)
2. NCSE occurring only in childhood
a. NCSE is early-onset benign childhood occipital epilepsy (Panayiotopoulos syndrome)
b. NCSE in other forms of childhood epileptic encephalopathies, syndromes, and etiologies (e.g., Ring chromosome 20,
Angelman syndrome, Rett syndrome, myoclonic-astatic epilepsy, and other childhood myoclonic encephalopathies
c. Electrical status epilepticus in slow wave sleep (ESES)
d. Landau–Kleffner syndrome
3. Convulsive SE occurring only in childhood
a. Febrille SE
4. NCSE occurring both in childhood and adult life with epileptic encephalopathy
a. NCSE in the Lennox–Gastaut syndrome
i. Atypical absence SE
ii. Tonic SE
b. Other forms of NCSE in patients with learning disability or disturbed decerebral development (cryptogenic or symptomatic)
without epileptic encephalopathy
c. Typical absence SE in idiopathic generalized epilepsy
d. Complex partial SE:
i. Limbic
ii. Nonlimbic
e. NCSE in the post-ictal phase of tonic–clonic seizures
f. Subtle SE (myoclonic SE occurring in the late stage of convulsive SE)
g. Aura continua [with (i) sensory, (ii) special sensory, (iii) autonomic, and (iv) cognitive symptoms]
5. Convulsive forms of SE occurring in childhood and adult life
a. Tonic–clonic status epilepticus
b. Epilepsia partialis continua 9 EPC; simple partial motor SE
c. Myoclonic SE
6. NCSE occurring in late adult life
a. De novo absence SE of late onset
7. Boundary syndromesa
a. Some cases of epileptic encephalopathy
b. Some cases of coma due to acute brain injury with epileptiform EEG changes
c. Some cases of epileptic behavioral disturbances or psychosis
d. Some cases of drug-induced or metabolic confusional state with epileptiform EEG changes
a
Boundary syndromes are defined as cases in which it is not clear to what extent the continuous epileptiform electrographic
abnormalities are contributing to the clinical impairment

age <18 years, history of prior epilepsy and electrographic seizure versus 5% of
convulsive seizures prior to monitoring were the non-comatose patients. Prevalence of NCSE by
risk factors for electrographic seizures. Seizures diagnosis is shown in Table 8.3 [3].
were detected during the first 24 h of CEEG The diagnosis of non-convulsive status
monitoring in 88% of patients, during day 2 of epilepticus in patients with altered mental status
monitoring in another 5%, and after 48 h of and ambivalent EEG can be aided with intra-
monitoring in 7%. 20% of comatose patients venous benzodiazepine bolus injection, e.g.,
required >24 h of monitoring to detect the first lorazepam 1 mg, which may abolish the
8 Status Epilepticus 117

Table 8.3 Prevalence of non-convulsive seizures/SE in Etiology of epilepsia partialis continua is


adult ICU by diagnosis (n = 570) [3] usually due to a fixed or progressive lesion
% involving the motor strip. These include tumors,
Overall 18 vascular lesions (CVA, AVM), infection (abscess
Unexplained altered MS 15 —especially TB, encephalitis, HIV, and subacute
Epilepsy 31 measles encephalopathy), autoimmune (Ras-
CNS infection 26 mussen), systemic lupus erythematosus (SLE),
Tumor 23 paraneoplastic, cortical dysplasia, Sturge–Weber
Neurosurgery 23 syndrome, traumatic brain injury (TBI), multiple
Traumatic brain injury 22 sclerosis, gliomatosis cerebri, or progressive
Toxic-metabolic 21 multifocal leucoencephalopathy.
Stroke-SAH 18
Medications that may cause SE include theo-
Stroke-hemorrhagic 13
phylline, lithium, isoniazid, cyclosporine, tacro-
limus, ifosfamide, amoxapine, flumazenil, and
Stroke-ischemic 13
among antiseizure medications (ASMs) tiaga-
Hypoxia 10
bine, vigabatrin and valproate.
Uncommon causes of SE include the
Table 8.4 Etiology of SE [4]
following:

Adult % Pediatric %
– Paraneoplastic etiology, with associated
CVA 25 Fever/infection 35 autoantibodies (i) Hu, (ii) Ma2, and
ASM change 19 ASM change 20 (iii) CRMP-5—all of them target intracellular
EtOH/recr. drugs 12 Unknown 9 antigens. Most common associated neo-
Anoxia 11 Metabolic 8 plasms are small cell lung carcinoma (asso-
Metabolic 9 Congenital 7 ciated with all of the above antibodies),
testicular germ call carcinoma (Ma2), and
Unknown 8 Anoxia 5
thymoma (CRMP5). In these conditions, SE
Fever/infection 5 CNS infection 5
may be refractory and respond to tumor
TBI 5 4
removal.
Tumor 4 CVA 3 – Autoimmune diseases including Hashimoto’s
CNS infection 2 EtOH/recr. drugs 2 thyroiditis, SLE, Rasmussen’s encephalitis
Congenital 1 Tumor 1 syndrome, with associated thyroid microso-
mal antibodies, voltage-gated K channels
antibodies, NMDA-receptor antibodies, all of
epileptiform discharges, sometimes with associ- which are extracellular antigens. Rasmussen’s
ated paradoxical improvement of the patient’s encephalitis syndrome is associated with
level of consciousness. anti-NR2A antibody (NMDA-receptor sub-
Etiology of SE for adults and children is unit GluRepsilon2).
shown in Table 8.4 [4]. – Infectious, ill-defined include the recently
A variable proportion of patients with status described new-onset refractory SE (NORSE)
have preceding the history of epilepsy, in some in adults and febrile infection-related epilepsy
studies estimated up to approximately 45%. In syndrome (FIRES) in previously normal
approximately 50% of patients with preceding children.
epilepsy, the epilepsy is acute symptomatic. It is – Chromosomal, genetic, or congenital
remote symptomatic in 20% cases, idiopathic in dysplastic and inborn errors of metabolism,
14%, and unclassified in 17%. all covered elsewhere in this book.
118 P. Klein

SE Clinical Stages Pathophysiology of epilepsia partialis con-


tinua is poorly understood. It may involve cor-
SE is divided into 4 phases (Table 8.5). Prodro- tical reflex myoclonus which originates from
mal phase may include confusion, myoclonus, hypersynchronous discharges of neuronal
and increasing seizure frequency without inter- aggregates in the cortex and may involve
vening loss of consciousness. Stage 1 is divided long-loop reflexes via the ventrolateral posterior
into incipient (continued seizure of >5 min nucleus of the thalamus to generate cortical
duration) and early (5–30 min duration). myoclonus [7].
EEG staging includes (i) discrete seizures
with interictal slowing; (ii) waxing/waning of
ictal discharges; (iii) continuous ictal discharge Metabolic Consequences of SE
evolving into continuous ictal discharges inter- (Table 8.6)
spersed by flat EEG; and (iv) Post-ictal:
PLEDs/PEDs with flat background [5]. During the initial acute stage of SE, there is an
increase in blood pressure, increase in cerebral
blood flow and oxygen utilization, increased
Pathophysiology of SE serum lactate, and, initially, increased glucose
levels. There may be associated respiratory and
SE evolves from an isolated seizure when there is metabolic acidosis. Subsequently, blood pressure
a failure of seizure containment leading to the normalizes and may fall, respiration becomes
transformation of isolated seizure(s) to SE. Ini- depressed, with falling oxygen and rising CO2
tially (ms/s), there is increased glutamate release levels, decrease in cerebral blood flow and brain
and ion channel activation receptor phosphory-
lation and desensitization. After approximately Table 8.6 Complications of tonic–clonic SE (adapted
30–45 min, there is receptor trafficking with from Ref. [2])
GABAA-R (b2-3, ɤ subunits) internalized from Cerebral Hypoxic/metabolic damage
synapse to cytosol where they are endocytosed
Excitotoxic damage
and destroyed, leading to reduced number of
Edema and " ICP
GABAA receptors at the synaptic membrane,
with simultaneous recruitment from cytosol to Venous thrombosis, infarction,
hemorrhage
the membrane of glutamatergic AMPA/NMDA
receptors (NR1 subunits). As a result of this Cardiac Hypo/hypertension
trafficking, the number of functional NMDA Cardiac failure/shock
receptors per synapse increases while the number Tachy-/brady-arrhythmia,
of functional GABAA receptors decreases [6]. arrest
This contributes to the resistance of prolonged Respiratory Apnea, respiratory failure
SE to GABAergic medication such as Pulmonary edema,
benzodiazepines. hypertension, pneumonia,
aspiration, PE
Autonomic Hyperthermia, sweating
Table 8.5 Clinical stages of status epilepticus Metabolic/systemic Hypoglycemia, # Na, # K,
Acidosis
Prodromal
Acute renal failure
Stage 1 (early) Incipient 5 min Acute hepatic failure
Early 5–30 min DIC
Rhabdomyolysis
Stage 2 (established) 30–60 min Infections
Stage 3 (refractory) >60 min Fractures
Post-ictal Labs (other) Leukocytosis; CSF pleocytosis
8 Status Epilepticus 119

oxygenation, and decrease in glucose level. Evaluations


There may be hyperthermia. These factors result
in energy mismatch, with higher brain energy Labs: Initially, check glucose, chemistry profile
utilization than supply and exacerbation of neu- —including calcium, magnesium, and phosphate
ronal injury. During later stages of both convul- —CBC, and urine toxicity screen ASM levels (if
sive SE and in NCSE, there is an increase in applicable). LP should be done if CNS infection,
serum levels of neuron-specific enolase, a marker vasculitis, autoimmune, paraneoplastic, or
of brain injury. Neuronal injury may occur even meningeal neoplastic disease is suspected as a
in the absence of metabolic derangement, and possible cause, after ruling out mass lesions with
without hypoxemia, hypotension, hypoglycemia, CT or MRI. Leukocytosis is commonly seen with
and hyperthermia. SE without any infection because of blood–brain
barrier breakdown during SE. CSF WBC counts
of up to 30  106 can be seen.
Management Continuous EEG monitoring should be star-
ted, if available, if SE has continued for >60 min.
Outline of the SE management and management Neuroimaging changes associated with SE
timeline is shown in Table 8.7 [8]. (Table 8.8; Figs. 8.1 and 8.2). MRI may be
focally abnormal during both convulsive and
non-convulsive status epilepticus. This may be
misdiagnosed as acute lesions, e.g., stroke or
Table 8.7 Status epilepticus: management timeline encephalitis. Possible MRI abnormalities during
(adapted from Ref. [8])
SE include increased FLAIR (fluid-attenuated
Time Treatment inversion recovery), T2 signal hyperintensity and
post-onset
high-intensity signal DWI (diffusion-weighted
0–5 min Diagnose
imaging), both local at seizure focus and remote,
ABC: airway, breathing, circulation
commonly in the ipsilateral posterior thalamus
Labs: glucose, chemistry, CBC, toxicity screen
ASM levels (if applicable)
(pulvinar), contralateral cerebellum, and bilateral
splenium of the corpus callosum (Table 8.8;
iv Glucose + thiamine 100 mg if applicable
Figs. 8.1 and 8.2). These changes may be due to
4–5 min Lorazepam 4 mg (0.1 mg/kg), or
Midazolam 10 mg im, or the prolonged ictal activity increasing glucose
Diazepam 10 mg (0.2 mg/kg) or rectal utilization, which is not adequately matched by
diazepam
the enhanced blood flow. Blood flow–metabo-
7–8 min Phenytoin or Fos-phenytoin 20 mg/kg i.v. at
lism uncoupling leads to a reduction of
 50 mg/min phenytoin or
150 mg/min Fos-phenytoin high-energy adenosine phosphates and tissue
(  0.75 mg/kg/min)
Pyridoxine 100–200 mg iv in children under
18 months Table 8.8 Peri-ictal SE MRI imaging abnormalities
10 min Repeat lorazepam or diazepam if seizures still Local" T2/DWI Remote −" DWI ± flair
ongoing
Mass effect Ipsi/bilat thalamic
30–60 min EEG monitoring unless status ended and patient lesions
waking up
Hippocampal swelling Cerebellar diaschisis,
40 min Phenobarbital 20 mg/kg at  5 mg/min
(0.75 mg/kg per min) contralateral
60+ min iv Anesthesia: Focal cortical lesions Splenium abnormalities
Propofol 3–5 µg/kg load, 5–10 mg/kg/h initial Migratory focal cortical " Reversible posterior
infusion then
T2/DWI lesions leukoencephalopathy
10–120 µg/kg/min, as tolerated/needed or
Midazolam 0.2 mg/kg load, then 0.05– BBB breakdown
0.3 mg/kg/h infusion
Pentobarbital 3–5 mg/kg load, then 1 mg/kg/h
" Blood vessel
infusion caliber/flow (MRA)
120 P. Klein

Fig. 8.1 Increased T2 signal (FLAIR protocol) in epilepsia partialis continua arising from the right hemisphere (a),
with resolution after i.v. anesthesia (b)

Fig. 8.2 MR images, FLAIR sequences. a Arrow shows hemisphere seizures. c Axial DWI showing expanded
left-sided region of change (shown only on the left side and hyper-intense splenium during bitemporal SE.
for clarity, but bilateral change is present). b Axial DWI d Axial DWI images showing gyral pattern of restricted
showing gyral pattern of restricted diffusion and ipsilateral diffusion and crossed cerebellar diaschisis in a right
thalamic (pulvinar) change in a patient with left hemispheric SE

hypoxia. The regional hyperperfusion serves as a Table 8.9 Treatment of early SE (first-line treatment)
(adapted from Ref. [2])
compensatory mechanism, but is insufficient to
prevent the stimulation of anaerobic glycolysis Route Adult Pediatric dose
dose
due to the prolonged ictal activity.
Lorazepam iv 4 mg 0.1 mg/kg
Midazolam im 5–10 mg 0.15–0.3 mg/kg

Treatment Diazepam iv (  2–5 mg/min) 10–20 mg 0.25–0.5 mg/kg


Rectal 10–20 mg 0.5–0.75 mg/kg

Early, Stage 1 (Table 8.9).


Primary treatment includes benzodiazepines,
midazolam, lorazepam, and diazepam. Midazo- Diazepam has more rapid onset because of
lam i.m. may be the most effective followed by greater lipid solubility. Lorazepam has longer
lorazepam followed by diazepam as shown in 2 anticonvulsant duration (12 h) than diazepam
large phase 3 studies ([9], Table 8.10). (30 min) and has less potential for respiratory
8 Status Epilepticus 121

Table 8.10 Comparative efficacy of diazepam, loraze- Phenobarbital may be more effective than
pam, and midazolam in the treatment of SE before arrival phenytoin in suppressing SE, but is less used in
in the hospital. Response = seizure termination prior to
arrival in ER without rescue treatment [9, 10] practice, possibly because of greater potential for
respiratory depression and intubation, particu-
Dose/route Response %
larly in combination with benzodiazepines.
Midazolam 10 mg im 73 Comparison of efficacy of lorazepam, pheno-
Lorazepam 4 mg iv 59–63 barbital, and phenytoin with or without benzo-
Diazepam 10 mg iv 43 diazepam is shown in Table 8.12.
Placebo Im 21 Phenytoin has the advantage of lack of seda-
tion or respiratory depression. Maximum CNS
concentration is reached in 20 min. Its potential
depression and sedation. Seizure recurrence is side effects include bradycardia (7%) and
common after benzodiazepine administration, hypotension (27%) [11]. Phenytoin is alkaline
especially in acute symptomatic SE. and with extravasation causes skin irritation and
In a study comparing the efficacy of intra- “purple glove syndrome” which occurs in 1–2%
muscular midazolam with intravenous lorazepam of patients [11]. Phenytoin needs to be adminis-
for children and adults in status epilepticus tered in normal saline as it precipitates in dex-
treated by paramedics outside the hospitals, sei- trose. Infusion rate is 50 mg/min for adults,
zures were stopped prior to arrival in the hospital 20 mg/min for the elderly, and  25 mg/min for
in 73% subjects treated with midazolam 10 mg i. children. The elderly have a higher risk for car-
m. and 63% subjects treated with lorazepam diovascular complications. Heart rate and blood
4 mg i.v. with 4.5 and 6.5 min to the cessation of pressure should be monitored with the reduction
convulsions, respectively. Seizures recurred in of infusion rate if hypotension occurs. Total
11% in both groups. Adverse-event rates were blood level and free phenytoin level should be
similar in the two groups [10]. checked at the end of the infusion, within 30 min
Secondary treatment includes phenytoin or of infusion if seizures persist or 1–2 h after
phenobarbital (Table 8.11). It should be admin- infusion if seizures stop, in order to help with
istered at the same time as the benzodiazepine. timing of maintenance treatment.

Table 8.11 Treatment parameters of SE treatment with phenytoin and phenobarbital


Route Adult dose Pediatric dose
Phenytoin/f-PHT iv infusion rate 20 mg/kg, Same
PHT  50 mg/min
F-PHT  100 mg/min
Phenobarbital iv infusion rate 15–20 mg/kg  100 mg/min Same 20 mg/min in neonates and infants

Table 8.12 Comparison of efficacy of SE treatment parameters phenytoin, phenobarbital, and lorazepam [11]
Response rate % (Sz Dose Maximal admin rate Infusion time
end  20 min) (mg/kg) (mg/min) (min)
Lorazepam 65 0.1 2 4.7
Phenobarbital 58 15 100 16.6
Phenytoin + diazepam 56 18 50 42
0.15 5
Phenytoin 44 18 50 33
122 P. Klein

Table 8.13 Use of valproate and levetiracetam in Stage 2 status epilepticus (adapted from Ref. [2])
Route Adult dose Pediatric dose
Valproate iv infus rate 15–30 mg/kg 20–40 mg/kg
10–15 min 10–25 mg/kg neonates
Levetiracetam iv infus rate Not established; 2000–4000 used 5–15 min Not established

Fos-phenytoin, a phosphate ester prodrug of defined as SE that continues  24 h after the


phenytoin, has replaced phenytoin in many onset of anesthesia, including SE recurrence after
institutions. Fos-phenytoin is given as phenytoin tapering of anesthesia. It occurs in approximately
equivalent (PE), with the dose of 20 mg/kg. It 10–15% of SE [14].
can be given in dextrose or normal saline. It is Pathophysiology of refractory SE (RSE):
water soluble and can be given i.m. as well as i.v. Pharmacoresistance develops after 30–45 min of
It may cause paraesthesias and pruritus at injec- continuous seizure. This is due to the afore-
tion site. Its bioavailability is 100% compared mentioned seizure-induced internalization of
with phenytoin. It is rapidly converted to synaptic GABA-A receptor (subunits b2-3, ɤ2)
phenytoin (PHT) by serum and tissue alkaline and simultaneous externalization of
phosphatases. Its conversion half-life to pheny- AMPA/NMDA receptors to the synapse. As a
toin is 7–15 min. Phenytoin levels should be result, there is decreased response to GABA and
checked 2 h after infusion. It may be difficult to GABA potentiating medications such as benzo-
maintain therapeutic levels in infants. diazepine. In animal models, the response to
Alternative treatments for Stage 2 SE are diazepam is reduced 20x in RSE [6].
shown in Table 8.13. Intravenously formulated Evaluation of RSE should include continuous
ASMs include—in addition to phenobarbital, video-EEG monitoring to (a) diagnose the con-
phenytoin, and benzodiazepine— sodium val- dition and (b) monitor the treatment response.
proate, levetiracetam, and lacosamide. Valproic The treatment goal during RSE is electrographic
acid (VPA) and levetiracetam (LEV) are some- seizure suppression and EEG burst suppression
times used for treatment of Stage 2 SE but their pattern or electrocerebral inactivity. Optimal
efficacy in SE has not been evaluated in con- parameters of burst suppression such as duration
trolled studies [12]. They have the advantage of interburst interval have not been determined.
over PHT and PB of lacking cardiovascular side Some investigators believe that an interburst
effects (lacosamide can rarely cause atrial fibril- interval of  5 s is desirable.
lation or tachy- or bradycardia). Treatment of RSE. Mainstay treatment is
“therapeutic coma” induced with intravenous
anesthetics such as propofol, midazolam (or
Refractory Status Epilepticus lorazepam), or pentobarbital, together with intu-
(RSE, Stage 3 SE) bation and mechanical ventilation. There are no
US RSE treatment guidelines, no randomized
It is defined as SE lasting for >1 h which has studies comparing different agents, and little
failed to respond to benzodiazepine + PHT or evidence to guide the choice of agent or duration
PB at adequate doses. Approximately 35% of all of treatment. Many centers resort to i.v. anes-
SE evolve into refractory SE [13]. Convulsive SE thesia after the failure of benzodiazepine +
may evolve into NCSE in approximately 15% of phenytoin or phenobarbital. Some, however, try
adults and 25% of children: Convulsions stop but a third-standard anticonvulsant such as VPA,
mental status does not improve and CEEG shows LEV, or lacosamide (LCM) before anesthesia. In
NCSE. Super-refractory status epilepticus is Europe, this approach is common. Current
8 Status Epilepticus 123

Table 8.14 Refractory SE treatment with iv anesthetics propofol, midazolam, and pentobarbital
Dose, bolus mg/kg Followed by infusion mg/kg/h SE control rate (%) [14]
Propofol 1–2 5–10 mg/kg/h 68
Midazolam 0.1–0.3 (at 25 mg/min) 0.05–0.4 mg/kg/h 78
Pentobarbital 5–20 0.1–3 64

European guidance recommends titration of It has a rapid onset of action: Seizure control
propofol and barbiturate to EEG burst suppres- occurs in 2–3 min versus 123 min with pento-
sion, and midazolam to seizure suppression, barbital. It often requires high doses (e.g.,  50–
maintained for at least 24 h [15]. 100 mcg/mg/kg/min) to induce burst suppres-
Therapeutic coma lowers metabolic activity of sion, often with associated hypotension requiring
brain tissue, removes the energy mismatch i.v. pressor support. It has common and poten-
between brain tissue energy use and supply, and tially lethal side effects, chiefly hypotension,
allows neuronal recovery, including recovery of metabolic acidosis, pneumonia, and the “propo-
normal neuronal synaptic receptor function. fol infusion syndrome” (PRIS). PRIS occurs at
The three most commonly used i.v. anesthetics high doses with prolonged infusion, e.g.,
are shown in Table 8.14, together with dosing and >4 mg/kg/h for more than 24 h, more so with
infusion rate. Following initial bolus injection, the co-treatment with catecholamines and steroids. It
rate of infusion/dose of the chosen agent should consists of unexplained lactic acidosis, rhab-
be titrated quickly up to electrographic seizure domyolysis with elevated creatinine kinase,
suppression and then EEG burst suppression. hypertriglyceridemia, and widespread ECG
The optimal duration of the treatment has not changes, including cardiac arrest. Prolonged
been determined in controlled studies. Different propofol infusion is associated with other serious
centers use variably 24–48 h of EEG burst sup- systemic complications, most commonly pneu-
pression on the i.v. anesthetic before attempting a monia. In one study of adults with RSE, there
taper. i.v. anesthetic is restarted if seizures recur. was 57% mortality with propofol treatment ver-
During i.v. anesthesia, non-anesthetic ASMs sus 17% with midazolam.
should be optimized in preparation for with- Midazolam rapidly enters brain tissue. It has a
drawal of the i.v. anesthetic. The duration of i.v. powerful short duration. 0.4 mg/kg/h infusion
anesthesia is empirical. Side effects are common, rate is more effective in RSE than 0.2 mg/kg/h
include hypotension, pneumonia, gastric paresis, infusion rate, with lower mortality of 40% versus
and immunosuppression, and contribute inde- 62%. There is a risk of development of acute
pendently to poor outcome and death. Mortality tolerance with risk of seizure relapse. Break-
and functional outcome is similar in those with through seizures may occur in 50% of patients.
and without EEG suppression. Side effects include hepatic and renal impair-
Propofol is the first-line intravenous anes- ment, respiratory, and cardiac depression,
thetic agent for RSE in many centers because its although the latter is less pronounced than with
rapid onset and short duration of action, even barbiturates.
after prolonged infusion, allow a greater control Pentobarbital has longer half-life, making
of the depth of anesthesia than with pentobarbital quick adjustments and evaluation of mental sta-
or midazolam [14]. Its t1/2 is 2 h, but its effect is tus after discontinuation of the infusion more
shorter (minutes) because of its rapid distribution difficult [16]. It is associated with the greatest
into peripheral tissues. 1–2 mg/kg load is fol- incidence of systemic complications, particularly
lowed by infusion at 5–120 mcg/kg/min, with hypotension, splanchnic hypoperfusion (leading
up-titration in increments of 10 mcg/kg/min to gastric, pancreatic, and hepatic sequelae),
every 10–15 min to EEG response/side effects. immunosuppression, with attendant risks of
124 P. Klein

infections most commonly pneumonia, but also TBI, encephalitis, and other infectious, inflam-
nosocomial iv sepsis via catheter, or UTI; and matory and paraneoplastic causes in previously
reduced GI motility. The side effects may limit non-epileptic patients—with RSE duration of
treatment dose and duration. >1 h. Good treatment response occurs in idio-
Inhalational agents: Inhalational halogenated pathic SE in previously non-epileptic patients;
anesthetics such as isoflurane and desflurane SE is associated with ASM non-compliance in
have been used successfully to control seizures in epileptic patients and SE duration of <1 h [13].
small numbers of patients who do not respond to SE prognosis: Overall mortality is 3–6% in
intravenous agents [14]. The logistical and safety children, 14% in young adults, 38% in the
implications of providing inhalational anesthesia elderly. It is 3% with SE duration of 30–60 min
in the ICU are substantial, and such treatment is and 32% with duration of >1 h. It is higher with
not a realistic option in most circumstances. an acute precipitant, in acute symptomatic epi-
Other agents are used in clinical practice in lepsy, after anoxic brain injury, in the elderly,
RSE. They include valproate, topiramate, leve- and in SE duration of >24 h. It is low in the
tiracetam, lacosamide, ketamine, and i.v lo- context of alcohol withdrawal or ASM
razepam infusion. Evidence of efficacy of VPA, non-compliance in an epilepsy patient. Approx-
TPM, LEV, LCM, and ketamine is based on imately 15% of patients have severe and 15% of
uncontrolled studies, retrospective reviews, and patients have mild neurological deficit. 35% of
case series reports. patients recover to baseline [14].
Other ancillary treatments used in continued In children with convulsive SE, mortality is
refractory status epilepticus unresponsive to 3–5% short term and further 3% long term, with
standard treatments have included hypothermia, similar risk factors for poor and favorable out-
ketogenic diet, immunotherapy—including IVIG come as with adults. 25–40% of children with SE
and plasmapheresis—resective surgery, and develop subsequent epilepsy. This is highest with
vagal nerve stimulation. Their use is based on acute symptomatic convulsive SE. 35% children
anecdotal and case series evidence only. with SE > 30 min go on to have neurodevelop-
RSE treatment monitoring includes monitor- mental decline.
ing of electrolytes, calcium, magnesium, blood RSE outcome: RSE has mortality of 39–48%
gases, and pH, and monitoring for and treatment in adults and 16–44% in children. 28% adults
of concurrent infection, fever, rhabdomyolysis, and 32% children return to baseline.
hypotension, and bradycardia, all of which may
worsen RSE outcome.
Refractory NCSE: Because the side effects of References
treatment might outweigh its potential benefits in
NCSE, there remains debate about whether 1. DeLorenzo RJ, Hauser WA, Towne AR, Boggs JG,
NCSE should be treated as aggressively as Pellock JM, Penberthy L, Garnett L, Fortner CA,
GCSE. Administration of anesthetic agents is Ko D. A prospective, population-based epidemio-
logic study of status epilepticus in Richmond, Vir-
often postponed until a trial of a third-line ginia. Neurology. 1996;46(4):1029–35.
non-anesthetic anticonvulsant has been com- 2. Shorvon S, Baulac M, Cross H, Trinka E. Walker M
pleted [15]. (for the Task Force ILAE Commission on European
Affairs. Gray Matters Epilepsia. 2008;49:1277–88.
3. Claassen J, Mayer SA, Kowalski RG, Emerson RG,
Hirsch LJ. Detection of electrographic seizures with
SE Treatment Outcome continuous EEG monitoring in critically ill patients.
Neurology. 2004;62:1743–8.
The two factors that best predict SE treatment 4. DeLorenzo RJ, Towne AR, Pellock JM, Ko D. Status
epilepticus in children, adults, and the elderly.
response versus resistance are etiology and RSE Epilepsia. 1992;33(S4):S15–25.
duration. Poor response is associated with SE 5. Treiman DM, Walton NY, Kendrick C. A progressive
caused by acute structural lesions such as CVA, sequence of electroencephalographic changes during
8 Status Epilepticus 125

generalized convulsive status epilepticus. Epilepsy Calabrese VP, Uthman BM, Ramsay RE, Mam-
Res. 1990;5(1):49–60. dani MB. A comparison of four treatments for
6. Wasterlain CG, Liu H, Naylor DE, Thompson KW, generalized convulsive status epilepticus. Veterans
Suchomelova L, et al. Molecular basis of Affairs Status Epilepticus Cooperative Study
self-sustaining seizures and pharmacoresistance dur- Group. N Engl J Med. 1998;339:792–8.
ing status epilepticus: the receptor trafficking hypoth- 12. Trinka E. What is the evidence to use new
esis revisited. Epilepsia 2009;50(Suppl 12):S 16–8. intravenous AEDs in status epilepticus? Epilepsia.
7. Guerrini R. Physiology of epilepsia partialis continua 2011;52(Suppl. 8):35–8.
and subcortical mechanisms of status epilepticus. 13. Lowenstein DH. The management of refractory
Epilepsia. 2009;50(Suppl 12):7–9. status epilepticus: an update. Epilepsia. 2006;47
8. Lowenstein DH, Alldredge BK. Status Epilepticus. (Suppl 1):35–40.
N Eng J Med. 1998;338:970–6. 14. Shorvon S, Ferlisi M. The outcome of therapies in
9. Alldredge BK, Gelb AM, Isaacs SM, Corry MD, refractory andsuper-refractory convulsive status
Allen F, Ulrich S, Gottwald MD, O’Neil N, epilepticus and recommendations for therapy. Brain.
Neuhaus JM, Segal MR, Lowenstein DH. A compar- 2012;135:2314–28.
ison of lorazepam, diazepam, and 23 Klein P. Status 15. Meierkord H, Boon P, Engelsen B, Göcke K,
Epilepticus placebo for the treatment of out-of- Shorvon S, Tinuper P, Holtkamp M. European
hospital status epilepticus. N Engl J Med. 2001;345 federation of neurological societies. EFNS guideline
(9):631–7. on the management of status epilepticus in adults.
10. Silbergleit R, Durkalski V, Lowenstein D, Conwit R, Eur J Neurol. 2010;17(3):348–55.
Pancioli A, Palesch Y, Barsan W. NETT Investiga- 16. Claassen J, Hirsch LJ, Emerson RG, Bates JE,
tors. Intramuscular versus intravenous therapy for Thompson TB, Mayer SA. Continuous EEG moni-
prehospital status epilepticus. N Engl J Med. toring and midazolam infusion for refractory non-
2012;366:591–600. convulsive status epilepticus. Neurology. 2001;57
11. Treiman DM, Meyers PD, Walton NY, Collins JF, (6):1036–42.
Colling C, Rowan AJ, Handforth A, Faught E,
EEG in Encephalopathy and Coma
9
Mohamad Z. Koubeissi, Nabil J. Azar and Peter W. Kaplan

The EEG is a very important part of the eval- may be detected within the first day of moni-
uation of patients with acutely altered mental toring, but in those with coma, the first seizure
status. Many of these patients are referred from is typically recorded after the first 24 h. Pre-
intensive care settings, and a routine initial step dictors of seizures on continuous EEG moni-
often involves obtaining a 20-min record to toring in the ICU include coma, age <18 years,
screen for seizures, status epilepticus, history of epilepsy, convulsive seizures prior to
encephalopathy, or even hypersomnolence. monitoring [1], CNS infection, brain tumor,
More recently, there has been a move to pro- recent neurosurgery, and periodic epileptiform
longed monitoring with continuous EEG moni- discharges [5].
toring (cEEG) also being made available in the Status epilepticus is discussed in Chap. 8 and
intensive care setting with the possibility of will not be discussed in this chapter. Rather, we
remote reviewing of the record. The exact place will explore the principal EEG findings in
for extended recordings remains under consid- encephalopathic patients and their clinical rele-
eration, but a recent study has revealed that vance. EEGs may be done for diagnosis, moni-
about 20% of comatose ICU patients have toring of the effects of treatment, or for
non-convulsive seizures [1]. There are some prognosis. Regarding the latter, there is an
data to suggest that non-convulsive seizures and increasing body of work that supports the notion
non-convulsive status epilepticus (NCSE) are that prognosis depends heavily on the etiology of
associated with worse outcome in patients with confusion and coma. For example, a particular
subarachnoid hemorrhage, or following head coma pattern due to medication toxicity may
trauma [1–4]. In 88% of ICU patients, seizures prove to be reversible, while conversely, it may
herald a very poor prognosis if due to hypoxic–
ischemic brain injury. Other EEG patterns may
predict outcome, independent of etiology. For
M.Z. Koubeissi (&)
Department of Neurology, George Washington example, EEG reactivity during which there may
University, Foggy Bottom South Pavilion, be a change in EEG amplitude, frequency, or the
22nd and I Street, NW, 7th Floor, appearance of other patterns following noxious
Washington DC 20037, USA
stimulation, noise or eye opening may indicate a
e-mail: koubeissi@gmail.com
more favorable prognosis. Spontaneous vari-
N.J. Azar
ability along the course of a recording typically
Vanderbilt University, Nashville, TN, USA
suggests a better prognosis than when the EEG is
P.W. Kaplan
suppressed, monotonous, and unreactive.
Department of Neurology and Pediatrics, Johns
Hopkins Medicine, Baltimore, MD, USA

© Springer Science+Business Media LLC 2017 127


M.Z. Koubeissi and N.J. Azar (eds.), Epilepsy Board Review,
DOI 10.1007/978-1-4939-6774-2_9
128 M.Z. Koubeissi et al.

Intermittent Rhythmic Delta Activity disturbances and with structural lesions, but did
(IRDA) not correlate with epilepsy [8]. More recently, a
large retrospective study found significant sta-
IRDA refers to diffuse, synchronous 2–3 Hz tistical correlations with strokes and noted its
sinusoidal waves that are often maximal anteri- favorable prognostic significance [9].
orly. It often occurs on a background of delta- or While IRDA is not associated with epilepsy,
theta-range slowing and can be occasionally OIRDA in children can have either an
asymmetric. As the term “intermittent” implies, it encephalopathic or epileptiform significance, as
often has an abrupt onset and offset, and is typ- it has been described in association with focal
ically a reactive pattern that is blocked by eye epilepsy and generalized epilepsy, especially
opening. In children, it may be maximum in the absence seizures [10].
posterior head regions and termed occipital
intermittent rhythmic delta activity, or OIRDA.
IRDA was long believed to be a pattern Triphasic Waves
indicative of deep-midline dysfunction originally
seen in children with midline tumors and raised As the name implies, triphasic waves consist of
intracranial pressure [6], and latter with structural three phases of increasing duration and decreas-
lesions involving both the cortical gray matter ing slopes. The initial phase is negative, brief,
and deep nuclei [7]. With time, it came increas- and steep, and is followed by a second positive
ingly to be seen in the elderly, typically with a trough of longer duration and slightly slower
frontal predominance (FIRDA), associated with slope. The magnitude of this second positive
background slowing. It signifies cerebral dys- phase is the largest, generally higher than 70 µV.
function, commonly diffuse encephalopathy, It is followed by a third phase that is negative and
although it can be seen with focal lesions. It is largest in duration. The duration of the triphasic
seen early in coma, soon after loss of the poste- wave is 150–500 ms. The distribution is gener-
rior basic rhythm, but is non-specific in terms of ally anterior-dominant, although they may be
pathology. As for all encephalopathic patterns, posterior-dominant in some cases. On the longi-
the prognosis of IRDA depends largely on the tudinal bipolar montage, the positive trough may
underlying pathology. show an anterior-to-posterior lag (Fig. 9.1).
Building on the work of others, Accolla and When they are maximal in the occipital region, a
colleagues in a prospective study noted that posterior to anterior lag may be seen (Fig. 9.2).
FIRDA occurred with toxic-metabolic Triphasic waves often occur with a frequency of

Fig. 9.1 Triphasic waves in an encephalopathic patient. Notice the anterior-to-posterior lag in the second (positive)
component of the triphasic wave (boxed)
9 EEG in Encephalopathy and Coma 129

Fig. 9.2 Notice the posterior-to-anterior lag in the second (positive) component of the triphasic wave (boxed)

once or twice per second on a slow background. in PDA becomes slower and loses its reactivity to
They can be bilaterally synchronous or appear stimulation.
independently on either side. PDA is believed to be generated in pyramidal
The cause of triphasic waves has classically neurons in cortical layers II, III, and V. Schaul
been attributed to metabolic encephalopathy such and colleagues found that this pattern was largely
as liver or kidney failure. However, they have also seen in dysfunctions of the subcortical white
been described in the other toxic and metabolic matter or with lesions which partially deaffer-
conditions, including lithium toxicity and ented white matter [12, 13]. Further studies
hyponatremia, or even with subcortical white revealed that it could be seen with metabolic,
matter disease [11]. Interestingly, they may toxic, or infectious encephalopathies [14], and
decrease upon administration of benzodiazepines less commonly with infratentorial lesions
without an improvement in sensorium, making the involving the thalamus and rostral brainstem [15,
distinction from epileptiform discharges difficult. 16]. Occasionally, PDA or more frequently
continuous RDA occurs with deep-seated
epileptic foci that are remote from the scalp
Continuous High-Voltage surface, with limbic encephalitis and with limbic
Polymorphic Delta (PDA) status epilepticus.
Figure 9.3 shows bilateral waxing and waning
PDA is 1–2 Hz, high-amplitude, arrhythmic medium-to-high voltage 1–3 Hz delta activity
slow-wave activity that is generally seen in the with some interspersed lower voltage theta fre-
later stages of coma than IRDA and triphasic quencies. There is little alpha or beta activity.
waves, but may still attenuate with stimulation. The patient had severe head trauma and remains
As the coma deepens, the predominant frequency alive but with little functional recovery.
130 M.Z. Koubeissi et al.

Fig. 9.3 Medium-to-high-voltage 1–3 Hz polymorphic delta activity in a patient with severe head trauma who had
little functional recovery

The EEG should raise the suspicion of CJD.


Generalized Periodic Epileptiform Patients with later stages of subacute sclerosing
Discharges encephalitis (SSPE) can have GPEDs with longer
inter-GPED interval.
GPEDs are generalized, usually high-voltage Figure 9.4 shows GPEDs at 1.2 Hz in a
sharp or spike morphologies, occasionally poly- patient after cardiac arrest. The record showed
morphic, occurring synchronously and bilater- little background and no reactivity to stimuli.
ally. The discharge frequency typically is 0.5 Hz There were no brainstem reflexes and the patient
or slower and occurs in an unreactive coma, died.
sometimes with low-amplitude face or limb
myoclonus. There is often little background
activity between discharges, but theta and delta Burst Suppression Pattern
may occur. As the coma deepens, amplitude of
the inter-GPED activity decreases. Burst suppression pattern refers to synchronous
There is a high association of clinical seizures bursts of high-voltage, mixed frequency activity,
or electrographic seizures before or after the separated by periods of EEG suppression to less
recording of periodic discharges, more so with than 10 µV (Fig. 9.5). The bursts contain spikes
GPEDs. The most frequent cause is cerebral and discharges of almost any other frequency. The
anoxia after cardiorespiratory arrest. A poor duration of the suppression increases with deep-
outcome (mortality or vegetative state) is >97%. ening coma or anesthetic agent dose. Etiologies
Severe metabolic disease and overdoses of that are commonly associated with burst suppres-
lithium and baclofen may also cause GPEDs. sion pattern include anoxic encephalopathy, drug
9 EEG in Encephalopathy and Coma 131

Fig. 9.4 GPEDs at 1.2 Hz in a patient after cardiac arrest

Fig. 9.5 Burst suppression pattern

intoxication, anesthetics, and hypothermia. This setting of anoxic encephalopathy, it is associated


pattern is generally reversible if induced by with poor prognosis. Indeed, this pattern occurs
hypothermia or anesthetics, including barbiturates shortly before progression to electrocerebral
or benzodiazepines. However, if it occurs in the inactivity.
132 M.Z. Koubeissi et al.

Low-Voltage Slow Non-reactive EEG types of artifacts, such as EKG, respiration, and
intravenous drips. The record must be run so as to
Low-voltage records (<20 lV) without variabil- conform to the Guidelines of the American Clin-
ity or reactivity occur with anoxia or less fre- ical Neurophysiology Society (ACNS) (http://
quently with severe metabolic and ischemic www.acns.org/pdf/guidelines/Guideline-3.pdf):
disturbances. Following cardiac arrest, it carries a
zero percent prognosis for return to conscious- – Minimum of 8 scalp electrodes and earlobe
ness, but care must be taken to exclude signifi- references
cant hypothermia and sedative or anesthetic – Electrode Impedance must be between 100
agents. Low-voltage fast patterns conversely may and 10,000 X
be seen in *5% of the normal population and – Interelectrode distance should exceed 10 cm
with alcoholism and sometimes with benzodi- – EEG must be read with sensitivity of
azepine use, but will also possess normal vari- 2 lV/mm and a
ability and reactivity. – s = 0.3–0.4 s
– Integrity of the whole system should be tested
– Monitoring techniques (EKG, Ventilator, etc.)
Electrocerebral Inactivity should be kept in mind as sources of artifact
– Reactivity to pain and loud sound must be
Electrocerebral inactivity (ECI) (Fig. 9.6) repre- checked
sents an EEG pattern where no activity of cortical – Assessment of adequate core body tempera-
origin can be seen. The EEG often shows many ture is required

Fig. 9.6 Electrocerebral inactivity (ECI)


9 EEG in Encephalopathy and Coma 133

– Recording should last for at least 30 min, and spindles occur in concert with generalized slow
done by qualified technologists negative waves constituting K-complexes. The
– Electroencephalographers should read the spindles may be prolonged and exceed 2 s Typ-
EEG at the bed side, and are advised to ically, the EEG tracing of sleep architecture is
repeat the following day if they suspect unreactive to external stimuli or at least returns to
electrocerebral silence. this pattern without a clinical return to
consciousness.
Once these criteria are satisfied, and if the First described by Jasper and Van Buren [17]
presence of anesthetic or suppressant drugs is in a patient with a midbrain tumor near the 3rd
excluded, the finding of ECI in concert with an ventricle, it has since been reported in >250
appropriate clinical examination (demonstrating patients with an aggregate prognosis for death
the lack of brainstem reflexes) indicates “brain of *25%. Spindle coma pattern is seen with
death.” These recordings are usually obtained head injury, anoxic encephalopathy, viral
after cardiorespiratory arrest, severe head trauma, encephalitis, drug intoxication, metabolic
and intractable malignant raised intracranial encephalopathy, and post-ictal states. It also may
pressure. be due to lesions in the pontomesencephalic
junction. Prognosis is often favorable, but gen-
erally depends on associated features, particu-
Spindle Coma larly reactivity. Studies have also shown the
prognosis to depend on the etiology of coma, and
Spindle coma is a pattern of sleep architecture in to be about 73% with structural abnormalities of
which bilateral bursts of fronto-central 9–14 Hz the brainstem, a third after hypoxia, about 15%
“spindles” are recorded (Fig. 9.7). If the coma after head trauma, and negligible when with
deepens, the spindle frequency slows. Often the following seizures or with drugs [18].

Fig. 9.7 Sleep-like bursts of spindles that are non-reactive to external stimuli. The patient recovered
134 M.Z. Koubeissi et al.

Alpha-Theta Coma reactivity is usually maintained, and it typically


evolves into a more favorable pattern. The
Alpha frequency patterns in coma, and “alpha overall mortality for the aggregate 335 cases with
coma” was first described by Loeb and Poggio in alpha coma was 76%, with mortality varying
1953 in a patient with brainstem hemorrhage according to etiology [20, 21]. Brainstem
[19]. Unlike a waking alpha rhythm, it is usually infarction and anoxia after cardiorespiratory
diffusely distributed and often anteriorly domi- arrest was *90%, while other causes, including
nant, and typically invariable and unreactive to drug intoxication, were much less (<10%) [18].
external stimuli (Fig. 9.8). The coma pattern is The alpha rhythm may be maximum posteri-
named after the predominant rhythm, which can orly (similar to the posterior-dominant rhythm) in
be in the alpha or theta range. The rhythm is comatose individuals after brainstem lesions at
generally diffuse, commonly anterior-maximum, the pontomesencephalic level. Like the posterior
and monomorphic. Alpha coma is seen in indi- basic rhythm, the posterior dominant pattern may
viduals with anoxic brain injury, in which case it be reactive to sensory stimulation and photic
is non-reactive to stimuli and signifies poor driving. However, the prognosis is poor.
prognosis. When alpha coma is due to toxic
encephalopathy, it is also anterior-maximum, but
with possible superimposed beta activity. It may Beta Coma
result from overdoses of benzodiazepines, bar-
biturates, anesthetic agents, imipramine, and Despite the ubiquity and abuse of benzodi-
meprobamate. When alpha coma occurs as a azepines, and previously of barbiturates, this
consequence of drug overdose, some degree of pattern remains infrequent, in contrast to the

Fig. 9.8 Low-voltage recording of a patient after car- Following noxious stimulation, there is no evidence of
diorespiratory arrest with alpha frequencies seen diffusely EEG reactivity. The patient died
bilaterally and prevalent in the frontal head regions.
9 EEG in Encephalopathy and Coma 135

Fig. 9.9 Beta coma pattern

frequently encountered excessive fast activity in stupor, coma, worsening rigidity, and progres-
the EEG non-comatose patients receiving ben- sion to death. The EEG shows slowing and dis-
zodiazepines or barbiturates. Beta coma or organization of the background rhythms in the
encephalopathy (Fig. 9.9) produces an EEG with first stage (Fig. 9.10). In the second stage, there
high-frequency spindle-like bursts at *20– are periodic bilaterally synchronous discharges
25 Hz, often diffusely, but typically involving of diphasic or triphasic morphology with volt-
the fronto-central regions. Waking background is ages reaching 300 µV. With more disease pro-
often seen, as is theta activity, and the patient is gression, multiphasic discharges or polyspikes
rarely deeply unresponsive. The beta activity is may appear. Classically, the frequency of these
usually little reactive to external stimuli. Causes periodic discharges is 1 per second, and they
include benzodiazepines and barbiturates. may be associated with myoclonic jerks. These
Excess EEG beta activity can be seen in awake or periodic discharges persist into the third stage
confused patients with alcohol or other with- with gradual increase in the interdischarge
drawal states. interval, gradually evolving into further attenua-
tion of the background activity.

Creutzfeldt–Jakob Disease (CJD)


Subacute Sclerosing Panencephalitis
CJD is a transmissible prion disease that can be (SSPE)
familial, sporadic, or iatrogenic. CJD is the most
common of all prion diseases, and 5–15% of the SSPE is a result of a delayed immunological
cases are clustered in families. Progressive reaction to measles infection, and the EEG has a
dementia is the hallmark of the first stage of the characteristic pattern of extremely high-voltage
disease. In the second stage, rigidity and the periodic discharges with very low interdischarge
myoclonus appear. In the third stage, there is frequency, occurring every 4–14 s. Unlike the
136 M.Z. Koubeissi et al.

Fig. 9.10 EEG in the early stages of Creutzfeld–Jakob disease

diphasic or triphasic discharges of CJD, the 4. Vespa PM, et al. Nonconvulsive seizures after
periodic complexes of SSPE are often slow traumatic brain injury are associated with hippocam-
pal atrophy. Neurology. 2010;75(9):792–8.
waves with or without sharply contoured wave- 5. Hirsch LJ. Continuous EEG monitoring in the
form components. They are generalized often intensive care unit: an overview. J Clin Neurophys-
with frontal predominance. Interestingly, these iol. 2004;21(5):332–40.
periodic discharges may initially appear while 6. Daly D, et al. The electroencephalogram in cases of
tumors of the posterior fossa and third ventricle.
the background is still within normal limits. This Electroencephalogr Clin Neurophysiol. 1953;5
is in contrast to CJD when the background dis- (2):203–16.
organizes invariably prior to the appearance of 7. Gloor P. Generalized cortico-reticular epilepsies.
the periodic discharges. Some considerations on the pathophysiology of
generalized bilaterally synchronous spike and wave
discharge. Epilepsia. 1968;9(3):249–63.
8. Accolla EA, et al. Clinical correlates of frontal
References intermittent rhythmic delta activity (FIRDA). Clin
Neurophysiol. 2011;122(1):27–31.
9. Sutter R, Stevens RD, Kaplan PW. Clinical and
1. Claassen J, et al. Detection of electrographic seizures imaging correlates of EEG patterns in hospitalized
with continuous EEG monitoring in critically ill patients with encephalopathy. J Neurol. 2013;260
patients. Neurology. 2004;62(10):1743–8. (4):1087–98.
2. Vespa PM, et al. Increased incidence and impact of 10. Watemberg N, et al. Clinical correlates of occipital
nonconvulsive and convulsive seizures after trau- intermittent rhythmic delta activity (OIRDA) in
matic brain injury as detected by continuous elec- children. Epilepsia. 2007;48(2):330–4.
troencephalographic monitoring. J Neurosurg. 11. Kaplan PW, Rossetti AO. EEG patterns and imaging
1999;91(5):750–60. correlations in encephalopathy: encephalopathy part
3. Claassen J, et al. Continuous EEG monitoring and II. J Clin Neurophysiol. 2011;28(3):233–51.
midazolam infusion for refractory nonconvulsive 12. Ball GJ, Gloor P, Schaul N. The cortical electromi-
status epilepticus. Neurology. 2001;57(6):1036–42. crophysiology of pathological delta waves in the
9 EEG in Encephalopathy and Coma 137

electroencephalogram of cats. Electroencephalogr electroencephalographic studies. Electroencephalogr


Clin Neurophysiol. 1977;43(3):346–61. Clin Neurophysiol Suppl. 1955;Suppl 4:168–88.
13. Gloor P, Ball G, Schaul N. Brain lesions that produce 18. Kaplan PW, et al. Clinical correlates and prognosis in
delta waves in the EEG. Neurology. 1977;27(4):326– early spindle coma. Clin Neurophysiol. 2000;111
33. (4):584–90.
14. Schaul N, et al. Structural determinants of electroen- 19. Loeb C, Poggio G. Electroencephalograms in a case
cephalographic findings in acute hemispheric lesions. with ponto-mesencephalic haemorrhage. Electroen-
Ann Neurol. 1986;20(6):703–11. cephalogr Clin Neurophysiol. 1953;5(2):295–6.
15. Schaul N, Gloor P, Gotman J. The EEG in deep 20. Austin EJ, Wilkus RJ, Longstreth WT Jr. Etiology
midline lesions. Neurology. 1981;31(2):157–67. and prognosis of alpha coma. Neurology. 1988;38
16. Schaul N, Lueders H, Sachdev K. Generalized, (5):773–7.
bilaterally synchronous bursts of slow waves in the 21. Kaplan PW, et al. Etiology, neurologic correlations,
EEG. Arch Neurol. 1981;38(11):690–2. and prognosis in alpha coma. Clin Neurophysiol.
17. Jasper H, Van Buren J. Interrelationship between 1999;110(2):205–13.
cortex and subcortical structures: clinical
Multiple Choice Questions for Part II

1. A 9-month-old-male infant has a seizure C. 12 h


that lasted for 20 min and started with D. 48 h
left-sided clonic jerking. He had a similar E. Data are not available
episode 6 h later. About 1 h before the first 4. A 6-year-old boy presented with his first
seizure, he had a fever of 102.7 °F, and he seizure, characterized by facial twitching on
was diagnosed in the Emergency Depart- the left-side and left-arm jerking, both of
ment with an otitis media. There is a family which lasting 45 s. The seizure occurred
history of febrile seizures but not epilepsy. 1 h after he went to sleep at night. Which of
Which of the following factors makes it the following is true about the most likely
most likely he will develop epilepsy? diagnosis?
A. This was a complex febrile seizure A. It typically persists until mid-
B. Temperature prior to seizure adulthood.
C. Presence of otitis media B. Most patients have 10–20 seizures
D. Family history over their lifetime.
E. Age C. Most patients with this syndrome have
2. Auras associated with neocortical temporal medically intractable epilepsy.
lobe seizures are characterized by all D. Patients can have associated
except: language-related developmental abnor-
malities.
A. Visual hallucinations
E. The EEG in this syndrome typically is
B. Clonic activity
normal.
C. Auditory hallucinations
D. Vertigo 5. Among ICU patients with altered mental
E. Inappropriate fear status, continuous EEG monitoring shows
non-convulsive status epilepticus to be
3. What duration of continuous EEG moni-
present in approximately:
toring in the ICU is needed to detect most of
the seizures in a comatose patient? A. 55%
B. 20%
A. 20 min
C. 5%
B. 1 h
140 Multiple Choice Questions for Part II

D. 75% 8. Which of the following are among the 3


E. None of the above most common causes of SE in children?
6. The following EEG illustrates which of the A. Change in antiepileptic drug treatment,
following findings? administration, or compliance
B. Congenital abnormalities
A. Sleep spindles C. Tumor
B. Left temporal sharp and slow wave D. CNS infection
C. Occipital lobe seizure E. None of the above
D. Triphasic waves
E. A&B

7. Which of the following is true about the 9. Clinical features of mesial temporal lobe
following EEG? epilepsy include all except:
A. It suggests a seizure focus in the left A. Early age of onset
inferior parietal region B. History of complex febrile seizures
B. It is consistent with the previous brain C. Stuttering course of seizure control
surgery D. Clonic movements
E. Hippocampal atrophy

C. It is indicative of encephalopathy 10. All of the following are true about initial
D. It shows triphasic waves investigation into patient with seizure in ER
E. C & D except:
Multiple Choice Questions for Part II 141

A. CK levels obtained between 6 and 13. In treatment of refractory status epilepticus all
12-h post-event show elevated levels of the following statements are true except:
in convulsive seizures and not in
PNES A. Continuous propofol infusion of
B. Prolactin level should be drawn within >50 mcg/kg/min is associated with a
20 min of event to differentiate risk of hypotension
organic event with the loss of con- B. Hypothermia may be used as adjunc-
sciousness with PNES tive treatment
C. Lactic acid levels drawn soon after an C. Levetiracetam dose used should not be
episode will show elevated levels after less than 2000 mg/day
convulsive seizure D. Mortality rate is higher with i.v.
D. Repeat EEG should be performed in midazolam than with i.v. propofol
patients with alcohol withdrawal sei- E. Lacosamide may be effective adjunc-
zures and EEG showing generalized tive treatment
spike-and-waves
E. All of the above are true 14. Seizures with generalized onset are seen in
which of the following genetic epilepsies?
11. Which of the following is true about
recurrence of febrile seizures? A. Benign childhood epilepsy with cen-
trotemporal spikes
A. Phenytoin is effective in preventing B. Autosomal dominant nocturnal frontal
recurrences. lobe epilepsy
B. The risk of recurrence is increased if C. Temporal lobe epilepsy with auditory
the temperature was high at the time of features
the seizure. D. Dravet syndrome
C. The older the age, the more likely the E. None of the above
febrile seizure is to recur.
D. Complex febrile seizures are more 15. Among the following factors, the one that is
likely to recur than simple febrile associated with the most favorable outcome
seizures. in SE is:
E. Frequency of febrile illness is directly
correlated with recurrence risk. A. Early treatment initiation
B. Non-compliance with anticonvulsant
12. During carotid endarterectomy, intraopera- medications in patients with estab-
tive EEG monitoring can help avoid: lished epilepsy
C. Epilepsy onset in the elderly
A. Seizures D. Early arrival in the hospital
B. Carotid clamping E. None of the above
C. Hypotension
D. Shunting 16. Which of the following is false about idio-
E. EEG is useless pathic generalized epilepsy (IGE)?
142 Multiple Choice Questions for Part II

A. Family history of epilepsy is often C. Ictal EEG can consist of generalized


present polyspike-and-wave activity
B. Motor seizures have a tendency to D. Ictal EEG may consist of low-voltage
occur upon awakening fast activity
C. Photosensitivity seen in *5% of E. All of the above are true
patients
D. EEG background is often abnormal 20. Which of the following statements is not
E. Family history of febrile seizures can true about ictal EEG?
be positive
A. It is marked by monomorphic features
17. Which of the following statements is true B. It is often not seen in association with
about slow spike-and-wave complexes? seizures that do not cause alteration of
awareness
A. They have a frequency that is faster C. When seen on the EEG before clinical
than that of absence epilepsy seizure onset, it tends to be of more
B. Seen in Lennox–Gastaut syndrome localizing value to the epileptogenic zone
C. Are deactivated by sleep D. When onset is characterized by
D. Are associated with normal posterior high-frequency activity, it indicates
basic rhythm proximity of the recording electrodes
E. None of the above to the epileptogenic zone
E. None of the above
18. A 5-year-old girl is seen in the first seizure
clinic after a nocturnal episode of vomiting, 21. Which of the following statements is true
pallor, behavioral irritability, and then eye about EEG in generalized epilepsies?
deviation to the right and unresponsiveness,
followed by a generalized tonic clonic sei- A. In absence epilepsy, the ictal discharge
zure. The total duration of the episode was has an abrupt onset and termination
5 min. Her development has been normal, B. In juvenile myoclonic epilepsy (JME),
and there is no family history of seizures. polyspike-and-slow-wave discharges
Which of the following is true? are seen at a frequency of around
5–6 Hz
A. This epilepsy syndrome has a poor C. Tonic seizures are often associated
prognosis for seizure control. with voltage attenuation
B. Treatment typically includes corticos- D. Tonic–clonic seizures may start with
teroids and vigabatrin. 10 Hz activity that gradually increases
C. Most patients with this disorder have in voltage
major structural brain malformations. E. All of the above are true
D. Trauma is a frequent antecedent of this
epilepsy syndrome. 22. A 6-month-old female is referred by a
E. The EEG likely will show paroxysms gastroenterology colleague for episodes of
of occipital spikes. trunk flexion that occur in clusters four
times per day. This has been occurring for
19. Which of the following is not true about the last 3 days. There have been some
atonic seizures: concerns about her vision, and her head
control is poor. Which of the following is
A. Slow waves correspond to atonia true?
B. Loss of muscle tone lasts <100 ms
Multiple Choice Questions for Part II 143

A. She has gastroesophageal reflux and B. Failure of release of GABA from


should be referred back to her pre-synaptic vesicles
gastroenterologist C. Failure of synaptic glutamate transport
B. She has an exaggerated startle reflex mechanism
and should be referred back to her D. Internalization and destruction of
pediatrician for routine care and synaptic GABA-A receptors
developmental follow-up E. None of the above
C. She has one of the benign epilepsy
syndromes of infancy and, after an 25. On day of life 3, you are consulted by the
EEG, should be followed routinely in neonatal intensive care unit about an infant
3 months in your clinic who has recurrent tonic spasms in clusters.
D. She has a malignant epilepsy syn- The EEG shows a suppression-burst pat-
drome of infancy and should have an tern. Which of the following is true about
EEG, MRI, and further genetic and/or this epilepsy syndrome?
metabolic workup, depending on the
initial findings A. The prognosis for normal psychomo-
E. She likely has seizures originating tor development is poor
from the left temporal region and, after B. Seizures stop after the first few days of
an EEG, should be started on life
levetiracetam C. Seizures are treated easily, regardless
of which medicine is used
23. Which of the following statements is not D. Further workup frequently is not
true about epileptiform discharges? indicated
E. Mortality in this condition is low
A. Anterior temporal spikes have higher
association with seizures than occipital 26. Postictal aphasia localizes the seizure focus
spikes to the:
B. Spikes can be seen in non-epileptic
conditions, such as congenital A. Right temporal region
blindness B. Left temporal region
C. Multifocal spikes are commonly C. Right parietal region
associated with a slow EEG D. Left parietal region
background E. Right frontal region
D. Multifocal spikes tend to be associated
with cognitive but not motor deficits 27. Advantages of Fos-phenytoin in compar-
E. In BECTS (Benign Epilepsy with ison with phenytoin include:
Centrotemporal Spikes), the positive
end of the dipole is in the frontal regions A. Lesser risk of allergic reaction
B. Water solubility
24. Recognized pathophysiological mecha- C. Subcutaneous administration
nisms of pharmacoresistance in refractory D. Lesser protein binding
status epilepticus include: E. None of the above

A. Failure of presynaptic synthesis of 28. Which of the following is not true about the
GABA following EEG?
144 Multiple Choice Questions for Part II

A. Suggestive of multifocal epilepsy A. When of neocortical origin, they are


B. Suggestive of right temporal always associated with a beta range
dysfunction ictal discharge

C. Suggestive of unilateral temporal lobe B. When of mesial temporal origin, they


epilepsy are always associated with a delta
D. Seizures are unlikely to be associated range ictal discharge
with postictal aphasia C. Two-thirds of all such seizures will
E. Comorbidities, including cognitive have a lateralizing ictal discharge
and psychiatric, are common in D. They are more likely to have an ictal
patients with such EEG discharge if they include motor
phenomena
29. Seizures from the dorsolateral frontal con- E. Their clinical semiology does not
vexity are characterized by all except: assist in their localization

A. Forced head turning 31. Established effective treatment of status


B. Unilateral clonic activity epilepticus treatment by EMS personnel on
C. Preservation of consciousness the way to the hospital includes:
D. Olfactory hallucinations
E. Forced eyes turning A. Diazepam 20 mg i.v. bolus
B. Midazolam i.m. 10 mg bolus
30. Which of the following is true about the C. Midazolam i.v. 10 mg bolus
ictal EEG of seizures that do not cause D. Lorazepam 1 mg i.v. bolus
alteration of awareness? E. Clonazepam 2 mg intranasally
Multiple Choice Questions for Part II 145

32. Which of the following is true about the A. Associated with clinical seizures
EEG shown above: B. The most common cause is drug
overdose
A. Pathognomonic of liver failure C. The outcome is very poor in <30%
B. Left hemispheric seizure focus D. Distinct from the typical EEG of CJD
C. Normal sleep EEG E. Distinct from the typical EEG of SSPE
D. Caused by medication toxicity
E. Requires prophylaxis with antiseizure 37. Metabolic consequences of convulsive sta-
medications tus epilepticus may include all of the fol-
lowing except:
33. Parietal lobe seizures are characterized by
all except: A. Hypoglycemia
B. DIC
A. Focal motor clonic activity C. Renal alkalosis
B. Tonic posturing of extremities D. Hypotension
C. Painful or thermal paresthesia E. Hyponatremia
D. Sexual paresthesia
E. Prominent autonomic features 38. A progressive change (increase or decrease)
in which of the following will help identify
34. According to the guidelines of the Ameri- ictal discharges on the EEG?
can Clinical Neurophysiology Society
A. Frequency
(ACNS), which of the following is needed
B. Amplitude
for the recording of electrocerebral
C. Field
inactivity?
D. Morphology
E. All of the above
A. Minimum of 12 scalp electrodes and
earlobe references
39. Celiac disease has been associated with:
B. Interelectrode distance should be less
than 10 cm
A. Thalamic bleeds
C. Reactivity to pain and loud sound
B. Occipital calcifications
must be checked
C. Chiari malformation
D. Electrode Impedance must be between
D. Optic Glioma
10 and 1000 X
E. Any of the above
E. Time constant must be 0.3–0.4 ms
40. Which of the following is true about sei-
35. The incidence of SE in the USA has been
zures in patients undergoing dialysis?
estimated to approximately
A. Previous history of epilepsy does not
A. 0.9% of population
increase risk of seizures in peritoneal
B. 0.01% of population
dialysis
C. Between 18 and 40 per 100,000
B. Hypocalcemia in these patients is a
population
significant cause of seizures
D. 6–7 per 100,000 population
C. Seizures can occur in patients who
E. None of the above
develop dialysis-associated dementia/
chronic dialysis encephalopathy
36. Which of the following is true about gen-
eralized periodic epileptiform discharges
(GPEDs)?
146 Multiple Choice Questions for Part II

D. Change in BUN pre- and post-dialysis C. Vomiting


is directly related to occurrence of D. Tonic eye and head deviation
seizures E. Simple visual hallucinations
E. In peritoneal dialysis, monitoring of
glucose content can prevent seizures in 45. Which of the following is not true about
nonketotic hyperosmolar coma frontal intermittent rhythmic delta activity
F. All of the above (FIRDA)?

41. Which of the following is not true about A. Occurs with toxic disturbances
triphasic waves? B. Occurs in liver failure
C. Correlates with epilepsy
A. May occur in lithium toxicity D. Occurs in stroke
B. May occur in renal failure E. Associated with good prognosis
C. Do not correlate with epilepsy
D. Worsen with administration of 46. Insular lobe seizures are characterized by all
benzodiazepines except:
E. May occur in white matter disease
A. Dysarthria
42. MRI scan of the brain during or after status B. Perioral warmth feeling
epilepticus may show an increase in T2 C. Hypersalivation
signal in D. Laryngeal constriction
E. Complex gestural automatisms
A. The cerebellum contralateral to the
side of ictus 47. The following is not true regarding gener-
B. The ipsilateral hippocampus alized epilepsies.
C. The ipsilateral posterior thalamus
D. The splenium A. Vagal nerve stimulator and discon-
E. All of the above nection of the two hemispheres are
options in certain refractory cases
43. The EEG target pattern during continuous B. Many patients with JME have seizure
EEG monitoring in refractory status relapse if antiepileptics are stopped
epilepticus (RSE) includes: even after years of seizure freedom
C. Almost half of the cases of childhood
A. Burst-suppression pattern with inter- absence epilepsy outgrow the seizures
burst intervals of at least 10 s by adulthood
B. Any burst-suppression pattern D. Prognosis of JAE is similar to CAE
C. Burst-suppression pattern with sup-
pression intervals of at least 3 s 48. In burst-suppression pattern:
D. Burst suppression with burst duration
of no longer than 2 s A. Suppression phases have voltage of
E. None of the above less than 10 µV
B. Bursts contain mixed frequency activity
44. Occipital seizures are characterized by all C. Bursts contain spikes
except: D. Pattern is reversible in hypothermia
E. All of the above
A. Complex visual hallucinations
B. Repeated eye blinking
Multiple Choice Questions for Part II 147

49. The incidence of photosensitivity is highest 54. Myoclonic absences differ from absence
in which of the following: seizures in childhood absence epilepsy by
all of the following except:
A. Juvenile myoclonic epilepsy
B. Juvenile absence epilepsy A. Faster frequency of ictal discharges
C. Childhood absence epilepsy B. Not precipitated by hyperventilation
D. Epilepsy with myoclonic absence C. Occur in older patients
E. Epilepsy with grand mal seizures upon D. Response to treatment is incomplete as
awakening in CAE
E. Patients more often have abnormal
50. Auras associated with mesial temporal lobe cognition
seizures are characterized by all except:
55. Ictal emeticus lateralizes the seizure focus
A. Déjà vu to:
B. Rising epigastric discomfort
C. Pallor A. Right temporal region
D. Vertigo B. Left temporal region
E. Borborygmi C. Right parietal region
D. Right occipital region
51. Ictal EEG with focal right temporal rhyth- E. Right frontal region
mic activity that evolves to 5–7 Hz rhythm
within 30 is consistent with an ictal focus in 56. Frontal lobe seizures are characterized by
the: all except:

A. Right neocortical temporal region A. Stereotypical pattern


B. Right mesial temporal region B. Nocturnal occurence
C. Right parietal region C. Nocturnal enuresis
D. Right occipital region D. Bizarre attacks
E. Right frontal region E. Aggressive sexual automatisms

52. The overall mortality in alpha coma is:

A. 10% Answers
B. 20%
C. 50% 1. (A). Risk factors for developing epilepsy
D. 75% after of febrile seizures include complex
E. 100% febrile seizures (i.e., lasting >15 min,
focal-onset seizures, and more than one
53. Hypermotor seizures with prominent seizure in 24 h), family history of epilepsy,
expression of fear localize the seizure focus abnormal development, and the presence of
to the: a post-ictal Todd’s weakness.
2. (E). Inappropriate fear is commonly seen
A. Supplementary motor region with mesial temporal lobe seizures because
B. Orbitofrontal region of the involvement of amygdala. The lead-
C. Cingulate region ing role of the amygdala, as well as that of
D. Dorsolateral frontal convexity the hippocampus and the parahippocampal
E. Parietal region gyrus, in the mechanisms of inducing a
148 Multiple Choice Questions for Part II

fearful perception is well established by (double-banana) montage in the third sec-


studies of electrical stimulation performed ond. The bilateral occipital waveforms are
intraoperatively or during presurgical eval- positive occipital sharp transients of sleep
uation using intracranial depth electrodes. occurring in runs, rather than a seizure
3. (D). Explanation: Continuous EEG moni- discharge. A sleep spindle is seen in the
toring in the ICU is essential since fourth second of this page.
approximately 20% of comatose patients in 7. (B). The asymmetry in the EEG amplitude
the ICU prove to have non-convulsive sei- between the two sides is consistent with a
zures detected on EEG. Studies have shown skull defect (breech artifact) on the left. The
that most EEG seizures can be detected patient indeed has a history of left
after 48 h of monitoring (88% of seizures in temporo-parietal craniotomy. The EEG is
the first day and another 5% in the second acquired during sleep and is not indicative
day). of encephalopathy.
4. (D). This patient has benign epilepsy with 8. (A). Please refer to Table 8.4 for explanation.
centrotemporal spikes, also known as 9. (D). Clonic activity is not seen mesial
benign rolandic epilepsy. Most patients temporal lobe complex partial seizures.
outgrow the diagnosis by mid-adolescence Lateral temporal lobe complex partial sei-
and have less than 5 seizures in their life- zures often evolve early to unilateral clonic
time (thus, not medically intractable). activity.
The EEG shows bilateral spikes maximal 10. (E). Generalized spike-and-wave discharges
over the central and temporal regions, with can be seen in the context of alcohol with-
an increase in the frequency of epileptiform drawal seizures and generalized seizures in
activity during drowsiness and sleep. On the context of metabolic encephalopathies
careful examination, many patients have such as hyponatremia and hence do not
developmental language disorders. Thus, necessarily support a diagnosis of Epilepsy.
consideration should be given to ordering Hence, repeat EEG is recommended once
neuropsychological testing in these patients. seizures have been controlled.
5. (B). In a study of 570 adults with altered 11. (E). Medication typically is not used to
mental status in the ICU who underwent prevent febrile seizure recurrences because
continuous EEG monitoring, seizures were the benefits do not outweigh the risks. Even
detected in 19% of patients. Seizures were if medicines were used, phenytoin would
exclusively non-convulsive in 92% of these not be effective. Recurrence risk of febrile
patients. Seizures were detected during the seizures is higher if the temperature was
first 24 h of CEEG monitoring in 88% low at the time of the seizure, in younger
patients, during day 2 of monitoring in children, and with increased illness fre-
another 5%, and after 48 h of monitoring in quency. Complex febrile seizures increase
7%. 20% of comatose patients required the risk of epilepsy but not recurrence.
>24 h of monitoring to detect the first 12. (D). During carotid endarterectomy, carotid
electrographic seizure versus 5% of shunting can be prevented if EEG moni-
non-comatose patients. toring remains symmetrical after carotid
6. (E). There are no triphasic waves in this clamping. Carotid shunting is associated
page. The EEG shows a sharp and slow with six fold increase in risk of embolic
wave in the left temporal region with infarcts.
phase-reversal over F7 in this bipolar 13. (C). Please refer to Chap. 8.
Multiple Choice Questions for Part II 149

14. (D). The common seizure types in Dravet occipital spikes that are best brought out by
syndrome that is associated with a mutation darkness but other types of epileptiform
of the alpha 1 subunit of the voltage-gated activity have been noted, as well. Because
sodium channel gene (SCA1) are general- the frequency of seizures is low, many
ized tonic–clonic and myoclonic seizures. patients do not need to be treated with
The remaining genetic epilepsy syndromes antiseizure medicines.
are characterized with focal-onset seizures. 19. (B). In atonic seizures, the slow wave cor-
Rolandic epilepsy presents with facial responds to atonia. The loss of muscle tone
motor seizures and has complex inheritance usually lasts for *400 ms. The ictal EEG
patterns. As the name implies, seizures in can have multiple patterns, including gen-
autosomal dominant nocturnal frontal lobe eralized polyspike-and-wave activity, bursts
epilepsy (associated with mutations in the of polyspike-and-wave followed by gener-
nicotinic acetylcholine receptor) are noc- alized slow activity, and low- or
turnal frontal lobe seizures. Seizures in high-voltage fast activity (this pattern can
temporal lobe epilepsy with auditory fea- be seen with tonic seizures).
tures or autosomal dominant lateral tempo- 20. (A). Ictal EEG is characterized by evolu-
ral lobe epilepsy (associated with the tion, rather than being monomorphic. By
leucine-rich, glioma inactivated-1 or LGI1 evolution, it is meant that the pattern
gene) are marked by auditory changes in terms of its frequency, ampli-
hallucinations. tude, field, or morphology as the seizure
15. (B). In SE, good response to treatment is occurs.
seen in idiopathic SE in previously 21. (E). Please refer to Chap. 5.
non-epileptic patients—SE associated with 22. (D). This patient has infantile spasms, one
medication non-compliance in epileptic of the most malignant (and frequently
patients and SE duration of less than an under-recognized) forms of epilepsy in
hour. infancy. Approximately 75% of patients
16. (D). The background in IGE is typically have an underlying cause of the syndrome,
normal. Myoclonic seizures tend to occur including structural, genetic, metabolic,
upon awakening. In generalized epilepsy post-ischemic, or post-infectious problems.
with febrile seizures plus (GEFS+) patients Developmental outcomes for patients with
experience febrile seizures early in child- these underlying abnormalities are very
hood and other types of seizures later in life. poor.
17. (B). Slow spike-and-wave complexes pre- 23. (D). The clinical significance of spikes of
sent with a typical frequency of 1.0–2.5 Hz, different locations is not the same. Seizures
with wider (less spiky) sharp component occur in 90% of children with anterior
than in the absence epilepsy frequency. temporal spikes, but in only 40% of those
They are a typical of Lennox–Gastaut syn- with rolandic spikes or occipital spikes.
drome. Sleep activates trains of such slow Occipital spikes can be in children with
complexes in the extent that they may congenital blindness. In BECTS, spikes are
appear continuous as in electrical status equally negative over the central and tem-
epilepticus during sleep (ESES). They are poral derivations with the positive end of
associated with slow EEG background. the dipole appearing typically in the frontal
18. (E). The patient has benign occipital epi- regions. Multifocal spikes are often associ-
lepsy or idiopathic childhood occipital epi- ated with background slowing and comor-
lepsy (most likely Panayiotopoulos bidities including cognitive and motor
syndrome). The symptoms and signs deficits.
described in the question are classic for this 24. (D). Pharmacoresistance in SE is due to
diagnosis. The EEG classically shows seizure-induced internalization of synaptic
150 Multiple Choice Questions for Part II

GABA-A receptor (subunits b2-3, ɤ2) and Cognitive and psychiatric comorbidities are
simultaneous externalization of common in temporal lobe epilepsy.
AMPA/NMDA receptors to the synapse. As 29. (D). Olfactory hallucinations/auras are not
a result, there is decreased response to seen with dorsolateral frontal convexity
GABA and GABA potentiating medica- seizures. Mesial temporal structures espe-
tions such as benzodiazepine. cially the amygdala play an important role
25. (A). This patient has early infantile epileptic in the genesis of olfactory auras. Selective
encephalopathy (EIEE), also known as amygdalectomy has a dramatic effect on the
Ohtahara syndrome. Age of onset typically olfactory aura in some patients.
is within the first few months and patients 30. (D). Seizures that do not cause alteration of
can have tonic spasms, partial seizures, and awareness will have EEG changes in only
myoclonus. The EEG shows a suppression- 21% of the cases. This increases to 33% if
burst pattern. A wide variety of structural, the seizures include motor phenomena and
metabolic, and genetic etiologies are asso- drops to 15% if they don’t.
ciated with EIEE. Although seizures may 31. (C). In a study comparing the efficacy of
resolve in up to 50% of survivors, the intramuscular midazolam with intravenous
prognosis for normal psychomotor devel- lorazepam for children and adults in status
opment is very poor. Many patients progress epilepticus treated by paramedics outside
to developing infantile spasms and Lennox– hospitals, seizures were stopped prior to
Gastaut syndrome. arrival in the hospital in 73% subjects
26. (B). Postictal aphasia in complex partial treated with midazolam 10 mg i.m. and
seizures is seen if the ictal focus is in the 63% subjects treated with lorazepam 4 mg
dominant temporal lobe while preserved i.v. with 4.5 and 6.5 min to cessation of
ictal language in complex partial seizures is convulsions, respectively. Seizures recurred
seen if the focus is in nondominant tem- in 11% in both groups. Adverse-event rates
poral lobe. were similar in the two groups.
27. (B). Fos-phenytoin, a phosphate ester pro- 32. (D). The EEG shows triphasic waves,
drug of phenytoin, has replaced phenytoin in which can occur in a variety of clinical
many institutions. Fos-phenytoin is given as settings including lithium toxicity, white
phenytoin equivalent (PE), with the dose of matter disease, hyponatremia, and meta-
20 mg/kg. It can be given in dextrose or bolic encephalopathy, among others. There
normal saline. It is water soluble and can be is no correlation with increased seizure risk.
given i.m. as well as i.v. Its bioavailability is 33. (E). Prominent autonomic features are
100% compared with phenytoin. It is rapidly commonly seen with insular cortex stimu-
converted to PHT by serum and tissue alka- lation and seen with insular seizures.
line phosphatases. Its conversion half-life to Autonomic features are not a feature of
phenytoin is 7–15 min. Phenytoin levels parietal lobe seizures.
should be checked 2 h after infusion. 34. (C). The ACNS guidelines include:
28. (A). The EEG shows a right temporal spike – Minimum of 8 scalp electrodes and
and slow wave and evidence of right tem- earlobe references
poral slowing before and after the spike, – Electrode Impedance must be between
compared with the left temporal tracings. 100 and 10,000 X
The slowing is indicative of regional dys- – Interelectrode distance should exceed
function. There are no other spike popula- 10 cm
tions to suggest multifocal epilepsy. – EEG must be read with sensitivity of
Postictal aphasia localizes to the dominant, 2 lV/mm and a
commonly left, hemispheric seizures. – s = 0.3–0.4 s
Multiple Choice Questions for Part II 151

– Integrity of the whole system should be 40. (F). Unlike hemodialysis, peritoneal dialy-
tested sis does not increase the risk of seizure
– Monitoring techniques (EKG, Ventila- occurrence.
tor, etc.) should be kept in mind as 41. (D). Triphasic waves have been classically
sources of artifact associated with metabolic encephalopathy
– Reactivity to pain and loud sound must such as liver or kidney failure. However, they
be checked are also seen in the other toxic and metabolic
– Assessment of adequate core body conditions, including lithium toxicity and
temperature is required hyponatremia, or even with subcortical white
– Recording should last for at least 30 min matter disease. Interestingly, they may
and done by qualified technologists decrease upon the administration of benzo-
– Electroencephalographers should read diazepines without an improvement in sen-
the EEG at the bedside and are advised sorium, making the distinction from
to repeat the following day if they sus- epileptiform discharges difficult.
pect electrocerebral silence. 42. (E). In SE, MRI may be focally abnormal
35. (C). The incidence of SE in the USA is showing increased FLAIR, T2 signal
between 18 patients per 100,000 popula- hyperintensity and high-intensity signal
tion, according to a retrospective epidemi- DWI (diffusion-weighted imaging), both
ological study in Rochester, MN, and local at seizure focus and remote, com-
41/100,000 population (with 50 SE monly in the ipsilateral posterior thalamus
episodes/year/100,000) in a prospective (pulvinar), contralateral cerebellum, and
epidemiological study in Richmond, VA. bilateral splenium of the corpus callosum.
36. (A). There is a high association clinical 43. (B). The treatment goal during RSE is
seizures or electrographic seizures before or electrographic seizure suppression and EEG
after the recording of GPEDs. The most burst-suppression pattern or electrocerebral
frequent cause is cerebral anoxia after car- inactivity. Optimal parameters of burst
diorespiratory arrest. A poor outcome suppression such as duration of interburst
(mortality or vegetative state) is >97%. interval have not been determined. Some
Severe metabolic disease and overdoses of investigators believe that an interburst
lithium and baclofen may also cause interval of  5 s is desirable.
GPEDs. The EEG should raise the suspi- 44. (A). Occipital lobe seizures are associated
cion of CJD. Patients with later stages of with simple visual hallucinations. Complex
subacute sclerosing encephalitis (SSPE) can visual hallucinations are commonly origi-
have GPEDs with longer inter-GPED nated in visual association cortex at tem-
interval. poro–occipital junction.
37. (C). Complications of SE include hypona- 45. (C). A prospective study noted that FIRDA
tremia, hypoglycemia, acidosis, acute renal occurred with toxic-metabolic disturbances
failure, acute hepatic failure, and DIC, and with structural lesions, but did not
among others. correlate with epilepsy. More recently, a
38. (E). Seizures are marked by evolution in large retrospective study found significant
any of the mentioned criteria. statistical correlations with strokes, and
39. (B). Celiac disease has been associated with noted its favorable prognostic significance.
bilateral occipital calcifications. This can 46. (E). Complex gestural automatisms are not
potentially result in visual symptoms and seen in insular seizures rather they are
occipital seizures. manifested in complex partial seizures from
152 Multiple Choice Questions for Part II

frontal lobe seizures (cingulated, orbito- indicated mesial temporal onset. In this case,
frontal) or temporal pole. it would mean right mesial temporal region.
47. (D). The prognosis of CAE is better than 52. (D). The overall mortality for the aggregate
JAE as patients with JAE have other seizure 335 cases with alpha coma was 76%, with
types and might not outgrow their seizures. mortality varying according to etiology.
Treatment response is incomplete compared Brainstem infarction and anoxia after car-
to CAE where most patients respond to diorespiratory arrest was *90%, while
ethosuximide or valproic acid. other causes, including drug intoxication,
48. (E). Burst-suppression pattern consists of were much less (<10%).
bursts of high-voltage, mixed frequency and 53. (C). Hypermotor seizures with prominent
spike activity, separated by the periods of facial expression of fear are primarily
EEG suppression to less than 10 µV. Eti- associated with a ventral-prefrontal epilep-
ologies include anoxic encephalopathy, drug tic zone (cingulate region), whereas those
intoxication, anesthetics, and hypothermia. presenting with lower agitation and
This pattern is generally reversible if tonic/dystonic posturing are associated with
induced by hypothermia or anesthetics, but a mesial premotor epileptic zone.
is indicative of poor prognosis in the setting 54. (B). In myoclonic absences, the frequency
of anoxic encephalopathy. of the ictal discharge is 3–5 Hz and dis-
49. (A). Incidence of photosensitivity is highest charges are time-locked to myoclonic jerks.
in patients with juvenile myoclonic Similar to seizures in CAE, myoclonic
epilepsy. absences are precipitated by hyperventila-
50. (D). Auras of vertigo and dizziness are tion. They occur in older patients—mean
more frequent in patients with extratempo- age of 7—and two-thirds of the times have
ral complex partial seizures than in patients abnormal cognition, and treatment response
with mesial temporal epilepsy. is incomplete.
51. (B). Risinger et al. tested the reliability and 55. (A). Ictal vomiting results from the activa-
accuracy of scalp ictal EEG with sphenoidal tion of nondominant mesial temporal
electrodes, as a predictor of seizure localiza- structures. Intense nausea and vomiting can
tion in 110 patients with suspected temporal occur with insular stimulation in animals.
lobe epilepsy who subsequently underwent Fiol et al. suggested that insular areas trig-
intracerebral EEG monitoring. Unilateral ger ictal vomiting but require the mesial
5-Hz or greater temporal or sphenoidal temporal cortex for completion.
rhythm was the first discernible ictal activity 56. (C). Nocturnal enuresis is not typically seen
or was evident within 30 s after seizure onset with frontal lobe seizures.
Part III
Specific Epilepsy Syndromes
Electro-Clinical Syndromes by Age
of Onset—Childhood, Adolescence, 10
and Adult

Tesfaye Zelleke

Pathophysiology:
Childhood Absence Epilepsy (CAE)
According to the cortical focus theory, spikes
are generated at a cortical focus and propagate
Childhood absence epilepsy occurs in children
rapidly via cortico-cortical networks to both
4–8 years of age with peaks around 6 years. It is
hemispheres resulting in rapid synchronization.
more frequent in girls. CAE is characterized by
Paroxysmal oscillation in the cortico-thalamic
very frequent absence seizures (pyknolepsy).
loops amplifies and sustains the spike-wave
Hyperventilation induces the absence seizures.
discharges.
Based on clinical and electrographic features,
Genetics:
absence seizures are classified as typical and
Genetic mutations currently account for a
atypical absence seizures.
small proportion of patients with absence epi-
Typical absence seizures are associated with
lepsy. Mutations in GABA receptor (GABRG2,
transient impaired consciousness (behavioral
GABRA1), calcium channels, and non-ion
arrest, staring) with or without eye fluttering and
channel proteins have been identified in CAE.
automatisms. Onset and cessation are abrupt.
Glut-1 transporter deficiency is described in
Ictal EEG shows >2.5 Hz generalized spike and
about 10% of children with early-onset (under
wave lasting >3 s (average <10 s).
the age of 4 years) absence epilepsy.
Atypical absence seizures tend to have less
Animal models (generalized epilepsy/
abrupt onset and cessation, greater change in
absence epilepsy):
tone, longer duration, and variable impairment of
Several animal models are available for gen-
consciousness. Interictally, the EEG shows gen-
eralized epilepsy and absence epilepsy, which
eralized irregular and asymmetric slow spike and
include acute pharmacologic models
waves (1.5–2.5 Hz). The ictal EEG is similar to
[pentylenetetrazole (PTZ), penicillin, THIP,
the interictal discharges but may be associated
GBL], chronic models [genetic absence epilepsy
with diffuse fast activity.
rats from Strasbourg (GAERS), Wistar Albino
Glaxo/Rijswijk (WAG/Rij)] and other models
[AY-9944, MAM-AY].
Treatment options:
These include ethosuximide, lamotrigine, and
valproate [1]. A double-blinded, randomized,
comparative clinical trial compared the three
T. Zelleke (&) medications in CAE and found that ethosuximide
Department of Neurology, Children’s National
provided the best combination of seizure control
Medical Center, 111 Michigan Avenue, NW,
Washington, DC. 20010, USA and fewest attentional side effects, making it the
e-mail: tzelleke@cnmc.org optimal initial monotherapy in CAE.

© Springer Science+Business Media LLC 2017 155


M.Z. Koubeissi and N.J. Azar (eds.), Epilepsy Board Review,
DOI 10.1007/978-1-4939-6774-2_10
156 T. Zelleke

Ethosuximide provided better seizure control A mix of generalized seizures occurs including
compared to lamotrigine and fewer attentional myoclonic, atonic, absence, GTC, and tonic (less
effects compared to valproic acid. Although CAE common). Non-convulsive status epilepticus may
is often perceived as “benign”, many children occur on awakening from sleep or nap. GTC is
with CAE have cognitive deficits and long-term usually the first seizure and is seen in 75–95% of
psychosocial problems. patients. Prognosis for seizure and cognitive
outcome is variable.
EEG shows bursts of 2–3 Hz generalized
Epilepsy with Myoclonic Absences spike wave and polyspike waves, which increase
with sleep. 4–7 Hz rhythmic theta activity over
The majority (70%) of children with epilepsy the central regions and vertex is a specific find-
with myoclonic absences are boys. The average ing. Myoclonic-atonic seizures are electrograph-
age of onset is 7 years. In about one-third of ically associated with a single generalized
patients a specific cause is identified. Family spike/polyspike wave (unlike LGS which shows
history of epilepsy is reported in 20%. Neu- secondary bilateral synchrony) or 3–4 Hz activ-
roimaging abnormalities, mainly some degree of ity lasting 2–6 s.
diffuse atrophy, are seen in 17% of patients. Neuroimaging is normal. Glut-1 deficiency
Patients have variable impairment of con- syndrome is identified in about 5 % of children
sciousness. Myoclonic jerks involving shoulders, with Doose syndrome. SCN1A mutation is
arms, and legs are seen. Tonic contractions, reported as a cause. Treatment options include
particularly elevation of the arms, are often valproate, lamotrigine, ethosuximide, topiramate,
noted. At times, the tonic activity may be levetiracetam, and the ketogenic diet.
asymmetric. Arrest of respiration and urinary
incontinence may occur. Seizures last 10–60 s
and frequent seizures may occur on awakening. Lennox-Gestaut Syndrome (LGS)
Other seizure types such as GTC, absence, and
drop seizures may also occur. Some children may LGS is characterized by the triad of 1. multiple
evolve to Lennox-Gastaut syndrome (LGS), and seizure types: tonic (nocturnal tonic seizures are
cognitive impairment may occur. characteristic), atonic, atypical absence, GTC,
The ictal EEG shows 3 Hz rhythmic, bilateral, and partial seizures; 2. EEG features of slow
synchronous, and symmetric spike-wave dis- background with generalized slow spike-wave
charges. Intermixed polyspikes may be seen. discharges at 1.5–2 Hz, multifocal discharges,
Bilateral myoclonias occur at same frequency as and generalized fast activity at 10–25 Hz in
spike waves, i.e., 3/second. Treatment options sleep; and 3. cognitive dysfunction and intellec-
include valproate and ethosuximide [2]. tual disability.
The peak age of onset is 3–5 years. Infantile
spasms precede LGS in 10–25%. LGS results
Epilepsy with Myoclonic-Atonic from structural/metabolic causes in 70–78%,
Seizures including meningo-encephalitis, cortical dys-
plasia, hypoxia, and traumatic brain injury,
[Also known as Doose syndrome, or Epilepsy among others. In 22–30% of children the cause is
with myoclonic astatic epilepsy] unknown. Family history of epilepsy is reported
Age of onset is 1–5 years, and the child is in 3–30%.
normal at onset. There is strong family history of Several medications are used in the treatment
idiopathic epilepsy in 15–32%. Myoclonic and of LGS, but seizure control is usually inadequate
atonic seizures with falls suggest the diagnosis. [3].
10 Electro-Clinical Syndromes by Age of Onset—Childhood, Adolescence, and Adult 157

Epileptic Encephalopathy Juvenile Myoclonic Epilepsy (JME)


with Continuous Spike and Wave
During Sleep (CSWS) The age of onset for JME is 12-18 years (mean
age 14.6). Seizures occur mostly upon awaken-
Electrical status epilepticus in sleep (ESES) is ing in the morning but can occur after waking
defined commonly as epileptiform activity from a nap. Sleep deprivation, stress, fatigue, and
occupying >85% of NREM sleep. Two syn- alcohol are major seizure provoking factors.
dromes, Landau–Kleffner Syndrome (LKS) and Seizure types in JME include myoclonic sei-
continuous spike and wave during sleep zures (may be repetitive), GTC seizures (occur in
(CSWS), are associated with ESES. 80–95% of patients; typically preceded by clus-
ters of myoclonic jerks; clonic–tonic–clonic sei-
1. Landau–Kleffner Syndrome (LKS): zures; frequency—one or two/year), and absence
LKS usually occurs in children 3–10 years of seizures (in 18–38% of patients).
age and manifests as language regression with Interictal EEG demonstrates high amplitude
verbal auditory agnosia (word deafness with generalized symmetric and synchronous 4–6 Hz
normal hearing test). Seizures occur in polyspike-wave discharges. Photosensitivity is
two-thirds. ADHD features are common. noted in about 30% of patients.
Brain MRI is normal; however, cases with Family history of epilepsy is reported in 40–
structural causes have been reported. Func- 50%. Inheritance is unclear, likely polygenic, but
tional studies, such as SPECT and PET, have both autosomal dominant and recessive inheri-
demonstrated temporal lobe abnormalities. tance have been reported. Gene mutations iden-
2. Continuous spike and wave during sleep tified in some families include GABRA1
(CSWS) (GABA-A receptor gene on chromosome 5q34),
CSWS manifests as global regression in CLCN2 (chloride channel 2 gene on chromo-
cognition and behavior. The majority of some 3q26), and myoclonin1/EFHC1 (EH-hand
patients have seizures. CSWS is sometimes motif protein on chromosome 6p12; found in 9%
associated with identifiable pathology; e.g., of JME).
neuronal migrational disorders, polymicro- Treatment: Valproate, lamotrigine (may wor-
gyria, shunted hydrocephalus, porencephaly, sen myoclonus), topiramate, and levetiracetam
and thalamic lesions. Family history of sei- are used in the treatment of JME.
zure is reported in 10–15%.
Treatment: Various medications, including
valproate, ethosuximide, and benzodi- Epilepsy with GTC Seizures Only
azepines; immune modulatory therapy (ster-
oids, IVIG); and surgery have been used in Seizures may predominate in the morning (the
the treatment of ESES. earlier classification of epilepsy with grand mal
on awakening included). Sleep deprivation is a
trigger. Family history of generalized epilepsy
and photosensitivity may be present.
Juvenile Absence Epilepsy (JAE)

JAE, unlike CAE, is associated with less frequent Progressive Myoclonic Epilepsy
absence seizures (one to a few per day) and is (PME)
more frequently associated with GTC seizure (in
80%). The impairment of consciousness is less PME is a group of disorders presenting with
severe in JAE. The age of onset ranges from 10– severe myoclonic seizures (and tonic–clonic
17 years. seizures) with progressive neurologic
158 T. Zelleke

deterioration (ataxia, dementia). EEG shows seizure with automatisms. The mean age
progressive slowing, generalized and multifocal of onset is 10 years. Asymptomatic
discharges, and photosensitivity typically at family members may have hippocampal
lower flash frequency. sclerosis. The course is benign in the
PME includes Lafora disease, Myoclonus majority.
Epilepsy with Ragged Red Fibers (MERRF). C. FMTLE associated with febrile seizures
Neuronal Ceroid Lipofuscinoses, Sialidosis, and starts in the first to second decade of life
Unverrricht-Lundborg Disease. Dentatorubral- and has a benign course.
pallidoluysian atrophy (DRPLA) may also cause
PME [4]. 2. Familial lateral temporal lobe epi-
lepsy (Autosomal Dominant Partial
Epilepsy with Auditory Features
Autosomal Dominant Nocturnal (ADPEAF)):
Frontal Lobe Epilepsy (ADNFLE) ADPEAF is associated with focal sei-
zures. Elemental or complex auditory
ADNFLE is caused by mutation of genes coding aura is prominent in 64% of patients.
for neuronal nicotinic acetylcholine receptor The epilepsy has a benign course.
subunits (CHRNA4, CHRNB2, CHRNA2). Sei- Leucine-rich glioma-inactivated 1
zure onset is in childhood with mean age of (LGI1) gene mutation is identified in
11.7 years, and seizures persist into adulthood. 50% of families.
The typical seizure manifests with sudden arou-
sal from NREM sleep (stage II) with hyperkinetic
or tonic movements. Seizures may cluster.
Awareness is usually retained. Auras, which may Familial Focal Epilepsy with Variable
be specific with sensory and psychic symptoms Foci
or non-specific, are frequently reported. Neu-
roimaging is normal. Interictal EEG is usually Different focal epilepsies are seen in different
normal. Ictal EEG may show bifrontal family members. Seizures and EEG abnormali-
discharges. ties are consistent in each affected family mem-
ber. Frontal lobe seizures predominate. Features
that help to differentiate from ADNFLE include
Familial Temporal Lobe Epilepsy less frequent seizures, daytime seizures, more
(Autosomal Dominant frequent secondary generalization, and rare
with Incomplete Penetrance) clusters and auras. Seizure foci may also be
temporal or occipital. Inheritance pattern is
1. Familial mesial temporal lobe epilepsy autosomal dominant with 70% penetrance;
(FMTLE): FMTLE may occur with or with- mapped to chromosome 22q12 [5].
out hippocampal sclerosis.

A. Benign FMTLE without hippocampal Reflex Epilepsies


sclerosis or febrile seizures has its onset in
adolescence or adulthood with aura of In reflex epilepsies, a specific stimulus or event
mesial temporal origin (psychic and auto- repeatedly elicits seizure. Trigger stimuli include
nomic features). Prognosis is excellent. visual stimuli (light, patterns), startle, reading,
B. FMTLE associated with hippocampal tactile, music, drawing/praxis, bathing, thinking,
sclerosis presents with complex partial arithmetics, decision making, and gaming.
10 Electro-Clinical Syndromes by Age of Onset—Childhood, Adolescence, and Adult 159

Gelastic Seizures—Hypothalamic without mirth and with no loss of consciousness;


Hamartoma (HH) autonomic signs may be present. Other seizure
types include crying (“dacrystic”) seizure, tonic,
Gelastic seizures are true diencephalic seizures, and atonic seizures. Interictal EEG shows slow
and the secondary epileptogenesis may be related background, and focal/multifocal and general-
to the connectivity of hypothalamus with frontal ized epileptiform activity. Ictal patterns are
lobes, limbic circuity, and thalamus. Seizures are variable, sometimes no epileptiform activity is
almost always drug resistant and associated with seen. Treatment includes laser surgery, gamma
encephalopathy, but symptoms resolve with knife, surgical resection [6].
surgical ablation of hypothalamic hamartoma.
HH is a developmental, non-neoplastic mal-
formation in the area located between References
infundibular stalk and mamillary bodies. It may
be intrahypothalamic (wide-based attachment to 1. Meeran H, et al. Evolving concepts on the pathophys-
hypothalamus) or parahypothalamic (attached iology of absence seizures: the cortical focus theory.
Arch Neurol. 2005;62:371–6.
with the floor of 3rd ventricle by a peduncle). HH 2. Bureau M, Tassinari CA. The syndrome of myoclonic
is usually sporadic but is rarely associated with absences. In: Epileptic syndromes in infancy, child-
autosomal dominant Pallister-Hall syndrome hood and adolescence. 4th ed. 2005; 337.
(GLI3 gene mutation). The hamartoma has neu- 3. Markland ON. Lennox-Gestaut syndrome (Childhood
Epileptic Encephalopathy). J Clin Neuorphysiol
rons and interspersed glial nuclei. GABA 2003;20(6):426–441.
expressing spontaneously firing neurons may 4. Shahwan A, et al. Progressive myoclonic epilepsies: a
drive the synchrony of large output neurons review of genetic and therapeutic aspects. Lancet
resulting in epileptogenicity. The clinical features Neurol. 2005;4:239–48.
5. Klein KM, et al. Familial focal epilepsy with variable
of HH include epilepsy, developmental retarda- foci mapped to chromosome 22q12: expansion of the
tion, behavioral disorders (PDD, ADHD), and phenotypic spectrum. Epilepsia. 2012;53(8):e151–5.
central precocious puberty. 6. Freeman JL. The anatomy and embryology of the
In HH the characteristic seizure type is hypothalamus in relation to hypothalamic hamar-
tomas. Epileptic Disord. 2003;5:177–86.
gelastic, a brief stereotyped mechanical laughter
Seizures and Epilepsy in Early Life
11
Adam L. Hartman

profound neurodevelopmental deficits in sur-


Neonatal Period
vivors. Seizures are typically resistant to treat-
ment but controlled by school age in half of
Benign Familial Neonatal Seizures
children. Many neonates later progress to West
(BFNS) [1]
syndrome (see below).
Also known as “3rd day fits,” this disorder has
mapped to chromosomes 20 and 8 (KCNQ2 and
3, K+ channels). Seizures consist of tonic pos-
Early Myoclonic Encephalopathy:
turing, apnea/cyanosis, autonomic signs, face and
EME [2]
limb clonus, and last 1–3 min. If treatment
The onset of EME is very early, typically in first
required, medications can be continued until 3–6
month of life (some cases are familial). The
mos of age. Patients may develop other types of
seizure semiology typically is myoclonus in the
seizures later in life.
limbs and face. Focal seizures and tonic spasms
are common. In terms of etiology, concurrent
Early Infantile Epileptic metabolic disorders are common (the classic one
is glycine encephalopathy but others have been
Encephalopathy: EIEE (Ohtahara
Syndrome) [2] noted, as well—a B6 trial is reasonable but it
typically is unsuccessful). The EEG (while
awake) shows multifocal spikes on slow back-
The primary semiology consists of frequent tonic
ground ± periodic activity. Unlike EIEE,
spasms in isolation or clusters (other seizure
types can occur, as well). The onset is in first suppression-burst is observed primarily during
sleep. Conventional treatments are typically used
3 months of life. Neonates can have hundreds of
with very limited success. Corticosteroids have
seizures per day. Structural brain lesions are the
only a minimal effect on seizures. Myoclonus
most common etiology but the following genes
usually resolves by weeks to months but focal
also have been associated with EIEE: STXBP1,
seizures persist. Mortality is high in the early
CDKL5, ARX, KCNQ2. There is a high mortality
ages. The prognosis is poor for resolution of
rate in infancy. Prognosis is characterized by
seizures and neurodevelopment.

A.L. Hartman (&)


Neurology & Pediatrics, Johns Hopkins Medicine,
600 N. Wolfe St., Meyer 2-147, Baltimore, MD
21287, USA
e-mail: ahartma2@jhmi.edu

© Springer Science+Business Media LLC 2017 161


M.Z. Koubeissi and N.J. Azar (eds.), Epilepsy Board Review,
DOI 10.1007/978-1-4939-6774-2_11
162 A.L. Hartman

Infancy vitamin B6, noted in different published case


series. Animal models are generated by inducing
Migrating Partial Seizures early injury and genetic manipulations.
of Infancy [3]

In this syndrome, development is initially nor- Myoclonic Epilepsy in Infancy (MEI) [2]
mal, and then seizure starts between 1 week and
7 mos (mean = 3 mos). Initially, there are spo- With an onset of 4 mos–3 yo, this syndrome is
radic focal motor seizures but eventually they eventually outgrown in most patients. Clinically,
become prolonged or occur in clusters and may there are axial or upper extremity myoclonic
secondarily generalize. The interictal EEG ini- jerks with head drops; trunk flexion or extension
tially shows multifocal slowing, which pro- has been noted, and the lower extremities are
gresses and later includes a disruption of sleep only involved rarely. The EEG shows general-
architecture. The ictal EEG shows multifocal ized spike/polyspikes lasting 1–3 s. This syn-
origins of seizures with migration to different drome is associated rarely with antecedent febrile
regions (morphologically, including rhythmical seizures. Reflex myoclonic seizures are a sub-
delta or sharp waves/spikes). There are associ- group (induced by auditory, tactile stimuli); some
ated extrapyramidal signs and tone worsens over patients are photosensitive. The differential
time. There is early intractability but seizure diagnosis includes infantile spasms and benign
control may improve with age in survivors. Early myoclonus. The EEG and normal development
deaths may be associated with respiratory differentiate MEI from infantile spasms (hypsar-
difficulties. rhythmia) and benign myoclonus (normal EEG).
Neurodevelopmental prognosis in survivors Neurodevelopment generally is normal but
generally is poor. The list of genetic mutations patients may develop other seizures later in life.
associated with this syndrome is expanding Treatment typically is with VPA, LEV, or CZP.
rapidly.

Benign Infantile Seizures [4]


Infantile Spasms [2]
The terminology used for this family of disorders
Clinically, this epilepsy syndrome is character- is in development as of this writing but both
ized by flexor or extensor spasms in clusters. The familial and non-familial forms have been noted.
peak onset is 5 mos (typically 4–8 mos). The The onset is between 3 and 20 mos in a devel-
eponym West syndrome is defined by the triad of opmentally normal infant. The non-familial form
spasms, the EEG appearance of hypsarrhythmia, onset can be in the 2nd year of life, with equal
and developmental delay. The etiologies may be sex predominance. In the familial form, onset
symptomatic (i.e., with an identifiable underlying typically is 4–7 mos, with a female predomi-
cause, representing 75–85% of patients) or nance. Seizures are characterized by focal onset
asymptomatic. Underlying conditions may be (head, face, limbs) clonic seizures, may secon-
genetic (ARX, STXBP1), metabolic, congenital darily generalize, and can occur in clusters with
infection, neonatal infection, among many oth- varying lateralization. Medicines usually are
ers. Intellectual disability is seen in 75–90% of prescribed but seizures generally are easy to treat.
patients. The differential diagnosis includes Structural and metabolic workup is negative.
benign myoclonus, benign myoclonic epilepsy, The EEG shows a focal ictal onset (posterior or
and gastroesophageal reflux (EEG easily distin- temporal); interictal EEG typically is normal. In
guishes these from one another). Treatment the familial form, mutations in PRRT2 (same
includes steroids (ACTH, prednisolone), viga- gene as paroxysmal kinesigenic dyskinesia),
batrin, the ketogenic diet, zonisamide, and ASC-1 (amino acid transporter), and SCN2A have
11 Seizures and Epilepsy in Early Life 163

been noted (there likely are others). The prog- seizures are the most commonly noted one. By
nosis is good for both seizures and development. definition, there is no evidence of intracranial
infection or defined cause for the seizure. The
incidence is 3–5% of the US population. The
Myoclonic Encephalopathy median age of presentation is 18 mos and half of
in Nonprogressive Disorders patients present between 12 and 30 mos. In terms
of genetics, 10–20% of siblings also have these
Three forms of this epilepsy syndrome have been seizures.
described [5]: Recurrence of febrile seizure is:

1. Absence + myoclonic seizures. The EEG *33% will have a second FS (range in studies is
shows theta–delta or delta with spikes. Typ- 23–42%);
ically, this is diagnosed in the 1st year of like. *½ of those will have a 3rd FS (range in studies
There usually is a genetic etiology (Angel- is 7–30%);
man, Prader–Willi, Rett, others). Treatment in *50% recur in 1st 6 months; 75–90% recur in
combination with ESM-VPA may work in 1st year.
some patients.
2. Alternating bilateral positive and negative Recurrence risk is influenced by: age (<1 year
myoclonus. The EEG shows diffuse rhythmic doubles the risk), FS in 1st degree relative (up to
slow spike-waves or multifocal spike-waves double the risk), low-grade fever at seizure onset,
or theta–delta. There may be dyskinetic and illness frequency. Risk factors for develop-
movements. The onset usually is  6 yo. ing epilepsy after a FS include a positive family
Seizures are medically intractable seizures, history of epilepsy, abnormal neurodevelopment,
and infants show poor neurodevelopment. occurrence of a complex febrile seizure, a pos-
Structural brain malformations have been tictal Todd’s paralysis, number of febrile seizures
noted in some patients. (more seizure = greater risk), and duration of
3. Mild onset with focal facial (then limbs) sei- febrile seizure (longer seizure = greater risk).
zures. Onset in this form typically is 7 mos–5
yo. The EEG shows generalized spike-waves
or bilateral continuous slow activity and then Genetic Epilepsy and Febrile Seizures
EEG and clinical deterioration (with both Plus (GEFS+) [7]
pyramidal and extrapyramidal signs, as well
as myoclonus). This form may be associated FS+ is defined as a FS after 6 yo or occurrence of
with neonatal anoxia. Seizures are medically other seizure types. Almost any type of seizure
intractable. has been documented for FS+. FS+ is associated
with mutations in SCN1A, SCN1B, or GABRG2
Because the prognosis is somewhat different, but importantly, mutations only are seen in 10–
this syndrome should be distinguished from the 20% of those with GEFS+ (so genetic testing
progressive myoclonus epilepsies. generally is not advised).

Childhood Dravet Syndrome (also called Severe


Myoclonic Epilepsy of Infancy
Febrile Seizures (FS): The Basics [6] or SMEI)7

By definition, febrile seizures have an onset Characterized by a prolonged FS in 1st year of


between 1 mo and 5 years. Any seizure semiol- life, there is a seizure-free period followed by the
ogy can be seen but generalized tonic–clonic appearance of myoclonic seizures at 1–4 years
164 A.L. Hartman

(8% of patients have an onset <3 yo). Develop- of spikes in sleep, fast spikes or polyspikes, or a
ment is normal early and then deteriorates clini- suggestion of burst suppression following spikes.
cally (including pyramidal signs and ataxia). In terms of prognosis, BECTS typically is out-
The EEG shows spike and wave or polyspike and grown by age 16. Nearly 80% of patients have
waves. Approximately 70–80% of patients have <6 seizures so treatment is not recommended for
mutations in SCN1A, so genetic testing generally most patients. There are concerns about language
is indicated for prognosis and to obviate the need development, which should be specifically
for further diagnostic workup. The so-called screened even if school performance is average
SMEI “borderland” (SMEB) lacks certain core (high-functioning students may not need this).
features of SMEI but has been documented in Occasionally, BECTS resolves only to have
various pedigrees but is no longer believed to be other types of seizures develop after 18 years of
a separate entity. Treatment with Na-channel age. Treatment, if needed, has included CBZ,
medications (e.g., carbamazepine, lamotrigine) OXC, LEV, VPA, GPN, and sulthiame.
may worsen seizures in these patients. Some
patients have clear seizure exacerbations when
exposed to extrinsic (or even generating intrinsic) Panayiotopoulos Syndrome
heat—thus, these environmental stimuli should (Early Childhood Onset “Occipital”
be avoided when possible and practical. Epilepsy) [8]

With a peak onset between 3 and 6 years, seizures


Benign Epilepsy with Centrotemporal start with behavioral agitation and then headache,
Spikes BECTS (“Rolandic Epilepsy”) [8] autonomic symptoms, and motor (hemiclonic or
generalized tonic clonic) seizures. Seizures tend
This is the most common focal epilepsy in to be prolonged. Autonomic symptoms (includ-
childhood, with an onset between 2 and 14 years ing status epilepticus) include vomiting, pallor,
(peak 7–10 years). Sensory symptoms may be cyanosis, among other symptoms. The interictal
seen in the tongue, lips, gums, or cheek; drooling EEG classically shows occipital spikes in salvos
also may be noted. Motor symptoms typically are (increased in sleep) but spikes can be seen over
in the tongue, larynx, or pharynx. Seizures usu- any region. This condition is treated only rarely
ally occur during sleep (1st part of the night) but because 85% of patients have  5 seizures over
10–20% of patients have seizures only while their lifetime. Approximately 2/3 of patients have
awake. The interictal EEG shows spikes that are seizures out of sleep.
diphasic with a phase reversal over temporal,
central, or parietal regions (longitudinal bipolar
montage). They typically are bilateral but one Gastaut Syndrome (Late Childhood
side may have more spikes than the other. Salvos Onset “Occipital” Epilepsy) [8]
of central and temporal spikes are seen, with an
increase in persistence during drowsiness and With a peak onset of 8–11 years, seizures start
sleep. Slowing after spikes suggests seizures will with elementary visual auras and may progress to
be more challenging to control. When seen by partial vision loss or classical focal-onset sei-
those unfamiliar with this syndrome, spikes may zures (only rarely will they generalize). In con-
be mistaken for multifocal epilepsy. The differ- trast to Panayiotopoulos syndrome, autonomic
ential diagnosis includes malformations of cor- symptoms are not prominent but headache is
tical development, vascular anomalies, and other seen commonly. Seizures are fairly frequent but
lesions. These conditions should be considered if of short duration. Also in contrast to Panayioto-
there are unilateral spikes, very frequent seizures, poulos syndrome, a daytime occurrence of sei-
or lack of improvement. EEG findings suggest- zures is common. The interictal EEG classically
ing other diagnoses include decreased frequency shows occipital spikes in salvos but spikes can be
11 Seizures and Epilepsy in Early Life 165

seen anywhere. There is an increased frequency Treatment initially is with antiseizure medici-
of epileptiform activity during sleep and with eye nes, although seizures become refractory to
closure. Because seizures are fairly frequent, this treatment. Immunomodulators such as corticos-
epilepsy syndrome usually requires treatment but teroids, intravenous immunoglobulin, and certain
seizures tend to remit 2–7 years after onset. chemotherapeutic agents (e.g., cyclophos-
phamide) have been used to mitigate the impact
Rasmussen Syndrome of the progressive nature of this disease, with only
limited success. Unfortunately, most patients
With an onset typically between 3 and 14 years, eventually need a hemispherectomy for seizure
patients are almost always neurodevelopmentally control (the unihemispheric nature of the disease,
normal prior to the onset of seizures. On occa- which remains unexplained, makes this surgery
sion, there will be an antecedent nonspecific an option). Prognosis is largely determined by
febrile illness days to weeks before the first sei- preoperative level of function. Postoperative
zure. Rarely, this syndrome will develop in rehabilitation is necessary to optimize outcomes.
patients with other autoimmune diseases. There
likely is an immune-mediated etiology but a
specific organism or trigger has not been identi- References
fied to date, despite exhaustive research (the
association with mGluR3 antibodies has since 1. Allen NM, Mannion M, Conroy J, et al. The variable
phenotypes of KCNQ-related epilepsy. Epilepsia.
been disproven because it is neither sensitive nor 2014. doi:10.1111/epi.12715.
specific, although depending on his/her perspec- 2. Wilmshurst JM, Gaillard WD, Vinayan KP, et al.
tive, examiners may ask this question on a test). Summary of recommendations for the management of
Diagnostic criteria include the following [9]: infantile seizures: task force report for the ILAE
commission of pediatrics. Epilepsia. 2015. doi:10.
Need either all three criteria in Part A or two 1111/epi.13057.
of three in Part B (start with Part A). 3. Coppola G. Malignant migrating partial seizures in
Part A: infancy. In: Handbook of clinical neurology. Amster-
dam: Elsevier; 2013. p. 605–609. doi:10.1016/B978-
1. Clinical: Focal seizures (±epilepsia partialis 0-444-52891-9.00062-2.
continua) and unilateral cortical deficit(s). 4. Vigevano F. Benign familial infantile seizures. Brain
2. EEG: Unihemispheric slowing ± epilepti- Dev. 2005;27(3):172–7. doi:10.1016/j.braindev.2003.
12.012.
form activity and unilateral seizure onset.
5. Elia M. Myoclonic status in nonprogressive encepha-
3. MRI: Unihemispheric focal cortical atrophy lopathies: an update. Epilepsia. 2009;50(Suppl 5):41–
and at least one of the following: 4. doi:10.1111/j.1528-1167.2009.02119.x.
– Gray or white matter T2/FLAIR hyperin- 6. Patel N, Ram D, Swiderska N, Mewasingh LD, New-
ton RW, Offringa M. Febrile seizures. BMJ. 2015;351:
tense signal
h4240.
– Hyperintense signal/atrophy of ipsilateral 7. Catterall WA. Sodium channel mutations and epi-
caudate head lepsy. In: Noebels JL, Avoli M, Rogawski MA,
Olsen RW, Delgado-Escueta AV, editors. Jasper’s
basic mechanisms of the epilepsies, 4th ed. Bethesda
Part B:
(MD): National Center for Biotechnology Information
4. Clinical: Epilepsia partialis continua or pro- (US); 2012.
gressive unilateral cortical deficit(s). 8. Sánchez Fernández I, Loddenkemper T. Pediatric focal
epilepsy syndromes. J. Clin. Neurophysiol. 2012;29
5. MRI: Progressive unihemispheric focal cor-
(5):425–40. doi:10.1097/WNP.0b013e31826bd943.
tical atrophy. 9. Bien CG, Granata T, Antozzi C, et al. Pathogenesis,
6. Histopathology: T-cell-dominated encephali- diagnosis and treatment of Rasmussen encephalitis: a
tis w/activated microglial cells (typically, but European consensus statement. Brain. 2005;128(Pt
3):454–71. doi:10.1093/brain/awh415.
not necessarily forming nodules) and reactive
astrogliosis.
Imitators of Epilepsy
12
Dewi Frances T. Depositario-Cabacar and Nabil J. Azar

List of the common nonepileptic paroxysmal • Behavioral disorders (rage attacks, inat-
disorders by age: tentiveness), and
• Syncope.
A. Infants and neonates: C. Adolescences and adults:
• Sandifer syndrome, • Nonepileptic psychogenic seizures (also
• Self-gratification syndrome, known as pseudoseizures),
• Benign myoclonus of early infancy, • Syncope,
• Shuddering attacks, • Panic attacks and hyperventilation,
• Startle disease or hyperekplexia, • Migraines, and
• Benign neonatal myoclonus, • Parasomnias and sleep disorders (such
• Jitteriness, head banging and body as narcolepsy and cataplexy).
rocking,
• Spasmodic torticollis, and Sandifer syndrome: This syndrome consists of
• Apnea. intermittent abnormal posturing such as stiffen-
B. Older children: ing and opisthotonic posturing usually primarily
• Breath-holding spells, due to gastroesophageal reflux. It is usually
• Movement disorders (tics, paroxysmal associated with feedings. It can be mistaken as
kinesigenic choreoathetosis, etc.), paroxysmal dystonia, epileptic spasms, or tonic
• Parasomnias and sleep disorders, seizures. This usually improves with antireflux
• Migraine headaches, medications.
• Psychogenic nonepileptic seizures, Self-gratification syndrome: This is also
referred to as infantile masturbation. This occurs
in infants and young children. It involves rubbing
of the thighs against each other, thrusting of the
pelvis associated with sweating, grunting, or
flushing of the face with variable degree of
D.F.T. Depositario-Cabacar responsiveness. After the event, the child is back
Department of Neurology and Pediatrics,
Children’s National Medical Center, to baseline. Treatment involves reassuring the
George Washington University, family of its self-limiting and benign nature.
111 Michigan Avenue, Washington Benign myoclonus of infancy: This usually
DC 20010, USA occurs in the first year of life, mostly seen
e-mail: Dcabacar@childrensnational.org
between the age of 3–8 months. It consists of
N.J. Azar (&) brief tonic or myoclonic contractions involving
Department of Neurology, Vanderbilt University
Medical Center, Nashville, TN 37212, USA the axial muscles. These spasms usually occur in
e-mail: dr.nabil.azar@gmail.com cluster and are different from benign neonatal

© Springer Science+Business Media LLC 2017 167


M.Z. Koubeissi and N.J. Azar (eds.), Epilepsy Board Review,
DOI 10.1007/978-1-4939-6774-2_12
168 D.F.T. Depositario-Cabacar and N.J. Azar

sleep myoclonus. Neurological examination, Bradycardia or asystole may occur. Main treat-
EEG, and development are all normal. Treatment ment is also parental reassurance, with some
involves reassurance since myoclonus usually studies suggesting a role for atropine.
resolves by 2 years of age. Migraines: Migraines are common causes of
Shuddering attacks: These usually occur recurrent headaches. The dilemma occurs if the
between the ages of 4–6 months. The events neurological event occurs without significant
consist of tremor in the head, arms, and trunk headaches and the event can be mistaken for
with adduction and flexion of the elbows. Com- seizures, such as in the case of confusional
munication and responsiveness are usually migraine or migraine with aura.
impaired. They can be triggered by emotions For example, confusional migraines are epi-
such as fear, anger, or frustration. They tend to sodes of confusion, agitation/hyperactivity, par-
gradually subside with age and completely tial or total amnesia, disorientation, lethargy, or
resolve by 10 years of age. Reassurance of the vomiting that may last for several minutes or
family is usually enough. hours. Headache and visual complaints may
Hyperekplexia: This is also referred as stiff occur but are not prominent symptoms. These
baby syndrome or startle disease. It appears more may be mistaken for temporal lobe seizures but
like an exaggerated startle response and may other etiologies such as encephalitis, substance
result in falling. It usually occurs at the age of abuse, metabolic causes, and vasculitis should be
6 months to 6 years and peaks at the age of 2– considered.
3 years old. It consists of a triad of generalized On the other hand, migraines with aura can
stiffness, nocturnal myoclonus, and tonic spasms appear as focal seizures if the visual disturbances
with auditory or tactile stimuli. In affected neo- are more prominent than the headache, such as
nates, tapping the tip of the nose or glabella may “Alice in wonderland” syndrome where visual
elicit a typical response. It is secondary to gene micropsia, macropsia, and distortions can all
mutations affecting inhibitory glycine receptor occur. This can be easily mistaken for temporal
(GLRA1 and GLRB). Commonly used treat- or occipital lobe seizures.
ments are clonazepam, followed by valproic acid Parasomnias and sleep disorders: Some sleep
or levetiracetam. disorders can be mistaken for complex partial
Breath-holding spells: Breath-holding spells seizures, commonly of frontal lobe origin due to
commonly occur between the ages of 6 and their hypermotor features and nocturnal occur-
18 months. Spells are divided as cyanotic or rence. These mostly include night terrors (oc-
pallid. curring in non-REM sleep) and REM behavior
Cyanotic breath-holding spells are usually disorder (occurring in REM sleep).
precipitated by an emotional cause such as frus- In the case of night terrors, the child wakes up
tration. The child cries and then holds the breath from sleep, appears terrified and agitated, is
in expiration resulting in cyanosis and may be inconsolable, and has no recollection of the
followed by loss of consciousness and loss of event. They occur around the age of 4 years and
tone. The precipitating event can be from the typically resolve by 8 years.
child being upset. If the apnea is prolonged, While in REM behavior, excessive and vio-
clonic jerks may be observed. Treatment is usu- lent motor activity with no recollection of events
ally reassurance and behavioral modification of is frequent. Polysomnography (with EEG normal
parental response. If recurrent, then screening for recording) is the gold standard diagnostic tool.
iron deficiency anemia can be done. On the other hand, narcolepsy is characterized
Pallid breath-holding spells usually occur in by a tetrad of cataplexy, sleep paralysis, hypna-
response to a minor trauma or fright. The child gogic hallucinations, and excessive daytime
stops breathing, becomes pale, may have a brief sleepiness. Cataplexy is a sudden loss of muscle
cry followed by loss of consciousness. Clonic tone precipitated by touch, emotional excitement,
jerks and incontinence may also occur. or laughter. It may occur several times during the
12 Imitators of Epilepsy 169

day and may often be mistaken for atonic sei- Psychogenic nonepileptic seizures (PNES):
zures. Two tests that are commonly used in PNES are by far the most common imitators of
diagnosing narcolepsy are the polysomnogram epilepsy. It has been reported that 25–40% of all
and multiple sleep latency test (MSLT). admissions to inpatient video-EEG studies end
Syncope: Syncope is very common and up to up with the diagnosis of PNES. However, despite
10% of the population will experience it at least the ability to diagnose PNES with high confi-
one time during their life. It is caused by a dence using video-EEG monitoring, the delay in
transient interruption of cerebral blood flow to diagnosis is long, averaging about 7–10 years.
the brain resulting in loss of consciousness. The patient’s history may suggest the diag-
Neurocardiogenic syncope (or vasovagal nosis. Several clues are useful in clinical practice
syncope) is the most common type of syncope. and should raise the suspicion that seizures may
The event is often preceded by prodromal be psychogenic rather than epileptic. One of the
symptoms of feeling of warmth, nausea, blurring first clues is resistance or worsening of seizures
or tunnel vision, diaphoresis, and lightheaded- to antiepileptic drug trials. The presence of
ness. Events are usually provoked by emotions prominent comorbid psychiatric conditions
(pain, anxiety, blood drawing) particular situa- should also raise the suspicion of PNES, partic-
tions (crowded environment, hot weather, pro- ularly the presence of depression, anxiety, or
longed standing, fatigue). It may be caused by PTSD (mainly secondary to physical or sexual
decreased blood volume or venous return, or abuse). Psychiatric comorbidities are found in the
parasympathetic cardioinhibitory response caus- majority of patients with PNES, with depression
ing vasodepression. Common reflex causes of being the most common.
syncope are coughing, micturition, and swal- In addition, several specific triggers that are
lowing. Tilt table tests are occasionally done in atypical for epilepsy may suggest the diagnosis
recurrent syncopal cases. Diagnosis is made by of PNES, such as emotional triggers (whether
history, orthostatic measurements, or tilt table positive or negative stress), headache or pain,
testing. Treatment includes reassurance, avoiding specific sounds or lights. Also, certain situations
precipitating factors, increasing fluid and salt may trigger PNES, for example, the presence of
intake. If needed, beta-blockers, alpha-adrenergic specific audience and occurrence in the physi-
agonists, anticholinergics, and mineralocorti- cian’s office or waiting room.
coids (such as fludrocortisone) can be used. A detailed description of events by witnesses
In contrast, cardiogenic syncope is less com- is always helpful since specific clinical features
mon but may be more life threatening. In such have been shown to favor the diagnosis of PNES.
case, syncopal episodes occur without warning Some of these clinical features include eye clo-
or prodromal symptoms and may occur during sure, side-to-side head or body shaking, bilateral
physical exertion. Common causes include asynchronous and often discontinuous move-
structural cardiac diseases (such as hypertrophic ments, crying, speech stuttering, and arching of
cardiomyopathy, aortic stenosis), or dysrhyth- the back. With convulsive attacks, rapid or short
mias. An initial cardiology workup including postictal recovery, shallow irregular, and soft
cardiac echocardiography and Holter monitoring postictal breathing pattern also favor PNES.
is warranted. Despite the fact that urinary incontinence, tongue
Convulsive syncope refers to syncope that is biting, bodily injuries can be commonly seen in
followed by brief tonic or rarely clonic activity, both epileptic and nonepileptic seizures, serious
which may occur in prolonged cerebral hypop- injuries and tongue biting at the side are more
erfusional states. typical for epileptic seizures.
During syncopal episodes, the EEG typically PNES diagnosis requires video-EEG recording
show transient high-voltage delta activity or of typical events. Withdrawal of antiepileptic
complete voltage attenuation. drugs is also essential for excluding a concomitant
170 D.F.T. Depositario-Cabacar and N.J. Azar

diagnosis of epilepsy, which can be seen in up to Successful treatment and seizure freedom may
10% of patients suffering from PNES. A formal be a challenging task, and in several instances
psychiatric evaluation with additional neuropsy- hard to reach. Some of the common treatment
chological testing is also an integral part of the obstacles include effective delivery of the diag-
initial assessment. nosis, acceptance of the diagnosis and patient
Treatment of PNES is targeted toward specific commitment to therapy and follow-up. In gen-
psychiatric conditions and is often very individ- eral, patients who accept their diagnosis and
ualized. Therapy typically includes psychother- follow through with therapy are more likely to
apy and the use of adjunctive medications to treat experience a successful outcome.
coexisting depression or anxiety.
Genetic Analysis of Epilepsies
13
Sara K. Inati

synaptic signaling. Contiguous gene syndromes


Background
are microdeletions leading to loss of several
neighboring genes. Chromosomal abnormalities
An ever larger number of gene mutations have
involve larger deletions, duplications, and translo-
been discovered to have a major effect on sus-
cations, usually causing polygenic dysfunction and
ceptibility to seizures and epilepsy. Epilepsy
producing severe symptoms such as mental retar-
associated with a single gene mutation (Men-
dation and/or growth failure. These are most fre-
delian or monogenic inheritance) is thought to
quently found in epilepsy patients with coexisting
occur in approximately 1–2% of patients with
multiple congenital abnormalities and/or intellec-
epilepsy. Up to 40% of epilepsies are thought to
tual disability. Most involve de novo parental
involve polygenic or complex genetic inheri-
germ-cell mutations, but familial rearrangements
tance, involving multiple possibly interacting
such as balanced translocations can occur. Once
genes and/or environmental influences [1, 2].
they reach a certain number, trinucleotide repeat
Genetic generalized epilepsies (formerly idio-
expansions may also lead to diseases associated
pathic generalized epilepsies), where generalized
with epilepsy [4].
seizures are the predominant feature, show mostly
Multiple methods are available to test for dif-
complex inheritance. However, specific muta-
ferent types of genetic changes. Standard kary-
tions have been associated with a variety of other
otyping and high-resolution chromosome analysis
epilepsy syndromes, most notably early-onset
can be used to identify chromosomal abnormalities
epileptic encephalopathies, syndromes associated
such as Trisomy 21 or ring chromosome 20.
with febrile seizures, several familial focal epi-
Array-comparative genomic hybridization (aCGH)
lepsy syndromes, as well as symptomatic epi-
can detect submicroscopic chromosomal rear-
lepsy syndromes, in which seizures are only part
rangements (deletions or duplications, also called
of a more widespread CNS disorder [3].
copy number variations or CNVs) and can inves-
There are several types of genetic changes that
tigate multiple loci simultaneously. Single nucleo-
may contribute to the development of seizures and
tide polymorphism (SNP) arrays can assess known
epilepsy. Small deletions, insertions, and point
SNPs throughout the genome. Testing for patho-
mutations can lead to problems in neuronal meta-
genic CNVs using aCGH and SNP arrays is also
bolism, network development, and membrane and
known as molecular karyotyping. These tests
identify CNVs that are felt to be causative in
approximately 15–20% of patients with intellectual
S.K. Inati (&) disability [5], and 8% of patients with early-onset
EEG Section, Office of the Clinical Director,
epileptic encephalopathies [6], and are becoming
NINDS, NIH Clinical Center, Building 10,
Rm 7-5680, Bethesda, MD 20892-1445, USA widely used in these settings. Other techniques,
e-mail: sara.inati@nih.gov such as fluorescent in situ hybridization (FISH) and

© Springer Science+Business Media LLC 2017 171


M.Z. Koubeissi and N.J. Azar (eds.), Epilepsy Board Review,
DOI 10.1007/978-1-4939-6774-2_13
172 S.K. Inati

multiplex ligation probe amplification (MLPA), are genetic change that is identified. When a muta-
also widely used [1]. tion in a single gene produces different epilepsy
phenotypes in different individuals, this is refer-
red to as variable expressivity and reduces the
Utility of Genetic Testing in Epilepsy positive predictive value of the test. For example,
missense mutations in SCN1A can be associated
When ordering genetic testing, it is important to be with phenotypes ranging from no seizures or
aware of factors that may affect the clinical validity simple febrile seizures (GEFS+) to severe
and utility of a given test in a specific clinical sit- epileptic encephalopathy (Dravet syndrome) [7].
uation. Clinical validity describes the ability of a Genetic (or locus) heterogeneity occurs when
test to determine whether a person is or will a single clinical phenotype can result from
become affected with a given disorder. Clinical mutations in different genes (e.g., in GEFS+ due
utility of a test refers to the risks and benefits of a to different sodium and/or GABA receptor
positive or negative test on patient care. mutations), or when different genetic mecha-
Clinical validity is determined by many fac- nisms can produce the same disease (e.g., IGE
tors. Genetic testing may be performed in several can show autosomal dominant or complex
types of laboratories. Clinical laboratories in the inheritance). Additionally, families with a given
USA that are certified under the Clinical Labora- syndrome may not have mutations in any previ-
tory Improvement Act (CLIA) are required to ously identified genes. Patients without affected
meet federal quality standards. Tests can also be relatives are also less likely to have a mutation in
carried out in research laboratories, which do not previously identified genes [2]. In such cases, a
require CLIA certification and perform analyses positive result may be informative, but a negative
for research only. There is also direct-to-consumer result is not helpful clinically.
genetic testing, usually ordered over the Internet Reduced penetrance describes when individ-
for a fee. This has the advantage of easy access uals with a mutation remain unaffected. Pene-
and privacy, but is not subject to the same quality trance for AD epilepsy is usually approximately
control measures as clinical laboratories, and 70% [2]. This lowers the positive predictive
generally does not include genetic counseling or value of testing and can present significant dif-
adequate epidemiological data about baseline ficulties in predictive testing in asymptomatic
genetic variation to assist with interpretation of individuals. Also, gene–environment interactions
the results [2]. may play a role in the expression of a trait. Each
The appropriate type of test must also be of these factors may lead to decreased clinical
ordered, as a given test will miss changes that it validity of a genetic test.
is not designed to detect (a mutation in a given Clinical utility depends on the clinical validity
gene may not affect a test for a particular SNP or of a test, but also on specific features of a given
CNV but still contribute to a disease). For each clinical situation. The relative risks and benefits
test, a negative result is most definitive if a of testing depend on the availability of an
positive result was obtained from an affected effective treatment, cost and accessibility of
family member. Additionally, the source of DNA testing, severity of the disease, age of onset
affects the validity of a test. A germ-line mutation (particularly as it may affect reproductive choi-
may be absent in a parent’s somatic cells (leading ces), family history and the implications of test-
to negative genetic testing), but can still be pas- ing on other family members, and potential
sed on to offspring. Somatic mosaicism (more ethical, legal, and social implications of genetic
than one genotype in the body) may lead to testing. The Federal Genetic Information
negative testing from one tissue and positive Non-Discrimination Act (GINA) passed in May
testing from another [2]. 2008 prohibits discriminatory use of genetic
Clinical validity is also impacted by the sen- information by employers and health insurers,
sitivity and specificity of the test and the type of but does not extend to life insurance, disability
13 Genetic Analysis of Epilepsies 173

insurance, or long-term case insurance [8]. All of in vitro fertilization). Of note, for women carry-
these issues should be addressed in pretest and ing a mitochondrial mutation, the risk of recur-
posttest genetic counseling, and informed con- rence cannot be accurately predicted and prenatal
sent should be obtained before ordering a genetic testing is not accurate. The only way to guarantee
test [2]. avoidance of recurrence is the use of a donor egg
[1].
In order to perform appropriate genetic test-
Clinical Application of Genetic ing, a full history must be obtained, including a
Testing in Epilepsy three-generation pedigree focused on seizures
and other seizure mimics, neurodevelopmental
Genetic testing in patients with epilepsy can be and psychiatric conditions, ancestral origins,
diagnostic or predictive. Diagnostic testing is outcomes of all pregnancies, and consanguinity.
done in a patient with epilepsy to clarify the Full neurologic examination, developmental
diagnosis and/or prognosis, to save a patient assessment, and appropriate further testing such
from further evaluation/testing, and rarely to as EEG, MRI, and/or metabolic workup can also
affect clinical management. It can also provide contribute to appropriate genetic counseling and
families with information about the risks of test selection [1].
recurrence and can help with reproductive For the epilepsies, the total number of genes
decision-making. Ideally, this testing may also in a differential diagnosis is often large and
lead to targeted therapy. available tests are rapidly changing, so it is
Predictive testing is performed to predict the important to know genetic diseases with a high
onset of epilepsy in asymptomatic patients risk for epilepsy and to become familiar with
(usually offspring or siblings of patients with online references such as GeneTests and OMIM
epilepsy). This also includes prenatal diagnostic [2]. Gene panels are also becoming more widely
testing. The risk of epilepsy in relatives of available. Tables 13.1 and 13.2 are a selected list
patients with epilepsy is increased if there is an of genes that have been identified in idiopathic
earlier age of onset (<35 years old), idiopathic and symptomatic epilepsies. These represent a
epilepsy, an increased number of affected rela- synthesis of information from the referenced
tives, and if a parent is affected, particularly the articles, and the reader is referred to epilepsy
mother [9]. Particularly for prenatal diagnostic textbooks and the references at the end of this
testing, epilepsy risk is increased if a parent is a section if a more complete list is desired.
carrier of a balanced chromosomal translocation, The clinical utility of performing these tests
if the mother is a carrier of an X-linked or remains highly variable. Identification of muta-
mitochondrial mutation, if both parents are car- tions in patients with early-onset epileptic ence-
riers for an autosomal recessive condition, or if a phalopathies such as Dravet syndrome (SCN1A),
parent carries an autosomal dominant disorder epilepsy limited to females with mental retarda-
[1]. Many severe pediatric neurogenetic condi- tion (EFMR) (PCDH19), and infantile spasms
tions result from de novo or spontaneous muta- (ARX in boys, CDKL5 in girls) can be very
tions. If parents have negative genetic testing, the useful, as they can limit further (often invasive)
risk of recurrence in future offspring is <1% and diagnostic testing, inform the prognosis, at times
would likely be attributable to undetectable guide treatment, and provide useful information
gonadal mosaicism [1]. for genetic counseling for the family [3].
Options for parents at high risk who desire In other clinical situations, genetic testing has
additional children include adoption, use of a less clinical utility. Although very useful in the
donor egg or sperm (depending on which parent setting of a baby with possible Dravet syndrome,
carries the mutation), prenatal testing (chorionic testing for SCN1A mutations in a family with
villous sampling or amniocentesis), or preim- GEFS+ has limited clinical utility due to the
plantation genetic diagnosis (which requires widely variable phenotype, so that testing cannot
174 S.K. Inati

Table 13.1 Genes identified in idiopathic epilepsy syndromes


Syndrome Genetic test Inheritance Comment
(a) Syndromes beginning in the 1st year of life
Benign familial neonatal KCNQ2 AD M-channel subunit of voltage-gated
seizures KCNQ3 potassium channel
Seizure onset often day of life 2–3
Diagnosis often clear without testing
Benign outcome
Benign familial SCN2A AD Sodium channel subunit
neonatal-infantile seizures Seizures from 2 days to 6 months, normal
development
Ohtahara syndrome STXBP1 Various Onset <6 months of age, tonic spasms,
ARX suppression burst on EEG
Early-onset spasms CDKL5 X-linked In girls (possibly lethal in males)
(consider Hypermotor-tonic-spasms
aCGH,
TSC1/2,
etc.)
X-linked infantile spasms ARX X-linked In boys
(b) Syndromes with prominent Febrile seizures
Generalized Epilepsy with SCN1A Can be AD or Sodium channel and GABAA receptor
Febrile Seizures plus SCN1B oligogenic subunits
(GEFS+) GABRG2 Early-onset febrile seizures and/or variable
afebrile seizure types including absence
Dravet syndrome (or severe SCN1A AD Sodium channel subunit
myoclonic epilepsy of Early febrile and then various afebrile seizure
infancy—SMEI) types; psychomotor delay usually apparent
during 2nd year of life
Childhood absence epilepsy GABRG2 AD GABAA receptor subunit
with febrile seizures
Epilepsy and mental PCDH19 X-linked Protocadherin
retardation limited to (present only Early-onset seizures, autism,
females in heterozygous developmental delay
females) Affects only heterozygous females
(c) Idiopathic generalized epilepsies
Early-onset absence epilepsy SLC2A1 AD GLUT1 (glucose transporter type 1)
Can also see mutation in early-onset
refractory seizures, movement disorders
Treat with ketogenic diet
Juvenile myoclonic epilepsy GABRA1 AD GABAA receptor
EFHC1 EF hand motif protein
(d) Focal epilepsies
Autosomal dominant CHRNA4 AD Nicotinic acetylcholine receptor subunits
nocturnal frontal lobe CHRNA2 Nocturnal clusters of brief motor seizures
epilepsy (ADNFLE) CHRNB2
Autosomal dominant partial LGI1 AD Also called autosomal dominant lateral
epilepsy with auditory temporal lobe epilepsy (ADLTLE)
features (ADPEAF) Simple partial seizures with mainly acoustic
hallucinations
13 Genetic Analysis of Epilepsies 175

Table 13.2 Genes identified in symptomatic epilepsy syndromes


Syndrome Genetic test Inheritance Comment
(a) Progressive myoclonic epilepsies
Myoclonic epilepsy with Mitochondrial Mitochondrial Best diagnosed by muscle biopsy
ragged red fibers (MERRF) DNA evaluation
(tRNA)
Unverricht-Lundborg EPM1 (CSTB: AR
cystatin B)
Lafora disease EPM2A, AR PME with rapidly progressive dementia
NHLRC1 Proteins: laforin and malin
(EPM2B) Lafora bodies present on biopsy
Neuronal ceroid lipofuscinosis CLN1-8 Adult onset With visual failure
(NCL) PPT1 Kufs—AD Lysosomal storage disease
All others AR
Sialidosis NEU AR Increased urinary oligosaccharides
Dentatorubral-pallidoluysian ATN1 AD Trinucleotide repeat disorder
atrophy (DRPLA)
(b) Epilepsies related to cortical malformations
Lissencephaly LIS1 AD Posterior predominant
(PAFAH1B1) Developmental delay, hypotonia,
seizures
X-linked lissencephaly/double DCX (XLIS) X-linked Males: lissencephaly, epilepsy, MR
cortex syndrome/subcortical dominant Females: broad heterotopic zone,
band heterotopia frontally predominant
Periventricular nodular FLNA X-linked ARFGEF2: PNH and microcephaly
heterotopia (PNH) ARFGEF2 dominant
(FLNA)
AR
(ARFGEF2)
(C) Other
Mitochondrial encephalopathy, Mitochondrial Mitochondrial Seizures initially with metabolic
lactic acidosis, and stroke-like DNA evaluation disarray, later due to structural lesions
episodes (MELAS) (tRNA-Leu)
Alpers syndrome POLG1 AR Nuclear gene for DNA polymerase
gamma
Intractable seizures, developmental
arrest, liver dysfunction
Tuberous sclerosis TSC1 AD, variable CNS, renal, cardiac, dermatologic,
TSC2 penetrance pulmonary, eye abnormalities
Proteins: hamartin and tuberin
Can have infantile spasms, MR, autism
Fragile X syndrome FMR1 AD Trinucleotide repeat disorder
Rett syndrome MECP2 X-linked Seizures and other “spells,” regression
Trisomy 21 Classical Epilepsy in up to 12 %, variable seizure
cytogenetics presentation including West syndrome
and Lennox–Gastaut syndrome
Angelman syndrome DNA Severe mental retardation, ataxic jerky
methylation movements, hypotonia, inappropriate
analysis laughter, absence of speech,
UBE3A microcephaly
176 S.K. Inati

accurately predict prognosis or necessarily 3. Scheffer IE. Genetic testing in epilepsy: what should
inform treatment or genetic counseling. Testing you be doing? Epilepsy Curr. 2011;11:107–11.
4. Engel J, Pedley TA, Aicardi J, Bonkowsky et al.
for KCNQ2 and 3 in benign familial neonatal Genetic diseases associated with epilepsy, Epilepsy.
seizures, although highly accurate, is also of Philadelphia, PA: Lippincott Williams & Wilkins;
limited usefulness clinically. The diagnosis is 2007 (Chapter 17 ).
often clear based on the history, as this disorder 5. Miller DT, et al. Consensus statement: chromosomal
microarray is a first-tier clinical diagnostic test for
is autosomal dominant with high penetrance. In individuals with developmental disabilities or congen-
addition, the prognosis is favorable, so diagnosis ital anomalies. Am J Hum Genet. 2010;86:749–64.
is unlikely to alter reproductive choices [2]. 6. Mefford HC, et al. Rare copy number variants are an
important cause of epileptic encephalopathies. Ann
Neurol. 2011;70:974–85.
7. Harkin LA, et al. The spectrum of SCN1A-related
References infantile epileptic encephalopathies. Brain.
2007;130:843–52.
8. Hampton T. Congress passes bill to ban discrimination
1. Pong AW, Pal DK, Chung WK. Developments in
based on individuals’ genetic makeup. JAMA.
molecular genetic diagnostics: an update for the
2008;299:2493.
pediatric epilepsy specialist. Pediatr Neurol.
9. Winawer MR, Shinnar S. Genetic epidemiology of
2011;44:317–27.
epilepsy or what do we tell families? Epilepsia.
2. Ottman R, et al. Genetic testing in the epilepsies-report
2005;46(Suppl 10):24–30.
of the ILAE Genetics Commission. Epilepsia.
2010;51:655–70.
Epilepsy Secondary to Specific
Mechanisms 14
Amar Bhatt

In any suspected cases, antibody testing and


Autoimmune Epilepsy
malignancy screening are necessary. Antibodies
should be tested in both serum and CSF, though
Epilepsy related to cerebral autoimmune disease
they may be found more consistently in the CSF.
has recently gained increased recognition. These
Routine CSF studies (cell counts, oligoclonal
are not “zebras” and remain an important, treat-
bands, and IgG index) are usually normal. MRI
able cause of epilepsy. Antiepileptic drugs
should be performed with contrast, but this can
(AEDs) alone will not suffice as treatment, and a
also be normal. Cancer investigation should
protracted course of multiple immunotherapies
include whole-body PET/CT; in select cases,
may be needed (often over weeks or months).
ultrasound, endoscopy, or mammography may be
The evidence basis for diagnosis and treatment of
needed.
autoimmune epilepsy is limited to expert opinion
Antibody testing in the serum and CSF is
and case series [1, 2].
available both commercially and through private
Autoimmune epilepsy should be considered early
universities. Newer and rarer antibodies may not
in cases of limbic encephalitis, new-onset refractory
be available commercially or may not be inclu-
epilepsy, or new-onset status epilepticus. Features
ded on the commercial panels. Antibodies can be
that favor autoimmune epilepsy include
classified as cellular (“onconeural”) or cell
encephalopathy, cognitive decline, personality
membrane. The cellular antibodies have a
changes, a movement disorder, or prominent psy-
stronger association with cancer, though these
chiatric symptoms (psychosis, catatonia, and agita-
are thought to represent an epiphenomenon and
tion). Additional “red flags” include autoimmune
are not necessarily pathogenic. Cellular
stigmata (type 1 diabetes mellitus, thyroid disease,
antibody-mediated diseases may be poorly
celiac disease, and vitamin B12 deficiency) or a
responsive to immunotherapy and require an
history of cancer (or strong cancer risk factors). Stiff
exhaustive search for malignancy. A recent
person syndrome, type 1 diabetes mellitus, and
review [3] has an excellent discussion of the
autoimmune encephalitis can all be associated with
most common antibodies, their classifications,
anti-glutamic acid decarboxylase (GAD) antibodies.
and common cancer associations. A brief over-
view is given in Table 14.1.
One important set of antibodies is directed
against the voltage-gated potassium channel
(VGKC) complex. This complex was previously
implicated in Isaac syndrome (neuromyotonia)
A. Bhatt (&)
which at times was paraneoplastic. These
Department of Neurology, Vanderbilt University
Medical Center, Nashville, TN, USA antibodies are associated with non-paraneoplastic
e-mail: amar.b.bhatt@vanderbilt.edu autoimmune limbic encephalitis, presenting with

© Springer Science+Business Media LLC 2017 177


M.Z. Koubeissi and N.J. Azar (eds.), Epilepsy Board Review,
DOI 10.1007/978-1-4939-6774-2_14
178 A. Bhatt

Table 14.1 Common antibody-related autoimmune epilepsies and associated presentations


Antibody/target Symptoms (other than seizures/limbic encephalitis) Associated cancer(s)
VGKC Personality or behavioral changes, myoclonus (CJD-like picture), SCLC, thymoma
complexa neuropathy, and hyponatremia
NMDA receptor Psychosis, extrapyramidal disorders (e.g., choreoathetosis), and Ovarian teratoma
dysautonomia
GAD Stiff person syndrome, ataxia, brainstem encephalitis, parkinsonism, and Thymoma
diabetes (DM-1) Breast
adenocarcinoma
Ma1, Ma2 Brainstem encephalitis Testicular
ANNA-1 (Hu) Brainstem encephalitis, autonomic or sensory neuropathy SCLC
CRMP-5 Dementia, personality change, chorea, ataxia, and neuropathy SCLC Thymoma
Amphiphysin Dementia, myelopathy, and neuropathy SCLC Breast
adenocarcinoma
Antibody/target GABA receptor; Symptoms: encephalopathy; Associated cancer(s) SCLC, thymoma
Antibody/target ANNA-2 (Ri); Symptoms: brainstem encephalitis, cerebellar ataxia; SCLC, breast,
Associated cancer(s) gynecological
Antibody/target AMPA receptor; Symptoms: psychiatric; Associated cancer(s) multiple solid cancers
a
Multiple antibody targets in this complex (LGI1, CASPR2, and contactin-2)
VGKC, voltage gated potassium channel; SCLC, small cell lung cancer; CJD, Creutzfeld-Jakob disease; NMDA,
N-methyl-D-aspartate; GAD, glutamic acid decarboxylase; DM, diabetes mellitus; ANNA, anti-neuronal nuclear
antibody; CRMP, collapsin response mediator protein; GABA, gamma-aminobutyric acid; AMPA,
alpha-amino-3-hydroxy-5-methyl-4-isoxazolepropionic acid

seizures, confusion, amnesia, and myoclonus non-paraneoplastic. Typical symptoms include


(thus mimicking Creutzfeldt–Jakob disease). seizures, confusion, catatonia, amnesia, choreoa-
There is often associated hyponatremia. Both thetosis, and dysautonomia. Anti-NMDA-R anti-
seizures and MRI abnormalities (T2 hyperinten- body titers may correlate to disease severity. The
sity, restricted diffusion, or contrast enhance- course may be protracted, have relapses, and
ment) are typically in the temporal regions, require hospitalization for weeks or months to
though generalized seizures may also occur. control drug-resistant seizures or
Variations in presentation may relate to the dif- immunotherapy-resistant symptoms. A recent
ferent antibody targets within the VGKC com- study suggests that the “extreme delta brush” pat-
plex; laboratory results may be reported by the tern on EEG may be a unique finding in
specific target (CASPR2, LGI1, and contactin-2). anti-NMDA-R encephalitis [4].
LGI1-associated disease may present with Management has a four-part approach: first, an
faciobrachial dystonic seizures (FBDS), charac- aggressive workup including MRI, EEG, CSF,
terized by repetitive, brief episodes of facial antibody testing, and cancer screening; second,
twitching and ipsilateral arm dystonia, with or early immunotherapy; third, concomitant AED
without EEG correlation. FBDS may occur treatment; and fourth, management of systemic
before, during, or after the development of cog- complications. First-line immunotherapy is usu-
nitive impairment, which can delay diagnosis. ally 3–5 days of IV methylprednisolone, IV
Another important autoimmune epilepsy is immunoglobulin, or both. If there is good
related to anti-N-methyl-D-aspartate receptor response, the treatment may be tapered and
(NMDA-R) antibodies. This is classically descri- replaced with mycophenolate or azathioprine. In
bed as a paraneoplastic syndrome associated with resistant cases, cyclophosphamide or rituximab
ovarian teratoma, though it is often may be considered.
14 Epilepsy Secondary to Specific Mechanisms 179

Brain Tumors and Epilepsy correlation is the absence of LGI1 gene product
in GBM, due to gene translocation [5]. This is a
Intracranial tumors are a common cause of adult tumor suppressor gene, but two non-neoplastic
—and childhood-onset epilepsies. In general, the epilepsies relate to LGI1: autosomal dominant
following tumors are more epileptogenic: lateral temporal lobe epilepsy with auditory fea-
adult-onset tumors (which tend to be supraten- tures caused by LGI1 gene mutation and
torial, as opposed to pediatric tumors), lower autoimmune epilepsy related to antibodies
grade tumors, cortical tumors, and tumors closer against an LGI1 gene product (VGKC complex).
to sensitive networks, such as hippocampus or The American Academy of Neurology
motor cortex [5]. Parietal tumors have the (AAN) guidelines recommend strongly against
strongest association with seizures, followed AED prophylaxis in brain tumor patients without
closely by temporal tumors. a history of seizures, since prophylaxis does not
Seizure semiology depends on tumor location, prevent the first seizure [7]. AED prophylaxis
but certain pathologies have stronger association may be used peri- and post-operatively, but
with seizures. Nearly all dysembryoplastic neu- usually only for one week. Once seizures have
roepithelial tumors will cause seizures, followed occurred, AEDs must be chosen carefully due to
by gangliogliomas and low-grade astrocytomas; interactions with chemotherapy and corticos-
higher grade or fast-growing tumors (such as teroids, as well as additive risk of bone marrow
glioblastoma multiforme [GBM] or primary CNS suppression. Thus, agents such as levetiracetam
lymphoma) do not cause seizures as often [6]. and lacosamide may be preferred.
A characteristic GBM is shown in Fig. 14.1. The goal of seizure freedom must be balanced
Additionally, hypothalamic hamartomas cause with tumor prognosis; seizure freedom may not
gelastic seizures. Regardless of tumor type, a be a goal with unresectable tumors. Surgical
seizure as the initial symptom of tumor presen- treatment must be divided into “tumor surgery”
tation may increase the risk of recurrent seizures (curative) or “epilepsy surgery” (palliative). Poor
and refractory seizures, possibly independent of prognostic factors for seizure control include
treatment. longer epilepsy duration, lower tumor grade,
Epileptogenicity may relate to both peritu- seizures at time of tumor diagnosis, and incom-
moral (non-neoplastic) tissue as well as genetic plete resection. Surgery can be considered even
factors. Higher grade tumors may have central in low-grade tumors with resistant epilepsy, even
necrosis and be electrically silent, whereas sur- if stable on imaging. Imaging alone should not
rounding hemosiderin or edematous tissue may guide surgery, since peritumoral tissue can be
be epileptogenic. One example of a genetic epileptogenic. Video-EEG, electrocorticography,

Fig. 14.1 Left temporal


glioblastoma multiforme,
on T2 FLAIR and
T1-contrasted MRI
180 A. Bhatt

and functional mapping (e.g., language or motor has important implications; males have the more
function) should be used to guide resection. severe phenotype of LIS, whereas females have
the milder phenotype of SBH (e.g., mild devel-
opmental delay and seizure onset in teenage
Malformations of Cortical years).
Development Polymicrogyria (PMG) is characterized by
excessive, small gyri. It may present as bilateral
Classification and understanding of malformations perisylvian polymicrogyria syndrome, consisting
of cortical development (MCDs) continues to of seizures, aphasia, and oromotor dysfunction.
evolve. Most definitions are based on genetics, Schizencephaly (SCZ) and porencephaly
imaging, molecular biology, and pathology [8, 9]. (POR) are both characterized by parenchymal
Stem cells not only differentiate into neurons and “clefts”; SCZ typically has gray matter along the
glia, but they also migrate radially outward from the clefts (which is often PMG), whereas POR has a
germinal matrix in the deep forebrain and white matter lining. When SCZ is associated with
periventricular regions. They also organize into optic nerve hypoplasia and absence of the septum
“cytoarchitectonic” patterns, creating the six layers pellucidum, this is known as septo-optic dys-
of neocortex. Any disruption in this process can plasia (de Morsier syndrome), and screening for
lead to MCDs (i.e., normal cells in the wrong place, hypopituitarism is important.
or abnormal cells in the right place). Periventricular nodular heterotopia (PVNH)
Many MCDs are named based on descriptive consists of gray matter nodules along the lateral
anatomic terms and do not indicate a specific ventricles due to failed neuronal migration
disease or genetic cause per se; in fact, many (Fig. 14.2), often causing intractable focal sei-
have overlapping pathology. Some occur in iso- zures. PVNH may be associated with abnormal
lation as well as in the context of larger syn- overlying cortex; there is debate as to whether
dromes, such as hemimegalencephaly (HMEG). both the nodule and cortex should be resected.
HMEG is characterized by a triad of intractable PVHN must be differentiated from the
partial seizures from infancy, hemiparesis, and subependymal nodules of TSC (Table 14.2).
developmental delay; imaging readily identifies
an enlarged, dysmorphic cerebral hemisphere.
HMEG may occur in neurocutaneous syndromes,
such as tuberous sclerosis complex (TSC) or
neurofibromatosis. Functional hemispherectomy
can improve seizure control and quality of life.
Lissencephaly (LIS) and subcortical band
heterotopia (SBH) are two distinct phenotypes
that may share similar genetic features. LIS is
characterized by a “smooth brain” with absent or
decreased convolutions (so-called agyria or
pachygyria). SBH consists of an extra band of
gray matter within the white matter (also known
as “double cortex”). The classical form of LIS
has a thickened, four-layer cortex and may have
associated SBH. The autosomal dominant form
of LIS is caused by LIS1 gene mutation and is
typically more severe posteriorly, whereas the
X-linked form is usually caused by DCX
(“doublecortin”) gene mutation and is typically Fig. 14.2 Bilateral periventricular nodular heterotopia,
more severe anteriorly. The X-linked inheritance as seen on T2-weighted MRI
14 Epilepsy Secondary to Specific Mechanisms 181

Table 14.2 Comparison of periventricular lesions in two distinct neurological disorders


Tuberous sclerosis complex (subependymal nodules) Periventricular nodular heterotopia
Smaller Larger
Less in number More in number, often bilateral
Heterogeneous Homogeneous
Calcified Not calcified
White matter intensity on MRI Gray matter intensity on MRI

Fig. 14.3 Left parietal


focal cortical dysplasia,
with thickened cortex on
T2 FLAIR and concordant
region of hypometabolism
on FDG-PET

PVNH can be familial, most often due to the


X-linked FLNA (filamin A) gene mutation. Post-traumatic Epilepsy
Genetic cases are typically female and have
bilateral PVNH (presumably the mutation is Head trauma is a leading cause of epilepsy,
lethal in males). especially in young adults. The challenge lies in
Focal cortical dysplasias (FCDs) can also cause differentiating post-traumatic epileptic seizures
intractable focal seizures. Typical MRI findings from psychogenic non-epileptic events (PNES),
include blurred gray–white junction, thickened though both may coexist in the same patient.
cortex (Fig. 14.3), or the transmantle sign (a band Post-traumatic seizures are classified as early
of T2 hyperintensity extending radially between (within the first week) and late (after the first
the cortex and ventricle). Many FCDs are subtle week).
or not visible on MRI and may be found on A single late unprovoked post-traumatic sei-
functional imaging (PET or interictal SPECT, see zure is nearly synonymous with epilepsy, and the
Fig. 14.3). FCDs can be classified by pathological terms may be used interchangeably. In one study,
severity [10, 11]; more severe pathology has better the risk of seizure recurrence after a single late
prognosis, possibly due to being more visible on seizure was 86% within two years [13]. Therefore,
MRI or due to having better defined resection only one late seizure is necessary to diagnose
margins [12]. The mildest type is known as epilepsy and strongly consider AED treatment.
microdysgenesis. The intermediate type (Type I) Approximately 10% of patients with early
may or may not be seen on MRI. The most severe post-traumatic seizures develop epilepsy; how-
type (Type II) can have “balloon cells” on ever, multivariate analysis has shown that this can
pathology (Type IIb). Type III refers to dual be explained by factors other than the early sei-
pathology (FCDs associated with other lesions, zures themselves [14]. Also, early status epilepti-
such as tumors or mesial temporal sclerosis). cus may have a higher risk for late seizures.
182 A. Bhatt

Head trauma may be classified as mild, though maintaining first-week seizure freedom
moderate, or severe (Table 14.3). The presence does not reduce mortality, disability, or late sei-
of early seizures in combination with moderate zures [18]. There is no evidence for the use of
or severe head trauma increases the risk of steroids to prevent seizures.
developing epilepsy [15]. Early seizures in mild
head trauma do not necessarily increase that risk.
In fact, there may be an association between mild Stroke and Epilepsy
head trauma and PNES. In children under five
years of age, early seizures after head trauma are Pediatric and adult epilepsy related to cere-
more common, but these are less predictive of brovascular disease differs in many regards.
epilepsy as compared to adults. Seizure as the presenting symptom of stroke is
Assuming that a past head injury is the cause very common in neonates (about 80%), relatively
of seizures can be detrimental; one may miss common in children (about 30%), and rare in
non-epileptic events or a diagnosis of adults; epilepsy risk after pediatric stroke can be
idiopathic/genetic generalized epilepsy. The cost up to 40%, whereas the risk in adults is less than
may be more severe in veterans; PNES is asso- 5% [19].
ciated with significantly more cumulative AED Post-stroke seizures are classified as early
exposure and delay in diagnosis in veterans as (within the first week) or late (after the first
compared to civilians [16]. Epileptiform dis- week), similar to post-traumatic seizures. Even
charges on EEG are not necessarily predictive of one late unprovoked post-stroke seizure has a
epilepsy and can be misleading, particularly high recurrence rate (>50%), and AED treatment
when events are unwitnessed or the history is should strongly be considered. Therefore, as in
vague. Video-EEG remains the gold standard in head trauma, late post-stroke seizures are nearly
the diagnosis of post-traumatic seizure-like synonymous with epilepsy.
events, and it should be considered in any Predictors of post-stroke epilepsy in adults
patient who is medication-resistant after one year include cortical location, presence of hemor-
of treatment. Video-EEG with AED withdrawal rhage, and stroke severity (based on examination
is important, since some patients have both and NIH stroke scale) [19]. The EEG is not
epileptic seizures and PNES. consistent in predicting epilepsy after stroke.
Prophylactic treatment is only recommended Lateralized periodic discharges (LPDs, formerly
in certain situations, and the evidence is strongest known as periodic lateralized epileptiform dis-
for prevention of early seizures in adults; data in charges or PLEDs) are considered a classic
children is insufficient. The AAN guidelines finding in stroke and may be predictive of sei-
recommend phenytoin prophylaxis in adults with zures, but they are not common, and may only
severe brain injury, but only for the first week; predict early seizures and not necessarily later
evidence does not suggest benefit of longer epilepsy. The most common finding is slow
duration of prophylaxis in preventing late sei- activity (focal or generalized), which is
zures [17]. Per a Cochrane review, the number non-specific and not predictive of seizures. A re-
needed to treat in preventing early seizures is ten, cent Cochrane review [20] has not found strong

Table 14.3 Definitions of the severity of head trauma


Mild Moderate Severe
Duration of loss of <30 min 30 min–24 h >24 h
consciousness, amnesia, or
coma
Presence of structural brain None Skull fracture without contusion Brain contusion, intracranial
injury or intracranial hemorrhage hemorrhage, or dural penetration
14 Epilepsy Secondary to Specific Mechanisms 183

evidence that treating early post-stroke seizures thin cut imaging (Fig. 14.4) [24]. When com-
prevents the development of epilepsy, although paring hippocampal volumes, asymmetry of the
only one of the studies reviewed met inclusion temporal horns of the lateral ventricle should not
criteria as a randomized controlled trial designed be over-interpreted as MTS. Additionally, even if
to address this question. MRI is normal, PET may show temporal hypo-
metabolism suggestive of MTS. A recent study
found good surgical outcomes after temporal
Mesial Temporal Sclerosis lobectomy in patients with PET-positive,
MRI-negative temporal lobe lesions [25], com-
Mesial temporal sclerosis (MTS), also known as parable to the typical MRI-positive MTS
hippocampal sclerosis, is one of the most com- patients.
mon causes of adult-onset epilepsy, especially The cause of MTS is unclear. There may be a
refractory epilepsy. However, it has been found relationship between MTS, early complex febrile
in up to 14% of adults without epilepsy [21]. seizures, and childhood head trauma, but cause
Classic semiology can include abdominal auras and effect remain controversial. Often there is a
(nausea, pressure, butterflies, and epigastric ris- latent period between the injury and seizure
ing), fear, an unpleasant taste or smell, oroali- onset, but it is not clear whether the MTS noted
mentary or (ipsilateral) limb automatisms, and on imaging was either not previously present, not
autonomic phenomenon. The typical ictal EEG apparent since the brain was still developing, or
pattern consists of anterior temporal rhythmic not able to be studied by current imaging
theta or alpha activity, which often exceeds 5 Hz techniques.
within 30 s of seizure onset [22, 23]. Though Histopathology in MTS usually involves
many cases may have bilateral temporal onset on neuronal loss and gliosis in the CA1, CA3, and
scalp (bisynchronous or independent), this does CA4 hippocampal regions, with relative sparing
not necessarily rule out surgery. However, since of CA2. Surgical experience has noted two
seizures may start elsewhere and spread to the important areas outside the hippocampus that
mesial temporal region, a primary extratemporal usually require resection to achieve a good sei-
localization may be the source of seizures, even zure outcome: the parahippocampal gyrus and
when semiology and ictal EEG patterns are pre- the amygdala. Because MTS is so common, and
dominantly temporal. so amenable to surgical resection, new-onset
The most common MRI finding in MTS is temporal lobe epilepsy at any age warrants
hippocampal hyperintensity on T2-weighted evaluation for MTS. Dual pathology (MTS with
sequences (e.g., FLAIR). However, this is not coexistent neoplasms, MCDs, or vascular
very reliable. Hippocampal atrophy is the most lesions) may require resection of both lesions for
specific finding, usually noted on T1-weighted, a good outcome.

Fig. 14.4 Right mesial


temporal sclerosis, with
hippocampal atrophy on
T1-weighted MRI and
anterior temporal
hypometabolism on
FDG-PET
184 A. Bhatt

Predictors of good postsurgical outcomes reported, usually with autosomal dominant


include later age at onset, shorter duration of inheritance; some patients also have cutaneous
epilepsy, presence of febrile seizures, positive and retinal involvement.
MRI (or positive PET with negative MRI), uni- In a recent large population study [27], risk of
lateral findings on PET, concordant data new-onset seizure over five years in patients with
(matching of localization based on semiology, incidental AVMs was 8%, but the risk increased
EEG, functional imaging, and anatomical imag- to 23% if the AVM had previously caused hem-
ing), and the lack of need for intracranial moni- orrhage or a focal neurological deficit. On the
toring. In a typical case of MTS, the chance of other hand, this study noted that CMs carried a
seizure freedom after resection is approximately similar risk (4–6%) of new-onset seizure, whether
60–70%. Surgical options include selective the CM was incidental or symptomatic. There was
amygdalohippocampectomy, tailored temporal a suggestion that AVMs (but not CMs) in the
lobectomy (sparing dominant eloquent function), temporal lobe were more likely to cause seizures.
hippocampal laser ablation, and standard anterior Venous angiomas and DVAs rarely cause
temporal lobectomy. seizures or hemorrhage and are most often inci-
dental. Resection should probably be avoided, as
the epileptogenic focus may be unrelated to these
Vascular Malformations lesions. A recent review of fifteen studies (with a
combined 714 patients at the time of DVA
Epilepsy is most strongly associated with arteri- diagnosis) found that 61% of DVAs were inci-
ovenous malformations (AVMs) and cavernous dental findings, 6% were associated with focal
malformations (CMs). Developmental venous neurologic deficits, 6% with symptomatic
anomalies (DVAs) are usually incidental findings bleeding, and 4% with seizures; the presenting
and not epileptogenic. In most vascular malfor- symptoms were unclear in the remaining 23%
mations, the surrounding hemorrhage, gliosis, [28]. This study also prospectively analyzed an
and encephalomalacia are the epileptogenic tis- adult DVA population in Scotland, noting that
sues; the vascular lesions themselves are silent 98% of DVAs (in 93 patients) were incidental.
since they do not contain neuronal structures.
Surgical management should have dual goals of
seizure freedom and hemorrhage prevention. References
Electrocorticography-guided resection may have
better seizure outcomes, as opposed to pure
1. Irani SR, Bien CG, Lang B. Autoimmune epilepsies.
structural lesion-guided resection, especially in Curr Opin Neurol. 2011;24:146–53.
temporal CMs [26]. Stereotactic radiosurgery is 2. Quek AM, Britton JW, McKeon A, et al. Autoim-
also an option in AVMs. mune epilepsy: clinical characteristics and response
An AVM is a direct connection between to immunotherapy. Arch Neurol. 2012;69(5):582–
93.
arteries and veins, without capillaries in between. 3. McKeon A, Pittock SJ. Paraneoplastic encephalo-
These appear as a small collection of signal void myelopathies: pathology and mechanisms. Acta
on MRI. CMs are also known as cavernous Neuropathol. 2011;122:381–400.
angiomas or cavernomas, consisting of small 4. Schmitt SE, Pargeon K, Frechette ES, Hirsch LJ,
Dalmau J, Friedman D. Extreme delta brush: a
bundles of brittle vascular endothelium (not true unique EEG pattern in adults with anti-NMDA
vessels) that lead to recurrent bleeding. On MRI, receptor encephalitis. Neurology. 2012;79
they are heterogeneous, with a core of mixed (11):1094–100.
signal intensity surrounded by a T2 or 5. Jehi L. Brain Tumors and Epilepsy. In: Wyllie E,
editor. Wyllie’s treatment of epilepsy: principles and
gradient-echo hypointense rim (presumably practice (Chap. 28). 5th ed. Philadelphia: Lippincott
hemosiderin). Familial CM syndromes have been Williams and Wilkins; 2011.
14 Epilepsy Secondary to Specific Mechanisms 185

6. van Breemen MS, Wilms EB, Vecht CJ. Epilepsy in 18. Schierhout G, Roberts I. Antiepileptic drugs for
patients with brain tumours: epidemiology, mecha- preventing seizures following acute traumatic brain
nisms, and management. Lancet Neurol. 2007;6 injury. Cochrane Database Syst Rev. 2012;6:
(5):421–30. CD000173.
7. Glantz MJ, Cole BF, Forsyth PA, et al. Practice 19. Hantus S, Friedman N, Pohlmann-Eden B. Epilepsy
parameter: anticonvulsant prophylaxis in patients in the setting of cerebrovascular disease (Chap. 30).
with newly diagnosed brain tumors. Neurology. In: Wyllie E, editor. Wyllie’s treatment of epilepsy:
2000;10(54):1886–993. principles and practice. 5th ed. Philadelphia: Lippin-
8. Leventer RJ, Guerrini R, Dobyns WB. Malforma- cott Williams and Wilkins; 2011.
tions of cortical development and epilepsy. Dialog 20. Sykes L, Wood E, Kwan J. Antiepileptic drugs for
Clin Neurosci. 2008;10(1):47–62. the primary and secondary prevention of seizures
9. Mirzaa G, Kuzniecky R, Guerrini R. Malformations after stroke. Cochrane Database Syst Rev. 2014;1:
of cortical development and epilepsy. In: Wyllie E, CD005398.
editor. Wyllie’s treatment of epilepsy: principles and 21. Benbadis SR, Wallace J, Reed Murtagh F. MRI
practice (Chap. 27). 5th ed. Philadelphia: Lippincott evidence of mesial temporal sclerosis in subjects
Williams and Wilkins; 2011. without seizures. Seizure. 2002;11(5):340–3.
10. Blümcke I, Thom M, Aronica E, et al. The clinico- 22. Risinger MW, Engel J Jr, Van Ness PC, Henry TR,
pathologic spectrum of focal cortical dysplasias: a Crandall PH. Ictal localization of temporal lobe
consensus classification proposed by an ad hoc Task seizures with scalp/sphenoidal recordings. Neurol-
Force of the ILAE Diagnostic Methods Commission. ogy. 1989;39(10):1288–93.
Epilepsia. 2011;52(1):158–74. 23. Ebersole JS, Pacia SV. Localization of temporal lobe
11. Krsek P, Maton B, Korman B, et al. Different foci by ictal EEG patterns. Epilepsia. 1996;37
features of histopathological subtypes of pediatric (4):386–99.
focal cortical dysplasia. Ann Neurol. 2008;63 24. Cendes F. Neuroimaging in investigation of patients
(6):758–69. with epilepsy. Continuum (Minneap Minn). 2013;19
12. Lerner JT, Salamon N, Hauptman JS, et al. Assess- (3):623–42.
ment and surgical outcomes for mild type I and 25. LoPinto-Khoury C, Sperling MR, Skidmore C, et al.
severe type II cortical dysplasia: a critical review and Surgical outcome in PET-positive, MRI-negative
the UCLA experience. Epilepsia. 2009;50(6):1310– patients with temporal lobe epilepsy. Epilepsia.
35. 2012;53(2):342–8.
13. Haltiner AM, Temkin NR, Dikmen SS. Risk of 26. Van Gompel JJ, Rubio J, Cascino GD, Worrell GA,
seizure recurrence after the first late posttraumatic Meyer FB. Electrocorticography-guided resection of
seizure. Arch Phys Med Rehabil. 1997;78(8):835–40. temporal cavernoma: is electrocorticography war-
14. Annegers JF, Hauser WA, Coan SP, Rocca WA. ranted and does it alter the surgical approach? J
A population-based study of seizures after traumatic Neurosurg. 2009;110(6):1179–85.
brain injuries. N Engl J Med. 1998;338(1):20–4. 27. Josephson CB, Leach JP, Duncan R, Roberts RC,
15. Schuele S. Post-traumatic epilepsy (Chap. 29). In: Counsell CE, Al-Shahi Salman R; SAIVMs steering
Wyllie E, editor. Wyllie’s treatment of epilepsy: committee and collaborators. Seizure risk from
principles and practice. 5th ed. Philadelphia: Lippin- cavernous or arteriovenous malformations: prospec-
cott Williams and Wilkins; 2011. tive population-based study. Neurology. 2011;76
16. Salinsky M, Spencer D, Boudreau E, Ferguson F. (18):1548–54.
Psychogenic nonepileptic seizures in US veterans. 28. Hon JM, Bhattacharya JJ, Counsell CE, et al. SIVMS
Neurology. 2011;77(10):945–50. collaborators. The presentation and clinical course of
17. Chang BS, Lowenstein DH. Practice parameter: intracranial developmental venous anomalies in
antiepileptic drug prophylaxis in severe traumatic adults: a systematic review and prospective,
brain injury. Neurology. 2003;60:10–6. population-based study. Stroke. 2009;40(6):1980–5.
Multiple Choice Questions for Part III

1. Which of the following is shared by Dravet 4. Which of the following is true about
syndrome and generalized epilepsy with familial lateral temporal lobe epilepsy with
febrile seizures plus (GEFS+)? auditory features?

A. Lamotrigine is helpful in both A. Autosomal recessive inheritance


conditions B. Typical seizure onset in early
B. Both result from mutations in SCN1A childhood
C. EEG shows multifocal spikes and C. Typical seizure onset in early
slowing childhood
D. Seizures decrease by late childhood D. Related to mutations in the leucine-rich
E. All of the above glioma-inactivated (LGI) gene on
chromosome 10q
2. During video-EEG monitoring, which of E. Epilepsy surgery is curative.
the following features are suggestive of
nonepileptic psychogenic seizures? 5. A 6-month-old-male infant has a hemi-
clonic convulsion lasting 5 min in the con-
A. Epigastric rising sensation, lip smack- text of a fever. He continues having
ing, and confusion monthly febrile seizures. Around
B. Visual hallucination, nystagmus, con- 12 months of age, he develops myoclonic
fusion, and vomiting seizures and generalized tonic clonic sei-
C. Eye closure, confusion, and speech zures. His development is noted to be
stuttering delayed by 18 months of age. Which of the
D. Head turning, confusion, and bicycling following is true?
E. Brief staring, unresponsiveness, and
subtle facial automatism A. Genetic testing for mutations in a
sodium channel is indicated.
3. The rate of GLUT1 deficiency in B. Seizures are easily controlled with the
early-onset absence epilepsy is closest to: first antiseizure medicine.
C. Nonconvulsive status epilepticus is
A. 1% very rare.
B. 10% D. Gait remains normal in all patients.
C. 50% E. Vaccination has never been reported as
D. 75% a trigger.
E. 100%
188 Multiple Choice Questions for Part III

6. Mutations in which of the following genes C. Unverricht–Lundborg disease


are associated with autosomal dominant D. Sialidosis
nocturnal frontal lobe epilepsy (ADNFLE)? E. Batten disease

A. STXBP1 11. Which of the following is true about epi-


B. LGI lepsy with myoclonic absences?
C. SCN1A
D. KCNQ2 A. Ethosuximide must be avoided
E. nicotinic acetylcholine receptor B. Seizure frequency increases in the
afternoon and evening
7. The risk of SUDEP in patients with C. Brain MRI may show abnormalities
drug-resistant convulsive epilepsy is: D. Tonic seizures do not occur
E. It is more frequent in girls
A. 1/100000
B. 1/10000 12. A 3 month-old full-term baby presents with
C. 1/1000 upper respiratory tract symptoms and one
D. 1/100 simple febrile seizure. His work-up should
E. 1/10 include:

8. Which of the following is true about A. EEG


childhood absence epilepsy (CAE)? B. LP
C. Brain MRI/MRA
A. Ethosuximide and valproate are equally D. Bronchoscopy
tolerable E. None of the above
B. Lamotrigine and ethosuximide are
equally efficacious 13. Mutations of which of the following genes
C. Children with CAE are cognitively are not associated with generalized epilepsy
intact with febrile seizures plus (GEFS+)?
D. GABA receptor mutations have been
identified A. GABRG2
E. It is more frequent in boys B. PCDH19
9. Convulsive psychogenic seizures are more C. SCN1B
likely to show the following except: D. SCN1A

A. Closed eyes 14. Which of the following statements is true


B. Side-to-side head movements regarding seizures after stroke?
C. Asynchronous motor activity
D. Back arching A. Neonatal strokes are more likely to
E. Postictal regular and noisy breathing present with focal neurologic deficits
rather than seizures
10. Mutations of the cystatin B gene are asso- B. Adult onset stroke is more likely to lead
ciated with which of the following pro- to epilepsy as compared to pediatric
gressive myoclonus epilepsies (PME)? stroke
C. At least two late seizures are necessary
A. Lafora body disease to diagnose post-stroke epilepsy and
B. Jansky–Bielschowsky disease consider treatment
Multiple Choice Questions for Part III 189

D. Focal slowing on EEG is not predictive 18. Which of the following is true regarding
of epilepsy after stroke seizures after traumatic brain injury?
E. Strokes are not correlated with seizures
A. Prophylaxis of seizures should continue
15. Which of the following is true about Doose for one month after severe brain injury
syndrome? B. Risk of epilepsy decreases with time after
the injury, especially after the first two
A. Positive family history of idiopathic years
generalized epilepsy C. At least two late seizures are necessary
B. Grand mal seizures are rare to diagnose post-traumatic epilepsy and
C. Brain MRI often shows abnormalities consider treatment
D. Glut-1 deficiency is identified in 50% D. A history of mild head trauma and
of cases abnormal EEG warrants anticonvulsant
E. Dietary therapies are not effective treatment
E. All of the above are correct
16. A 6-year-old male is seen for occasional
nocturnal seizures consisting of strange 19. Which of the following is true about Len-
throaty noise and contraction of the left face. nox–Gastaut syndrome (LGS)?
The yield of the EEG will be highest during:
A. Infantile spasms precede LGS in >50%
A. Photic stimulation of cases
B. Sleep B. The EEG shows spike-wave discharges
C. Wakefulness at 3–5 Hz
D. Hyperventilation C. Sleep normalizes the EEG
E. Studying D. EEG background is often normal
E. 70% of the patients have structural/
17. Which of the following does not have an metabolic causes
autosomal dominant inheritance?
20. A 26 year old lady with two-year history of
A. Early-onset absence epilepsy (due to diarrhea and weight loss, presented with
GLUT-1) new onset seizures. Which of the following
B. Juvenile myoclonic epilepsy (due to can help most with the diagnosis?
GABA-A receptor)
C. Lateral temporal lobe epilepsy with A. Pelvic CT scan
auditory features (due to LGI1) B. D-Xylose test
D. Dentatorubral-pallidoluysian atrophy C. Sural biopsy
(due to trinucleotide repeats) D. Urine epinephrine test
E. Sialidosis (due to Increased urinary E. None of the above, EEG is the only
oligosaccharides) helpful test
190 Multiple Choice Questions for Part III

21. A 16-year-old girl with intractable partial 25. A 17-year-old high school student presented
epilepsy has a brain MRI which reveals with recurrent weekly episodes of odd
periventricular lesions. Which of the fol- behavior at night. During these episodes, he
lowing radiologic features is more likely to would wake up 3 h after falling asleep,
suggest periventricular nodular heterotopia, walk through the dormitory, or be sitting
rather than the subependymal nodules of quietly in the kitchen room. His classmates
tuberous sclerosis complex? report that he would appear confused and
clumsy and afterward has no recollection of
A. Unilateral lesion the event. Which of the following is the
B. Calcified lesion most likely diagnosis?
C. Homogenous lesion
D. White matter intensity lesion A. Nocturnal absence seizures
E. Ventricular lesion B. Confusional arousal disorder
C. Frontal hypermotor seizures
22. Which of the following is an identifiable D. Sleep-induced syncope
pathology in continuous spike-and-wave E. Early-onset dementia
during sleep (CSWS)?
26. Which of these tumor types is most likely to
A. Polymicrogyria be associated with seizures?
B. Porencephaly
C. Thalamic lesions A. Primary CNS lymphoma (PCNSL)
D. Migrational disorders B. Dysembryoplastic neuroepithelial
E. All of the above tumor (DNET)
C. Meningioma
23. A 6-month-old boy has epileptic spasms, D. Astrocytoma
hypotonia, and difficulty feeding. His MRI E. Schwannoma
reveals lissencephaly. Which genetic muta-
tion is also known to be responsible for a 27. Which of the following is true about juve-
milder phenotype, especially in girls? nile myoclonic epilepsy (JME)?

A. LIS1 on chromosome 17 A. Photosensitivity is noted in 75%


B. DCX (doublecortin) on the X B. Family history is positive in <20%
chromosome C. Inheritance is likely polygenic
C. FLNA (filamin A) on the X chromosome D. Mutations in myoclonin1/EFHC1 have
D. TSC2 (tuberin) on chromosome 16 been identified in 50%
E. WD 40 E. All of the above

24. Which of the following would NOT be an 28. A 65-year-old male is referred for the
appropriate treatment for this patient (in evaluation of frequent nocturnal stereotyped
question 23), if started within the first events of brief violent motor activity. EEG
4 days of onset? recording during these events is normal. His
brain MRI is also normal. Which neuro-
A. Vigabatrin logical condition is likely associated with
B. Carbamazepine this sleep disorder?
C. Ketogenic diet
D. Oral corticosteroids A. Alzheimer’s dementia
E. ACTH B. Parkinson’s disease
Multiple Choice Questions for Part III 191

C. Multiple sclerosis 33. A 46-year-old woman has been suffering of


D. Pompe’s disease frequent complicated attacks of migraines
E. Tuberous sclerosis since her early teens. She has no history of
epilepsy. If she undergoes an EEG, the
29. Pallister-Hall syndrome (GLI3 gene muta- tracing may show the following:
tion) is associated with which of the
following? A. Normal background
B. Generalized slowing
A. Autosomal dominant temporal lobe C. spikes
epilepsy D. Enhanced photic drive
B. hypothalamic hamartoma E. All of the above
C. Juvenile myoclonic epilepsy
D. Electrical status epilepticus during 34. A 55-year-old man presents with a
sleep one-month history of progressive cognitive
E. None of the above decline and episodes of unresponsiveness
(which are diagnosed as nonconvulsive sei-
30. Glucose transporter type I deficiency syn- zures by video-EEG monitoring). Despite
drome (Glut1 DS) is associated with which cessation of seizures after treatment with
of the following? four anticonvulsants, he becomes comatose.
On examination, he is noted to have myo-
A. Infantile onset seizures clonic jerks. Laboratory studies reveal
B. Microcephaly hyponatremia. Which serum antibody is
C. Ataxia most likely to be positive in his condition?
D. Mixed-type seizures
E. All of the above A. N-methyl-D-aspartate receptor
(NMDA-R) antibody
31. Which of the following reduces the likeli- B. LGI1 antibody (voltage-gated potas-
hood of remission in psychogenic seizures? sium channel [VGKC] complex)
C. Glutamic acid decarboxylase (GAD)
A. Normal neuropsychological testing antibody
B. Longer duration of illness D. Anti-Amphiphysin antibody
C. Frequent events E. Anti-cardiolipin antibody
D. Willingness to seek psychiatric help
E. All of the above 35. Which of the following statements is false
regarding early seizures after traumatic
32. In primary brain tumors, which of the fol- brain injury?
lowing factors is least likely to have higher
association with seizures? A. Early seizures occur within the first
week after injury
A. Adult onset of brain tumor B. Early seizures may be associated with
B. Tumor situated near the hippocampus increased occurrence of late
C. Tumor situated near primary motor seizures/epilepsy
cortex C. Early status epilepticus may be associ-
D. High-grade tumor pathology ated with increased occurrence of late
E. All the above have similar seizure seizures/epilepsy
association
192 Multiple Choice Questions for Part III

D. Early seizures should be treated in D. Medications are effective in about half


order to lower the risk of late of the cases
seizures/epilepsy E. Dietary therapies are ineffective.
E. All of the above are correct
39. A 46-year-old woman presents with
36. A 5-year-old girl is seen in first seizure new-onset episodes of cold sweat, pallor,
clinic after a nocturnal episode of vomiting, and heart racing lasting for few minutes.
pallor, behavioral irritability, then eye Occasionally, these episodes are associated
deviation to the right and unresponsiveness, with loss of consciousness. She had a nor-
followed by a generalized tonic clonic sei- mal neurological examination and negative
zure. The total duration of the episode was prior routine EKG, EEG, and brain imaging.
5 min. Her development has been normal She was started on lacosamide with partial
and there is no family history of seizures. relief. Which of the following test is most
Which of the following is true? likely to provide a definitive diagnosis?

A. This epilepsy syndrome has a poor A. Inpatient video-EEG monitoring


prognosis for seizure control. B. Tilt-table testing
B. Treatment typically includes corticos- C. Cardiac Holter monitoring
teroids and vigabatrin. D. Lumbar puncture
C. Most patients with this disorder have E. 24-hour urine metanephrine
major structural brain malformations.
D. Trauma is a frequent antecedent of this 40. An 8-year-old boy has epilepsy and partial
epilepsy syndrome. blindness. He has a history of events where
E. The EEG likely will show paroxysms he feels palpitations and lightheadedness,
of occipital spikes. falls down, and passes out for 30–45 s.
An EEG during the event reveals general-
37. A 2-year-old boy presented with occasional ized slow activity. MRI shows clefts in the
episodes of generalized stiffness and tonic cortical tissue and the absence of the septum
spasms since birth. His development, neu- pellucidum. What is the most appropriate
rological examination, and EEG are normal. test to order in this patient?
Which of the following is true about this
boy’s condition? A. Continuous video-EEG monitoring
B. Tilt-table testing
A. Inhibitory glycine receptor mutation C. 24-hour cardiac rhythm monitoring
B. LGL1 gene mutation D. Serum cortisol level
C. Calcium T-channel disfiguration E. Sedimentation rate
D. Leucine antibodies
E. Anti-Gad antibodies 41. A 43-year-old woman is being evaluated for
intermittent convulsions. Which feature
38. Glucose transporter type I deficiency syn- would be more suggestive of epileptic sei-
drome (Glut1 DS) is associated with which zures as opposed to psychogenic nonepilep-
of the following except? tic seizures?

A. Dysarthria A. Tongue biting at the side


B. Episodic weakness B. Opisthotonic posturing
C. CSF glucose <40 mg/dl C. Side-to-side head movements
Multiple Choice Questions for Part III 193

D. Postictal shallow and rapid breathing A. Prominent motor features


E. Ictal eye closure B. Male gender
C. Longer duration of illness
42. A 4-year-old boy has severe oral and facial D. Older age at diagnosis
dysfunction since birth, associated with E. Lower educational achievement
difficulty feeding. He has intractable partial
epilepsy with bihemispheric epileptiform 46. A patient with a brain tumor has never had a
discharges on EEG. His MRI shows seizure and is requesting anticonvulsant pro-
excessive, small convolutions in the bilat- phylaxis when being admitted for her tumor
eral frontal and temporal opercular regions. resection. The most appropriate response is:
What is his diagnosis?
A. Anticonvulsant prophylaxis is not
A. Polymicrogyria recommended before the first seizure,
B. Lissencephaly and it is also not recommended peri-
C. Schizencephaly operatively.
D. Pachygyria B. Anticonvulsant prophylaxis is not rec-
E. Septum pellucidum ommended before the first seizure, but
it is recommended for the first six
43. Anti-NMDA receptor encephalitis is asso- months postoperatively.
ciated with which of the following except? C. Anticonvulsant prophylaxis is not rec-
ommended before the first seizure, but
A. A prodrome of viral-like illness it is recommended for the first one
B. Ovarian teratoma week postoperatively.
C. Mildly elevated protein in CSF D. Anticonvulsant prophylaxis is recom-
D. CSF rarely shows signs of inflammation mended before the first seizure, and it
E. Anti-NMDA receptor antibodies in also should be continued indefinitely
both serum and CSF postoperatively.
E. Steroids are the anticonvulsant of
44. Propofol infusion syndrome is more choice
common:
47. In a child presenting with seizures, CSF
A. Children analysis can be helpful if which of the fol-
B. Critically-ill patients lowing is suspected?
C. Concomitant use of catecholamines
D. Prolonged infusions A. Nonketotic hyperglycinemia
E. All of the above B. Alpers’ syndrome
C. Pyridoxamine 5′-phosphate oxidase
45. Which of the following characteristics is (PNPO) deficiency
most associated with a good outcome in D. Primary cerebral folate deficiency
psychogenic nonepileptic seizures? E. All of the above
194 Multiple Choice Questions for Part III

48. A 4-month-old full-term baby has periodic 2. (C). For the diagnosis of nonepileptic psy-
episodes of arching of the back and stiff- chogenic seizures, several clinical features
ening every 2–3 h. His neurological exam- are helpful in confirming this diagnosis.
ination and EEG between attacks is normal. Highly predictive features include eye clo-
He has not been gaining weight at the sure, out of phase or discontinuous motor
expected pace and has occasional vomiting. activity, and forward pelvic thrusting, among
The treatment of choice is: others. Confusion is not helpful in distin-
guishing epileptic from nonepileptic seizures.
A. Pyridoxine 3. (B). Most cases of absence epilepsies of
B. Vigabatrin childhood have complex inheritance. In one
C. ACTH study, only 12% of screened patients with
D. Omeprazole early-onset absence epilepsy had mutations
E. Observe in SLC2A1, the gene encoding the GLUT1
glucose transporter. Some mutations are
49. A 35-year-old woman without a history of familial and others are de novo.
epilepsy is hospitalized with confusion and 4. (D). Familial lateral temporal lobe epilepsy
psychosis. EEG reveals frequent partial with auditory features is autosomal dominant
seizures, resistant to four anticonvulsants. with seizure onset in adolescence or early
She is noted to have choreiform movements adult life. Typical aura is a simple auditory
on examination. Which test is most likely to hallucination with the evolution to dyscog-
provide a specific diagnosis? nitive and grand mal seizures. It is associated
with mutations in the leucine-rich glioma-
A. PET-CT of abdomen and pelvis inactivated (LGI) gene on chromosome 10q.
B. LGI1 antibody (voltage-gated potas- 5. (A). This is a classical description of an
sium channel complex) infant with Dravet syndrome, also known as
C. MRI of the brain with contrast severe myoclonic epilepsy of infancy
D. Continuous video-EEG monitoring (SMEI). More than 70% of patients have
E. Lumbar puncture mutations in SCN1A, although girls with
negative testing should undergo analysis of
PCDH19. Seizures are very difficult to con-
Answers trol. Many recommend avoidance of seizure
medicines that act on sodium channels.
1. (B). Dravet syndrome presents in infancy Nonconvulsive status epilepticus is not
with convulsive seizures often triggered uncommon. Deterioration in gait patterns
by fever. It also includes other seizure types has been noted in studies of older patients.
such as myoclonic, atonic, focal dyscognitive Vaccinations (especially those containing
seizures, and nonconvulsive status epilepti- whole-cell pertussis) have been reported as a
cus. Development slows in the second year. trigger in case series.
Intellectual disability and intractable seizures 6. (E). Mutations in CHRNA2, CHRNA4, and
ensue. In Dravet, *75% of subjects have CHRNB2, genes encoding the nicotinic
mutations in the voltage-gated sodium acetylcholine receptor, are seen in patients
channel gene, SCN1A (95% of which are de with autosomal dominant nocturnal frontal
novo). Sodium channel agents, such as lam- lobe epilepsy. Please refer to Table 12.1 for
otrigine, tend to worsen the seizures. In 10% other details.
of familial GEFS+, there are mutations in the 7. (D). The risk of Sudden Unexpected Death
SCN1A gene; others have GABRG2 muta- in Epilepsy (SUDEP) in patients with
tions affecting the GABA channel. drug-resistant epilepsy is about 1/100, while
Multiple Choice Questions for Part III 195

the risk of SUDEP in community-based can be asymmetric. Treatment options


epilepsy patients is 1/1000. include valproate and ethosuximide.
8. (D). CAE is more frequent in girls, and in 12. (B). Infants younger than five months of age
some cases mutations in GABA receptor presenting with febrile seizures should
(GABRG2, GABRA1), calcium channels, undergo a thorough infectious work-up
and non-ion channel proteins have been including a lumbar puncture to rule out
identified. A double-blinded, randomized, CNS infections. chest Xray, urine testing
comparative clinical trial compared etho- and
suximide, lamotrigine, and valproate in CAE 13. (B). Please refer to Table 13.1.
and found that ethosuximide provided the 14. (D). Unlike adult stroke, pediatric stroke
best combination of seizure control and more commonly presents with seizure as the
fewest attentional side effects, making it the presenting symptom and has higher risk for
optimal initial monotherapy in CAE. Etho- developing epilepsy. In fact, about 80% of
suximide provided better seizure control neonatal strokes present with seizure (as
compared to lamotrigine and fewer atten- opposed to with focal neurological deficits).
tional effects compared to valproic acid. Similar to post-traumatic epilepsy, even one
Although CAE is often perceived as “be- late unprovoked post-stroke seizure has a
nign,” many children with CAE have cog- high recurrence rate and is tantamount to
nitive deficits and long-term psychosocial post-stroke epilepsy. Predictors of
problems. post-stroke epilepsy include cortical stroke
9. (E). When evaluating convulsive attacks, location, presence of hemorrhage, and
several clinical features favor the diagnosis examination severity, but not presence of
of psychogenic nonepileptic seizures. Most focal slowing (a nonspecific finding).
studied features include eye closure, 15. (A). In 15–32% of subjects with Doose
side-to-side head and body movements, syndrome, there is strong family history of
asynchronous motor activity, and postictal idiopathic epilepsy. GTC is usually the first
irregular shallow breathing. seizure and is seen in 75–95% of patients.
10. (C). PME is characterized by myoclonic and Neuroimaging is typically normal. Glut-1
grand mal seizures, as well as progressive deficiency is identified in about 5% of chil-
cognitive dysfunction deterioration and dren with Doose syndrome. Treatment
ataxia. Unverricht–Lundborg disease is options include valproate, lamotrigine,
associated with mutations of the cystatin B ethosuximide, topiramate, levetiracetam, and
gene, Lafora body disease with mutations of the ketogenic diet.
EPM 2A (coding for laforin) and EPM2B 16. (B). The age and description of the seizures
(coding for malin) genes, and sialidosis with is consistent with benign epilepsy with cen-
mutations of NEU1 gene that codes for trotemporal spikes (BECTS) also known as
neuraminidase. A number of gene mutations benign rolandic epilepsy. Onset of seizures
have been identified in neuronal ceroid occurs between the ages of 4–10 years.
lipofuscinosis of which Jansky–Biel- Clinically, seizures are common during sleep
schowsky disease and Batten disease are (80% of times) consisting of unilateral tonic
subtypes. or clonic activity of the face and excessive
11. The majority (70%) of children with epi- salivation often followed by secondarily
lepsy with myoclonic absences are boys. generalization.
Neuroimaging abnormality, mainly some 17. (C). Contrary to all listed conditions, Siali-
degree of diffuse atrophy, is seen in 17% of dosis is inherited in an autosomal recessive
patients. Frequent seizures may occur on pattern.
awakening. Tonic contractions, particularly 18. (B). Prophylaxis is recommended only for
elevation of the arms, are often noted, which the first week after severe traumatic brain
196 Multiple Choice Questions for Part III

injury. Recurrence risk of seizures after one and thalamic lesions. Family history of sei-
late post-traumatic seizure is high; therefore, zure is reported in 10–15%.
this is essentially considered to be epilepsy, 23. (B). LIS1 gene mutation causes the autoso-
and treatment should be strongly considered. mal dominant form of lissencephaly; the
Early seizures in combination with moderate DCX gene mutation has an X-linked inher-
or severe head trauma increase risk of itance pattern which may present with
developing epilepsy, but mild head trauma in (milder) subcortical band heterotopia phe-
combination with abnormal EEG is not notype in girls. FLNA is related to periven-
diagnostic of epilepsy, particularly if tricular nodular heterotopia, and TSC2 is
the clinical events are not convincingly related to tuberous sclerosis complex.
epileptic by history. Video-EEG monitoring 24. (B). A recent evidence-based guideline out-
should be considered in such cases, as lined the evidence for different treatments for
non-epileptic events are in the differential infantile spasms (Go et al., Neurol
diagnosis. 2012;78:1974). Of all the treatments listed,
19. (E). The peak age of onset of LGS is 3– carbamazepine would not be expected to lead
5 years, and infantile spasms precede LGS to resolution of the spasms. Earlier onset of
in 10–25%. EEG features of slow back- treatment is associated with better outcomes.
ground with generalized slow spike-wave 25. (B). The history of stereotyped nocturnal
discharges at 1.5–2 Hz, multifocal dis- odd behavior occurring during the first half
charges, and generalized fast activity at 10– of the night, and amnesia of the events is
25 Hz in sleep. LGS results from suggestive of confusional arousal disorder.
structural/metabolic causes in 70–78%. Sleep deprivation, excessive stress, or alco-
20. (B). Common clinical signs and symptoms hol intake can trigger these episodes. EEG
of Whipple’s disease include diarrhea, recording during these events may be normal
steatorrhea, abdominal pain, weight loss, and or show generalized slowing of background
migratory arthropathy. In about 10–30% of activity.
patients, CNS symptoms of dementia, 26. (B). Of the primary brain tumors, DNET has
movement disorders or seizures may occur. the highest association with seizures (nearly
D-Xylose test can help with confirming 100%), followed by gangliogliomas and
intestinal malabsorption syndrome. Overall, astrocytomas. Higher grade or fast-growing
duodenal biopsy is considered the gold tumors such as glioblastoma or PCNSL have
standard for diagnosis. a lower association with seizures.
21. (C). Both tuberous sclerosis complex 27. In JME, photosensitivity is noted in about
(TSC) and periventricular nodular hetero- 30% of patients. Family history of epilepsy
topia (PVNH) can present with seizures and is reported in 40–50%. Inheritance is
periventricular lesions. The lesions of PVNH unclear, likely polygenic, but both autoso-
are typically not calcified, are homogenous, mal dominant and recessive inheritance have
are gray matter (by definition), and may be been reported. Gene mutations identified in
bilateral (particularly in familial cases, which some families include GABRA1 (GABA-A
are usually X-linked). receptor gene on chromosome 5q34),
22. (E). CSWS manifests as global regression in CLCN2 (chloride channel 2 gene on chro-
cognition and behavior. The majority of mosome 3q26), and myoclonin1/EFHC1
patients have seizures. CSWS is sometimes (EH-hand motif protein on chromosome
associated with identifiable pathology, e.g., 6p12; found in 9% of JME).
neuronal migrational disorders, polymicro- 28. (B). The clinical description is most consis-
gyria, shunted hydrocephalus, porencephaly, tent with REM behavior disorder. This sleep
Multiple Choice Questions for Part III 197

disorder has been closely linked to the which LGI1 is a common target. NMDA-R
development of Parkinson’s disease. antibody-mediated disease often presents
29. (B). Hypothalamic hamartoma is usually with associated movement disorder and
sporadic but is rarely associated with auto- ovarian teratoma; GAD antibody-mediated
somal dominant Pallister–Hall syndrome disease may also present with type I diabetes
(GLI3 gene mutation). mellitus or stiff-person syndrome; anti-
30. (E). Glut1 is found in microvessels and Amphiphysin antibody-mediated disease
astrocytes, and it facilitates glucose transport often presents with neuropathy and can be
into the brain. Initial patients with Glut1 DS associated with small cell lung cancer or
had refractory epilepsy (infantile onset), breast adenocarcinoma.
encephalopathy, acquired microcephaly, 35. (D). There is no evidence that treating early
cognitive impairment, and motor abnormal- post-traumatic seizures lowers the risk of
ities (spasticity, ataxia, and dystonia). Clin- developing epilepsy/late seizures, although
ical features include seizures, movement/gait treating and preventing early seizures may
disorders, and cognitive/behavioral distur- help with acute recovery, as early seizures
bances. Seizure starts in infancy (average could lead to further brain injury. Although
age of onset 8 months). GTC, absence, multivariate analyses have shown that early
myoclonic, and focal seizures may occur. seizures do not themselves increase risk of
Only 8% will be seizure free with medica- late seizures, early seizures are associated
tions alone. with development of late seizures due to
31. (B). Longer duration of illness, abnormal confounding factors that increase risk for
neuropsychological testing, and presence of both early and late seizures (brain contusion,
psychiatric comorbidities are the predictors subdural hematoma, and loss of conscious-
of negative outcome in psychogenic ness or amnesia > 24 h).
nonepileptic seizures. 36. (E). The patient has benign occipital epilepsy
32. (D). Tumors in adults, tumors near sensitive or idiopathic childhood occipital epilepsy
or irritable structures and networks (limbic (most likely Panayiotopoulos syndrome).
pathways, motor cortex), and tumors that are The symptoms and signs described in the
low grade (e.g., those without central question are classic for this diagnosis.
necrosis) are more likely to be associated The EEG classically shows occipital spikes
with seizures. that are best brought out by darkness but
33. (E). Patients with history of migraines may other types of epileptiform activity have been
have normal EEGs or nonspecific EEG chan- noted, as well. Because the frequency of
ges including generalized slowing (or focal in seizures is low, many patients do not need to
hemiplegic migraines), loss of normal alpha be treated with antiseizure medicines.
rhythm, enhanced photic drive, and attenuated 37. (A). Age and clinical description are consis-
beta activity. Occipital spikes can also be seen tent with the diagnosis of hyperekplexia or
in basilar migraines. In the absence of suspi- stiff baby syndrome. It consists of a triad of
cion for epilepsy, there is no justification in generalized stiffness, nocturnal myoclonus,
obtaining EEGs for migraineurs. and tonic spasms that are usually triggered by
34. (B). The presence of rapid cognitive decline auditory or tactile stimuli. It is caused by
and myoclonus could suggest Creutzfeldt– gene mutations affecting the inhibitory gly-
Jacob disease, but new-onset intractable cine receptor (GLRA1 and GLRB). The
seizures more strongly suggest autoimmune treatment of choice is clonazepam.
epilepsy. Hyponatremia is typical of VGKC- 38. Glut1 deficiency is associated with spastic or
complex antibody-mediated disease, of ataxic gait, dystonia, chorea, tremor,
198 Multiple Choice Questions for Part III

exertional dyskinesia, and episodic weak- 41. (A). During convulsions, tongue biting
ness (triggered by fasting and exercise). (especially at the side of tongue) is more
Cognitive and behavioral disturbances likely to occur with epileptic seizures. The
including learning/intellectual disability, remaining features are highly suggestive of
language deficit (predominantly expressive), psychogenic nonepileptic seizures.
and dysarthria are common. The diagnosis of 42. (A). This is a classic description of bilateral
Glut1 DS is based on hypoglycorrhachia perisylvian polymicrogyria syndrome; the
with normoglycemia and low-to-normal CSF perisylvian localization explains the symp-
lactate. CSF glucose is <40 mg/dl toms, EEG findings, and imaging abnormal-
(2.2 mmol/L); in all reported cases, CSF ities. Schizencephaly can be seen with
glucose is <60 mg/dl (3.3 mmol/L). CSF: polymicrogyria but is not typically part of this
blood glucose ratio is <0.4 (fasting state). syndrome; the MRI would show parenchy-
Gene mutation (SLC2A1 gene on chromo- mal clefts in schizencephaly. Pachygyria and
some 1p35-31.3) is identified in about 90% lissencephaly are seen in combination with
of patients. 90% is de novo mutation; *10% each other, and the MRI description is
AD; rare AR. Erythrocyte glucose uptake inconsistent with these two options.
assay may be used to confirm the diagnosis. 43. (D). Anti-NMDA receptor encephalitis may
The ketogenic diet is effective. Only 8% will present with a prodrome of viral-like syn-
be seizure free with medications alone. drome precedes psychiatric and behavioral
39. (A). The initial symptoms are suggestive of problems. The symptoms progress to altered
presyncope but the associated change in mental state, seizures, dyskinesia, choreoa-
awareness and the partial relief on lacosa- thetosis, and breathing and autonomic insta-
mide are both suggestive of epileptic sei- bility. CSF shows signs of inflammation in
zures. Normal routine EEG and brain over 90% of patients. Lymphocytic pre-
imaging do not exclude the diagnosis of dominant pleocytosis, mildly elevated pro-
epilepsy. An inpatient video-EEG monitor- tein, and oligoclonal bands may be present.
ing with lacosamide withdrawal is the gold The diagnosis of anti-NMDA receptor
standard test, in addition to full cardiac encephalitis is made by the demonstration of
telemetry will be valuable too. anti-NMDA receptor antibodies in serum and
40. (D). The constellation of partial blindness, CSF. The level of antibody in CSF correlates
epilepsy, schizencephaly, and an absent with symptom and outcome. Anti-NMDA
septum pellucidum suggests septo-optic receptor encephalitis may be associated with
dysplasia (de Morsier syndrome). Optic ovarian teratoma in females.
nerve hypoplasia may also be seen. This is 44. (E). Propofol infusion syndrome is a rare
typically associated with hypopituitarism, syndrome which affects patients undergoing
and the events described are probably syn- long-term treatment with high doses of
cope and not seizures. Therefore, a serum propofol. This can lead to cardiac failure,
cortisol level is most likely to give a specific metabolic acidosis, rhabdomyolysis, and
diagnosis. Although the remaining answer kidney failure, and is often fatal. The syn-
choices may suggest a cause of syncope, drome occurs more commonly in children,
screening for pituitary dysfunction should and critically ill patients receiving glucocor-
not be overlooked. ticoids and catecholamines.
Multiple Choice Questions for Part III 199

45. (B). Several characteristics were shown to be Pyridoxamine 5′-phosphate oxidase (PNPO)
associated with good outcomes (event reso- deficiency, caused by mutation of PNPO on
lution) in psychogenic nonepileptic seizures. chromosome 17q21.32 and inherited as
These include male gender, younger age at autosomal recessive trait, results in the
onset, short duration of illness, higher edu- decreased synthesis of PLP and clinically
cational and socioeconomic status, and manifests with severe refractory neonatal
minimal motor involvement during events. epileptic encephalopathy with prenatal sei-
46. (C). The American Academy of Neurology zures. EEG may show burst-suppression
guidelines recommend anticonvulsant treat- pattern. CSF neurotransmitter metabolite
ment only after a seizure has occurred. Pro- levels are abnormal: low HVA and 5-HIAA
phylaxis in brain tumor patients without a concentration (degradation products of
history of seizures is only recommended for dopamine and serotonin, respectively), and
one week post-operatively; longer-term high L-dopa (precursor of dopamine),
prophylaxis in patients (regardless of oper- 5-hydoxytryptophan (precursor of sero-
ative history) has not been shown to prevent tonin), and 3-ortho-methyldopa concentra-
the first seizure. tions. CSF PLP level is reduced. Patients
47. (E). Glycine encephalopathy (also known as respond to pyridoxal phosphate but not to
nonketotic hyperglycinemia) is an autosomal pyridoxine.Primary cerebral folate deficiency
recessive condition caused by defect in the results from defect in transport of folate
glycine cleavage system (intramitochondrial across blood-CSF/blood–brain barrier. There
enzyme complex) resulting in excess glycine are 2 possible causes for the transport defect:
in all tissues, especially CNS. Glycine is both blocking auto-antibodies against folate
excitatory (cortex) and inhibitory (brainstem receptors of the choroid plexus and folate
and spinal cord) neurotransmitter in the CNS. receptor gene mutation (FOLR1). Clinically
Infants presented with neonatal hypotonia, symptoms start at 4–6 months of age with
seizures (often myoclonic), encephalopathy irritability and disturbed sleep. Slow head
or coma, and hiccups. The diagnosis of gly- growth, ataxia, choreoathetosis, and ballistic
cine encephalopathy is confirmed by elevated movements are additional features. By
glycine in plasma and CSF, and CSF to 2 years of age, mental retardation, ataxia,
plasma glycine ratio >0.08. Seizures occur in involuntary movements, and spastic diplegia
35–60% of patients with mitochondrial dis- will ensue. Later visual disturbance and
order. Examples include MERRF (myoclonic progressive sensorineural hearing loss occur.
epilepsy and red ragged fibers), Alpers syn- About a third of patients have seizures which
drome [POLG1 (polymerase gamma) muta- includes myoclonic-atonic, absence, and
tion: explosive focal epilepsy, EPC or status GTC seizures.
epilepticus, and predominant occipital dis- 48. (D). The clinical description is most consis-
charges on EEG may suggest the diagnosis], tent with the diagnosis of Sandifer syndrome
and MELAS (mitochondrial myopathy or infantile gastroesophageal reflux disease.
encephalopathy, lactic acidosis and stroke- Since he has been losing weight and vom-
like episodes). Elevated plasma and/or CSF iting, a trial of antacids such is omeprazole is
lactate suggests mitochondrial disorder but needed. This condition usually resolves after
normal lactate does not rule out the diagno- the age of 6 months.
sis. Elevated alanine level may indicate 49. (A). The patient most likely has anti-NMDA
mitochondrial disorder. Pyridoxal 5′ phos- receptor encephalitis, characterized by
phate (PLP) is an essential cofactor in the new-onset refractory seizures, prominent
synthesis of dopamine and serotonin. psychiatric features, and a movement
200 Multiple Choice Questions for Part III

disorder. This can be associated with ovarian Anti-LGI1 (VGKC-complex) encephalitis


teratoma. Testing for malignancy (PET-CT can have a similar presentation but may have
or ovarian ultrasound) is indicated. MRI and hyponatremia, myoclonus, and faciobrachial
EEG can be abnormal but are not likely to dystonic seizures and in most cases is not
reveal a specific diagnosis or etiology. paraneoplastic.
Part IV
Management of Epilepsy
Principles of Epilepsy Diagnosis
and Management 15
Hasan H. Sonmezturk, Mohamad Z. Koubeissi
and Nabil J. Azar

Epidemiology First Seizure

Epilepsy is a fairly common condition compared i. Provoked seizures:


to other neurological disorders. Approximately
3.1 % of the US population (about 9 millions) This term is often misinterpreted as all seizures
suffers from epilepsy. About 5 % of the US are provoked. However, in epilepsy, the term
population (about 15 millions) will have a sei- “provoked seizure” often implies the presence of a
zure at some time in their lives and only just a bit simple fixable cause for the seizure. The term
more than half of those will progress into epi- “provoked seizure” is mostly used for a selected
lepsy. Incidence rate for epilepsy is about 44 per group of seizures. These are the seizures triggered
100,000 in the US. This rate is 61 for first by some reversible process, such as electrolyte
unprovoked seizures, 39 for acute symptomatic abnormality, severe sleep deprivation, or a medi-
seizures, and 100 for all seizures including sei- cation adverse effect. A provoked seizure is pre-
zures from epileptic patients Hauser et al. [1]. ventable after reversing the provocation. In contrast,
The incidence of epilepsy tends to be higher in intractable epilepsies are often caused either by a
males and is highest at or after age 75. The most cortical malformation, hippocampal sclerosis, or
common known etiology is cerebrovascular dis- some other structural abnormalities such as acute
ease at 11 %, followed by neurologic deficits cerebral insults. These are called symptomatic sei-
from birth, mental retardation, or cerebral palsy zures (related to a structural abnormality) rather than
at 8 % Hauser et al. [1]. provoked seizures. Even when we cannot identify a
clear etiology, we assume there is always a reason
for seizures including genetic causes.

ii. First unprovoked seizure:

H.H. Sonmezturk This is defined as a seizure with no clearly


Vanderbilt Department of Neurology, Vanderbilt identified provocation or etiology at the initial
University, Nashville, TN, USA
work up.
M.Z. Koubeissi  N.J. Azar (&)
Department of Neurology, Vanderbilt University
iii. Appropriate work up for the first
Medical Center, 1161 21st Ave. South a-0118
Medical Center North, Nashville, unprovoked seizure:
TN 37232-2551, USA
e-mail: nabil.azar@vanderbilt.edu There is level B evidence supporting the use of
M.Z. Koubeissi EEG and brain imaging mainly MRI for the initial
e-mail: koubeissi@gmail.com

© Springer Science+Business Media LLC 2017 203


M.Z. Koubeissi and N.J. Azar (eds.), Epilepsy Board Review,
DOI 10.1007/978-1-4939-6774-2_15
204 H.H. Sonmezturk et al.

work-up of a first unprovoked seizure. MRI is Table 15.1 Seizure recurrence rates
usually preferred over CT scans because of its Recurrence
higher resolution for soft tissue and malforma- rate (%)
tions. In one pooled analysis of 928 subjects with After first unprovoked seizure 40
brain MRI after first seizure, 15 % were found to After two unprovoked seizures 73
have brain MRI abnormalities. In the same study, After three unprovoked seizures 76
51 % of 1766 patients (pooled analysis) had an
After treating first unprovoked seizure 15
abnormal initial routine EEG. An additional 35 %
were identified to have abnormalities on the sec-
ond sleep-deprived EEG. An abnormal EEG
predicts a higher recurrence rate, and a normal FIRST Trial (Italy) in 1993 found a 51 %
EEG predicts a lower recurrence rate but does not recurrence by two years.
rule out epilepsy. On average, about 50 % of UK (MESS Trial) in 2005 found 39 %
individuals clinically diagnosed with a seizure recurrence by two years.
have a normal first routine EEG. Laboratory The recurrence rates were higher years after a
testing, such as complete blood counts, blood remote symptomatic and after having two or
glucose, and electrolyte panels (particularly more seizures (Tables 15.1 and 15.2). Treatment
sodium), cerebrospinal fluid analysis, and toxi- after the first seizure decreases the recurrence rate
cology screening may be helpful as determined by 30–60 % but does not eliminate the risk
by the specific clinical scenario based on history, completely (Table 15.3).
physical, and neurologic examination, but there
v. Seizure recurrence after the first unpro-
are insufficient data to support or refute recom-
voked seizure according to etiologic factors
mending any of these tests for the routine evalu-
and EEG findings Berg and Shinnar [4]:
ation of adults presenting with an apparent first
unprovoked seizure (Krumholz et al.).
Etiology
In one study by King et al., brain MRI
Idiopathic 32 %
abnormalities were seen in 14 % (38 of 277) of
Symptomatic 57 %
patients with a first unprovoked seizure. Tumors
EEG
were the most commonly identified etiology for
Normal 27 %
seizures followed by developmental anomalies
Epileptiform 58 %
then hippocampal pathologies and vascular
Etiology + EEG
malformations King et al. [2]. In another similar
Idiopathic + Normal 24 %
study done 15 years later, 23 % (177 of 764) of
Symptomatic + Abnormal EEG 65 %
patients had MRI abnormalities. This time
Focal onset seizures had higher risk of
developmental anomalies were equally common
recurrence than generalized-onset seizures.
as tumors. The higher yield was attributed to the
A first seizure occurring during sleep also had
use of 3 T MRIs as opposed to 1.5 T in the
higher risk for recurrence than a seizure occur-
original study Hakami et al. [3].
ring during wakefulness. The other risk factors
for seizure recurrence included status epilepticus
iv. Seizure recurrence after first unprovoked
as first seizure, abnormal interictal neurological
seizure:
examination, abnormal brain imaging, multiple
The recurrence rate after the first unprovoked or clustered seizures at onset, and strong family
seizure averages around 40 % within the first two history of seizures.
years. This was concluded based on three large Treatment decision after first seizure
pivotal studies: This subject has always been controversial as
Meta-analysis (USA) in 1991 which found a scholars usually differ in their practices of treat-
36 % recurrence by the two years. ing or not treating the first seizure. In general,
15 Principles of Epilepsy Diagnosis and Management 205

Table 15.2 Risk of Acute (%) Remote (%)


seizure recurrence after first
symptomatic seizure Stroke 33 71.5
(Hesdorffer et al.) TBI 13.4 46.6
CNS Infection 16.63 63.5
Acute: within one week, remote: after one week

Table 15.3 Seizure Seizure rate


recurrence rates at 6 and
24 months, after treatment Untreated (%) Treated (%)
or no treatment Italian FIRST study 1993 (n = 397) 6 months 41 17
24 months 51 25
UK MESS study 2005 (n = 1443) 6 months 26 18
24 months 39 32

about 60 % of the patients who had their first epilepsies. On the contrary, some narrow spec-
seizure will never have another one. In this case, trum AEDs could cause worsening of seizures if
it is appropriate to give the facts to the patient given to a patient with generalized-onset seizures
and let them make their decisions. If the patient (see Table 15.4).
prefers not to be treated and understands the risk Mechanism of action (MOA) is another factor
of receiving no treatment, it is perfectly accept- that may influence the selection of an AED
able not to treat. Some patients will insist on (Table 15.5). However, the role of MOA is very
being treated since they do not want to risk limited, and the existing data do not support
another seizure. In this case, treatment is offered consideration of MOA as a major criterion in
for these patients provided that they understand choosing an AED. Many AEDs have multiple
the seizure recurrence risk is decreased but not MOAs, and some have unknown MOAs. This
necessarily eliminated with treatment. After an makes it even more difficult to make a judgment
acute or remote symptomatic (cerebral insult) based on MOA alone [5, 6]. According to a study
seizure in a clinically unstable patient, treatment by Deckers CL et al., there is some evidence
is usually indicated without waiting for a second showing that AED polytherapy based on MOA
seizure. Selecting an appropriate antiepileptic may enhance effectiveness. In particular, com-
(AED) is quite challenging task in most cases. bining a sodium channel blocker with a drug
This is due to the presence of multiple AED enhancing GABAergic inhibitor could be an
options and very little predictability of good advantageous combination. Combining two
response. In addition, efficacy and tolerability of GABA mimetic drugs or combining an AMPA
AEDs may strongly differ among individuals. antagonist with an NMDA antagonist may
There are several available factors that could enhance efficacy, but tolerability may be reduced
help us make an educated decision of what AED with this combination.
would be best to start with. The spectrum of a
particular AED (narrow or broad) is one of these
factors (Table 15.4). In general, narrow spectrum Other Factors Influencing AED
AEDs are reserved for seizures with known. Selection
When seizure classification is unclear, it is wiser
to initiate a broad spectrum agent AED. The • Seizure type/classification
reason for this approach is that most narrow • Epilepsy syndrome
spectrum AEDs will not benefit patients • Established drug efficacy for a particular sei-
with generalized-onset seizures or generalized zure type or epilepsy syndrome
206 H.H. Sonmezturk et al.

Table 15.4 Grouping of Broad spectrum AEDs Narrow spectrum AEDs


AEDs according to their
spectrum of activity Valproate Phenytoin*
Felbamate Ezogabine*
#
Phenobarbital Perampanel*
Lamotrigine Lacosamide*
Topiramate Carbamazepine*
Zonisamide Gabapentin*
Levetiracetam Pregabalin*
Benzodiazepines Tiagabine*
Primidone Oxcarbazepine*
Vigabatrin@
Rufinamide
Ethosuximide
Rufinamide is minimally effective against focal seizures, Ethosuximide is only approved
for absence seizures, * = Spectrum not yet fully identified or mixed, * = May exacerbate
some generalized seizures such as myoclonus and absence, @ = Considered narrow
spectrum but exceptionally useful in infantile spasms, # = May trigger absence seizures
or worsen Lennox-Gastaut syndrome or myoclonic epilepsies

• Safety were no new AEDs during the period of 1978


• Tolerability profile (approval of VPA) until 1993 (approval of Fel-
• Co-morbidities (weight, cognition, psychi- bamate). The AEDs which came after 1993 are
atric, other) considered to be newer AEDs. The newer AEDs
• Side-effect profile have shown comparable efficacy to older ones.
• Metabolic status (renal, hepatic) However, newer AEDS offer numerous advan-
• Drug-drug interactions tages over the older AEDs. These include minimal
• Drug formulations drug-drug interactions, better pharmacokinetic
• Ease of administration/titration profiles, and minimal or no long-term side effects.
• Pregnancy or contraception They are often associated with less teratogenicity
• Prior allergies and cross reactivity and less immediate side effects. Most importantly,
• Cost they usually do not need routine blood level
• Availability monitoring because of low protein binding and
negligible hepatic induction or inhibition proper-
Available AEDs: Currently, there are about ties. The availability of generic formulations of
25 FDA-approved AEDs in the USA. The graph some of the newer AEDs helped boost their use.
below lists AEDs in chronological order AED selection is still determined on a
(Fig. 18.1). Commonly used abbreviations, case-by-case basis but overall, the newer AEDs
brand names, and common side effects of current mostly replaced the older ones as first choice
AEDs are listed in Table 15.6. AEDs.

Older (Classic) Versus Newer AEDs FDA Indications

The classic AEDs include Phenobarbital, Pheny- Drug trials primarily aim to prove superiority
toin, Primidone, Ethosuximide, Benzodiazepines, over placebo, or no inferiority (conversion to
Carbamazepine, and Sodium Valproate. There monotherapy studies). Drug trials tend to use
15 Principles of Epilepsy Diagnosis and Management 207

Table 15.5 Mechanism of action of AEDs


AED Enhancement of Blockade of Blockade of Inhibition of Other
GABA-mediated sodium calcium glutamate
excitation channels channels
Benzodiazepines& + +*
Carbamazepine +
Ethosuximide + (T-type)
&
Phenobarbital + + +
Phenytoin +
Valproate + + + (T-type) +
Gabapentin + (L-type)$
Pregabalin + (L-type)$
Felbamate + + +
Lamotrigine + + (L-type)
Levetiracetam +@
Oxcarbazepine + + (L-type)
Tiagabine^ +
Topiramate* + + + (L-type) + +
Zonisamide* + + (L-type) + +
Lacosamide +#
Rufinamide +#
Vigabatrin% +
+
= a documented mechanism of action, * = at high benzodiazepine concentrations, % = GABA transaminase inhibitor,
#
= enhancers of slow inactivation of sodium channel, ~=Carbonic anhydrase inhibitors, ^ = GABA reuptake inhibitor,
@
= acts on synaptic vesicle 2A receptors, $ = acts on alpha-2-delta receptors, & = GABA-A receptor enhancers
(Phenobarbital causes sustained opening of GABA channels, benzodiazepines cause frequent opening of GABA
channels

Fig. 15.1 There are about


25 FDA-approved AEDs in
the USA. This graph lists
AEDs in chronological
order. Adapted from
Brodie MJ. Seizure 19
(2010) 650–655. Since this
graph was created three
additional AEDs got
approved by FDA
(Clobazam, Ezogabine,
Perampanel, and
eslicarbazepine)
208 H.H. Sonmezturk et al.

Table 15.6 Commonly used abbreviations, brand names, and most frequent side effects of AEDs
ABR Generic Brand Watch for
Older (Classic) AEDs
PHB Phenobarbital (Luminal) Sedation, rash, liver failure, aplastic anemia, osteopenia, CT d/o
PHT Phenytoin (Dilantin) SJS, blood dyscrasia, liver failure, gingival hyperplasia, hirsutism,
osteopenia
PRM Primidone (Mysoline) Metabolized to phenobarbital
ESX Ethosuximide (Zarontin) Stomach upset, abdominal pain/cramps
CLZ Clonazepam (Klonopin) Somnolence, lethargy, sexual dysfunction, tolerance is an issue
CZP Clorazepate (Tranxene) Somnolence, lethargy
DZM Diazepam (Valium) Not for prevention, can be used as abortive medication (watch for resp
suppression)
LRZ Lorazepam (Ativan) Not for prevention, can be used as abortive medication (resp
suppression)
CBZ Carbamazepine (Tegretol) Sedation, #WBC, #Na, bradycard, SJS, agranulo, hepatic fail,
pancreatitis
VPA Valproate (Depakote) Weight gain, tremor, #Plt, pancreatitis, liver failure, "ammonia, hair
loss
Newer AEDs
VGB Vigabatrin (Sabril) Permanent visual field deficit 30–40 %, reversible subcortical edema
ZNS Zonisamide (Zonegran) Cross reacts with sulfa, hypohidrosis, renal stones
LTG Lamotrigine (Lamictal) Nonsedating, insomnia, SJS (needs very slow titration), myoclonus
FBM Felbamate (Felbatol) Aplastic anemia, liver failure, weight loss, "PHT, VPA, PHB
GBP Gabapentin (Neurontin) Sedation, weight gain, myoclonus
TPM Topiramate (Topamax) Weight loss, severe cognitive slowing, dysesthesias, glaucoma, renal
stones
TGB Tiagabine (Gabitril) Sedation, cognitive slowing, worsens gen szs (myoclonic, absence)
OXC Oxcarbazepine (Trileptal) #Na in elderly, #OCP levels, sedation, rash
LEV Levetiracetam (Keppra) Irritability (hateful, anger issues, Vit B6 100 mg/day may help),
depression
PGB Pregabalin (Lyrica) Sedation, swelling in lower extremities, blurred vision
RFM Rufinamide (Banzel) Loss of appetite, aggravated seizures, status epilepticus
LCM Lacosamide (Vimpat) Dizziness, fatigue
ESL Eslicarbazepine (Aptiom) Nausea, Dizziness, diplopia, hyponatremia (1–2 %)
CLB Clobazam (Onfi) 1, 5-benzodiazepine, somnolence, lethargy, (less addiction potential)
EZG Ezogabine (Potiga) Urinary retention, tremors, bluish skin coloration
PER Perampanel (Fycompa) Ataxia, severe mood issues (hostility, homicidal ideation, aggression)

higher doses to reach higher efficacy. Tolerability


at these higher doses tends to be lower. Reference Studies on AED Selection
Table 15.7 summarizes studies and level of evi-
dence for each seizure type and epilepsy syn- In one study, Mattson et al. compared CBZ,
drome adopted from a special report by the PHB, PHT, or PRM in partial seizures and
International League Against Epilepsy. secondarily generalized tonic-clonic seizures
15 Principles of Epilepsy Diagnosis and Management 209

Table 15.7 Summary of studies and level of evidence for each seizure type and epilepsy syndrome
Seizure type or epilepsy syndrome Class I Class Class Level of efficacy and effectiveness
studies II III evidence
studies studies (in alphabetical order)
Adults with partial-onset seizures 4 1 34 Level A: CBZ, LEV, PHT, ZNS
Level B: VPA
Level C: GBP, LTG, OXC, PHB,
TPM, VGB
Level D: CZP, PRM
Children with partial-onset seizures 1 0 19 Level A: OXC
Level B: None
Level C: CBZ, PHB, PHT, TPM,
VPA, VGB
Level D: CLB, CZP, LTG, ZNS
Elderly adults with partial-onset 1 1 3 Level A: GBP, LTG
seizures Level B: None
Level C: CBZ
Level D: TPM, VPA
Adults with generalized-onset tonic– 0 0 27 Level A: None
clonic seizures Level B: None
Level C: CBZ, LTG, OXC, PHB,
PHT, TPM, VPA
Level D: GBP, LEV, VGB
Children with generalized-onset tonic– 0 0 14 Level A: None
clonic seizures Level B: None
Level C: CBZ, PB, PHT, TPM, VPA
Level D: OXC
Children with absence seizures 1 0 7 Level A: ESM, VPA
Level B: None
Level C: LTG
Level D: None
Benign epilepsy with centrotemporal 0 0 3 Level A: None
spikes (BECTS) Level B: None
Level C: CBZ, VPA
Level D: GBP, LEV, OXC, STM
Juvenile myoclonic epilepsy (JME) 0 0 1 Level A: None
Level B: None
Level C: None
Level D: TPM, VPA

(SGTCS). This was a multi-center double-blind (Mattson et al. 1992). Glauser et al. compared
trial for initiation monotherapy. A total of 622 ESX, VPA, and LTG monotherapies for the
adult epilepsy patients were recruited. Treatment treatment of childhood absence epilepsy in a
success was highest with CBZ or PHT, inter- double-blind randomized controlled trial. This
mediate with PHB, and lowest with PRM study was done on 453 children with newly
(Mattson et al. 1985). In another multi-center diagnosed absence epilepsy. ESX and VPA had
double-blind study, VPA was compared to CBZ similar efficacy which was better than LTG. ESX
for the treatment of complex partial seizures and had fewer attentional dysfunction than VPA.
SGTCS in adult patients. Both drugs were ESX was re-established as the first choice drug in
equally effective in controlling SGTCS. For the childhood absence epilepsy despite the
complex partial seizures, outcome measures availability of multiple newer AEDs. No
favored CBZ which also had less adverse effects prospective double-blind controlled study
210 H.H. Sonmezturk et al.

Table 15.8 Indication


Indication Try Avoid
Seizure type
Focal or secondary LTG, LEV, OXC, LCM,
generalized TPM > CBZ, > VPA, ESL, PHT
Primary Generalized (GTC) VPA, LEV, LTG, TPM, ZNS
Primary Generalized ESX, VPA > LTG PHT, CBZ, GBP, TGB, VGB
(Absence)
Primary Generalized LEV, VPA, CLZ PHT, CBZ, GBP, TGB, VGB,
(Myoclonic) PGB
Rolandic (centrotemporal) LEV, OXC
Other factors
Young women LTG, LEV, LCM VPA > > CBZ, PHT
Depression LTG PHT, PHB, PRM,
Labile, impulsive VPA, CBZ, LTG, OXC, TPM LEV
Liver disease LEV, LTG, PGB VPA, PHT, CBZ
Obesity TPM, ZNS VPA, GPN, PGB
Pain GBP, PGB, CBZ, OXC
Headache TPM, VPA, GBP, PGB
Type A personality (baseline LEV
irritability)
Polytherapy (non AEDs) LEV, PGB, GBP Enzyme Inducers
Asian (Han Chinese or CBZ, OXC (if have to use check
Taiwanese) HLA-b 1502)

showed superior efficacy of newer drugs com- highest with OXC and TPM and lowest with
pared to the older ones. Several studies showed GBP and LEV Costa et al. [8]. Table 15.8
better tolerability and less discontinuation rates summarizes ideal AED(s) selection based on
with newer AEDs. In one study, PGB had similar seizure type and comorbidities.
tolerability but inferior efficacy to LTG for the
treatment of newly diagnosed partial seizures in
adults Kwan et al. [7]. Another study demon- Drug-Resistant Epilepsy
strated PGB to be non-inferior to LTG in the and Polytherapy
treatment of refractory partial seizures. In a
meta-analysis by Costa J. et al., clinical compa- Drug resistant epilepsy is defined as failure to
rability of the newer AEDs in refractory partial achieve seizure freedom after adequate trials of
epilepsy was analyzed. A total of 62 randomized two appropriately chosen AEDs used as thera-
controlled trials comparing a new AED to a peutic levels in monotherapy or combination
placebo as adjunctive therapy, and 8 randomized therapy (2010 consensus document by ILAE,
controlled trials comparing a new AED to Kwan et al. 2010). Overall, about one third of all
another AED as add were reviewed. In this epilepsies prove to be drug resistant. Among all
meta-analysis, LEV shined as the AED with high epilepsies, symptomatic or cryptogenic epilepsies
responder rate as well as low withdrawal rate. have higher rate of drug resistance compared to
Looking at the results overall, TPM and LEV had idiopathic epilepsies Kwan et al. [9]. Drug
the highest responder rates. GBP and TGB had resistance also tends to be higher in patients who
the lowest responder rates. Withdrawal rate was had more than 20 seizures prior to starting
15 Principles of Epilepsy Diagnosis and Management 211

Table 15.9 Specific rash AEDs Cross-sensitivity rates (%)


cross-sensitivity rates
(Hirsch et al. 2008) CBZ – OXC 33–71
CBZ – PHT 42–57
CBZ – PHB 27–66
PHT – ZNS 21

treatment. When the initial AED fails, adding a study, 1617 seizure-free epilepsy patients on
second AED or switching to another AED did polytherapy were identified. The majority of
not differ statistically; however, common practice seizure-free patients (81.3 %) on polytherapy
favors the combination therapy Kwan et al. [9]. were on two AEDs only. With 64 effective dual
Wide range of combinations of two or perhaps therapy regimens, VPA and LTG combination
three AEDs can be effective in some patients. was the most commonly successful combination
With the available 25 different AEDs, about 300 at 24.3 %. About 17.5 % of seizure-free patients
possible double therapy regimen and more than were on three AEDs. There were 57 effective
2000 triple therapy regimens can be achieved. triple therapy regimens, and the most commonly
While choosing an appropriate AED combina- successful combinations were (LTG + TPM +
tion, drug-drug interactions should be avoided. VPA) or (LEV + LTG + VPA) Stephen et al.
Enzyme inducing AEDs will increase their own [10].
clearance as well as most of the other AEDs One should also consider the specific rash
(Table 15.10). For example, combining PHB and cross-sensitivity rates among AEDs (Table 15.9).
VPA would increase sedation and weight gain; There is no known specific cross-sensitivity
combining PHT and CBZ would result in between LTG and other AEDs. AEDs with low
increased side effects such as dizziness and risk of rash include VPA, GBP, PGB, LEV, and
diplopia and bidirectional induction of metabo- TPM.
lism would make it difficult to maintain thera- Eliminating AEDs which are deemed inef-
peutic blood levels. VPA may triple LTG blood fective is necessary to decrease the drug burden,
levels and result in increased chances of allergic to allow higher doses of more effective drugs,
reactions. If used cautiously, VPA and LTG and also to avoid drug-drug interactions. There
combination therapy is shown to be one of the may be certain patients who would have
most successful combinations in generalized increased seizures even when an ineffective drug
epilepsy. Also it is recommended to avoid com- is decreased or stopped. These patients usually
bination of AEDs with similar side effects such end up on multiple AEDs up to six or seven
as dizziness, imbalance, and diplopia common to without seizure control. Withdrawal seizures
sodium channel blockers. In particular, these should not discourage clinician to simplify AED
combinations include CBZ + LTG, CBZ + regimens. If needed, epilepsy monitoring unit
LCM, OXC + LCM, or LTG + LCM. In one could be used to safely taper the ineffective AED.

Table 15.10 P450 Enzyme inducing AEDs (strong) Enzyme inhibiting AEDs (strong)
enzyme modulation of
several AEDs Phenobarbital Valproic acid
Primidone Topiramate (weak)
Phenytoin
Carbamazepine
Oxcarbazepine (doses > 900 mg)
Lamotrigine (weak)
80–90 % of all existing medications will be affected by the enzyme inducing AEDs
above. In particular efficacies of steroids, estrogens, digoxin, warfarin, furosemide, and
doxycycline will be decreased
212 H.H. Sonmezturk et al.

Stopping AEDs When Seizure these patients. Young women with 2–3 year
Freedom Achieved seizure freedom could prefer drug withdrawal
during pregnancy. Some patients have intolerable
About 70 % of epilepsy patients will eventually side effects which may necessitate drug with-
achieve seizure freedom with AEDs. There is drawal. Some patients have difficulty finding a
often an illusion of cure after seizures are con- job while on AEDs, and we may chose to taper
trolled for long term. However, this is not the AEDs off in these patients. In general 2–5 years
case in most patients. In general, depending on of seizure freedom is needed. And the tapering of
the seizure or epilepsy and etiology, 11–41 % of the AEDs should be slowly spread over
patients will relapse after the AED discontinua- 6–12 months.
tion. The relapse rate tends to be lower in chil-
dren (* 20 %) and higher in adults (* 40 %).
In one large study, patients in long-term remis- References
sion were randomized either to withdraw or
continue the treatment. In the first two years, 1. Hauser et al. Rochester Minnesota 1955–1984. 1993.
41 % of the patients coming off the treatment 2. King, et al. Lancet. 1998;352(9133):1007–11.
relapsed versus 22 % of the patients continuing 3. Hakami, et al. Neurology. 2013;81(10):920–7.
4. Berg and Shinnar Neurology. 1991;41(7):965–72.
on medication. Most relapses occurred within the 5. Dodrill CB, et al. Epilepsy Res. 2000;42(2–3):123–
first year of treatment reduction or AED with- 32.
drawal. The more severe and long lasting a 6. Biton V, et al. Epilepsia. 1998;39(suppl 6):125.
patient’s active epilepsy was before remission, 7. Kwan, et al. Lancet Neurol. 2011;10(10):881–90.
8. Costa J, et al. Epilepsia. 2011;52:1280.
the greater the risk of relapse. Diagnosis of 9. Kwan P, Brodie MJ. N Engl J Med. 2000;342
Juvenile myoclonic epilepsy (JME) or the pres- (5):314–9.
ence of a structural lesion underlying the epi- 10. Stephen LJ, et al. Epilepsy Res. 2012;98(2–3):194–8.
lepsy also increased the relapse risk (MRC 11. Krumholz et al. Neurology. 2007;69(21):1996–2007.
12. First Seizure Trial Group Neurology. 1993;43(3 Pt
Antiepileptic Drug Withdrawal Group (1991). 1):478–83.
Clinicians should choose a patient specific 13. Marson A et al. Lancet. 2005;365(9476):2007–2013.
approach in discontinuing AED therapy. Also, 14. Hesdorffer DC, et al. Epilepsia. 2009;50(5):1102–8.
the epilepsy classification and seizure type 15. Deckers CL, et al. Epilepsia. 2000;41(11):1364–74.
16. Tracy Glauser et al. Special Report Epilepsia, **
should be considered. Relapse rate in rolandic (*):1–13, 2013.
epilepsy is about 2 %, so the threshold to with- 17. Baulac M, et al. Epilepsy Res. 2010;91(1):10–9.
draw the treatment after long-term remission 18. Hirsch LJ, et al. Neurology. 2008;71(19):1527–34.
should be low. On the other hand, the relapse rate 19. MRC Antiepileptic Drug Withdrawal Group. Lancet.
1991;337:1175–1180.
in JME is about 85 %, so there should be a much
higher threshold to withdraw the treatment in
Old Generation Antiepileptic Drugs
16
Bassel Abou-Khalil

and others. It reduces the serum concentration of


Phenobarbital
carbamazepine but may increase the carba-
mazepine epoxide to carbamazepine ratio. It
The first of old-generation antiepileptic drugs
reduces the efficacy of warfarin, steroids, and the
(AEDs) that are currently marketed was pheno-
oral contraceptive.
barbital, which came into clinical use in 1912. Its
Phenobarbital has a variable effect on pheny-
initial use was as a sedative and sleep aid. Its
toin concentration; while it may induce pheny-
efficacy against seizures was discovered later
toin metabolism, it may also compete with
(Table 16.1).
phenytoin for metabolic enzymes CYP 2C9 and
Phenobarbital exerts its action by binding to
2C19. Phenobarbital serum concentration may be
the GABA-A receptor and enhancing chloride
increased by the inhibitors valproate and
currents by prolonging the opening of the chlo-
felbamate.
ride channel. It may also have other actions
The main adverse effects of phenobarbital are
including blocking high-voltage-activated cal-
sedation, mood changes (particularly depres-
cium channels and AMPA subtype glutamate
sion), hyperactivity and irritability in children,
receptors.
and decreased memory and concentration. It also
Phenobarbital is available as oral preparations
has long-term adverse effects. Long-term use of
as well as parenteral solution. Its oral availability
phenobarbital is associated with decreased bone
is greater than 90%. Its protein binding is about
density and some connective tissue disorders,
45%. Its volume of distribution is approximately
particularly Dupuytren’s contractures, plantar
0.6 L/kg. Phenobarbital is mostly metabolized in
fibromatosis, and frozen shoulder. Phenobarbital
the liver but 20–25% is eliminated unchanged in
is associated with increased risk of cardiac mal-
the urine. The half-life in adults is 80–100 h. The
formations in the exposed fetus and reduced
half-life is longer in newborns and shorter in
cognitive abilities in the exposed male offspring.
young children. Phenobarbital is a potent inducer
It is assigned to pregnancy category D.
of P4 50 enzymes. It does accelerate the meta-
Phenobarbital is effective against partial onset
bolism and reduces the levels of anti-epileptic
(focal) seizures, generalized tonic–clonic sei-
drugs processed by this enzyme system. For
zures, and other generalized onset seizures
example, phenobarbital reduces serum concen-
except for absence. The recommended thera-
trations of valproate, ethosuximide, lamotrigine,
peutic concentration is 50 and 2 14 mg/L.
Phenobarbital is not a drug of first choice in
developed countries, because of its adverse
B. Abou-Khalil (&)
effects, namely sedation, and because of its
Department of Neurology, Vanderbilt University
Medical Center, Nashville, TN, USA enzyme-inducing properties. However, it is
e-mail: bassel.abou-khalil@vanderbilt.edu inexpensive and widely available, and may be

© Springer Science+Business Media LLC 2017 213


M.Z. Koubeissi and N.J. Azar (eds.), Epilepsy Board Review,
DOI 10.1007/978-1-4939-6774-2_16
214 B. Abou-Khalil

Table 16.1 Select pharmacokinetic parameters of main old-generation antiepileptic drugs


AED Phenytoin Carbamazepine Oxcarbazepine Phenobarbital Valproate Ethosuximide
MHD
Protein High Intermediate Low Low High Low
bindinga
Half-lifeb Intermediate (can Intermediate Short Long Intermediate Long
become long in
toxicity)
Metabolism Extensive, Extensive Extensive >70% Extensive Extensive
Nonlinear
Enzyme +++ +++ + +++ – –
Induction autoinduction (CYP 2C19)
Enzyme – – + – +++ –
Inhibition (CYP 3A4)
a
Low < 50%; intermediate: 50–85%, high > 85%
b
Short < 10 h; intermediate 10–30 h; long > 30 h

the only affordable antiepileptic drug in much of unchanged in the urine. In the presence of
the developing world. enzyme induction, only about 40% is excreted
unchanged. In monotherapy, primidone half-life
is 10–15 h; it is shorter (6.5–8.3 h) in the pres-
Primidone ence of enzyme inducers.
In the presence of inducers, particularly
Primidone is converted to phenobarbital and phenytoin, the ratio of primidone-to-
phenylethylmalonamide (PEMA), which is also phenobarbital is reduced due to the acceleration
an active metabolite. Unlike phenobarbital, of primidone-to-phenobarbital conversion.
primidone does not have a direct effect on Primidone and phenobarbital are potent enzyme
GABA receptors. Primidone and phenobarbital inducers, decreasing the efficacy of drugs
may act synergistically to reduce sustained metabolized by the p450 enzymes system. Since
high-frequency repetitive firing, at clinically rel- phenobarbital will be present when primidone is
evant concentrations. This is an effect on the used, all phenobarbital interactions are also pre-
sodium channel that neither drug has when used sent by necessity.
alone. As mentioned earlier, phenobarbital acts at Primidone has acute toxic reactions that are
the GABA-A receptor to prolong the opening of different from phenobarbital. It can produce
the chloride channel. The mechanism of action of transient drowsiness, dizziness, ataxia, nausea,
PEMA is unknown, and its anti-seizure activity is and vomiting that can be debilitating. These
modest. Primidone is available only as an oral reactions are present even before phenobarbital
preparation. It is poorly soluble, precluding an IV has appeared as a metabolite. Therefore, a slow
preparation. titration of primidone is necessary. Tolerance to
Primidone oral bioavailability is fairly com- these acute adverse experiences develops rapidly
plete. Its volume of distribution is *0.7 L/kg. Its within hours to days. Otherwise, primidone has
protein binding is low, less than 10% for primi- similar adverse experiences to phenobarbital,
done and for PEMA. including adverse experiences from long-term
Primidone is metabolized in the liver. PEMA use.
is the first detected metabolite. When used in Primidone is effective against the same seizure
monotherapy, about 25% of oral primidone is types as phenobarbital. The recommended
converted to phenobarbital. After one dose, primidone therapeutic plasma concentration is 5–
approximately 64% of primidone is excreted 12 mg/L. A phenobarbital level should also be
16 Old-Generation Antiepileptic Drugs 215

monitored. Since about 25% of oral primidone is lead to phenytoin accumulation. Similarly, some
converted to phenobarbital, the dose of primi- alleles are associated with decreased activity
done required for a certain level is about 4–5 leading to accumulation.
times the dose of phenobarbital required for that The phenytoin half-life is dependent on the
same level. Primidone was found to have equal serum concentration. The initial half-life is
efficacy but lower tolerability in comparison with approximately 22 h (with the range of 8–60 h).
phenobarbital, phenytoin, and carbamazepine. The half-life increases as the serum concentration
increases within and above the recommended
therapeutic range (10–20 mg/L).
Phenytoin The mechanism of the nonlinear elimination
kinetics is that enzymes responsible for most of
Phenytoin has been used since 1938 when phenytoin elimination are partially saturated with
Houston and Merritt discovered its efficacy in the concentrations within the recommended range
maximum electroshock animal model. Phenytoin (with individual variation as to the concentration
acts by binding to active state of the sodium at which this phenomenon first appears). These
channel and reducing high-frequency firing (as enzymes are not able to increase their activity in
might occur during a seizure), while allowing proportion to phenytoin concentration, as the
normal action potentials to occur. concentration increases in the recommended
Phenytoin is available as oral preparations and therapeutic range. Therefore, steady-state
parenteral solution. There is also a phenytoin phenytoin level increases disproportionately as
prodrug for parenteral administration, the maintenance dose is increased within and
fosphenytoin. above the recommended therapeutic range.
Phenytoin absorption is variable. The rate and Below are two examples of the consequences of
extent of absorption may also differ among for- phenytoin nonlinear kinetics.
mulations and is affected by a variety of factors Example 1: A daily dose of 300 mg per day
including age and food. While oral bioavailabil- results in a concentration of 9 mg/L, with some
ity can be greater than 90% in adults, it is residual seizures. Increasing the dose to 400 mg
decreased in neonates and is also decreased in the per day, a 30% increase in dose would have been
presence of nasogastric feedings, calcium, and expected to increase the steady-state concentra-
antacids. There is limited absorption in the tion by 30%, to 12 mg/L, if phenytoin were to
stomach. Absorption is primarily in the duode- follow linear elimination kinetics. However, with
num where the higher pH increases the phenytoin its nonlinear kinetics, the concentration increases
solubility. The time to maximal concentration is disproportionately, by more than 300%, to
shorter with immediate release preparations and 31 mg/L, with associated toxicity. Therefore,
longer with extended-release formulations. The when increasing phenytoin dose within the
volume of distribution is approximately therapeutic range, small increments should be
0.75 L/Kg. Protein binding is approximately used (e.g., 30–60 mg).
90%. Example 2: A patient presents with phenytoin
The major pathway of elimination of pheny- toxicity and a serum concentration of 40 mg/L.
toin is hydroxylation, mediated mainly by the The half-life was previously estimated at 24 h
cytochrome p450 enzymes CYP 2C9 and to a when the serum concentration was 13 mg/L.
lesser extent CYP 2C19. Phenytoin follows However, after phenytoin was stopped, it took
nonlinear kinetics. Small changes in CYP 2C9 3 days for the serum concentration to drop below
activity may have clinically significant effects. 20 mg/L. The reason for this is that the half-life
Some CYP 2C9 alleles are associated with was markedly prolonged in the presence of
reduced clearance of phenytoin. The importance toxicity.
of CYP 2C19 increases with higher levels. Some Phenytoin is affected by drugs that decrease
inhibitors such as ticlopidine and isoniazid may absorption (e.g., nasogastric tube feedings) drugs
216 B. Abou-Khalil

that compete for protein binding (such as val- A phenytoin level of 15 mg/L may actually be
proate) and by enzyme inducers or inhibitors. toxic with a free level of 3–4.5 mg/L, equivalent
Drugs that cause phenytoin accumulation include to a total level of 30–45 mg/L in the presence of
amiodarone, azoles, fluoxetine/fluvoxamine, and normal protein binding.
isoniazid. Phenytoin is a potent enzyme inducer When a low-protein state is present as the
that reduces the efficacy of other drugs metabo- only factor affecting protein binding, the total
lized by the p450 enzyme system, including a phenytoin level can be corrected using the fol-
number of other antiepileptic drugs. lowing formula: Cn = Co/[(0.02  albu-
Phenytoin protein binding plays an important min) + 0.1], where Cn is the normal total level
role in some phenytoin interactions. The and Co is the observed total level.
protein-free phenytoin level is responsible for its Phenytoin concentration-dependent adverse
therapeutic effect and for its toxicity. The free effects include nystagmus, ataxia, incoordination,
fraction increases in the presence of low-protein diplopia, dysarthria, and drowsiness. In addition,
state, renal failure, hepatic failure, old age, or exacerbation of seizures may occur with con-
with co-administration of valproate. Therapeutic centrations above 30 mg/L. Some individuals
decisions are usually made based on the total may experience prominent adverse effects within
phenytoin level, assuming that 10% is free. the recommended therapeutic range, including
However, a free level should be obtained in cognitive adverse effects.
clinical situations where an increase in the free Idiosyncratic reactions may be related to the
fraction is expected. formation of an arene oxide, the active metabo-
As an example of the potential consequences lite that forms due to inadequate epoxide
of altered protein binding of phenytoin, a patient hydrolase activity. Allergic rash occurs in up to
with epilepsy and renal impairment may be 8.5% of patients. Serious severe rash such as
having uncontrolled seizures with a total Stevens–Johnson syndrome or toxic epidermal
phenytoin concentration of 15 mg/L. The deci- necrolysis are much less common. A hypersensi-
sion has to be made whether the dose should be tivity syndrome may occur rarely, with rash,
increased to improve seizure control or should be fever, lymphadenopathy, eosinophilia, elevated
decreased because of toxicity causing a para- liver enzymes, and renal failure.
doxical increase in seizure frequency. The Phenytoin also has long-term adverse effects
protein-free concentration turns out to be including gingival hyperplasia, hirsutism, acne,
4.5 mg/L, equivalent to a total phenytoin con- cerebellar atrophy (which may also occur after
centration of 45 mg/L under the condition of acute intoxication), reduced bone density,
normal protein binding. In this case, holding reduced folate levels, anemia, and macrocytosis.
phenytoin was the correct approach. While renal Phenytoin also has potential teratogenicity and is
and hepatic failures are frequently associated classified with pregnancy category D.
with the decreased albumin concentration, the The IV preparation is associated with local
reduction in protein binding may also occur as a reactions such as pain and burning at the infusion
result of small molecules that compete for protein site, phlebitis, cellulitis, or necrosis from
binding. extravasation, and the purple glove syndrome
Valproate competes with phenytoin for pro- with discoloration then petechial rash. Cardio-
tein binding. In monotherapy, each of phenytoin vascular adverse experiences include hypoten-
and valproate are approximately 90% sion, conduction abnormality, and arrhythmia.
protein-bound, and the free phenytoin level is They are related in part to the vehicle, propylene
about 10% of the total level. With concomitant glycol. They can be avoided with the slowing of
use, each 1 mg/L of valproate increases the free the infusion rate, which should not exceed
fraction of phenytoin by approximately 50 mg/m.
0.1–0.2%. At a valproate level of 100 mg/L, the Phenytoin is effective against partial onset
free phenytoin fraction may increase to 20–30%. (focal seizures) and generalized tonic–clonic
16 Old-Generation Antiepileptic Drugs 217

seizures. Efficacy against tonic and atonic sei- rapidly after the end of infusion. It is not seen
zures is less well established. Phenytoin is not with intramuscular administration.
effective against generalized myoclonic or gen-
eralized absence seizures and may even exacer-
bate these seizures. Carbamazepine
Phenytoin can be loaded orally (18 mg/kg
divided into 3 doses given 2–3 h apart). IV Carbamazepine is a related structure to tricyclic
loading dose for status epilepticus is 18– antidepressants. Its mechanism of action is
20 mg/kg. Phenytoin should be evaluated in reducing high-frequency neuronal firing through
normal saline, not dextrose 5% in water. It the blocking of the sodium channel in a voltage
should be administered into a large vein with a and use dependent fashion.
maximum rate not exceeding 50 mg/min. It has good bioavailability of 80–90%, and it
Intramuscular injection is not recommended is lipophilic but poorly water-soluble, making
due to slow and erratic absorption as well as parenteral formulation difficult. Its protein bind-
crystallization at the injection site causing pain ing is about 75%, usually not of clinical
and a sterile abscess. importance.
Carbamazepine is cleared almost entirely via
hepatic metabolism. The most important meta-
Fosphenytoin bolic product is carbamazepine-10,11-epoxide,
produced via oxidation through CYP 3A4 and
Fosphenytoin is a water-soluble phenytoin pro- CYP 2C8. It is an active metabolite which is also
drug. It can be given intravenously or intramus- responsible for some adverse effects. Carba-
cularly. It is rapidly and completely converted to mazepine induces its own metabolism. This
phenytoin by the cleavage of the phosphate process, known as autoinduction, causes
group by nonspecific phosphatases. Its conver- increased clearance, shortened half-life, and
sion half-life is 8–18 min, and the conversion is lower serum concentration of carbamazepine
completed in a little more than 1 h. It is highly over time. The process typically takes 2–
bound to serum albumin (95–99%). It displaces 4 weeks. As a result, carbamazepine cannot be
phenytoin from protein-binding sites after IV started at the target maintenance dose. It has to be
administration, increasing unbound phenytoin titrated gradually.
concentrations as a function of fosphenytoin Carbamazepine is a potent inducer of p450
concentration. It is indicated for a replacement of enzyme system (particularly CYP 3A4, CYP
oral fosphenytoin or for intravenous or intra- 2C9, CYP 2C19, and CYP 1A2), increasing the
muscular loading. It is marketed in phenytoin clearance of agents metabolized by these
equivalents, so the loading dose is the same as enzymes. The list of drugs affected includes
phenytoin. The maximum rate of intravenous hormonal contraceptives, warfarin, and several
infusion is much higher, at 150 mg/min, in view antiepileptic drugs, including valproate and
of the absence of propylene glycol. A therapeutic lamotrigine.
phenytoin level is usually reached within 10 min Carbamazepine is affected by agents that
after IV loading and within 30 min after intra- induce or inhibit CYP 3A4. The list includes
muscular administration. erythromycin and other macrolide antibiotics
Fosphenytoin has a lower incidence of local (except azithromycin), fluoxetine, propoxyphene,
reactions. However, intravenous administration and grapefruit juice among others. The level of
in the awake individual is commonly associated carbamazepine epoxide is increased by the con-
with paresthesias and itching, most often in the comitant use of valproate, felbamate, oxcar-
groin and perianal region, as well as the trunk bazepine, and zonisamide.
and the back of the head. This adverse experi- The most common adverse experiences with
ence is related to infusion rate and subsides carbamazepine are nausea, GI discomfort,
218 B. Abou-Khalil

headache, dizziness, incoordination, unsteadi- until the drug failed due to uncontrolled seizures
ness, vertigo, sedation, tiredness, blurred vision, or unacceptable side effects. The overall treat-
diplopia, nystagmus, and tremor. Benign ment success was highest with carbamazepine
leukopenia is common, occurring in 10–20% of and phenytoin, intermediate with phenobarbital
instances. It is most often transient and may be and lowest with primidone. The drugs had
persistent though not progressive. This is to be overall equal efficacy, and the difference in
distinguished from the more serious but very rare treatment success was related to tolerability.
aplastic anemia. Carbamazepine can cause Primidone caused more intolerable acute toxic
hyponatremia. Cognitive impairment has been effects, mainly nausea, vomiting, dizziness,
reported on neuropsychological testing. sedation, decreased libido, and impotence. When
Long-term use is associated with weight gain and specific seizure types were analyzed, control of
decreased bone density. Carbamazepine has been secondarily generalized tonic–clonic seizures did
found to increase sex hormone binding globulin not differ significantly with the 4 drugs, but
and decrease testosterone concentration. carbamazepine provided complete control of
Idiosyncratic adverse experiences include partial seizures more often than primidone or
rash, rare Stevens–Johnson syndrome and toxic phenobarbital. As a result, carbamazepine
epidermal necrolysis, as well as very rare became the drug against which new antiepileptic
hypersensitivity syndrome with fever, rash, end drugs were compared.
organ involvement. Lupus-like syndrome is rare,
as are hepatotoxicity and aplastic anemia (esti-
mated at 1 in 200,000). There is a strong asso- Oxcarbazepine
ciation between the HLA-B1502 allele and
carbamazepine-induced Stevens–Johnson syn- Even though oxcarbazepine was first introduced
drome in Asian populations and individuals of in the USA in 2000, it is listed with the
Asian descent. The FDA has issued an alert and old-generation drugs since it was introduced in
updated product labeling recommending genetic some European countries as early as 1963.
testing of HLA-B polymorphisms to predict Oxcarbazepine is structurally related to carba-
carbamazepine-induced serious skin reactions in mazepine, but different from carbamazepine in its
individuals of Asian descent. metabolism and in the induction of metabolic
Carbamazepine has been assigned pregnancy pathways. Oxcarbazepine has a similar mecha-
category D due to increased risk of spina bifida nism of action to carbamazepine, inhibiting
when used in polytherapy. high-frequency repetitive neuronal firing by
Carbamazepine is effective against partial blocking voltage-gated sodium channels. It also
onset (focal) seizures and against generalized modulates high-voltage-activated calcium
tonic–clonic seizures. However, it may exacer- channels.
bate absence and myoclonic seizures as well as Oxcarbazepine absorption is almost complete
atonic seizures. The recommended therapeutic with a bioavailability of about 99%. It is very
range is 4–12 mg/L. rapidly metabolized to a monohydroxy deriva-
The efficacy and tolerability of carba- tive, an active metabolite responsible for oxcar-
mazepine, phenobarbital, phenytoin, and primi- bazepine activity. Oxcarbazepine protein binding
done were compared in a large, multicenter, is about 60% while the monohydroxy derivative
double-blind, cooperative veterans administra- protein binding is about 40%. The oxcarbazepine
tion trial [9]. Six hundred and twenty-two adults half-life is 1–3.7 h. The monohydroxy derivative
with partial and secondarily generalized tonic– is further metabolized and has a half-life of 8–
clonic seizures were randomly assigned to one of 10 h. Oxcarbazepine does not induce its own
the four drugs and were followed for two years or metabolism.
16 Old-Generation Antiepileptic Drugs 219

The monohydroxy derivative level decreases Valproate/Divalproex


in the presence of enzyme-inducing drugs. One
major advantage of oxcarbazepine over carba- Valproate discovery was serendipitous as it was
mazepine is that it is not affected by CYP 3A4 used as a solvent for antiepileptic drugs in test-
inhibitors such as erythromycin, fluoxetine, pro- ing. Valproate is a short chain, branched fatty
poxyphene, and grapefruit juice. Oxcarbazepine acid. It has multiple mechanisms of action,
does not induce the metabolism of other including the blocking of sodium channels,
antiepileptic drugs or warfarin. It weakly induces gabapentin potentiation, and blocking of T cal-
CYP 3A4 which is responsible for estrogen cium channels. The main form of valproate used
metabolism, thus reducing the efficacy of the clinically is divalproex sodium, a complex
birth control pill at high doses. Oxcarbazepine composed of equal parts of valproate and sodium
also weakly inhibits CYP 2C19, thus raising valproate. Preparations include immediate
phenytoin level when used at high doses. release valproate capsules, tablets, and syrup;
The most common adverse effects of oxcar- delayed-release enteric coated tablets of dival-
bazepine are somnolence, headache, dizziness, proex sodium; divalproex sodium enteric coated
blurred vision, diplopia, fatigue, nausea, vomit- sprinkles; extended-release divalproex sodium;
ing, and ataxia. Rash has been reported in 2–4% and parenteral sodium valproate. The
of individuals. Oxcarbazepine can cause delayed-release enteric coated tablets are rapidly
hyponatremia, which is more likely in older absorbed after the coating is dissolved, or the
individuals and those taking a diuretic. Oxcar- extended-release divalproex sodium is absorbed
bazepine does not has the effect on sex hormone slowly. Oral bioavailability is almost complete
binding-globulin and testosterone that carba- for most of valproate preparations, but 90% for
mazepine has. the extended-release preparation. The time to
Oxcarbazepine was assigned pregnancy cate- maximal concentration is very dependent on the
gory C. preparation. It is 2 h for the syrup and up to 17 h
Oxcarbazepine is effective against partial onset for the extended-release divalproex. The volume
(focal) seizures. The recommended therapeutic of distribution is 0.13–0.19 L/kg in adults. Pro-
range is 15–35 mg/L. Oxcarbazepine is a narrow tein binding is about 90%. The free fraction
spectrum drug that should be avoided in indi- increases with increasing total concentration and
viduals with generalized epilepsy. Oxcarbazepine can reach 30% 150 mg/L.
may exacerbate absence and myoclonic seizures. Valproate is extensively metabolized by p450
Multiple comparative monotherapy trials for enzymes, including CYP 2A6, CYP 2B6, CYP
new onset partial epilepsy have demonstrated 2C9, and CYP 2C19. The half-life in adults is
that oxcarbazepine is equal in efficacy to 13–16 h without induction and about 9 h with
phenytoin and carbamazepine but with superior enzyme-inducing drugs.
tolerability [1, 3, 5, 6, 11–13]. Conversion to Valproate metabolism is induced by pheny-
oxcarbazepine from carbamazepine can be made toin, carbamazepine, and phenobarbital. Val-
overnight using a 1.5-to-1 ratio when the carba- proate levels will increase after the withdrawal of
mazepine dose is 800 mg or less. At higher these enzyme-inducing drugs. Valproate levels
carbamazepine doses, a slower conversion and a increase with coadministration of felbamate and
lower ratio are advisable. Conversion from car- clobazam. Valproate can inhibit the metabolism
bamazepine to oxcarbazepine will be accompa- of phenobarbital, lamotrigine, rufinamide, and
nied by enzyme de-induction and possible carbamazepine epoxide, causing increased serum
elevation of some medication levels. In addition, concentrations. Valproate competes with pheny-
sodium level may decrease after conversion from toin for protein binding (look under phenytoin
carbamazepine. for potential consequences of this interaction).
220 B. Abou-Khalil

Valproate adverse effects include gastric irri- Ethosuximide


tation with nausea, vomiting, GI distress, and
anorexia. These adverse effects are most likely Ethosuximide mechanism of action is blockade
with the enteric coated preparation and the of the T-type calcium currents in the thalamus. It
extended-release formulations. Other adverse has an excellent oral bioavailability (greater than
effects include tremor, weight gain, hair loss, 90%), and volume of distribution is 0.65 L/kg.
peripheral edema, thrombocytopenia, and Protein binding is very low, less than 10%.
drowsiness/lethargy/confusion. A reversible Ethosuximide is extensively metabolized in the
dementia and brain atrophy have been described liver with oxidative biotransformation to inactive
in some individuals, particularly in seniors. metabolites, mainly by CYP 3A4 has a long
Encephalopathy may occur when valproate is half-life of 30–60 h (shorter in children).
used in polytherapy. Hyperammonemia may be Ethosuximide has no effect on hepatic p450
seen in some individuals. Carnitine deficiency enzymes and is unlikely to affect other drugs.
has been reported and associated with tiredness. However, it is susceptible to interactions from
Idiosyncratic adverse experiences include inducers and inhibitors of p450 enzyme system.
fatal hepatotoxicity and pancreatitis. The risk Its clearance is increased with enzyme inducers
factors for severe hepatotoxicity are polytherapy and may decrease with valproate and isoniazid.
and young age. Hepatotoxicity is most likely to Most ethosuximide adverse effects are
occur below age 3, with a risk of 1 in 600. Above dose-related and helped by dividing the dose and
age 40, the risk is less than 1 in 100,000. Mon- administration with meals. Among these, GI side
itoring of liver enzymes is recommended in effects include nausea, abdominal discomfort,
young children. anorexia, vomiting, and diarrhea; CNS adverse
Valproate is teratogenic and was assigned effects include drowsiness, insomnia, nervous-
pregnancy category D. Valproate teratogenicity ness, dizziness, hiccups, fatigue, ataxia, and
is dose-dependent. The risk of major congenital behavior changes such as aggression, irritability,
malformations can be higher than 30% at doses and hyperactivity; hematologic side effects
greater than 1100 mg per day [15]. At doses include granulocytopenia. Headaches, psychosis,
below 1000 mg per day, the malformation rate depression, and hallucinations (visual or audi-
was about 3.2%. In addition in utero exposure to tory) are not clearly dose-related.
valproate has been associated with reduced ver- Idiosyncratic adverse experiences include
bal IQ and autism [2, 10]. This developmental rash, Stevens–Johnson syndrome, SLE, rare
toxicity is also dose-dependent. aplastic anemia, thrombocytopenia, or agranulo-
Valproate has a wide spectrum of efficacy cytosis, and rare autoimmune thyroiditis.
against partial onset (focal) and all generalized Ethosuximide has been assigned a pregnancy
onset seizures, including absence and myoclonic category D due to increased risk of birth defects.
seizures. It is also FDA-indicated for migraine Ethosuximide has a narrow spectrum of
prophylaxis and bipolar disorder. The recom- activity with efficacy limited to generalized
mended therapeutic range is 40–100 mg/L. An absence seizures. It is not effective against any
important class III comparative study showed other seizure type. A large, multicenter,
that valproate was more effective than lamotrig- double-blind, randomized, controlled trial to
ine and better tolerated than topiramate for the compare the efficacy, tolerability, and neuropsy-
treatment of generalized epilepsy [7]. However, chological effects of ethosuximide, valproic acid,
in another cooperative Veterans Administration and lamotrigine favored ethosuximide [4]. After
Study, valproate was less effective than carba- 16 weeks of therapy, the freedom-from-failure
mazepine for complex partial seizures, even rates for ethosuximide and valproic acid were
though the two drugs were equally effective for similar and higher than the rate for lamotrigine.
secondarily generalized tonic–clonic seizures [8]. However, attentional dysfunction was more
In addition, carbamazepine was better tolerated. common with valproic acid than with
16 Old-Generation Antiepileptic Drugs 221

ethosuximide. As a result, ethosuximide became specific metabolic pathway. Inhibition of the


the drug of choice for pure generalized absence major pathway may cause accumulation, while
seizures. The recommended therapeutic range is inhibition of a minor pathway has limited effect.
40–100 mg/L. It is recommended that a complete On the other hand, induction of either a major or
blood count be checked before and after 2– minor pathway will reduce the benzodiazepine
3 months of treatment. concentration. The clinical effect of induction
and inhibition also depends on the presence of
active metabolites and metabolic pathways
Benzodiazepines (Table 16.3).
Benzodiazepines have similar adverse expe-
Benzodiazepines as a family act mainly on the riences, particularly drowsiness, to which toler-
GABA-A receptor, increasing the frequency of ance may develop. With higher doses,
GABA-mediated chloride channel openings. nystagmus, incoordination, ataxia, and dysarthria
Benzodiazepines have a wide spectrum of effi- may occur. Behavioral disturbances may occur,
cacy. Among the benzodiazepines most com- more commonly in children (aggression, hyper-
monly used for epilepsy, diazepam and activity, paranoia). Most benzodiazepines are
lorazepam are primarily used for acute seizure assigned pregnancy category D. Tolerance may
emergencies, particularly status epilepticus and develop to the therapeutic effect of benzodi-
acute repetitive seizures, while clonazepam, azepines, so that after a few weeks or months of
clorazepate, and clobazam are used mainly for treatment, efficacy is lost, or any higher dose is
chronic epilepsy management. required to maintain efficacy. Withdrawal sei-
Most benzodiazepines have good oral zures may occur with abrupt discontinuation.
bioavailability (larger than 80%). One exception Below is specific information on individual
is midazolam (not discussed in this chapter), benzodiazepines.
which is metabolized in intestinal epithelium, Clonazepam has minimal interactions, except
resulting in a bioavailability of about 40% [14]. that its clearance is increased by inducers.
All benzodiazepines rapidly cross the blood– Clonazepam is used for long-term treatment as
brain barrier, with diffusion rate and onset of well as acute seizure management. Only an oral
action determined by lipid solubility. Benzodi- form is available in the USA, while an intra-
azepines have large volumes of distribution and venous formulation is available abroad. It has an
are characterized by 2-compartment distribution official FDA indication for myoclonic seizures.
model; after initial rapid distribution in the blood, However, it has a wide spectrum of efficacy
benzodiazepines diffuse into a second compart- against partial and generalized seizure types.
ment. For example, diazepam redistributes to Diazepam is metabolized to desmethyl-
adipose tissue after intravenous administration. diazepam (DMD). Like other benzodiazepines, it
While the true half-life is 36 h, the redistribution is highly protein-bound. Valproate may increase
half-life is less than 1 h. Benzodiazepines are free diazepam levels due to displacement from
also highly protein-bound. protein binding. Diazepam is available in oral
While benzodiazepines are generally similar tablet and liquid form, rectal gel, and parenteral
in absorption and distribution, they vary consid- solution. It is also being investigated for nasal
erably in their metabolism and elimination rate administration. It is used for acute repetitive
[14]. Diazepam, clorazepate, and clobazam are seizures and for status epilepticus. When used for
converted to active metabolites while lorazepam status epilepticus through intravenous route, an
and clonazepam are converted to inactive additional agent needs to be administered to
metabolites (Table 16.2). maintain seizure control beyond the first 15–
Benzodiazepines have both pharmacokinetic 30 min, because of diazepam’s short duration of
and pharmacodynamic interactions. Pharma- action due to redistribution. Diazepam is not
cokinetic interactions are dependent on the usually adequate for chronic use except that a
222 B. Abou-Khalil

Table 16.2 Benzodiazepine metabolism (adapted from [14])


Benzodiazepine Primary metabolic Active metabolite T1/2 of parent T1/2 of active
pathway drug (hrs) metabolite (hrs)
Diazepam Demethylation, Desmethyldiazepam (DMD), 21–70 DMD: 49–179
hydroxylation, oxazepam, temazepam Oxazepam: 6–24
glucuronidation Temazepam: 8–
24
Lorazepam Glucuronidation None 7–26 NA
Clonazepam Nitroreduction, None 19–60 NA
acetylation, hydroxylation
Clorazepate Decarboxylation DMD, oxazepam NA DMD: 20–160
Oxazepam: 6–24
Clobazam Demethylation N-desmethylclobazam 10–30 36–46

Table 16.3 Enzymes Enzyme Diazepam Lorazepam Clonazepam Clorazepate Clobazam


involved in metabolism of
select benzodiazepines CYP X X
(adapted from [14]) 2B6
CYP X
2C9
CYP X
2C18
CYP X X* X
2C19
CYP X X X* X
3A4
CYP X
3A5
UGT X
NAT X
UGT—uridine diphosphate glucuronosyltransferase
NAT—N-acetyltransferase
*
applies to DMD

diazepam course can be used in some syndromes be used orally to stop mild seizure clusters/acute
such as Landau–Kleffner syndrome and electrical repetitive seizures.
status epilepticus during sleep (ESES). Clorazepate is a prodrug, as it is rapidly
Lorazepam is metabolized in the liver through decarboxylated in the stomach to form the active
glucuronidation and excreted by the kidneys. It desmethyldiazepam (DMD). It is FDA-approved
does not have active metabolites. Its clearance is for management of anxiety disorders and as
reduced by valproate and other inhibitors. It is adjunctive therapy in the management of partial
available in oral and parenteral forms. It is not seizures. It is available in oral form only, in
appropriate for chronic use. Its main use is for immediate and extended-release preparations.
status epilepticus. It has a longer duration of Clobazam was only approved in the USA in
action than diazepam despite its shorter half-life, 2009, but is listed with the old-generation
as a result of less lipid solubility and less redis- antiepileptic drugs because it has been used
tribution to adipose tissue. Lorazepam can also in Europe since 1975. It is the only
16 Old-Generation Antiepileptic Drugs 223

1,5–benzodiazepine (referring to position of seizures. Cochrane Database Syst Rev, CD006453,


nitrogen atoms in the heterocyclic ring), while 2009.
7. Marson AG, Al-Kharusi AM, Alwaidh M, et al.
other benzodiazepines are 1,4–benzodiazepines. It The SANAD study of effectiveness of valproate,
is metabolized in the liver to the active N– lamotrigine, or topiramate for generalised and
desmethylclobazam. It is less sedating than unclassifiable epilepsy: an unblinded randomised
1,4-benzodiazepines. It is available in tablets and controlled trial. Lancet. 2007;369:1016–26.
8. Mattson RH, Cramer JA, Collins JF. A comparison
syrup. It is FDA-indicated for seizures associated of valproate with carbamazepine for the treatment of
with the Lennox–Gastaut syndrome, but it has a complex partial seizures and secondarily generalized
wide spectrum of efficacy as with other tonic-clonic seizures in adults. The Department of
benzodiazepines. Veterans Affairs Epilepsy Cooperative Study
No. 264 Group. N Engl J Med. 1992;327:765–71.
9. Mattson RH, Cramer JA, Collins JF, et al. Compar-
ison of carbamazepine, phenobarbital, phenytoin, and
References primidone in partial and secondarily generalized
tonic-clonic seizures. N Engl J Med. 1985;313:145–
51.
1. Bill PA, Vigonius U, Pohlmann H, et al. A dou- 10. Meador KJ, Baker GA, Browning N, et al. Fetal
ble-blind controlled clinical trial of oxcarbazepine antiepileptic drug exposure and cognitive outcomes
versus phenytoin in adults with previously untreated at age 6 years (NEAD study): a prospective obser-
epilepsy. Epilepsy Res. 1997;27:195–204. vational study. Lancet Neurol. 2013;12:244–52.
2. Christensen J, Gronborg TK, Sorensen MJ, et al. 11. Muller M, Marson AG, Williamson PR. Oxcar-
Prenatal valproate exposure and risk of autism bazepine versus phenytoin monotherapy for epilepsy.
spectrum disorders and childhood autism. JAMA. Cochrane Database Syst Rev. 2006;CD003615.
2013;309:1696–703. 12. Nolan SJ, Muller M, Tudur Smith C et al. Oxcar-
3. Dam M, Ekberg R, Loyning Y, et al. A double-blind bazepine versus phenytoin monotherapy for epilepsy.
study comparing oxcarbazepine and carbamazepine Cochrane Database Syst Rev. 2013;5:CD003615.
in patients with newly diagnosed, previously 13. Reinikainen KJ, Keranen T, Halonen T, et al.
untreated epilepsy. Epilepsy Res. 1989;3:70–6. Comparison of oxcarbazepine and carbamazepine: a
4. Glauser TA, Cnaan A, Shinnar S, et al. Ethosux- double-blind study. Epilepsy Res. 1987;1:284–9.
imide, valproic acid, and lamotrigine in childhood 14. Riss J, Cloyd J, Gates J, et al. Benzodiazepines in
absence epilepsy. N Engl J Med. 2010;362:790–9. epilepsy: pharmacology and pharmacokinetics. Acta
5. Guerreiro MM, Vigonius U, Pohlmann H, et al. Neurol Scand. 2008;118:69–86.
A double-blind controlled clinical trial of oxcar- 15. Vajda FJ, O’Brien TJ, Hitchcock A, et al. Critical
bazepine versus phenytoin in children and adoles- relationship between sodium valproate dose and
cents with epilepsy. Epilepsy Res. 1997;27:205–13. human teratogenicity: results of the Australian reg-
6. Koch MW, Polman SK. Oxcarbazepine versus ister of anti-epileptic drugs in pregnancy. J Clin
carbamazepine monotherapy for partial onset Neurosci. 2004;11:854–8.
New Generation Antiepileptic Drugs
17
Bassel Abou-Khalil

The new-generation antiepileptic drugs (AEDs) half-life in monotherapy is 20–23 h. The half-life
became available in the USA after 1993, fol- is shorter in children and also shorter in the
lowing a 15-year hiatus during which no new presence of enzyme inducers. Felbamate has
drugs were introduced for the treatment of epi- many interactions. It is an inhibitor of CYP
lepsy. In general, the newer AEDs have not 2C19, CYP 1A2, and beta-oxidation. As a result,
improved on the efficacy of carbamazepine, the it inhibits the metabolism and increases the
AED to which they have been most often com- serum concentration of phenobarbital, phenytoin,
pared. However, many of the newer antiepileptic valproate, carbamazepine epoxide, and warfarin.
drugs have advantages in terms of pharmacoki- On the other hand, felbamate induces CYP 3A4
netics (Table 17.1), interactions, and tolerability and thus decreases carbamazepine level and also
[1, 2]. The new drugs will be discussed in the reduces oral contraceptive efficacy.
order that they were introduced to the US market. Felbamate is affected by enzyme-inducing
The exceptions are oxcarbazepine and clobazam, antiepileptic drugs which accelerate felbamate
which were discussed in the previous chapter. clearance and reduce its serum concentration.
Common adverse effects of felbamate are
anorexia, nausea, vomiting, and weight loss.
Felbamate Stomach irritation can be improved by adminis-
tration with food and by the use of H2 blockers
Felbamate was first approved in the USA in or proton pump inhibitors. Felbamate may also
1993. Felbamate has several mechanisms of cause insomnia, irritability, and headache.
action, including NMDA antagonism, enhancing Felbamate was discovered to have serious
GABA, blocking sodium channels, and blocking idiosyncratic potential adverse effects. It may
high-voltage activated calcium channels. cause aplastic anemia with an estimated risk of 1
It has excellent oral bioavailability, greater in 5000–8000 patients. Aplastic anemia has not
than 90%. It is only 25% protein-bound, which is been reported below age 13. The onset of aplastic
not clinically significant. It is metabolized via anemia is after 2.5–6 months of treatment. It is
CYP 3A4. Between 40 and 50% of the absorbed highly unlikely to occur after one year of treat-
dose appears unchanged in the urine and the rest ment. It has risk factors including prior cytope-
as inactive metabolites and conjugates. Its nia, allergy to or significant toxicity with other
antiepileptic drugs, and underlying autoimmune
disease [3]. Another serious idiosyncratic
potential adverse effect is hepatic failure, with an
B. Abou-Khalil (&)
estimated risk of 1 26,000-1 and 54,000. The
Department of Neurology, Vanderbilt University
Medical Center, Nashville, TN, USA
e-mail: bassel.abou-khalil@vanderbilt.edu

© Springer Science+Business Media LLC 2017 225


M.Z. Koubeissi and N.J. Azar (eds.), Epilepsy Board Review,
DOI 10.1007/978-1-4939-6774-2_17
226 B. Abou-Khalil

Table 17.1 Select pharmacokinetic parameters of new-generation antiepileptic drugs


AED Felbamate Gabapentin Lamotrigine Topiramate Tiagabine Levetiracetam
Protein bindinga Low Low Intermediate Low High Low
b
Half-life Intermediate Short Intermediate Intermediate Short Short
Metabolism *50% None Extensive *30% Extensive *30%
Not hepatic
Enzyme + – – + – –
induction CYP 3A4 CYP 3A4
Enzyme +++ – – + – –
inhibition CYP 2C19
AED Zonisamide Pregabalin Lacosamide Vigabatrin Rufinamide Ezogabine
a
Protein binding Low Low Low Low Intermediate Intermediate
Half-lifeb Long Short Intermediate Intermediatec Short Long
Metabolism *65% None *60% None Extensive Extensive
Enzyme – – – + + –
induction CYP 2C9 UDP–GT
Enzyme – – – – + –
inhibition CYP 2E1
AED Perampanel Eslicarbazepine
a
Protein binding High Low
b
Half-life Long Intermediate
Metabolism Extensive *40%
Enzyme induction – +
CYP 3A4
Enzyme inhibition – +
CYP 2C19
a
Low <50%; intermediate: 50–85%; high >85%
b
Short <10 h; intermediate 10–30 h; long >30 h
c
Vigabatrin duration of action is longer than predicted by half-life

onset of this toxicity has been after 25–959 days adjunctive therapy in the treatment of partial sei-
of treatment, with a mean of 217 days. Neither zures, with or without generalization, in adults
aplastic anemia nor liver failure can be prevented with epilepsy and adjunctive therapy in the treat-
by monitoring of CBC and liver enzymes. Nev- ment of partial and generalized seizures associated
ertheless, it is recommended to check CBC and with Lennox-Gastaut syndrome in children.”
liver function tests prior to starting felbamate, The FDA indication specifies that “felbamate is
then every 2 weeks for 6 months, then every 2– not indicated as a first-line treatment; it is rec-
3 months after 6 months, and then every ommended only in those who respond inade-
6 months after the first year. quately to alternative treatments and whose
Felbamate is a wide-spectrum antiepileptic epilepsy is so severe that the risk of aplastic ane-
drug, although its efficacy in generalized seizure mia and/or liver failure is deemed acceptable.” A
types of idiopathic generalized epilepsy has not written informed consent is needed.
been evaluated in class I studies. The official The suggested felbamate therapeutic range is
FDA indications are: “Either monotherapy or 40–100 mg/L.
17 New-Generation Antiepileptic Drugs 227

Gabapentin Lamotrigine

Gabapentin was first approved in the USA in 1994. Lamotrigine was first approved in the USA in
The mechanism of action is binding to the alpha-2 1995, but was licensed in Europe in 1991. Its
delta subunit of voltage-gated calcium channels. mechanism of action is blocking sodium chan-
This binding reduces the influx of calcium and nels. This secondarily results in blocking the
reduces neurotransmitter release under hyperex- release of glutamate. Lamotrigine also inhibits
citable conditions. Despite its name, gabapentin high-voltage activated calcium channels.
does not interact with GABA receptors. Lamotrigine has an excellent oral bioavail-
Gabapentin bioavailability is low, with con- ability of about 98%. The time to maximum
siderable inter-subjective variability. In addition, concentration is 1–1.5 h with the immediate
oral bioavailability decreases with increasing release preparation and 4–11 h with the
gabapentin dose. For example, bioavailability is extended-release preparation. Its protein binding
60% after a single 300 mg dose, but only 29% is only 55%.
for 1600 mg t.i.d. and 36% for 1200 mg p.o. q.i. Lamotrigine is extensively metabolized in the
d. (bioavailability improves with dividing the liver, predominantly by glucuronidation (to lam-
dose) [4]. The reason for the above is that otrigine 2–N-glucuronide), then excreted by the
gabapentin is transported from the gut into the kidney. About 94% is eliminated in the urine,
bloodstream by the L-amino acid transport sys- about 10% as unchanged drug, and 90% as glu-
tem, which is saturable. Gabapentin protein curonide conjugates. The half-life is about 24 h in
binding is minimal at less than 50%. monotherapy, 48–60 h when used with valproate,
Gabapentin is not metabolized in humans. It is and 12 h when used with an enzyme inducer.
eliminated unchanged in the urine as a result Lamotrigine is associated with a mild autoin-
requires dose reduction with renal impairment. duction. It is a weak inhibitor of dihydrofolate
Its half-life is 5–7 h. reductase (not clinically relevant). Lamotrigine
Gabapentin has no known interactions, which slightly increases topiramate level and slightly
is predicted by the absence of metabolism, the decreases valproate level. However, it is mark-
absence of enzyme induction or inhibition, and edly affected by some other drugs. Its clearance is
the absence of protein binding. However, antacids increased in the presence of enzyme-inducing
including aluminum hydroxide or magnesium drugs, estrogen containing oral contraceptives,
hydroxide may reduce gabapentin bioavailability and pregnancy. On the other hand, lamotrigine
if taken within 2 h from gabapentin intake. clearance is markedly decreased by valproate.
Gabapentin adverse effects include sedation, Dose-related adverse effects include dizziness,
dizziness, ataxia, asthenia, and weight gain. It ataxia, blurred vision, diplopia, nausea, and
may cause myoclonus. It may be associated with vomiting. Headache and tremor may also occur.
cognitive slowing in the elderly, and emotional Rash is seen in about 3%, but the risk is higher in
lability or hostility in children. It has been children, with coadministration of valproate, with
assigned pregnancy category C. faster titration, and with higher doses. As a result
Gabapentin is a narrow-spectrum agent of increased risk of rash with faster titration,
against focal seizures. It failed clinical trials lamotrigine has to be titrated very slowly. The rate
against absence and primary generalized tonic– of titration is slower in the presence of valproate.
clonic seizures [5, 6]. It may cause exacerbation Rare serious idiosyncratic adverse effects
of myoclonus [7]. The official FDA indication is include Stevens–Johnson syndrome, toxic epi-
for adjunctive therapy for partial seizures. It is dermal necrolysis, or hypersensitivity syndrome
also approved for the treatment of postherpetic (1 in 4000). It is assigned pregnancy category C.
neuralgia. An extended-release preparation Lamotrigine is a wide-spectrum antiepileptic
(gabapentin enacarbil) has been approved for the drug effective against partial–onset as well as
treatment of restless leg syndrome. generalized tonic–clonic seizures. It is FDA
228 B. Abou-Khalil

indicated as adjunctive therapy or for conversion word-finding difficulty, and cognitive slowing.
to monotherapy for partial seizures, adjunctive There may be depression. Kidney stones occur in
therapy for generalized tonic–clonic seizures, and about 1.5% of individuals. Acute myopia and
adjunctive therapy for Lennox–Gastaut syndrome. secondary angle-closure glaucoma are reported
Efficacy against generalized absence seizures is rarely. Paresthesias in the hands and feet may
less than valproate and ethosuximide. Its efficacy occur as a result of the carbonic anhydrase activ-
against myoclonic seizures is variable, and lam- ity. Oligohydrolysis, hyperthermia, and metabolic
otrigine may exacerbate myoclonic seizures in acidosis are more common in children. Weight
some individuals. The recommended therapeutic loss may occur.
range for lamotrigine is 2–20 mg/L [8]. Topiramate is assigned pregnancy category D
Lamotrigine is also FDA indicated for main- due to increased risk of oral clefts in exposed
tenance treatment of bipolar I disorder to delay a infants [9].
mood episode. Topiramate is a wide-spectrum antiepileptic
drug. However, it is not effective against general-
ized absence seizures as demonstrated in a ran-
Topiramate domized controlled clinical trial [10]. The FDA
indications are for initial monotherapy or adjunc-
Topiramate is a sulfamate-substituted monosac- tive therapy for partial–onset or primary general-
charide. It was approved in the USA in 1996. It has ized tonic–clonic seizures in adults and children
multiple mechanisms of action including blocking 2 years or older, and as adjunctive therapy for adult
of voltage-gated sodium channels, augmentation and pediatric patients with seizures associated with
of GABA activity, antagonism of AMPA/kainate Lennox–Gastaut syndrome. Topiramate is also
receptors, inhibition of high-voltage activated indicated for prophylaxis of migraine. Topiramate
calcium channels, and weak inhibition of carbonic requires a slow titration to improve tolerability.
anhydrase activity.
Topiramate has a good oral bioavailability of
80–95%. Its protein binding is only 15–40%. Tiagabine
Topiramate is not extensively metabolized.
About 70% is eliminated unchanged in the urine. Tiagabine was first approved in the USA in 1997.
Its hepatic metabolism by the p450 enzyme It is a designer drug, the mechanism of which is
system is via hydroxylation, hydrolysis, and inhibition of GABA reuptake at the synapse.
glucuronidation, to form inactive metabolites. Tiagabine has an excellent oral bioavailabil-
There is evidence of renal tubular reabsorption. ity. It is highly protein-bound (96%). It is
The half-life is about 21 h. extensively metabolized in the liver mainly by
Drug interactions are minimal. Topiramate is CYP 3A4. Only 2% is excreted unchanged; 63%
a mild inhibitor of CYP 2C19, so that it may is excreted in the feces and 25% in the urine. The
increase phenytoin levels when used at a higher half-life is 7–9 h in monotherapy and 2–5 h in
dose. It is also a mild inducer of CYP 3A4, so the presence of enzyme inducers. As a result of
that it may reduce the efficacy of the oral con- the short half-life, it requires t.i.d. dosing.
traceptive when used at a dose of 200 mg per day Tiagabine does not affect other medications.
or more. Hyperammonemia may occur when Even though it is highly protein-bound, its serum
topiramate is used in conjunction with valproate. concentration is low and it is unlikely to compete
Enzyme-inducing antiepileptic drugs may reduce for protein binding; in addition, tiagabine dosing
topiramate levels by up to 50%. decisions are not usually based on serum con-
Topiramate adverse effects include sedation, centration. Tiagabine metabolism, however, is
fatigue, dizziness, and ataxia, which are helped by accelerated by enzyme-inducing drugs.
slower titration. Topiramate may cause cognitive The most commonly reported tiagabine
difficulties including memory disturbance, adverse effects were dizziness, asthenia,
17 New-Generation Antiepileptic Drugs 229

nervousness, tremor, depression, and emotional levetiracetam titration [13]. There have been rare
lability. Adverse effects are more common dur- reports of psychosis [14].
ing titration, and slow titration is thus required. Levetiracetam is a wide-spectrum drug. The
Tiagabine may be associated with dose-related official FDA indications are adjunctive therapy
episodes of nonconvulsive status epilepticus or for partial–onset seizures in adults and children
encephalopathy, which may occur even in the 4 years or older; adjunctive therapy for myo-
absence of epilepsy [11, 12]. It has been assigned clonic seizures in adults and adolescents 12 years
pregnancy category C. or older with juvenile myoclonic epilepsy; and
Tiagabine is a narrow-spectrum agent effec- adjunctive therapy for primary generalized tonic–
tive against focal seizures only. It is not effective clonic seizures in adults and children 6 years or
against and may exacerbate generalized absence older with idiopathic generalized epilepsy.
or myoclonic seizures. Its FDA indication is for Levetiracetam is not FDA approved for
adjunctive therapy only. It is used off label in the monotherapy in the USA, but it is approved for
treatment of addiction, to increase proportion of initial monotherapy in Europe. The optimal
deep sleep, and in the management of spasticity therapeutic level is unknown. One study sug-
in multiple sclerosis. gested that 11 mg/L may be a threshold con-
centration for therapeutic response [15]. The
upper limit of the therapeutic range is unknown.
Levetiracetam

Levetiracetam was first approved in USA in 1999. Zonisamide


Its mechanism of action is binding to the synaptic
vesicle protein SV2A. This seems to result in Zonisamide is structurally related to sulfon-
nonspecific decrease in neurotransmitter release. amides. It was approved in Japan in 1989. How-
There is a functional correlation between SV2A ever, it was first approved in the USA in 2000.
binding affinity and anticonvulsant potency of The mechanism of action is blocking sodium
levetiracetam analogues. channels, reducing T-type calcium currents, and
Levetiracetam is available in oral and IV weak inhibition of carbonic anhydrase activity (it
formulations. It has an excellent oral bioavail- is 100–200 times less potent than acetazolamide).
ability of about 100%. Time to maximum con- Oral bioavailability is about 100%. Protein
centration is about 1 h (1.5 h with food). Its binding is only 40–50%. It is metabolized in the
protein binding is less than 10%. liver by acetylation and reduction, mediated by
Levetiracetam has no hepatic metabolism. It is CYP 3A4, then glucuronidation. Its metabolites
partly hydrolyzed to inactive compounds; 66% is are inactive and cleared by renal excretion. It has
excreted unchanged in the urine. The half-life is a long half-life of about 60 h.
6–8 h, but shorter in children and longer in the Zonisamide is not a hepatic enzyme inducer
elderly. or inhibitor and has no effect on the pharma-
There are no known significant pharmacoki- cokinetics of other commonly used antiepileptic
netic interactions. However, some studies have drugs. However, it is affected by CYP 3A4
suggested lower levetiracetam levels in the inducers or inhibitors. The addition of enzyme-
presence of enzyme inducers. inducing antiepileptic drugs decreases zon-
Levetiracetam adverse effects include som- isamide half-life and plasma level. On the other
nolence, dizziness, and asthenia. Irritability and hand, zonisamide concentration is increased by
hostility may occur, more commonly in children. CYP 3A4 inhibitors such as ketoconazole or
Risk factors for these behavioral adverse effects cyclosporine.
include symptomatic generalized epilepsy, his- Zonisamide adverse effects include sedation,
tory of psychiatric diagnosis, and faster ataxia, dizziness, nausea, fatigue, agitation/
230 B. Abou-Khalil

irritability, and anorexia. Weight loss may occur. particularly with higher doses. Pregabalin is
Cognitive slowing and difficulty with concen- classified with pregnancy category C.
tration may be seen, particularly at higher doses. Pregabalin is a narrow-spectrum drug against
Kidney stones occur as frequently as 4%. Rarely, partial–onset seizures. The official FDA epilepsy
depression and psychosis may occur. Serious indication is an adjunctive therapy for adult
rash such as Stevens–Johnson syndrome and patients with partial–onset seizures. Pregabalin is
toxic epidermal necrolysis occur rarely. Oligo- also indicated for neuropathic pain associated
hydrolysis, hyperthermia, and metabolic acidosis with diabetic peripheral neuropathy, postherpetic
may occur, more often in children. Zonisamide neuralgia, and fibromyalgia.
was assigned pregnancy category C.
Zonisamide is a wide-spectrum antiepileptic
drug but has undergone class I trials only for Lacosamide
partial–onset seizures. The official FDA indica-
tion is for adjunctive therapy in the treatment of Lacosamide was first approved in the USA in
partial seizures in adults with epilepsy. In Eur- 2008. Its mechanism of action is enhancing slow
ope, it is indicated as initial monotherapy for inactivation of sodium channels. This is to be
partial seizures. In Japan, it is also indicated as distinguished from other antiepileptic drugs that
monotherapy for generalized seizures. The sug- interact with the sodium channel, all of which
gested therapeutic range is 10–14 mg/L. enhance fast inactivation of sodium channels.
Lacosamide is available in oral and intra-
venous formulations. The oral bioavailability is
Pregabalin about 100%. Protein binding is less than 15%.
Lacosamide is metabolized by demethylation
Pregabalin was first approved in USA in 2005. in the liver to inactive O-desmethyl metabolite
Its mechanism of action is similar to gabapentin. via CYP 2C19. Approximately 95% is excreted
It binds to the alpha-2 delta subunit of voltage- in the urine, 40% as unchanged drug, and 30% as
gated calcium channels, reducing the influx of O-desmethyl metabolite. The half-life is
calcium and reducing neurotransmitter release approximately 13 h.
under hyperexcitable conditions. Lacosamide has no known pharmacokinetic
Unlike gabapentin, pregabalin has very good interactions despite the CYP 2C19 metabolism.
oral bioavailability, greater than 90%. The However, it does have pharmacodynamic inter-
bioavailability is also independent of dose. The actions with other antiepileptic drugs that act on
time to maximum concentration is 1 h, but is the sodium channel.
delayed up to 3 h when ingested with food. It has The dose-related adverse effects include
no protein binding. dizziness, headache, nausea, diplopia, and seda-
Pregabalin is not metabolized in humans. It is tion. All these are more likely when lacosamide
excreted unchanged in the urine, thus requiring is used in conjunction with other sodium channel
dose reduction in patients with renal impairment. blockers. It may also cause a small asymptomatic
Pregabalin half-life is about 6 h. increase in the PR interval.
Pregabalin has no known pharmacokinetic Lacosamide is a narrow-spectrum antiepilep-
interactions, which is predicted by the absence of tic drug against partial–onset seizures. The offi-
metabolism, the absence of enzyme induction or cial FDA indication is for adjunctive therapy of
inhibition, and the absence of protein binding. partial–onset seizures in patients 17 years or
Pregabalin can cause increased appetite and older. The parenteral formulation is indicated as
weight gain. There may be peripheral edema. short-term replacement when oral administration
Myoclonus may occur in some individuals, is not feasible in patients taking oral lacosamide.
17 New-Generation Antiepileptic Drugs 231

Vigabatrin outweigh the risk of vision loss” and


“monotherapy for pediatric patients with infantile
Vigabatrin was initially licensed in Europe in spasms one month to 2 years of age for whom
1989, but was first approved in the USA in 2009. the potential benefits outweigh the potential risk
Its mechanism of action is irreversible inhibition of vision loss.” Because of the visual toxicity,
of GABA transaminase, resulting in the accu- treatment with vigabatrin should be continued
mulation of GABA. only if there is considerable benefit observed in
Vigabatrin has excellent oral bioavailability, the first 3 months of treatment.
which is nearly complete. It has no protein
binding.
It is not significantly metabolized and is Rufinamide
eliminated unchanged in the urine. The half-life
is 10.5 h in young adults and 5–6 h in infants. Rufinamide was first approved in USA in 2008.
Vigabatrin is a weak inducer of CYP 2C9. Its mechanism of action is binding to the sodium
This results in slight reduction of phenytoin channels, prolonging the inactive state of sodium
levels with the addition of vigabatrin. channels.
Vigabatrin adverse effects include sedation, Oral bioavailability is about 85% with food,
fatigue, dizziness, and ataxia. There may be but less without food. Food increases the
irritability, behavioral changes, psychosis, and absorption by more than 30%. Protein binding is
depression. Weight gain may occur. The most about 55%.
concerning adverse effect is bilateral concentric Rufinamide is metabolized by enzymatic
visual field constriction, which is progressive and hydrolysis to an inactive metabolite. This is not
permanent. This occurs in up to 30–40% of dependent on the P450 enzyme system. The
individuals [16]. The risk increases with inactive metabolites are eliminated by excretion in
increased daily dose and increased duration of the urine. The half-life is approximately 6–10 h.
therapy [17]. As a result of the retinal visual Rufinamide is a weak inhibitor of CYP 2E1
toxicity, periodic visual assessment is required at (it increases olanzapine level) and a weak
baseline and every 3 months. In cooperative inducer of CYP 3A4 (it decreases oral contra-
adult and pediatric patients, the monitoring can ceptive efficacy). Rufinamide is a weak inducer
be accomplished with perimetry. Optional testing of UDP–GT (it increases the clearance of
includes electroretinography and retinal imaging lamotrigine). The addition of enzyme-inducing
with optical coherence tomography. MRI chan- antiepileptic drugs increases rufinamide clear-
ges may occur in treated infants, consisting of ance and decreases rufinamide level. On the
increased T2 and restricted diffusion in deep other hand, the addition of valproate decreases
white matter, basal ganglia, thalamus, and corpus rufinamide clearance and increases rufinamide
callosum. These MRI changes are asymptomatic levels by up to 70%.
and reversible. Vigabatrin was assigned preg- Rufinamide adverse effects include dizziness,
nancy category C. fatigue, somnolence, and headache. Vomiting
Vigabatrin is a narrow-spectrum drug effec- may occur in children. Rufinamide may cause a
tive against focal seizures. It may worsen shortening of the QT interval. It was assigned
absence and myoclonic seizures in idiopathic pregnancy category C.
generalized epilepsy [7]. The official FDA indi- Rufinamide is FDA indicated as adjunctive
cations are “adjunctive therapy for adults and treatment of seizures associated with Lennox–
pediatric patients 10 years of age or older with Gastaut syndrome in children 4 years and older
refractory complex partial seizures who have and adults. Although rufinamide was found to be
inadequately responded to several alternative effective for partial seizures, it has not been FDA
treatments and for whom the potential benefits approved for this indication.
232 B. Abou-Khalil

Ezogabine (Known as Retigabine Perampanel has excellent oral bioavailability


Outside the USA) of about 100%. It is 95% protein-bound.
Perampanel is extensively metabolized by
Ezogabine was first approved in the USA in primary oxidation mediated by CYP 3A4, fol-
2011. It has a normal mechanism of action as a lowed by glucuronidation. It is excreted as inac-
potassium channel opener. It enhances the tive metabolites, 30% in the urine and 70% in the
activity and prolongs the opening of neuron feces. It has a long half-life of 105 h on average.
specific KCNQ2/3 (Kv7.2/7.3) voltage-gated Perampanel does not affect other antiepileptic
potassium channels, thereby activating the M drugs. However, perampanel at a dose of 12 mg
current. Ezogabine also potentiates GABA- per day reduces levonorgestrel by about 40%.
evoked currents in cortical neurons at much This effect is not seen at the dose of 8 mg per
higher concentration than that needed to activate day. Enzyme inducers will decrease perampanel
potassium currents. levels.
Ezogabine oral bioavailability is about 60%. It Perampanel adverse effects include dizziness,
is about 80% protein-bound. somnolence, headache, fatigue, ataxia, and blur-
Ezogabine is extensively metabolized, pri- red vision. Aggression and hostility may occur—
marily via glucuronidation and acetylation. It is at a dose of 12 mg per day, its incidence is
metabolized to the active N-acetyl metabolite estimated at 20%. Perampanel is assigned preg-
(NAMR), which is also subsequently glu- nancy category C.
curonidated. About 85% of the absorbed dose is Perampanel is FDA indicated for the treat-
recovered in the urine, 36% as unchanged ezo- ment of partial–onset seizures with or without
gabine, and 18% as NAMR. The half-life is secondary generalization in patients with epi-
7–11 h for both ezogabine and NAMR. lepsy aged 12 years or older. It is not yet known
Ezogabine does not significantly affect other whether perampanel is effective against general-
antiepileptic drugs except for a 22% increase in ized onset seizures.
lamotrigine clearance. NAMR may inhibit
renal clearance of digoxin. Enzyme-inducing
antiepileptic drugs reduce ezogabine levels. Eslicarbazepine
The most common ezogabine adverse effects
are dizziness, somnolence, fatigue, confusion, Eslicarbazepine acetate was approved for mar-
blurred vision, tremor, and nausea, most often keting in the USA in 2014. It is rapidly converted
during titration. Urinary retention may occur. to the active metabolite (S)-licarbazepine by
There may be QT prolongation. Long-term use hydrolytic first-pass metabolism. (S)-licarbaze-
has been associated with skin, nail, and retinal pine is the active enantiomer of the monohy-
pigmentation. Weight gain may occur. Ezo- droxy derivative, which is the active metabolite
gabine was assigned pregnancy category C. for oxcarbazepine. The monohydroxy derivative
Ezogabine is FDA indicated for adjunctive from oxcarbazepine is a racemic mixture of the
treatment of partial–onset seizures in patients active (S)-licarbazepine and the inactive (R)-
aged 18 years and older. It is not known whether licarbazepine. Eslicarbazepine acts by blocking
ezogabine is effective against other seizure types. sodium channels and stabilizing the inactive state
of the voltage-gated sodium channel.
Eslicarbazepine has excellent bioavailability,
Perampanel greater than 90%. Its protein binding is less than
40%.
Perampanel was approved in the USA in 2012. Eslicarbazepine is metabolized to inactive
Its mechanism of action is noncompetitive compounds. About 60% of the absorbed dose is
antagonism of AMPA glutamate receptors. excreted in the urine as unchanged eslicarbazepine,
17 New-Generation Antiepileptic Drugs 233

30% as the glucuronide conjugates, and 10% as frequency of administration in adult patients with
other metabolites. The half-life of eslicarbazepine epilepsy. Epilepsy Res. 1998;31:91–9.
5. Chadwick D, Leiderman DB, Sauermann W, et al.
is 13–20 h in plasma and 20–24 h in CSF, justi- Gabapentin in generalized seizures. Epilepsy Res.
fying once daily dosing used in clinical trials. 1996;25:191–7.
Eslicarbazepine is not subject to autoinduc- 6. Trudeau V, Myers S, LaMoreaux L, et al. Gabapentin
tion. However, it can induce CYP 3A4, thus in naive childhood absence epilepsy: results from two
double-blind, placebo-controlled, multicenter studies.
decreasing plasma concentrations of estrogen and J Child Neurol. 1996;11:470–5.
drugs metabolized by this enzyme. It also has a 7. Perucca E, Gram L, Avanzini G, et al. Antiepileptic
moderate inhibitory effect on CYP 2C19, drugs as a cause of worsening seizures. Epilepsia.
potentially increasing plasma concentration of 1998;39:5–17.
8. Hirsch LJ, Weintraub D, Du Y, et al. Correlating
phenytoin and other drugs metabolized by this lamotrigine serum concentrations with tolerability in
enzyme. Enzyme inducers may reduce her esli- patients with epilepsy. Neurology. 2004;63:1022–6.
carbazepine serum concentration. 9. Hunt S, Russell A, Smithson WH, et al. Topiramate
The most common eslicarbazepine adverse in pregnancy: preliminary experience from the UK
epilepsy and pregnancy register. Neurology.
effects are dizziness, headache, diplopia, som- 2008;71:272–6.
nolence, vertigo, nausea, vomiting, fatigue, and 10. Pina-Garza JE, Schwarzman L, Wiegand F, et al.
ataxia. Hyponatremia (defined as less than A pilot study of topiramate in childhood absence
125 mEq per liter) is reported in up to 1.5% of epilepsy. Acta Neurol Scand. 2011;123:54–9.
11. Azar NJ, Bangalore-Vittal N, Arain A, et al.
individuals taking 1200 mg per day. Rash occurs Tiagabine-induced stupor in patients with psy-
in up to 3% of individuals at 1200 mg per day. chogenic nonepileptic seizures: nonconvulsive status
Eslicarbazepine is effective against partial– epilepticus or encephalopathy? Epilepsy Behav.
2013;27:330–2.
onset (focal) seizures. It is currently FDA indi-
12. Koepp MJ, Edwards M, Collins J, et al. Status
cated as adjunctive treatment for partial–onset epilepticus and tiagabine therapy revisited. Epilepsia.
seizures. Monotherapy trials have been com- 2005;46:1625–32.
pleted but have not yet been considered to 13. White JR, Walczak TS, Leppik IE, et al. Discontin-
uation of levetiracetam because of behavioral side
change the FDA indication.
effects: a case-control study. Neurology.
2003;61:1218–21.
14. Kossoff EH, Bergey GK, Freeman JM, et al. Leve-
References tiracetam psychosis in children with epilepsy. Epilep-
sia. 2001;42:1611–3.
15. Lancelin F, Franchon E, Kraoul L, et al. Therapeutic
1. Marson AG, Al-Kharusi AM, Alwaidh M, et al. drug monitoring of levetiracetam by
The SANAD study of effectiveness of carba- high-performance liquid chromatography with pho-
mazepine, gabapentin, lamotrigine, oxcarbazepine, todiode array ultraviolet detection: preliminary obser-
or topiramate for treatment of partial epilepsy: an vations on correlation between plasma concentration
unblinded randomised controlled trial. Lancet. and clinical response in patients with refractory
2007;369:1000–15. epilepsy. Ther Drug Monit. 2007;29:576–83.
2. Rowan AJ, Ramsay RE, Collins JF, et al. New onset 16. Kalviainen R, Nousiainen I, Mantyjarvi M, et al.
geriatric epilepsy: a randomized study of gabapentin, Vigabatrin, a gabaergic antiepileptic drug, causes
lamotrigine, and carbamazepine. Neurology. concentric visual field defects. Neurology.
2005;64:1868–73. 1999;53:922–6.
3. Pellock JM, Faught E, Leppik IE, et al. Felbamate: 17. Toggweiler S, Wieser HG. Concentric visual field
consensus of current clinical experience. Epilepsy restriction under vigabatrin therapy: extent depends on
Res. 2006;71:89–101. the duration of drug intake. Seizure. 2001;10:420–3.
4. Gidal BE, DeCerce J, Bockbrader HN, et al.
Gabapentin bioavailability: effect of dose and
Dietary Therapies
18
Amy Kao

was the production of “Do No Harm,” directed


History
by Charlie’s father and narrated by and starring
Meryl Streep in 1997. Reflective of the subse-
In the Hippocratic medical writings of ancient
quent increase in interest is the increase in
Greece from 500 BC, fasting was described as
medical literature; there were 2–8 publications
the treatment for epilepsy. Then in biblical times,
per year between 1970 and 2000 on the KD
it was again mentioned as an effective cure for
and >40 per year since [1]. There are 102 KD
epilepsy. In 1911, two physicians in Paris used
programs listed on the Charlie Foundation Web
starvation in 20 children and adults and found
site (www.charliefoundation.org); the majority
that the severity of their seizures decreased. In
are in North America, but centers are also located
1921, Wilder at the Mayo Clinic proposed a diet
in Europe, South America, Asia, and Australia.
consisting of excessive fat and sparse carbohy-
drates, which would produce ketones, just as
starvation does, and coined the term “ketogenic
diet.” The ketogenic diet was subsequently
Overview of Types of Dietary
extensively used as an epilepsy treatment.
Therapies
In 1938, the discovery of diphenylhydantoin
Ketogenic Diet (“Classic”)
lessened the interest in the KD. In the late 1980s,
after learning about the KD, a family of a
The KD simulates the ketosis seen with starva-
20-month-old boy with refractory epilepsy des-
tion while providing necessary calories for
perately approached the Johns Hopkins Hospital,
growth and development. When a “normal” diet
requesting the treatment; the KD was effective
is consumed, glucose is the sole source of energy
for him. There was subsequently a Dateline
for the brain; fatty acids do not cross the blood–
episode on the effectiveness of the KD for this
brain barrier. When carbohydrate consumption is
boy, Charlie. The family also established the
limited, glucose supplies are low, so fat is then
Charlie Foundation, a nonprofit organization that
used as the alternative source of energy. Specif-
began providing information to parents and
ically, when glucose is low, oxaloacetate is
instructional videos to practitioners and dieti-
shunted from the Krebs cycle to gluconeogene-
tians. Further publicizing and supporting the KD
sis, to produce and maintain glucose levels. This
decreases the efficiency of the Krebs cycle to
metabolize the abundant acetyl-coA generated
A. Kao (&) from fatty acid metabolism, so acetyl-coA is
Center for Neurosciences and Behavioral Medicine, instead converted to ketone bodies, specifically
Children’s National Health System, 111 Michigan
Ave NW, Washington, D.C. 20010, USA to acetoacetate, which is then degraded to ace-
e-mail: akao@childrensnational.org tone and also converted to beta-hydroxybutyrate.

© Springer Science+Business Media LLC 2017 235


M.Z. Koubeissi and N.J. Azar (eds.), Epilepsy Board Review,
DOI 10.1007/978-1-4939-6774-2_18
236 A. Kao

Ketone bodies can cross the blood–brain barrier carbohydrates. The MCT diet provides 60–70%
and so are used instead of glucose for energy [2]. calories from fat. The downside of the MCT diet
The “classic” KD utilizes long-chain fatty is gastrointestinal side effects including diarrhea,
acids (as opposed to medium-chain fatty acids) vomiting, and abdominal pain [4]. However, a
and is the most widely used dietary treatment. randomized controlled trial comparing the toler-
The composition of the diet is calculated using a ability and efficacy of the MCT diet versus
ratio of the weight of fat (in grams) to the sum of classical KD showed no significant differences in
the weight of protein and carbohydrates. Typical either [5].
ratios are 3:1 or 4:1 (compared to the ratio in a
standard North American diet which is 0.3:1). In a
3:1 ratio, there are 3 grams of fat for every 1 g of Modified Atkins Diet (MAD)
protein plus carbohydrate combined. The amount
of protein is calculated to meet dietary reference The MAD was developed at Johns Hopkins
intake which is 1 g per kilogram body weight, so Hospital, first reported in 2003, and aimed at
a 3:1 ratio is typically used for older children with children with behavioral difficulties, adolescents,
greater body mass. Calories are also measured, and adults [6]. The standard Atkins diet has a
such that they are sufficient to support growth but goal of weight loss and includes an induction
controlled to prevent excessive weight gain. phase in which there is limitation of carbohy-
Roughly 90 % of the calories are obtained from drates to induce ketosis. In the MAD, this initial
fat consumption in the classic KD. phase of carbohydrate restriction is continued,
Historically, caloric restriction, fluid restric- and weight loss is not encouraged unless nutri-
tion, and an initial fasting period of 24–48 h, tionally indicated.
until large ketones are demonstrated, have been The fat to protein plus carbohydrate ratio of
features of the KD, but there is limited evidence the MAD is 0.9:1, providing approximately 60–
that these are necessary. Initial fasting seems to 65% of calories from fat, 30% from protein
shorten the time to the first reported seizure (higher than the KD and “normal” diet), and 10%
reduction, but long-term outcomes are not from carbohydrates (higher than the KD). Based
impacted. A randomized controlled trial com- upon the protocol at Johns Hopkins, in children,
paring fasting versus gradual initiation (gradually initial carbohydrate restriction is to 10 grams per
increasing ratios from 1:1 to 2:1 to 3:1 to 4:1) day for a month, with liberalization to 15 g, then
showed equivalent efficacy at 3 months, with 20–30 g per day. Adults are started at 15 g of
decreased weight loss, decreased episodes of carbohydrates per day and then increased to
hypoglycemia, and decreased treatment neces- 20–30 g per day after a month. This initial
sary for acidosis or dehydration in those children stricter period is based upon data from a ran-
initiated without fasting [3]. domized, prospective cross-over design study
which showed higher incidence of >50% seizure
reduction at 3 months in patients who initially
Medium-Chain Triglyceride consumed 10 grams per day (60%) versus 20 g
(MCT) Diet per day (10%). The glycemic index of carbohy-
drates is not controlled. Fiber is subtracted from
In 1971, Peter Huttenlocher at the University of the total carbohydrate count.
Chicago introduced the MCT Diet. Whereas the Initiation of the MAD is done as an outpatient
classic KD uses standard foods as source of fat, process and no weighing of foods is required,
this utilizes MCT oil as a source of although counting carbohydrates is required.
medium-chain fatty acids, which is more easily Low-carbohydrate multivitamin and calcium
absorbed and delivered directly to the liver; supplementation is prescribed. Medications may
thereby, more efficiently generating ketones and be changed to lowest carbohydrate formulations.
allowing greater consumption of protein and Urine ketones are checked twice per week and
18 Dietary Therapies 237

may be elevated to the “large” level, but some- breadth of issues regarding the KD including the
times are lower than those generated by the KD. practical details of management.
Monitoring includes phone follow-up in a month This consensus statement states that the “KD
and clinic follow-up at 3 and 6 months, with should be strongly considered in a child who has
complete blood count (CBC), complete meta- failed 2–3 anticonvulsant therapies, regardless of
bolic panel (CMP), fasting lipid profile at base- age or gender, and particularly in those with
line, 3 and 6 months [7]. symptomatic generalized epilepsies.” In addition,
the committee members reviewed publications
on efficacy in particular conditions. For the KD
Low-Glycemic-Index Treatment to be considered as having probable benefit in
(LGIT) that condition, the existence of at least 2 publi-
cations was required; those included myoclonic
This is the least restrictive dietary therapy pre- astatic epilepsy, Dravet syndrome, tuberous
sently used in epilepsy management and, thus, sclerosis, Rett syndrome, and infantile spasms.
may be seen as more palatable and better toler- Formula feeding through gastrostomy tube or
ated than the other diets. It is also initiated in the bottle would also be a favorable factor, as it
outpatient setting, without weighing of foods. It would simplify food preparation and minimize
was first reported in 2005 at Massachusetts chances of intolerance.
General Hospital, based upon the theory that one Also, based upon specific metabolic abnor-
mechanism of the KD could be stabilization of malities, there are 2 conditions in which the KD
blood sugar levels. This arose from observations could be considered treatment of choice, to be
that seizure control of children on the KD can be used before 2 or 3 antiseizure medications have
very sensitive to intake of extra carbohydrates, failed. Those would be glucose transporter defi-
and that blood glucose levels are extremely ciency syndrome, in which glucose transport
stable in children on the KD. The LGIT allows across the blood–brain barrier is impaired, and
higher carbohydrate intake than the other 2 diets pyruvate dehydrogenase deficiency in which
but limits the type of carbohydrates to those that pyruvate cannot be metabolized into acetyl-coA.
are low in glycemic index, foods that result in Other conditions in which the KD may pos-
lower postprandial blood sugar and insulin pro- sibly be beneficial (based upon single case report
files. Larger particle size, less gelatinization of a or case series) include such conditions as Landau
starch, presence of fat, higher acidity, and Kleffner syndrome, Lafora body disease, suba-
increased fiber content lead to lower glycemic cute sclerosing panencephalitis, mitochondrial
indices. The type of starch is also a factor. respiratory chain complex disorders, phospho-
In LGIT, fat contributes 60% of calories, protein fructokinase deficiency, febrile infection-related
20–30%, and carbohydrates are limited to 40– epilepsy syndrome (FIRES)/status epilepticus,
60 g per day. All carbohydrates are foods with Lennox-Gastaut syndrome, hypoxic-ischemic
glycemic index less than 50 [8]. encephalopathy, and focal malformations of
cortical development.
Contraindications are essentially disorders
Patient Selection that involve fatty acid metabolism defects due to
various enzyme deficiencies. Long-chain fatty
In December 2006, the Charlie Foundation acids are shuttled across the outer and inner
commissioned a panel of 26 pediatric epileptol- mitochondrial membranes by carnitine, facili-
ogists and dietitians from 9 countries with tated by first carnitine palmitoyltransferase I,
expertise in the KD, to create a consensus then carnitine translocase, then CPT2, after
statement on the clinical management of the KD. which beta oxidation occurs within the mito-
It was endorsed by the Child Neurology Society chondrion, generating acetyl-coA, which then
and serves as an excellent reference for a wide enters the Krebs cycle or forms ketones. Pyruvate
238 A. Kao

carboxylase converts pyruvate to oxaloacetate, so model studies, has investigated this. Proposed
deficiency impairs Krebs cycle function. Thus, mechanisms have included numerous processes,
contraindications include primary carnitine defi- such as direct anticonvulsant effect of ketones/
ciency, carnitine palmitoyltransferase I or II free fatty acids, antioxidant/anti-inflammatory
deficiency, carnitine translocase deficiency, effects by decreasing reactive oxygen species,
Beta-oxidation defects (medium-chain acyl action on mitochondrial uncoupling proteins,
dehydrogenase deficiency or MCAD, long-chain increase of mitochondrial biogenesis, decrease of
acyl dehydrogenase deficiency or LCAD, glutamate, and increase of GABA.
short-chain acyl dehydrogenase deficiency or
SCAD), and pyruvate carboxylase deficiency.
The lack of carbohydrates in the KD can also Duration
exacerbate acute intermittent porphyria. Relative
contraindications include “failure to thrive,” an When the KD is the effective, evidence of benefit
identifiable surgical focus, and predictors of occurs fairly quickly. The range of time until
noncompliance such as lack of supervision, benefit is 1–65 days and is typically within the
access to or tendency toward forbidden foods, first 2 weeks after initiation. Similarly, in a
and lack of parental readiness and commitment prospective study in adults on the MAD, median
[9]. time to improvement was 2 weeks (range 1–8
Typically, an outpatient clinic visit is where weeks) [10]. Based on these data, continued KD
screening and patient selection takes place. administration for 3 months is encouraged before
Nutritional history is obtained, complicating deciding if the KD should be resumed or
medical factors and other potential barriers to discontinued.
success are identified, education regarding the If seizure freedom is obtained on the KD,
particular diet is given, and expectations and weaning and discontinuation of the KD is con-
goals are discussed. Laboratory testing is per- sidered after 2 years, similar to the approach with
formed to screen for metabolic contraindications; antiseizure medications. If seizure freedom is not
these may include complete blood count, elec- obtained, but improved seizure control occurs,
trolytes, magnesium, phosphorous, zinc, sele- the balance between benefit and long-term risks
nium, liver and kidney function tests, fasting needs to be considered when discussing duration
lipid profile, urinalysis, urine calcium/creatinine, of the KD.
urine organic acids, serum amino acids, and
acylcarnitine profile.
The concomitant use of carbonic anhydrase Complications
inhibitors is not a contraindication. Topiramate
and zonisamide may increase the likelihood of Short-term complications of the KD may include
profound acidosis and associated lethargy and vomiting, dehydration, hypoglycemia, and
vomiting during the period of initiation of the excessive acidosis. Therefore, patients are typi-
KD. However, they do not need to be discon- cally hospitalized for initiation of the KD. Con-
tinued before initiation of the KD. Daily enteral stipation is a frequent side effect, which is treated
citrate may be administered to counteract this by increasing hydration, giving conducive foods
acidosis. such as avocado, or using bulk-forming laxatives.
Monitoring for long-term side effects of the
KD must take place on a standard basis, at least
Mechanisms every 3 months. This includes measurements and
laboratory testing for weight and height, hyper-
The exact mechanisms by which dietary thera- lipidemia (fasting lipid profile), nutritional and
pies treat seizures have not been precisely electrolyte deficiencies (electrolytes with bicar-
delineated. Much research, including animal bonate, calcium, magnesium, phosphorous,
18 Dietary Therapies 239

complete blood count with platelets, free and 90% seizure reduction and 28% with >90% sei-
total carnitine, zinc selenium, liver function tests, zure reduction. In an adult study, efficacy was
vitamin D), and kidney stones (urinalysis, urine quick if present (median 2 weeks), with 47%
calcium and creatinine). Also, as urine ketone experiencing >50% seizure reduction by
testing is less accurate than serum testing, serum 3 months and 33% by 6 months [7(p39)].
beta-hydroxybutyrate levels may be drawn to
correlate with home urine results. Bone mineral
density scan, to assess for osteopenia/ Low-Glycemic-Index Treatment
osteoporosis, may be considered, particularly in
patients who are high risk (immobile, multiple Review of 60 patients has shown 38% of patients
antiseizure medications, history of fractures) and with a >50% decrease in seizures at 1 month,
on the KD longer than 2 years. with 24% having >90% seizure decrease. Of the
Evidence of longer-term adverse events with patients who continued on LGIT through
the MAD is limited. In theory, risk of growth 6 months, 60% had a >50% seizure decrease,
limitation, kidney stones, dyslipidemia, and and 38% had a >90% decrease in seizures [8
gastroesophageal reflux would be less common (p43)].
than with the KD. Approximately, 25–50 mg/dl
increases in total cholesterol have been noted.
Blood urea nitrogen also has been shown to Future Directions
increase, likely related to increased protein intake
[7(p39)]. The use of the KD is being investigated in sev-
eral neurologic conditions beyond epilepsy, and
in traumatic brain injury, Alzheimer’s disease,
Outcomes amyotrophic lateral sclerosis, autism, glial
tumors, diabetic nephropathy, and Parkinson’s
A Cochrane Review in 2012 yielded 4 random- disease. In addition, in development is 2-deoxy-
ized controlled trials (versus no randomized (D)-glucose (2-DG), an agent which is a non-
controlled trials in a similar review in 2003). metabolizable glucose analog that inhibits
These trials were heterogeneous, namely one glycolysis.
compared 3:1 and 4:1 ratios of the KD, one
compared MCT and classical KD, one KD versus
no diet, one fasting versus gradual initiation, and Further Reading
one MAD with 10 versus 20 grams of carbohy-
drates. However, taken together, all showed that Lee PR, Kossoff EH. Dietary treatments for epilepsy:
at least 38% of patients had a 50% decrease in Management guidelines for the general practitioner.
seizures at 3 months, with the benefit maintained Epilepsy and Behavior 2011; 21:115–121.
Nangia S, Caraballo RH, Kang HC, Nordli DR, Schef-
at 1 year. fer IE. Is the ketogenic diet effective in specific
epilepsy syndromes? Epilepsy Research 2012;
100:252–257.
Modified Atkins Diet Levy RG, Cooper PN, Giri P, Pulman J. Ketogenic diet
and other dietary treatments for epilepsy. Cochrane
Database of Systematic Reviews 2012, Issue 3. Art
Efficacy of the MAD may be comparable to the No:CD001903.
KD in children and adults. In two studies
including children, 43–65% had at least >50%
reduction of seizures at 6 months, with 35–36% References
having >90% seizure reduction [7(p38)].
Meta-analysis of existing literature as of 2008 1. Wheless JW. History of the ketogenic diet. Epilepsia.
showed 45% of patients on the MAD with 50– 2008;49(Suppl 8):3–5.
240 A. Kao

2. Hartman AL, Gasior M, Vining EPG, Rogawski MA. 7. Kossoff EH, Dorward JL. The modified Atkins diet.
The neuropharmacology of the ketogenic diet. Pedi- Epilepsia. 2008;49(suppl 8):37–41.
atr Neurol. 2007;36(5):281–92. 8. Pfeifer HH, Lyczkowski DA, Thiele EA. Low
3. Bergqvist AGC, Schall JI, Gallagher PR, Cnaan A, glycemic index treatment: implementation and new
Stallings VA. Fasting versus gradual initiation of the insights into efficacy. Epilepsia. 2008;49(suppl
ketogenic diet: a prospective, randomized clinical 8):42–5.
trial of efficacy. Epilepsia. 2005;46(11):1810–9. 9. Kossoff EH, Zupec-Kania BA, Amark PE, et al.
4. Zupec-Kania BA, Spellman E. An overview of the Optimal clinical management of children receiving
ketogenic diet for pediatric epilepsy. Nutr Clin Pract. the ketogenic diet: recommendations of the Interna-
2008;23:589–96. tional Ketogenic Diet Study Group. Epilepsia.
5. Neal EG, Chaffe H, Schwartz RH, et al. A random- 2009;50:304–17.
ized trial of classical and medium-chain triglyceride 10. Kossoff EH, Rowley H, Sinha SR, Vining EPG.
ketogenic diets in the treatment of childhood A prospective study of the modified Atkins diet for
epilepsy. Epilepsia. 2009;50(5):1109–17. intractable epilepsy in adults. Epilepsia. 2008;49
6. Kossoff EH, Krauss GL, McGrogan JR, Freeman JM. (2):316–9.
Efficacy of the Atkins diet as therapy for intractable
epilepsy. Neurology. 2003;61:1789–91.
Other Pharmacological Therapies:
Investigational Antiepileptic Drugs, 19
Animal Models of Epilepsy,
Hormonal Therapy, Immunotherapy

Bassel Abou-Khalil

than for intravenous lorazepam. Another mar-


Investigational Antiepileptic Drugs
keted product that is being tested for an alternate
route of delivery is intravenous carbamazepine
The investigation of antiepileptic drugs (AEDs)
[24].
includes trials of new routes of delivery for
A number of new AEDs have become avail-
marketed drugs, trials of marketed drugs for new
able in extended release formulations. The list
indications, and trials of novel compounds.
includes gabapentin (for restless leg syndrome),
New routes of delivery are being tested in
lamotrigine, topiramate, levetiracetam, and
particular for benzodiazepines, in the treatment
oxcarbazepine [2, 3, 5, 9, 11, 28, 29, 35]. These
of seizure clusters by family members and other
formulations allow the convenience of once daily
nonmedical caregivers. A large pivotal trial was
dosing or steadier serum concentrations with
completed for intramuscular diazepam adminis-
twice-daily dosing [20].
tration by autoinjector, demonstrating intramus-
Several new AEDs are in trials for use in
cular diazepam to be superior to placebo in
monotherapy (lacosamide and eslicarbazepine)
preventing additional seizures and in obviating
or for use in primary generalized tonic-clonic
the need for other rescue treatment or emergency
seizures (lacosamide and perampanel). Intra-
room visit [1, 32]. Intranasal midazolam and
venous lacosamide is undergoing investigation
intranasal diazepam are also in testing for the
for use in nonconvulsive seizures or nonconvul-
treatment of seizure clusters by nonmedical
sive status epilepticus.
caregivers [7, 12, 16, 34]. Intramuscular mida-
While there are many experimental drugs in
zolam by autoinjector was tested as a prehospital
testing, the ones in phase 3 trials will be dis-
treatment of status epilepticus by emergency
cussed mainly. Brivaracetam is an analogue of
medical personnel. Intramuscular midazolam was
levetiracetam which has a sodium blocking
found to be noninferior to intravenous lor-
mechanism in addition to its SV2A binding [27].
azepam; in fact, patients treated with midazolam
It appears to have a similar profile to levetirac-
more often had stopped seizing upon arrival to
etam in general, with a higher potency. Two
the emergency department [33]. One important
novel compounds are, in advanced stages of
advantage of intramuscular midazolam is faster
testing, VX765 (an anti-inflammatory agent) and
delivery, with a shorter time to active treatment
YKP3089 (an agent with unknown mechanism,
probably an inhibitor of slow inactivated state of
sodium channels which may also facilitate the
B. Abou-Khalil (&) release of GABA).
Neurology, Comprehensive Epilepsy Program,
It is worthwhile pointing out a couple com-
Vanderbilt University Medical Center, Nashville,
TN, USA pounds that are still in early testing. Cannabidiol,
e-mail: bassel.abou-khalil@vanderbilt.edu a nonpsychoactive compound derived from

© Springer Science+Business Media LLC 2017 241


M.Z. Koubeissi and N.J. Azar (eds.), Epilepsy Board Review,
DOI 10.1007/978-1-4939-6774-2_19
242 B. Abou-Khalil

cannabis, is undergoing early trials in epilepsy initially produced only subclinical after dis-
after a media campaign based on anecdotal charges eventually result in full-fledged gener-
reports of efficacy in various epilepsy syndromes, alized tonic-clonic seizures [10]. Though
including Dravet syndrome [8]. Intravenous laborious, this model has been increasingly used
allopregnanolone, a neurosteroid that modulates in the preclinical testing of candidate drugs.
synaptic and extrasynaptic GABAA receptors, is However, this model has also been criticized for
undergoing testing in super-refractory status the absence of spontaneous seizures that are
epilepticus, where there is a decreased synaptic typical of human epilepsy.
expression of benzodiazepine-sensitive GABAA True animal models of epilepsy should have
receptors [31]. spontaneously recurrent seizures. Such models
may have genetic or acquired epilepsy. Com-
monly used genetic models are DBA/2 mice with
Animal Models of Epilepsy audiogenic seizures and the genetic absence
epilepsy rats from Strasbourg (GAERS) [22, 23].
The development of AEDs has depended con- Efficacy in DBA/2 mice with audiogenic seizures
siderably on animal models of epilepsy helps predict efficacy against human generalized
(Table 19.1). Perhaps, the most important appli- tonic-clonic seizures, while efficacy on GAERS
cation was historically screening of compounds helps predict efficacy against human generalized
for anti-seizure activity. The two main animal absence seizures. Animal models of acquired
models used in the screening of antiepileptic chronic epilepsy include post-status epilepticus
drug candidates were the maximal electroshock models, in which chemically or electrically
(MES) model and the subcutaneous pentyl- induced status epilepticus is followed by spon-
enetetrazole (PTZ) model [21]. In the MES taneously recurring seizures.
model, electrical stimulation is applied, usually Because newly introduced AEDs have had
through the cornea, with an intensity sufficient to limited impact on the proportion of patients with
elicit tonic hind limb extension in all control drug-resistant epilepsy, it is now recognized that
animals [4]. The electrical stimulus is 50–60 Hz there is a need for animal models of epilepsy that
in frequency, 0.6 ms in pulse width, and 0.2 s in are drug-resistant [21]. One such model is the
stimulus train duration. The usual intensity nee- 6-Hz psychomotor seizure model in mice. In this
ded is 50 mA for mice and 150 mA for rats. In model, 6-Hz pulses of 0.2-ms duration are
the PTZ model, PTZ is injected subcutaneously delivered through the cornea for 3 s, resulting in
at a dose known to induce at least 5 s of clonic a seizure that resembles limbic seizures in
seizure activity in 97% of the animals. The MES humans. At an intensity of 44 mA (twice that
model has been said to be predictive of efficacy necessary to produce seizures in 97% of mice),
against generalized tonic-clonic seizures, many AEDs become ineffective [21]. The
whereas the PTZ model has been thought to be methylazoxymethanol acetate (MAM) rat model
predictive of efficacy against absence seizures. of cortical dysplasia can also serve as a model of
However, these models may miss some effective pharmacoresistant epilepsy. In this model, MAM
AEDs, most notably levetiracetam. In addition, in utero exposure to results in a cortical
these models have been criticized for being dysplasia-like lesion, and seizures induced in
models of seizures in healthy animals, rather than these rats by kainate are resistant to several
models of epilepsy. AEDs. Pharmacoresistant epilepsy can also be
One animal model that is predictive of effi- produced by exposure to low doses of lamotrig-
cacy against partial (focal) seizures is the kin- ine during kindling or by selection of subgroups
dling model. In this model, repeated electrical of rats that are resistant to specific AEDs from a
stimuli are applied to the amygdala or hip- large group of epileptic rats.
pocampus of rats, resulting in permanent lower- All the currently marketed AEDs are used as
ing of the seizure threshold, so that stimuli that symptomatic treatment to suppress seizures.
19 Other Pharmacological Therapies: Investigational … 243

Table 19.1 Select animal models of seizures/epilepsy and the proposed corresponding human seizure/epilepsy type
that is modeled
Animal model MES PTZ Kindling DBA/2 mice GAERS rats
Human Generalized Generalized Partial Generalized Generalized
seizure/epilepsy tonic-clonic absence seizures tonic-clonic, reflex absence
type seizures seizures (limbic) seizures, SUDEP seizures
Animal model 6-Hz psychomotor seizure MAM seizure model Post-status epilepticus
model in mice model in rats
Human Pharmacoresistant limbic Pharmacoresistant seizures, Chronic focal epilepsy
seizure/epilepsy seizures cortical dysplasia
type

There is no clear evidence that any current AED cycle [14]. Synthetic progestins and clomiphene
is effective in the prevention of epilepsy. There is citrate have also been reported beneficial as
increasing interest in the identification of disease treatments for catamenial epilepsy in small
modifying treatments that could prevent the studies.
development of epilepsy after an insult or pre- Ganaxolone is a derivative of allopreg-
vent the progression of epileptogenesis. Chronic nanolone that lacks hormonal activity. It has been
animal models of epilepsy can be used to study tested in a number of clinical trials. There was a
potential anti-epileptogenic treatments. The most suggestion that women with catamenial epilepsy
commonly used models in this setting are the were a subgroup that benefited in particular [30].
kindling model and the post-status epilepticus Ganaxolone was also tested in infantile spasms
model [21]. and found helpful in some patients [19]. It is not
known if this compound will eventually be
available for clinical use.
Hormonal and Immunological ACTH and steroids are first-line short-term
Treatment treatments for infantile spasms/West syndrome.
They help control seizures and improve behavior
Hormonal therapy may be considered in women and EEG. They are most effective in the idio-
with catamenial epilepsy, in whom seizures seem pathic syndrome. A high dose seems to be more
to follow a cyclical pattern related to the men- effective. When it comes to ACTH, one approach
strual cycle [13, 15]. Three patterns of catamenial is to begin with 40 IU per day for 1–2 weeks and
epilepsy have been described: C1 pattern where increase to 60 or 80 IU per day thereafter if the
seizures increase in frequency just before and response is incomplete. If it is effective, it is then
during menses, C2 pattern where seizures tapered over 1–4 months. ACTH and steroids are
increase around the time of ovulation, and C3 less commonly used to treat Lennox–Gastaut
pattern where seizures occur with anovulatory syndrome and Landau–Kleffner syndrome.
cycles. Catamenial epilepsy is thought to be Steroids and IVIG may be considered in the
related to progesterone and estrogen fluctuations. treatment of Rasmussen’s syndrome and other
Estrogen appears to be proconvulsant, and pro- epilepsies suspected to be of immune origin, to
gesterone appears to be anticonvulsant. In cata- treat the underlying cause of epilepsy. Limbic
menial epilepsy, seizures are more likely to occur encephalitis, usually autoimmune, is increasingly
when the ratio of progesterone to estrogen recognized as a cause of chronic epilepsy. An
decreases, as seen around the time of menstrua- immune basis of epilepsy should be considered
tion and the time of ovulation. The C1 pattern of when there is no other clear etiology, the onset
catamenial epilepsy responds to progesterone was acute or subacute, and there is a prior history
200 mg tid administered on days 14–28 of the of autoimmunity (or autoimmunity is present in a
244 B. Abou-Khalil

first-degree relative), in the presence of a neo- 5. Clark AM, Halvorsen MB, Braun TL, et al. USL255
plasm, when there is CSF or imaging evidence of extended-release topiramate: dose-proportional phar-
macokinetics and tolerability in healthy volunteers.
inflammation, and when neuronal autoantibodies Epilepsia. 2014;55:1069–76.
are detected [36]. Faciobrachial dystonic seizures 6. Dalmau J, Gleichman AJ, Hughes EG, et al.
are brief seizures that predominantly affect the Anti-NMDA-receptor encephalitis: case series and
arm and ipsilateral face. They are an early sign in analysis of the effects of antibodies. Lancet Neurol.
2008;7:1091–8.
anti-LGI1 encephalitis that should prompt 7. de Haan GJ, van der Geest P, Doelman G, et al.
investigation for immune etiology and early A comparison of midazolam nasal spray and
immune therapy [17, 18]. The autoantibodies diazepam rectal solution for the residential treatment
most commonly associated with immune of seizure exacerbations. Epilepsia. 2010;51:478–82.
8. Devinsky O, Cilio MR, Cross H, et al. Cannabidiol:
epilepsy are anti-LGI1 antibodies (anti-voltage- pharmacology and potential therapeutic role in
gated potassium channel complex antibodies), epilepsy and other neuropsychiatric disorders.
anti-GAD antibodies, and anti-thyroid antibodies Epilepsia. 2014;55:791–802.
[25, 26]. Anti-NMDA antibodies are associated 9. French JA, Baroldi P, Brittain ST, et al. Efficacy and
safety of extended-release oxcarbazepine (Oxtellar
with a distinctive limbic encephalitis syndrome XR) as adjunctive therapy in patients with refractory
that usually includes seizures, but is unlikely to partial-onset seizures: a randomized controlled trial.
be a cause of pure chronic epilepsy [6]. When an Acta Neurol Scand. 2014;129:143–53.
immune origin is confirmed, first-line 10. Goddard GV, McIntyre DC, Leech CK. A permanent
change in brain function resulting from daily electri-
immunotherapies include oral or IV steroids cal stimulation. Exp Neurol. 1969;25:295–330.
(IV methylprednisolone 1000 mg daily for 3– 11. Gordi T, Hou E, Kasichayanula S, et al. Pharma-
5 days, then weekly for 4–6 weeks), IVIG cokinetics of gabapentin after a single day and at
steady state following the administration of
(0.4 g/kg/day for 3–5 days then weekly for 4–
gastric-retentive- extended-release and
6 weeks), or plasmapheresis [36]. If there has immediate-release tablets: a randomized, open-label,
been incomplete benefit and there is strong evi- multiple-dose, three-way crossover, exploratory
dence of autoimmune etiology, chronic study in healthy subjects. Clin Ther. 2008;30:909–
16.
immunosuppression could be considered with
12. Henney HR, 3rd, Sperling MR, Rabinowicz AL,
mycophenolate mofetil, azathioprine, or ritux- et al. Assessment of pharmacokinetics and tolerabil-
imab [36]. ity of intranasal diazepam relative to rectal gel in
healthy adults. Epilepsy Res. 2014.
13. Herzog AG. Catamenial epilepsy: definition, preva-
lence pathophysiology and treatment. Seizure.
References 2008;17:151–9.
14. Herzog AG, Fowler KM, Smithson SD, et al.
1. Abou-Khalil B, Wheless J, Rogin J, et al. A dou- Progesterone vs placebo therapy for women with
ble-blind, randomized, placebo-controlled trial of a epilepsy: A randomized clinical trial. Neurology.
diazepam auto-injector administered by caregivers to 2012;78:1959–66.
patients with epilepsy who require intermittent 15. Herzog AG, Klein P, Ransil BJ. Three patterns of
intervention for acute repetitive seizures. Epilepsia. catamenial epilepsy. Epilepsia. 1997;38:1082–8.
2013;54:1968–76. 16. Holsti M, Dudley N, Schunk J, et al. Intranasal
2. Anonymous. Topiramate extended-release (Trokendi midazolam vs rectal diazepam for the home treatment
XR) for epilepsy. Med Lett Drugs Ther. 2013;55: of acute seizures in pediatric patients with epilepsy.
87–88. Arch Pediatr Adolesc Med. 2010;164:747–53.
3. Biton V, Shneker BF, Naritoku D, et al. Long-term 17. Irani SR, Michell AW, Lang B, et al. Faciobrachial
tolerability and safety of lamotrigine dystonic seizures precede Lgi1 antibody limbic
extended-release: pooled analysis of three clinical encephalitis. Ann Neurol. 2011;69:892–900.
trials. Clin Drug Investig. 2013;33:359–64. 18. Irani SR, Stagg CJ, Schott JM, et al. Faciobrachial
4. Castel-Branco MM, Alves GL, Figueiredo IV, et al. dystonic seizures: the influence of immunotherapy on
The maximal electroshock seizure (MES) model in seizure control and prevention of cognitive impair-
the preclinical assessment of potential new ment in a broadening phenotype. Brain.
antiepileptic drugs. Methods Find Exp Clin Pharma- 2013;136:3151–62.
col. 2009;31:101–6. 19. Kerrigan JF, Shields WD, Nelson TY, et al. Ganax-
olone for treating intractable infantile spasms: a
19 Other Pharmacological Therapies: Investigational … 245

multicenter, open-label, add-on trial. Epilepsy Res. 29. Peltola J, Coetzee C, Jimenez F, et al. Once-daily
2000;42:133–9. extended-release levetiracetam as adjunctive treat-
20. Leppik IE, Hovinga CA. Extended-release ment of partial-onset seizures in patients with
antiepileptic drugs: a comparison of pharmacokinetic epilepsy: a double-blind, randomized,
parameters relative to original immediate-release placebo-controlled trial. Epilepsia. 2009;50:406–14.
formulations. Epilepsia. 2013;54:28–35. 30. Pennell PB. Hormonal aspects of epilepsy. Neurol
21. Loscher W. Critical review of current animal models Clin. 2009;27:941–65.
of seizures and epilepsy used in the discovery and 31. Rogawski MA, Loya CM, Reddy K, et al. Neuroac-
development of new antiepileptic drugs. Seizure. tive steroids for the treatment of status epilepticus.
2011;20:359–68. Epilepsia. 2013;54(Suppl 6):93–8.
22. Loscher W, Schmidt D. Which animal models should 32. Rogin J, Wheless J, Abou-Khalil B, et al. Safety and
be used in the search for new antiepileptic drugs? A effectiveness of long-term treatment with diazepam
proposal based on experimental and clinical consid- auto-injector administered by caregivers in an out-
erations. Epilepsy Res. 1988;2:145–81. patient setting for the treatment of acute repetitive
23. Marescaux C, Vergnes M, Depaulis A. Genetic seizures. Epilepsia. 2014.
absence epilepsy in rats from Strasbourg–a review. 33. Silbergleit R, Durkalski V, Lowenstein D, et al.
J Neural Transm Suppl. 1992;35:37–69. Intramuscular versus intravenous therapy for prehos-
24. Marino SE, Birnbaum AK, Leppik IE, et al. pital status epilepticus. N Engl J Med.
Steady-state carbamazepine pharmacokinetics fol- 2012;366:591–600.
lowing oral and stable-labeled intravenous adminis- 34. Sperling M, Seif Eddeine H, Haas K, et al. Dosing
tration in epilepsy patients: effects of race and sex. feasibility and pharmacokinetics by seizure type and
Clin Pharmacol Ther. 2012;91:483–8. status, and tolerability of intranasal diazepam in
25. McKnight K, Jiang Y, Hart Y, et al. Serum adults with epilepsy. Epilepsy Currents. 2014;14
antibodies in epilepsy and seizure-associated disor- (Supplement—2013 Abstracts):106–7.
ders. Neurology. 2005;65:1730–6. 35. Steinhoff BJ. Oxcarbazepine extended-release for-
26. Miro J, Fortuny R, Juncadella M, et al. Antithyroid mulation in epilepsy. Expert Rev Clin Pharmacol.
antibodies as a potential marker of 2009;2:155–62.
autoimmune-mediated late onset temporal lobe 36. Toledano M, Britton JW, McKeon A, et al. Utility of
epilepsy. Clin Neurol Neurosurg. 2014;121:46–50. an immunotherapy trial in evaluating patients with
27. Mula M. Brivaracetam for the treatment of epilepsy presumed autoimmune epilepsy. Neurology.
in adults. Expert Rev Neurother. 2014;14:361–5. 2014;82:1578–86.
28. Naritoku DK, Warnock CR, Messenheimer JA, et al.
Lamotrigine extended-release as adjunctive therapy
for partial seizures. Neurology. 2007;69:1610–8.
Epilepsy Management in Special
Populations 20
Amir M. Arain

hand, in institution-based studies including


Epilepsy in Individuals
patients with severe MR, the prevalence of epi-
with Intellectual and Developmental
lepsy varies from 32–34% [3].
Disability
Regarding the seizure semiology in patients
with IDD, a population-based study reported
Individuals with intellectual disability, a lifelong
generalized tonic-clonic (GTC) seizures to be the
condition, are dependent on others for their daily
most common type [2]. However, in
care needs. Intellectual and developmental dis-
institution-based studies, focal dyscognitive
ability (IDD) is defined by mental retardation
(complex partial) seizures with or without sec-
(MR) and can be mild, moderate, or severe.
ondary generalization were reported to be the
Individuals with mild MR, with IQs between 50–
most common [3]. Other seizure types, such as
55 and 70, are considered the educable mentally
tonic, atonic, myoclonic, and atypical absence
retarded and are placed in special classes. Indi-
seizures are also seen. Seizure types are also
viduals with moderate MR, with IQs between
age-related with GTC and focal dyscognitive
35–40 and 50–55, are often institutionalized, and
seizures becoming predominant at later ages.
their training is focused on self-care rather than
Only one-third of the individuals with IDD have
development of intellectual skills. Finally, those
seizures that can be classified per ILAE classifi-
with severe MR, with IQs between 20–25 and
cation [3]. The reason for this is partly the fact
35–40, cannot care for themselves, have major
that such individuals may have complex
problems with communication, and are often
non-epileptic behaviors that are confused with
listless and inactive.
epilepsy, and only video-EEG monitoring can
Individuals with IDD have higher incidence
discern the nature of such behaviors. This con-
of epilepsy in comparison with those with normal
stitutes a major problem in these individuals
intellect. Incidence of epilepsy increases with the
because often antiseizure medications are added,
severity of mental retardation, but it varies
or their dosages increased, to treat non-epileptic
depending on epidemiological methodologies. In
behaviors, impacting the patient’s quality of life.
population-based studies, 21% of those with mild
About two-thirds of institutionalized patients
MR had epilepsy [1]. Another study reported
with IDD have stereotypical behaviors, including
epilepsy in 11% of subjects with mild MR and in
head movements, rocking, and jerking [4]. Other
23% in those with severe MR [2]. On the other
motor behaviors may result from medication
adverse events, including tardive dyskinesia,
which may be confused with seizures.
A.M. Arain (&)
Video-EEG monitoring is often very helpful in
Vanderbilt University, A-0118 Medical Center
North, Nashville, TN 37232, USA confirming the diagnosis [5]. Caregivers can be
e-mail: amir.arain@vanderbilt.edu advised to make a home video of the spells in

© Springer Science+Business Media LLC 2017 247


M.Z. Koubeissi and N.J. Azar (eds.), Epilepsy Board Review,
DOI 10.1007/978-1-4939-6774-2_20
248 A.M. Arain

question as a first screening tool. Several factors Epilepsy in the Elderly


have an impact on the caring of individuals with
intellectual disability such as maladaptive The incidence of epilepsy increases with age,
behavior of the patients [6], severity of intellec- with a steep rise after age 60 [11, 12]. Epilepsy
tual disability, presence of multiple disabilities, has higher incidence (134/100,000 person-years)
and level of social support. Caregivers of indi- in the elderly than Alzheimer’s disease
viduals with IDD are at-risk of stress [7]. (123/100,000 person-years). Seizures may pre-
Seizure control is often brittle in this popula- sent with staring, disorientation, and subtle lip
tion. Seizure exacerbations occur with fever, smacking. Atypical presentations are also com-
infections, metabolic abnormalities [8, 9], or any mon in the elderly, including altered mental
stressful condition. At times, change in care- status, memory lapses, and intermittent confu-
givers or environment may have detrimental sion. In addition, auras are less common while
effect on seizure control. Patients with IDD and postictal states can be prolonged mimicking
epilepsy have risks of morbidity and mortality, dementia.
including a higher incidence of vitamin D defi- The most common causes of seizures in the
ciency. Their limited mobility and the effects of elderly are stroke, dementia, and head trauma
antiepileptic drugs (AEDs), especially older [13]. Other risk factors for epilepsy include
generation ones, can worsen their bone health. major depression, hypertension, and sleep apnea.
Patients with epilepsy and IDD have a higher risk Both ischemic and hemorrhagic cerebrovascular
of fractures. The risk of mortality individuals events increase the incidence of epilepsy [14,
with IDD and epilepsy exceeds that of individ- 15]. Epilepsy was a concomitant diagnosis in
uals with epilepsy alone, which, in turn, exceeds 10% of individuals with Alzheimer’s disease in
that of the healthy population. an autopsy-verified study, and seizures can occur
Treatment principle in the population is to at any stage of Alzheimer’s disease [16]. In
improve their seizure control without compro- treating patients with dementia and epilepsy, it is
mising their quality of life. Excessive sedation is important to keep in mind that acetyl-
not an acceptable cost for seizure control. At cholinesterase inhibitors may worsen seizures
times, patients may not like the color or taste of [17]. Hypertension is another independent risk
medicine. Caregivers should be astute enough to factor for developing epilepsy in the elderly
try different formulations. AEDs available as patients [18]. Aggressive treatment of hyperten-
liquid, in soluble or granular form, or as powder sion in elderly patients, particularly with diuret-
may be useful. Caregivers of these patients are ics, may have a protective effect [19].
the most important liaison between the physician Depression in the elderly is associated with an
and the patient as often these patients are non- increased risk of developing seizures. In a study
verbal. In these patients, assessment of treatment of patients aged 55 or older, there was a sixfold
is often hampered by lack of communication. increased risk of unprovoked seizures [20].
CNS side effects of AEDs may be masked. Similarly, sleep apnea can be a risk factor for
Caregivers should be tuned to assess any change new onset seizures or seizure exacerbations in the
in their behavior as drowsiness or mood changes elderly. Treating comorbid sleep apnea in elderly
may be a manifestation of side effects. patients with epilepsy may improve seizure
Patients with IDD and epilepsy who have control. In a study of elderly patients with epi-
been seizure-free for several years may undergo lepsy evaluated by polysomnography, patients
AED medication tapering trial. However, the risk with new onset or worsening seizures had sig-
of seizure recurrence is high. Predictors of suc- nificant high Apnea–Hypopnea Index compared
cessful discontinuation of AED include history to stable or seizure-free patients [21].
of few documented seizures, no gross neurolog- Diagnosis of epilepsy in elderly can be chal-
ical abnormalities, and normal EEG before and lenging because of higher percentage of partial
after AED discontinuation [10]. seizures than generalized tonic-clonic seizures
20 Epilepsy Management in Special Populations 249

that can be readily diagnosed. Moreover, the favorable pharmacokinetic profiles. Medications
extratemporal lobe seizures with subtle features must be initiated at a low dose and slowly titrated
and prominent postictal confusion are more up. Older AEDs with enzyme-inducing proper-
common in elderly. Misdiagnosis may also result ties should be avoided, and newer generation
from difficulties in history taking, comorbidities, AEDs with significant cognitive adverse events,
polypharmacy, and the fact that the EEG is less including topiramate and zonisamide, should also
helpful for diagnosis than in younger individuals. be avoided [25]. Valproate may be a good choice
Epilepsy in elderly can be delayed because of in elderly, but one has to keep in mind the
these factors. Physicians treating elderly patients potential side effects of parkinsonism (which is
should have epilepsy in their differential usually reversible) and dementia [26]. Lamot-
diagnosis. rigine, gabapentin, and levetiracetam are appro-
Once the diagnosis of epilepsy is established, priate medications to be started as monotherapy
then the treatment should be started as the risk of in elderly patients with epilepsy [27, 28]. Sei-
recurrence after the first unprovoked seizure is zures in the elderly can often be well controlled
*80% [22]. Clinicians should have a low with monotherapy. In patients with refractory
threshold in starting AEDs. Physiological chan- epilepsy, epilepsy surgery must be considered.
ges of aging may affect AED pharmacokinetics.
These include decreased albumin, decreased liver
metabolism, and decreased glomerular filtration Women with Epilepsy
and excretion. These changes result in longer
half-lives of the medication and greater risks of Women with epilepsy have some unique char-
drug–drug interactions [23]. Several AEDs acteristics that can significantly affect the course
including carbamazepine, phenytoin, and val- and management of epilepsy. Epilepsy and
proate are highly protein bound and will compete female hormones reciprocally influence one
with other medications, including digoxin, for another. Similarly, AEDs and female hormones
adherence to serum proteins resulting in toxicity. also influence one another. Such interactions can
Changes in protein binding result in misleading affect seizure control and medication adverse
measurements of total AED concentration. events. The specific issues in women with epi-
Checking free levels of phenytoin and valproate lepsy warrant special attention to their menstrual
is recommended to the elderly. Elderly patients cycle regularity, fertility and ovulatory function,
are more sensitive to side effects of AEDs, sexuality, hormonal contraception, pregnancy
including peak toxicity. Thus, extended release and breast-feeding, and bone health. Unfortu-
formulations can be helpful as they result in nately, the awareness to these issues is not
lower peaks. prevalent among healthcare providers [29].
Older generation AEDs have higher incidence Estrogen and progesterone have different
of drug–drug interactions with other medications. effects on epilepsy. Estrogen may be procon-
Many AEDs induce liver enzymes, thus lowering vulsant as it may reduce inhibition at the GABAA
the levels of other medications. Administration receptor and also inhibits the synthesis of
of carbamazepine, for example, can lower sim- GABA. On the other hand, progesterone may be
vastatin level [24]. Newer AEDs have relatively anticonvulsant as it enhances inhibition at the
better pharmacokinetics profile with less poten- GABAA receptor and increases the GABA syn-
tial for drug–drug interactions. Grapefruit juice thesis [30]. Progesterone also may attenuate the
can increase the level of many medications, action of the brain’s major excitatory neuro-
including carbamazepine, resulting in dizziness, transmitter, glutamate, in the hippocampus. Thus,
lack of coordination, sedation, and other side the hormonal fluctuations of the menstrual cycle
effects. may result in fluctuation in seizure frequency.
It is recommended that newer AEDs be started This pattern is seen in catamenial epilepsy, which
in elderly patients with epilepsy because of their is present in approximately half of all women
250 A.M. Arain

with epilepsy. In these patients, exacerbations of of cognitive teratogenicity with valproic acid. In a
seizure frequency occur at certain points in the study that assessed cognition in 3- and 6-year-old
menstrual cycle, either before the start of their children who were exposed in utero to AEDs,
menstruation, during menses, or around the time those who were exposed to valproate had signif-
of ovulation possibly because of higher icantly lower IQ scores than those exposed to
estrogen-to-progesterone ratios [31]. other medications [36, 37]. Thus, valproate must
Some AEDs may affect the female hormones be avoided in monotherapy or polytherapy during
and contraception. Enzyme-inducing AEDs, such the first trimester of pregnancy. However, other
as phenytoin, carbamazepine, phenobarbital, authors suggest that if it is imperative to use
primidone (and at higher doses, oxcarbazepine valproate during pregnancy, then a low dose of
>900 mg/day and topiramate >200/day), can the extended release formulation should be used,
reduce the levels of contraceptives [32, 33]. In starting with 500 mg per day and not exceeding
such situations, alternative or supplementary 1000 mg per day [38]. It is recommended to use
contraceptive methods should be used. On the AEDs in monotherapy rather than polytherapy as
other hand, estrogen decreases lamotrigine level the risk of major congenital malformations is
by about 50%. Dosage adjustments of lamotrig- increased by polytherapy. Based on the data, the
ine may be necessary to maintain appropriate probable safest medications are lamotrigine,
response when starting or stopping estrogen— levetiracetam, oxcarbazepine, zonisamide, gaba-
containing oral contraceptives in these women. pentin followed by carbamazepine and phenytoin
Fertility issues are common in women with [39]. Folic acid supplementation is recommended
epilepsy. In epidemiologic studies, women with to women with epilepsy prior to pregnancy. There
epilepsy are approximately two-thirds less likely is still controversy on the optimal dose of folic
to have children than women without epilepsy. acid. For healthy women, a dose of 0.4 mg/day is
Factors that may contribute to the lower fertility optimal, while for high-risk patients, especially
rates in women with epilepsy include decreased ones with the previous history of major congenital
libido, reduced marriage rates, increased anovu- malformations, a dose of 4–5 mg/day is recom-
latory menstrual cycles, menstrual disorders such mended [40]. Prenatal testing, at 14–20 weeks of
as amenorrhea or oligomenorrhea, and increased gestation, should be considered in pregnant
early miscarriages. AEDs, specifically women with epilepsy. Since then by seizure
enzyme-inducing ones, may also have an effect medication levels drop during pregnancy, check-
on fetal survival. Moreover, women with epi- ing the levels on a monthly basis is recommended.
lepsy are more likely to have polycystic- Breast-feeding risks and benefits should be
ovary-syndrome-like ovulatory dysfunction with assessed for every individual patient. AED
clinical evidence of hyperandrogenemia, a risk excretion into breast milk is adversely propor-
that is increased by valproic acid treatment [34]. tional to the degree of protein binding, with
Pregnancy can affect seizures, and although it higher protein binding resulting in less excretion
is a high progesterone state, some women may into breast. Phenobarbital and other AEDs may
have worsening of seizures during pregnancy. cause sedation or poor feeding. The American
This worsening could be an effect of pregnancy Academy of Neurology encourages breast-
itself or because of decreasing AED levels as the feeding in conjunction with close observation
pregnancy progresses. The risk of congenital for any excessive sedation or irritability [41].
malformations may be increased by epilepsy. Bone health is another important aspect of epi-
Teratogenicity can be worsened by AEDs, and the lepsy in women. AEDs, especially enzyme-
rates of major congenital malformations (ranging inducing ones, can worsen bone health and pre-
from 4.6% with carbamazepine to 10.7% with dispose to fractures [42]. Valproate, although not an
valproate monotherapy) are 2–3 times higher than enzyme inducer, can cause osteopenia [43–45].
in untreated women with epilepsy (*3%) [35]. Among newer generation AEDs, oxcarbazepine
Besides anatomical teratogenicity, there is a risk and topiramate at high doses can adversely affect
20 Epilepsy Management in Special Populations 251

the bone health [46, 47]. Dual-energy X-ray 11. Hauser WA, Annegers JF, Kurland LT. Prevalence of
absorptiometry scan can be used to diagnose epilepsy in Rochester, Minnesota: 1940–1980.
Epilepsia. 1991;32:429–45.
osteoporosis and low bone mineral density in 12. Olafsson E, Hauser WA. Prevalence of epilepsy in
women with epilepsy at risk. Women at risk should rural Iceland: a population-based study. Epilepsia.
be counseled to take vitamin D and calcium sup- 1999;40:1529–34.
plementation. If female patients are on AEDs that 13. Hauser WA. Seizure disorders: the changes with age.
Epilepsia. 1992;33(Suppl 4):S6–14.
worsen bone health, then switching them to other 14. Bladin CF, Alexandrov AV, Bellavance A, Born-
medications may reduce the risk of ongoing bone stein N, Chambers B, Cote R, Lebrun L, Pirisi A,
loss. Otherwise, a referral should be made to their Norris JW. Seizures after stroke: a prospective
primary care providers for possible institution of multicenter study. Arch Neurol. 2000;57:1617–22.
15. Kilpatrick CJ, Davis SM, Hopper JL, Rossiter SC.
bisphosphonates or calcitonin. Early seizures after acute stroke. Risk of late seizures.
Arch Neurol. 1992;49:509–11.
16. Hauser WA, Morris ML, Heston LL, Anderson VE.
References Seizures and myoclonus in patients with Alzheimer’s
disease. Neurology. 1986;36:1226–30.
17. Fisher RS, Bortz JJ, Blum DE, Duncan B, Burke H.
1. Gustavson KH, Hagberg B, Hagberg G, Sars K. A pilot study of donepezil for memory problems in
Severe mental retardation in a Swedish county. epilepsy. Epilepsy Behav. 2001;2:330–4.
I. Epidemiology, gestational age, birth weight and 18. Ng SK, Hauser WA, Brust JC, Susser M. Hyperten-
associated CNS handicaps in children born 1959–70. sion and the risk of new-onset unprovoked seizures.
Acta Paediatr Scand. 1977;66:373–9. Neurology. 1993;43:425–8.
2. Forsgren L, Edvinsson SO, Blomquist HK, Heijbel J, 19. Hesdorffer DC, Hauser WA, Annegers JF,
Sidenvall R. Epilepsy in a population of mentally Rocca WA. Severe, uncontrolled hypertension and
retarded children and adults. Epilepsy Res. adult-onset seizures: a case-control study in Roche-
1990;6:234–48. ster Minnesota. Epilepsia. 1996;37:736–41.
3. Mariani E, Ferini-Strambi L, Sala M, Erminio C, 20. Hesdorffer DC, Hauser WA, Annegers JF, Cascino G.
Smirne S. Epilepsy in institutionalized patients with Major depression is a risk factor for seizures in older
encephalopathy: clinical aspects and nosological adults. Ann Neurol. 2000;47:246–9.
considerations. Am J Ment Retard. 1993;98 21. Chihorek AM, Abou-Khalil B, Malow BA. Obstruc-
(Suppl):27–33. tive sleep apnea is associated with seizure occurrence
4. Berkson G, Davenport RK Jr. Stereotyped move- in older adults with epilepsy. Neurology.
ments of mental defectives. I. Initial survey. Am J 2007;69:1823–7.
Ment Defic. 1962;66:849–52. 22. Ramsay RE, Rowan AJ, Pryor FM. Special consid-
5. Arain A, Shihabuddin B, Niaz F, Modur P, Taylor H, erations in treating the elderly patient with epilepsy.
Fakhoury T, Abou-Khalil B. Epilepsy and the impact Neurology. 2004;62:S24–9.
of an epileptology clinic for patients with mental 23. Lackner TE, Cloyd JC, Thomas LW, Leppik IE.
retardation and associated disabilities in an institu- Antiepileptic drug use in nursing home residents:
tional setting. Epilepsia. 2006;47:2052–7. effect of age, gender, and comedication on patterns of
6. Hoare P. The quality of life of children with chronic use. Epilepsia. 1998;39:1083–7.
epilepsy and their families. Seizure. 1993;2:269–75. 24. Ucar M, Neuvonen M, Luurila H, Dahlqvist R,
7. Espie CA, Paul A, Graham M, Sterrick M, Foley J, Neuvonen PJ, Mjorndal T. Carbamazepine markedly
McGarvey C. The epilepsy outcome scale: the reduces serum concentrations of simvastatin and
development of a measure for use with carers of simvastatin acid. Eur J Clin Pharmacol.
people with epilepsy plus intellectual disability. 2004;59:879–82.
J Intellect Disabil Res. 1998;42(Pt 1):90–6. 25. Lee S, Sziklas V, Andermann F, Farnham S, Risse G,
8. Lerman P. Seizures induced or aggravated by Gustafson M, Gates J, Penovich P, Al-Asmi A,
anticonvulsants. Epilepsia. 1986;27:706–10. Dubeau F, Jones-Gotman M. The effects of adjunc-
9. Guerrini R, Belmonte A, Genton P. Antiepileptic tive topiramate on cognitive function in patients with
drug-induced worsening of seizures in children. epilepsy. Epilepsia. 2003;44:339–47.
Epilepsia. 1998;39(Suppl 3):S2–10. 26. Jamora D, Lim SH, Pan A, Tan L, Tan EK.
10. Alvarez N. Discontinuance of antiepileptic medica- Valproate-induced Parkinsonism in epilepsy patients.
tions in patients with developmental disability and Mov Disord. 2007;22:130–3.
diagnosis of epilepsy. Am J Ment Retard. 27. Ferrendelli JA, French J, Leppik I, Morrell MJ,
1989;93:593–9. Herbeuval A, Han J, Magnus L. Use of levetiracetam
252 A.M. Arain

in a population of patients aged 65 years and older: a prospective observational study. Lancet Neurol.
subset analysis of the KEEPER trial. Epilepsy Behav. 2013;12:244–52.
2003;4:702–9. 38. Vajda FJ, Hitchcock A, Graham J, Solinas C,
28. Rowan AJ, Ramsay RE, Collins JF, Pryor F, Board- O’Brien TJ, Lander CM, Eadie MJ. Foetal malfor-
man KD, Uthman BM, Spitz M, Frederick T, mations and seizure control: 52 months data of the
Towne A, Carter GS, Marks W, Felicetta J, Australian pregnancy registry. Eur J Neurol.
Tomyanovich ML. New onset geriatric epilepsy: a 2006;13:645–54.
randomized study of gabapentin, lamotrigine, and 39. Harden CL. Pregnancy and epilepsy. Continuum
carbamazepine. Neurology. 2005;64:1868–73. (Minneap Minn). 2014;20:60–79.
29. Morrell MJ, Sarto GE, Shafer PO, Borda EA, 40. Pittschieler S, Brezinka C, Jahn B, Trinka E, Unter-
Herzog A, Callanan M. Health issues for women berger I, Dobesberger J, Walser G, Auckenthaler A,
with epilepsy: a descriptive survey to assess knowl- Embacher N, Bauer G, Luef G. Spontaneous abortion
edge and awareness among healthcare providers. and the prophylactic effect of folic acid supplemen-
J Womens Health Gend Based Med. 2000;9:959–65. tation in epileptic women undergoing antiepileptic
30. Woolley CS, Schwartzkroin PA. Hormonal effects on therapy. J Neurol. 2008;255:1926–31.
the brain. Epilepsia. 1998;39(Suppl 8):S2–8. 41. Harden CL, Pennell PB, Koppel BS, Hovinga CA,
31. Herzog AG, Harden CL, Liporace J, Pennell P, Gidal B, Meador KJ, Hopp J, Ting TY, Hauser WA,
Schomer DL, Sperling M, Fowler K, Nikolov B, Thurman D, Kaplan PW, Robinson JN, French JA,
Shuman S, Newman M. Frequency of catamenial Wiebe S, Wilner AN, Vazquez B, Holmes L,
seizure exacerbation in women with Krumholz A, Finnell R, Shafer PO, Le Guen C.
localization-related epilepsy. Ann Neurol. Practice parameter update: management issues for
2004;56:431–4. women with epilepsy–focus on pregnancy (an
32. Harden CL, Leppik I. Optimizing therapy of seizures evidence-based review): vitamin K, folic acid, blood
in women who use oral contraceptives. Neurology. levels, and breastfeeding: report of the Quality
2006;67:S56–8. Standards Subcommittee and Therapeutics and Tech-
33. Wilbur K, Ensom MH. Pharmacokinetic drug inter- nology Assessment Subcommittee of the American
actions between oral contraceptives and Academy of Neurology and American Epilepsy
second-generation anticonvulsants. Clin Pharma- Society. Neurology. 2009;73:142–9.
cokinet. 2000;38:355–65. 42. Pack AM, Morrell MJ, Marcus R, Holloway L,
34. Isojarvi JI, Laatikainen TJ, Pakarinen AJ, Jun- Flaster E, Done S, Randall A, Seale C, Shane E.
tunen KT, Myllyla VV. Polycystic ovaries and Bone mass and turnover in women with epilepsy on
hyperandrogenism in women taking valproate for antiepileptic drug monotherapy. Ann Neurol.
epilepsy. N Engl J Med. 1993;329:1383–8. 2005;57:252–7.
35. Meador K, Reynolds MW, Crean S, Fahrbach K, 43. Sheth RD, Gidal BE, Hermann BP. Pathological
Probst C. Pregnancy outcomes in women with fractures in epilepsy. Epilepsy Behav. 2006;9:601–5.
epilepsy: a systematic review and meta-analysis of 44. Samaniego EA, Sheth RD. Bone consequences of
published pregnancy registries and cohorts. Epilepsy epilepsy and antiepileptic medications. Semin Pediatr
Res. 2008;81:1–13. Neurol. 2007;14:196–200.
36. Meador KJ, Baker GA, Browning N, Clayton-Smith 45. Sheth RD, Wesolowski CA, Jacob JC, Penney S,
J, Combs-Cantrell DT, Cohen M, Kalayjian LA, Hobbs GR, Riggs JE, Bodensteiner JB. Effect of
Kanner A, Liporace JD, Pennell PB, Privitera M, carbamazepine and valproate on bone mineral den-
Loring DW. Cognitive function at 3 years of age sity. J Pediatr. 1995;127:256–62.
after fetal exposure to antiepileptic drugs. N Engl J 46. Mintzer S, Boppana P, Toguri J, DeSantis A. Vita-
Med. 2009;360:1597–605. min D levels and bone turnover in epilepsy patients
37. Meador KJ, Baker GA, Browning N, Cohen MJ, taking carbamazepine or oxcarbazepine. Epilepsia.
Bromley RL, Clayton-Smith J, Kalayjian LA, Kan- 2006;47:510–5.
ner A, Liporace JD, Pennell PB, Privitera M, Lor- 47. Ali II, Herial NA, Orris M, Horrigan T, Tietjen GE.
ing DW. Fetal antiepileptic drug exposure and Migraine prophylaxis with topiramate and bone
cognitive outcomes at age 6 years (NEAD study): a health in women. Headache 2011;51:613–6.
Multiple Choice Questions for Part IV

1. Which of the following antiepileptic drugs C. The audiogenic seizure model in mice
would be the best initial choice in an adoles- predicts efficacy against focal seizures
cent girl with juvenile myoclonic epilepsy? D. Levetiracetam was effective in both MES
and PTZ models
A. Carbamazepine
E. The 6 Hz model is predictive of efficacy
B. Pregabalin
against absence seizures
C. Lacosamide
D. Valproic acid
4. The cooperative VA studies showed:
E. Levetiracetam
A. Carbamazepine and phenytoin were
2. A 26-year-old female had frequent episodes
overall more efficacious than phenobar-
of focal left hand shaking followed by gen-
bital and primidone
eralized tonic–clonic seizure activity. On
B. Phenobarbital and primidone were less
two occasions, she broke her jaw and her
well tolerated than carbamazepine and
right shoulder. Routine EEG is normal, and
phenytoin
brain imaging is unremarkable. Which
C. Phenobarbital was more efficacious than
would be most appropriate antiepileptic drug
primidone
for this patient?
D. Primidone was better tolerated than
A. Oxcarbazepine phenobarbital
B. Lamotrigine E. Primidone was more efficacious than
C. Topiramate phenobarbital
D. Phenobarbital
E. Valproate 5. Which of the following is not a 1,4
benzodiazepine?
3. Which is true about animal models of
epilepsy? A. Diazepam
B. Clonazepam
A. The PTZ model helps predict efficacy C. Lorazepam
against absence seizures D. Clobazam
B. The kindling model typically has spon- E. Clorazepate
taneously recurring seizures
254 Multiple Choice Questions for Part IV

6. Which of the following benzodiazepines has 11. Urinary retention is most likely to occur with
important active metabolites?
A. Perampanel
A. Clonazepam B. Levetiracetam
B. Clobazam C. Ezogabine
C. Diazepam D. Felbamate
D. B and C E. Pregabalin
E. All the above
12. Which is not likely to happen with the
7. Dupuytren’s contractures and plantar fibro- addition of felbamate?
matosis are chronic adverse effects of
A. Increased phenobarbital level
A. Tiagabine B. Increased phenytoin level
B. Phenobarbital C. Increased carbamazepine level
C. Vigabatrin D. Increased valproate level
D. Clobazam E. Toxicity related to carbamazepine epoxide
E. Clorazepate
13. A 4-month-old infant is diagnosed with
8. Simvastatin efficacy is reduced with: infantile spasms. He did not respond to
2-week course of ACTH at 150 units/m2.
A. Lamotrigine Which of the following antiepileptic medi-
B. Carbamazepine cations would you use next?
C. Levetiracetam
D. Pregabalin A. Vigabatrin
E. Ezogabine B. Levetiracetam
C. Topiramate
9. An adolescent boy with epilepsy presents D. Clobazam
with overheating and fever following exer- E. Tiagabine
cise. The likely cause is as follows:
14. Irritability is most common with the fol-
A. Levetiracetam lowing AEDs:
B. Lamotrigine
C. Zonisamide A. Lamotrigine and oxcarbazepine
D. Valproate B. Perampanel and levetiracetam
E. Phenobarbital C. Felbamate and zonisamide
D. Phenobarbital and primidone
10. The following antiepileptic drug is FDA E. Valproate and ethosuximide
approved for once-daily dosing:
15. Insomnia is most likely with which of the
A. Perampanel following?
B. Zonisamide
C. Ethosuximide A. Gabapentin
D. Clobazam B. Pregabalin
E. Topiramate C. Ezogabine
D. Perampanel
Multiple Choice Questions for Part IV 255

E. Felbamate 21. The optimal dosage of folic acid in high-risk


pregnant patients with the history of major
16. The mechanism of action of levetiracetam is congenital malformations is as follows:
as follows:
A. 0.4 mg/day
A. Binding to the synaptic vesicle protein B. 1 mg/day
B. Binding to the GABA A receptor C. 2 mg/day
C. Antagonism of NMDA receptors D. 5 mg/day
D. Blocking of sodium channels E. No need for folic acid
E. Opening of potassium channels
22. Following are all risk factors for the devel-
17. Which AED is an NMDA receptor opment of epilepsy in elderly patients except:
antagonist?
A. REM behavior disorder
A. Perampanel B. Hypertension
B. Topiramate C. Depression
C. Phenobarbital D. Cardiovascular disease
D. Zonisamide E. Obstructive sleep apnea
E. Felbamate
23. A 19-year-old neurologically challenged
18. Q-T interval prolongation is a potential male had ongoing complex partial seizures
adverse effect of: in spite of a therapeutic dose of depakote.
The addition of topiramate reduced his sei-
A. Levetiracetam zures significantly. However, his caregiver
B. Ezogabine reports progressive somnolence and
C. Lacosamide decreased responsiveness. The next most
D. Vigabatrin appropriate step is as follows:
E. Tiagabine
A. Urgent EEG to rule out subclinical status
19. All is true about perampanel except: B. Lumbar puncture
C. Urgent brain imaging
A. Weight gain D. Ammonia level
B. Short half-life E. Topiramate blood level
C. Selective AMPA antagonist
D. Black box warning on behavioral 24. All of these AEDs are highly protein-bound
abnormalities except
E. High protein bounding
A. Carbamazepine
20. In general, antiepileptic drug levels in B. Valproate
pregnancy: C. Tiagabine
D. Phenytoin
A. Increase E. Lamotrigine
B. Decrease
C. No changes 25. Which antiepileptic drug has an FDA indi-
D. Antiepileptic drugs should not be used cation for myoclonic seizures?
during pregnancy
E. None of the above A. Pregabalin
B. Valproate
256 Multiple Choice Questions for Part IV

C. Ethosuximide 30. Which of the following is not true about


D. Levetiracetam brivaracetam?
E. Lamotrigine
A. Analog of levetiracetam
26. Physiologic changes with advanced age B. Less potent than levetiracetam
include all of the following except: C. Binds to SV2A
D. Blocks sodium channels
A. Decreased renal clearance E. All of the above is true about
B. Decreased hepatic clearance brivaracetam
C. Decreased and erratic drug absorption
D. Increased protein binding 31. A 77-year-old man with newly diagnosed
E. Increased blood levels epilepsy started having tremors suspicious
for Parkinson’s disease. The likely cause:
27. Progesterone therapy was effective in the
following patient subgroup: A. Carbamazepine
B. Valproate
A. Patients with catamenial epilepsy with C. Phenytoin
seizure exacerbation at ovulation D. Clonazepam
B. Postmenopausal women E. Ethosuximide
C. Women with catamenial epilepsy and
seizure exacerbation around 32. All of the following are animal models of
menstruation partial epilepsy except?
D. Women with anovulatory cycles
E. All women with catamenial epilepsy A. Pentylenetetrazole
B. Kindling
28. Which of the following is an animal model C. Pilocarpine
of drug-resistant epilepsy? D. 6-Hz psychomotor seizures
E. Poststatus epilepticus model in rats
A. 6-Hz psychomotor seizures
B. PTZ (pentylenetetrazole) 33. The visual field changes with vigabatrin:
C. Maximal electroshock (MES)
D. Penicillin A. Are reversible
E. All of the above B. Represent an optic neuropathy
C. Are related to dose and duration of
29. Which of the following is true about cata- treatment
menial epilepsy? D. Typically occur within 2 months of
treatment
A. Seizure cluster around ovulation in C1 E. Are less likely when vigabatrin is com-
pattern bined with a sodium channel blocking
B. Seizure cluster before and during menses drug
in C2 pattern
C. Seizure cluster in anovulatory cycles in 34. Which of the following is true about
C3 pattern tiagabine?
D. Estrogen is anticonvulsant
E. Progesterone is proconvulsant A. Can be given once daily at bedtime
B. Is effective against absence seizures
Multiple Choice Questions for Part IV 257

C. Is approved as initial monotherapy 39. Incidence of epilepsy in institutionalized


D. Increases GABA levels at the synapse patients with intellectual developmental
E. Is associated with retinal toxicity disability is as follows:

35. Which is not true of felbamate idiosyncratic A. 5%


toxicity? B. 9%
C. 21%
A. Aplastic anemia has not been reported D. 32%
below age 13 E. 80%
B. Aplastic anemia and hepatic failure are
most likely within one month of initiat- 40. Which antiepileptic requires monitoring of
ing therapy visual fields during treatment?
C. Underlying autoimmune disease increa-
ses the risk of aplastic anemia A. Clobazam
D. Aplastic anemia is highly unlikely to B. Vigabatrin
occur after one year of therapy C. Tiagabine
E. The risk of aplastic anemia is estimated D. Primidone
at one in 5000 exposures E. Clorazepate

36. Which of the following is unlikely to exac- 41. Screening for potential psychosocial barriers
erbate absence seizures? to the safety and success of the ketogenic
diet should take place before initiation of
A. Carbamazepine therapy. Which of the following features
B. Oxcarbazepine makes the ketogenic diet a better/easier
C. Valproate treatment option?
D. Tiagabine
E. Vigabatrin A. Presence of a gastrostomy tube
B. Severe failure to thrive/inability to
37. Which of the following antiepileptic drug is maintain adequate nutrition
not known to modulate GABA? C. Potential surgically resectable seizure
focus
A. Phenobarbital D. Multiple siblings in an unstructured
B. Tiagabine home environment
C. Clonazepam E. Available premanufactured formulas
D. Pregabalin
E. Topiramate 42. Prolonged episodes of altered responsive-
ness are most likely with:
38. For which of the following conditions is the
ketogenic diet indicated? A. Tiagabine
B. Perampanel
A. Primary carnitine deficiency C. Vigabatrin
B. Pyruvate carboxylase deficiency D. Clobazam
C. Pyruvate dehydrogenase deficiency E. Pregabalin
D. Porphyria
E. None of the above
258 Multiple Choice Questions for Part IV

43. Screening for disorders of fatty acid meta- D. Deficiency of calcium and vitamin D,
bolism should be performed prior to initia- leading to bone mineralization loss
tion of the ketogenic diet. Specifically, this E. Encephalopathy due to hypoglycemia,
testing could include which of the dehydration, and excessive acidosis
following?
46. Epilepsy is most commonly associated with:
A. Complete blood count and complete
metabolic panel including liver function A. Dementia
tests and BUN and creatinine B. Stroke
B. Acylcarnitine profile, urine organic acids, C. Birth defect
and carnitine D. Bipolar disorder
C. CSF glucose, lactate, folate metabolites, E. Psychosis
amino acids, and neurotransmitters
D. Kidney ultrasound and nephrology 47. There is a need for more research and data
consult regarding the incidence and prevention of
long-term complications in patients who
44. A 23-year-old man with idiopathic general- have been on the ketogenic diet for long
ized epilepsy was initially maintained on durations, for instance greater than 5 years.
lamotrigine. Valproate was added as adjunct Which of the following tests would be rea-
therapy. Which of the following effects sonable to perform in such a patient, who is
may result from interaction between these otherwise asymptomatic?
drugs?
A. Blood draw for amylase, lipase
A. Lamotrigine toxicity because of glu- B. Renal ultrasound looking for renal stone
curonidation inhibition C. Abdominal X-ray for stool
B. Decreased lamotrigine efficacy due to D. Blood draw for 25-hydroxy-vitamin D
inhibition of cytochrome 2C9 and 2C19 and consideration of bone density scan
C. Decreased valproate efficacy due to E. CRP
induction of CYP2C9
D. Increased risk of liver disease 48. The literature supports the probable benefit
E. Valproate toxicity due to inhibition of its of the ketogenic diet in which of the fol-
b-oxidation lowing conditions?

45. With rare exceptions, the ketogenic diet is A. Benign myoclonus of infancy
initiated during an inpatient hospitalization. B. Juvenile myoclonic epilepsy
Complications during the initiation period C. Glucose transporter protein 1 deficiency
could include all of the below, except: D. Pyruvate carboxylase deficiency

A. Vomiting due to hypoglycemia, dehy- 49. A 36-year-old male from Nepal is evaluated
dration, excessive acidosis, constipation, for a generalized tonic seizure. Few weeks
or exacerbation of gastroesophageal ago, he started taking rifampin and isoniazid
reflux for a positive tuberculin test. His neurolog-
B. Precipitation of deterioration in a patient ical examination, EEG, and brain MRI are
with an undiagnosed disorder of fat all normal. What is the likely mechanism
metabolism responsible for his seizure?
C. Excessive metabolic acidosis in a patient
also treated with a carbonic anhydrase A. Reversible inhibition of GABA reuptake
inhibitor B. Impaired pyridoxine synthesis
Multiple Choice Questions for Part IV 259

C. Activation of glutamate B. Valproate exposure during gestation is


D. Hypermagnesemia associated with autism
E. Prolactin surge C. Valproate teratogenicity is
dose-dependent
50. A trial comparing valproate, ethosuximide, D. Folate supplementation reduces
and lamotrigine for absence demonstrated valproate-associated teratogenicity rate to
control value
A. All three AEDs were equally effective E. Malformation rate with valproate expo-
B. Valproate and ethosuximide were more sure is elevated with both monotherapy
effective than lamotrigine and polytherapy
C. Valproate was better tolerated than
lamotrigine 54. A 58-year-old man was started on phenytoin
D. Valproate had less neuropsychological for weekly partial seizures. His seizure fre-
adverse effects than ethosuximide quency dramatically decreased but has not
E. Ethosuximide was more effective than reached seizure freedom. He developed
valproate postherpetic neuralgia in the left abdominal
dermatome 6 months ago with persistent
51. Which of the following statements accu- pain. He also had a history of diabetes and a
rately conveys the typical recommendations history of a previous lacunar infarct.
(by consensus) for discontinuation of the Attempts to increase the dose of phenytoin
ketogenic diet? resulted in intolerable drowsiness. Which of
the following would be the most appropriate
A. Discontinue the ketogenic diet if it seems adjunct therapy in this patient?
ineffective by 1 month following
initiation A. Carbamazepine
B. Wait for 3 months following initiation B. Lamotrigine
before deciding to discontinue the diet C. Pregabalin
C. Abrupt discontinuation is preferred over D. Levetiracetam
gradual weaning over 2–3 months E. Lacosamide
D. Wean after 1 year of seizure freedom
55. Which is incorrect of valproate and pheny-
52. A 23-year-old male from El Salvador is seen toin protein binding?
for new onset–partial seizures. His brain
MRI shows multiple ring-enhancing lesions. A. Both phenytoin and valproate are highly
The best treatment is as follows: protein-bound
B. When used together, phenytoin free
A. Ceftriaxone fraction is increased
B. Acyclovir C. Valproate protein binding is increased at
C. Albendazole and steroids higher concentration
D. Natalizumab D. Phenytoin free fraction is increased in
E. Aspirin low protein states
E. Intravenous valproate may displace war-
53. Which is not true of valproate farin from protein binding
teratogenicity?
56. The potential benefit of the ketogenic diet
A. Valproate exposure during gestation is has been suggested for and is being inves-
associated with decreased verbal IQ tigated in all of the following except:
260 Multiple Choice Questions for Part IV

A. Amyotrophic lateral sclerosis B. Ceftriaxone


B. Traumatic brain injury C. Grapefruit juice
C. Alzheimer’s disease D. Fluoxetine
D. Brain tumors E. Propoxyphene
E. Epileptologists studying for the epilepsy
boards 62. Estrogen affects seizure control by:

57. Which of the following antiepileptic drugs A. Enhancing inhibition at GABA A


does not affect bone health in women with receptor
epilepsy? B. Increasing GABA synthesis
C. Accentuating the action of glutamate
A. Topiramate D. Inhibiting synthesis of GABA
B. Lamotrigine E. Estrogen in protective against seizures
C. Phenytoin
D. Phenobarbital 63. In patients with epilepsy, the yield of first
E. All of the above routine EEG is as follows:

58. All of the following AEDs are appropriate to A. 25%


use in elderly with epilepsy except: B. 50%
C. 75%
A. Lamotrigine D. 100%
B. Gabapentin E. EEG is not needed for diagnosis
C. Levetiracetam
D. Topiramate 64. Autism has been associated with the
E. Pregabalin intrauterine exposure to:

59. Faciobrachial dystonic seizures are an early A. Phenobarbital


manifestation of: B. Lamotrigine
C. Phenytoin
A. Anti-NMDA antibody limbic encephalitis D. Carbamazepine
B. Anti-LGI1 antibody limbic encephalitis E. Valproate
C. Anti-GAD antibody limbic encephalitis
D. Hashimoto’s encephalitis 65. Which has the shortest half-life for parent
E. Landau–Kleffner syndrome drug and active metabolite?

60. Which of the following does not cause A. Diazepam


phenytoin accumulation? B. Lorazepam
C. Clorazepate
A. Amiodarone D. Clonazepam
B. Carbamazepine E. Clobazam
C. Cimetidine
D. Fluconazole 66. The half-life of the following AED is pro-
E. Felbamate longed when the serum concentration is
above the recommended therapeutic range:
61. Which of the following does not cause car-
bamazepine accumulation? A. Phenobarbital
B. Carbamazepine
A. Erythromycin C. Phenytoin
Multiple Choice Questions for Part IV 261

D. Valproate 71. All of the following features are associated


E. Oxcarbazepine with higher seizure recurrence except:

67. The half-life of the following AED becomes A. Focal onset


shorter after two weeks of treatment: B. Cluster seizures
C. Status epilepticus
A. Phenobarbital D. Intermittent temporal slow activity
B. Carbamazepine E. Abnormal brain imaging
C. Phenytoin
D. Valproate 72. Oral contraceptive medications can be
E. Oxcarbazepine affected by all except:

68. A 42-year-old man is brought to the ED A. Oxcarbazepine


because of a generalized tonic–clonic sei- B. Phenytoin
zures. He has no previous history of sei- C. Primidone
zures, and his MRI is normal. His EEG D. Topiramate
reveals left temporal sharp waves. His sei- E. Levetiracetam
zure recurrence rate is as follows:
73. Ophthalmological adverse effects can be
A. 10% seen with:
B. 20%
C. 50% A. Rufinamide, ezogabine, and vigabatrin
D. 80% B. Ezogabine, vigabatrin, and topiramate
E. 100% C. Zonisamide, topiramate, and felbamate
D. Vigabatrin, perampanel, and zonisamide
69. Which of the following is incorrect about E. Clobazam, rufinamide, and pregabalin
primidone?
74. An adolescent boy treated for seizures suf-
A. Same acute adverse effects as fers from a heatstroke during a soccer game.
phenobarbital The boy is most likely taking:
B. Similar chronic adverse effects as
phenobarbital A. Ezogabine
C. The primidone dose needed to produce a B. Pregabalin
certain phenobarbital level is about 5 C. Zonisamide
times that of phenobarbital D. Valproate
D. Similar interactions to phenobarbital E. Carbamazepine
E. Produces two active metabolites
75. Risk of seizure recurrence after first symp-
70. Which of the following antiepileptic drugs tomatic seizure is highest in:
requires dose adjustment in a patient with
newly diagnosed creatinine of 4 mg/dl? A. Acute Stroke
B. Remote stroke
A. Phenytoin C. Acute head trauma
B. Carbamazepine D. Remote head trauma
C. Phenobarbital E. All of the above carry the same seizure
D. Gabapentin recurrence rate
E. Perampanel
262 Multiple Choice Questions for Part IV

76. Which is not true about vigabatrin’s visual 81. Which antiepileptic drug blocks T-type cal-
defect? cium channel:

A. Most visual field defects are irreversible A. Ethosuximide and valproate


B. Central vision is most affected B. Carbamazepine and valproate
C. Risk of visual defect increases with the C. Felbamate and pregabalin
treatment duration D. Valproate and felbamate
D. Periodic eye examination may detect E. Lamotrigine and valproate
early damage
E. May occur in up to one-third of patients 82. Before starting a Taiwanese man on carba-
mazepine for trigeminal neuralgia, which
77. All of the following antiepileptic drugs are genetic testing is recommended:
considered to be broad spectrum except:
A. HLA-B 23
A. Valproate B. HLA-A 322
B. Zonisamide C. HLA-B 1502
C. Pregabalin D. HLA-A 150
D. Lamotrigine E. No need for genetic testing
E. Felbamate
83. Which of the following is known to be an
78. Clinical trials have been completed or are inducer of the cytochrome P450 (CYP):
underway for all but one of the following
treatments for acute repetitive seizures: A. Aspartame
B. White chocolate
A. Intramuscular diazepam by autoinjector C. Aloe Vera
B. Intranasal diazepam D. Yohimbine
C. Intranasal lorazepam E. St John’s wort
D. Intranasal midazolam
E. Buccal midazolam 84. In seizure-free patients, seizure recurrence
after antiepileptic drug withdrawal is lower
79. All but one of the following has an extended in:
release preparation:
A. Children
A. Topiramate B. Adults
B. Zonisamide C. Men
C. Levetiracetam D. Women
D. Oxcarbazepine E. Same in all of the above
E. Lamotrigine
85. In December 2006, at the American Epilepsy
80. As compared to older AEDs, newer AEDs Society meeting, the Charlie Foundation
tend to have: commissioned a panel of 26 pediatric
epileptologists and dietitians from 9 coun-
A. Similar safety tries with expertise in the ketogenic diet, in
B. Better efficacy order to create a consensus statement
C. Lower drug–drug interaction regarding the clinical management of the
D. Lower cost ketogenic diet. Which of the following
E. Higher protein binding statements accurately conveys the
Multiple Choice Questions for Part IV 263

International Ketogenic Diet Study Group’s epilepsy syndromes showed that only
recommendation? 20% had a greater than 75% decrease in
seizures
A. The ketogenic diet should be considered D. There are currently no randomized con-
in a patient whose epilepsy is controlled trolled trials published on the efficacy of
by a single medication, but whose family dietary therapies in epilepsy treatment
prefers “natural” therapies
B. The ketogenic diet should be considered 88. Which antiepileptic drug should be avoided
only as a last resort, since it is hard to in a patient with known sulfa drug allergy?
administer, is unpalatable, and there is no
data from randomized controlled trials to A. Pregabalin
support it B. Zonisamide
C. The ketogenic diet should be considered C. Carbamazepine
in a child who has failed 2–3 anticon- D. Vigabatrin
vulsant therapies, as long as he/she is E. Lamotrigine
older than 1 year and younger than
12 years of age 89. The following statement is false:
D. The ketogenic diet should be considered
in a child who has failed 2–3 anticon- A. A common ratio of the ketogenic diet is
vulsant therapies, regardless of age or 4 g of fat to 1 g of protein plus carbo-
gender hydrate (4:1)
B. In a 4:1 ratio ketogenic diet, approxi-
86. Brivaracetam has the following mechanism mately 90% of energy comes from fat
of action: C. A fasting period during initiation of the
ketogenic diet is necessary to achieve
A. Potassium channel opening ketosis
B. Blocking of sodium channels D. The modified Atkins diet is approxi-
C. Binding to the synaptic vesicle protein mately a 1:1 ratio
D. Binding to the GABAA receptor E. In the low glycemic index treatment,
E. Binding to the synaptic vesicle protein foods are chosen which produce
and blocking of sodium channels slower/steadier changes in blood glucose

87. There has been growing interest and confi- 90. A 72-year-old man is seen in clinic for a new
dence in the use of the ketogenic diet and diagnosis of epilepsy, with recurrent partial
subsequently a growing body of literature. seizures secondary to a right MCA ischemic
Which of the following statements is true? infarct. His medical history includes chronic
afib (for which he takes warfarin), osteo-
A. Four randomized controlled trials porosis, and a history of kidney stones.
showed that at least 38% of patients had Which of the following is the most appro-
a 50% reduction in seizures compared to priate antiepileptic medication for this
controls at 3 months, with this response patient?
maintained for up to a year
B. A meta-analysis has shown that a third of A. Carbamazepine
all patients on the ketogenic diet may B. Levetiracetam
become seizure-free C. Primidone
C. A recent retrospective, multicenter study D. Ethosuximide
assessing the ketogenic diet for various E. Zonisamide
264 Multiple Choice Questions for Part IV

91. In addition to the traditional ketogenic diet, does not typically have spontaneous sei-
alternative dietary therapies have been zures. The audiogenic seizure model is pre-
developed for epilepsy treatment. Which of dictive of efficacy against generalized tonic–
the following is an alternative dietary ther- clonic seizures. Levetiracetam efficacy was
apy for epilepsy treatment? missed by the maximal electroshock model
(MES) and PTZ models. The 6 Hz model is
A. The low glycemic index treatment a model of pharmacoresistant epilepsy.
B. The Atkins diet 4. (B). In the first VA cooperate, all four
C. The Paleo diet antiepileptic drugs compared had equal effi-
D. The short-chain triglyceride diet cacy, but phenobarbital and primidone were
less well tolerated.
92. The blood levels of which antiepileptic will 5. (D). In contrast to the listed benzodi-
not be altered after the addition of azepines, Clobazam is a 1,5 benzodiazepine;
phenytoin? the brand name “On-fi” is derived from
“One-five.”
A. Felbamate 6. (D). Diazepam and clobazam have important
B. Topiramate active metabolites (desmethyldiazepam and
C. Zonisamide desmethylclobazam). Clonazepam is con-
D. Tiagabine verted to an inactive metabolite.
E. Pregabalin 7. (B). Connective tissue diseases in particular
Dupuytren’s contractures and plantar fibro-
matosis may be seen with long-term phe-
Answers nobarbital use.
8. (B). Enzyme-inducing antiepileptic medica-
1. (E). In this case, levetiracetam is the most tions can induce the metabolism of other
appropriate choice for treating juvenile concomitant medications such as carba-
myoclonic epilepsy in a woman of child- mazepine and phenytoin and can lower the
bearing age. Pregabalin and carbamazepine serum concentration of simvastatin by about
are typically efficacious against partial–onset 50%.
seizures and may potentially exacerbate 9. (C). Both zonisamide and topiramate are
generalized seizures, notably absence and mild carbonic anhydrase inhibitors. The
myoclonic seizures. Lacosamide lacks evi- resulting decreased sweat may lead to over-
dence to support its use in this condition, heating and possible heatstrokes especially
while valproate should be avoided in women in children.
of childbearing age due to significant risk of 10. (A). While some of the other antiepileptic
teratogenicity. drugs have a long half-life (such as zon-
2. (A). The clinical scenario describes partial isamide, clobazam, and ethosuximide), they
seizures with secondarily generalization. do not have FDA approval for once-daily
Oxcarbazepine is an appropriate drug for dosing. The main reasons are the potential
treating this condition. Lamotrigine and sedation and cognitive or gastrointestinal
topiramate would not be a good choice due side effects that may incur from high doses
to the slow titration schedule. Phenobarbital given once daily.
and valproate must be avoided in women of 11. (C). Ezogabine may be associated with uri-
childbearing age. nary retention in about 2% of patients. It is
3. (A). The pentylenetetrazole (PTZ) model is generally reported within the first six months
predictive of efficacy against absence sei- of treatment, but can also be observed later.
zures, although some effective drugs may be 12. (C). Felbamate reduces carbamazepine level
missed by this model. The kindling model through induction of CYP3A4, but may
Multiple Choice Questions for Part IV 265

cause accumulation of carbamazepine start folic acid 3–6 months before


epoxide leading to carbamazepine toxicity. conception.
13. (A). Treatment of infantile spasms should be 22. (A). REM behavior disorder is a risk factor
started as soon as the diagnosis is confirmed. for degenerative neurologic disease but not
The first-line recommended therapy includes for epilepsy while all the other choices are
high-dose ACTH or vigabatrin. If the cause established risk factors for developing epi-
is tuberous sclerosis, vigabatrin tends to be lepsy in the elderly.
more effective than ACTH. 23. (D). The addition of topiramate to valproate
14. (B). Among the listed antiepileptic drugs, may cause accumulation of ammonia, which
both levetiracetam and perampanel are more could explain the increased responsiveness
associated with irritability, which may limit and somnolence.
their use. 24. (E). Lamotrigine protein binding is low
15. (E). In contrast to most antiepileptic drugs (55%) compared to the other choices that
which are sedating, felbamate and lamot- have higher protein binding (>70%) causing
rigine are stimulating drugs and may be high potential for drug–drug interactions.
associated with insomnia. 25. (D). Levetiracetam is indicated as adjunctive
16. (A). Levetiracetam was first approved in treatment for myoclonic seizures.
USA in 1999. Its mechanism of action is 26. (D). With advancing age, several physio-
binding to the synaptic vesicle protein logical changes have a direct effect on drug
SV2A. This seems to result in nonspecific pharmacokinetics. These include decreased
decrease in neurotransmitter release. There is renal and hepatic clearance, reduced and
a functional correlation between SV2A erratic absorption, and decreased protein
binding affinity and anticonvulsant potency binding. All these changes can result in
of levetiracetam analogues. higher antiepileptic blood levels with higher
17. (E). Felbamate is anti-NMDA receptor risks of adverse effects and toxicity.
antagonist. Topiramate and perampanel are 27. (C). Progesterone can be effective in the
AMPA antagonists. subgroup of women with catamenial epi-
18. (B). Q-T prolongation may occur with the lepsy who have the C1 pattern, with seizure
use of ezogabine, while P-R prolongation exacerbations around menstrual periods.
may occur with the use of lacosamide. 28. (A). The 6-Hz psychomotor seizure model in
19. (B). All of the listed properties are true about mice and the methylazoxymethanol acetate
perampanel except for a short half-life. Per- (MAM) rat model of cortical dysplasia serve
ampanel’s half-life is long, about 105 h on as a model of pharmacoresistant epilepsy.
average. 29. (C). In catamenial epilepsy, seizures tend to
20. (B). Changes in AED pharmacokinetics are follow a cyclical pattern related to the
common during pregnancy due to increased menstrual cycle. There are three cyclical
clearance most prominent for lamotrigine, patterns of catamenial epilepsy: C1 pattern
but other AEDs show substantially more where seizures increase in frequency just
modest increases in clearance. Seizures before and during menses, C2 pattern where
worsen when AED levels fall >35% from seizures increase around the time of ovula-
preconception levels. Regular monthly tion, and C3 pattern where seizures occur
monitoring of AED levels during pregnancy with anovulatory cycles. Catamenial epi-
is recommended to adjust the dose to prevent lepsy is thought to be related to progesterone
seizure recurrence. and estrogen fluctuations. Estrogen appears
21. (D). For high-risk pregnant patients with to be proconvulsant, and progesterone
previous history of major teratogenicity, appears to be anticonvulsant.
higher folic acid dose of 4–5 mg/day is 30. (B). Brivaracetam is an analog drug of
recommended. It is also recommended to levetiracetam which has a sodium blocking
266 Multiple Choice Questions for Part IV

mechanism in addition to its SV2A binding. tricarboxylic acid cycle, to be used for
In general, it exhibits a similar profile to energy production or ketone body produc-
levetiracetam general but with a higher tion. A shift to use of fats as the primary
potency (about 10 times higher). energy source in disorders of fat metabolism
31. (B). Valproate can produce tremors and would precipitate deterioration. Lack of
Parkinsonism that is usually dose-dependent carbohydrates would exacerbate acute inter-
and more common in the elderly. It is usu- mittent porphyria.
ally reversible with reduction or elimination 39. (D). Incidence of epilepsy increases with the
of the drug. severity of mental retardation (MR), but it
32. (A). Pentylenetetrazole (PTZ) is an animal varies depending on epidemiological
model of absence seizures, while the other methodologies as well. In population-based
listed animal models are specific for partial studies, 21% of those with mild MR had
seizures. epilepsy. Another study reported epilepsy in
33. (C). Vigabatrin visual toxicity is a slowly 11% of subjects with mild MR and in 23%
progressive, usually irreversible retinopathy in those with severe MR. On the other hand,
that is related to dose and duration of in institution-based studies including
treatment. patients with severe MR, the prevalence of
34. (D). Tiagabine has a short half-life; it is a epilepsy varies from 32 to 34%.
narrow-spectrum agent for partial (focal) 40. (B). Vigabatrin use is associated with visual
seizures, approved only as adjunctive ther- field constriction in about one-third of
apy; it increases GABA levels by inhibiting patients. Periodic visual field monitoring is
its reuptake in the synapse. required for the prescription of vigabatrin.
35. (A). Aplastic anemia and hepatic failure are 41. (E). Premanufactured ketogenic formulas are
unlikely within one month of initiating fel- available, ensuring more accuracy of mea-
bamate therapy. Known risk factors include surements and minimizing barriers such as
prior cytopenia, allergy to or significant food refusal or aversions.
toxicity with other antiepileptic drugs, and 42. (A). Prolonged encephalopathy or noncon-
underlying autoimmune disease. vulsive status epilepticus may be seen as a
36. (C). Valproate is a wide-spectrum dose-related adverse effect of tiagabine.
antiepileptic drug effective against absence 43. (B). This should adequately screen for dis-
seizures; the other listed AEDs may exac- orders of fatty acid metabolism including
erbate absence seizures. carnitine deficiency, CPT I or II deficiency,
37. (D). Unlike other listed antiepileptic drugs, carnitine translocase deficiency, and the
pregabalin does not interact with GABA beta-oxidation defects. The other choices are
receptors. also reasonable considerations for preinitia-
38. (C). In pyruvate dehydrogenase deficiency, tion screening, but for other conditions.
pyruvate cannot be metabolized into 44. (A). Valproate inhibits uridine glucosyl
acetyl-coA. The ketogenic diet bypasses this transferase, the enzyme that metabolizes
step and provides ketones as an alternative lamotrigine. Initiation of valproate therapy
fuel for the brain. All of the other choices are will result in lamotrigine toxicity. This
contraindications to the ketogenic diet. addition usually requires immediate reduc-
Long-chain fatty acids are transported across tion of the dose of lamotrigine by about
the mitochondrial membrane by carnitine 50%.
(helped by CPT I and II and carnitine 45. (D). This would be a longer-term compli-
translocase); once in the mitochondrion, cation. Osteoporosis in the ketogenic diet is
fatty acids are beta-oxidized to 2 carbon contributed to by calcium/vitamin D defi-
units of acetyl-CoA that can then enter the ciency as well as acidosis.
Multiple Choice Questions for Part IV 267

46. (B). Most common known etiology for epi- of choice for pure generalized absence
lepsy is cerebrovascular disease at 11%, seizures.
followed by neurologic deficits from birth, 51. (B). Although the benefit on seizure control
mental retardation, or cerebral palsy at 8%. can be seen within 2 weeks after initiation
47. (D). Long-term complications in children on (in 75% of children in one study), it is rec-
the ketogenic diet for >2 years have not ommended that the ketogenic diet be con-
been systematically reviewed. There may be tinued for 3 months before deciding to
increased fractures and kidney stones. continue or discontinue. Gradual weaning
Symptoms, however, would be expected in rather than abrupt discontinuation is pre-
the setting of pancreatitis, renal calculi, or ferred and may assist with determining
severe constipation. Specific guidelines for whether there has been benefit of the keto-
monitoring of bone health, however, still genic diet on seizure control. The recom-
need to be delineated. mendation is to discontinue after 2 years of
48. (C). In GLUT1 deficiency syndrome, glu- seizure freedom, similar to the time period
cose transport across the blood-brain barrier used for anticonvulsant medications.
is impaired. Since the ketogenic diet pro- 52. (C). The likely diagnosis of this patient is
vides ketones that bypass the metabolic neurocysticercosis. Epilepsy is the most
defect, serving as an alternative fuel to the common presentation (70% of patients) fol-
brain, the ketogenic diet is the treatment of lowed by headache, stroke, and psychiatric
choice for this syndrome. Such epilepsy manifestations. It is more common in
treatment is not necessary for benign myo- Southern America due to ingestion of
clonus of infancy. Although the ketogenic uncooked egg-infected pork meat. Brain
diet may be particularly helpful for gener- imaging often reveals several ring-enhancing
alized epilepsies, there has not been data lesions. Treatment consists of anthelminthic
supporting its use in JME as of yet. The medication (such as albendazole) and ster-
ketogenic diet is contraindicated for pyru- oids (e.g., dexamethasone) to suppress the
vate carboxylase deficiency, which would inflammatory response induced by destruc-
impair tricarboxylic acid cycle function and tion of live cysticerci.
energy production in the ketogenic diet. 53. (D). Valproate exposure during pregnancy is
49. (B). Isoniazid is an antibiotic commonly associated with decreased verbal IQ and
used in treating tuberculosis. It may trigger autism in offsprings. The teratogenic effect is
de novo seizures by competing with the dose-dependent and is irrespective of
mechanism of pyridoxine and its metabo- monotherapy or polytherapy use. Supple-
lites. Pyridoxine is an essential cofactor for mentation with folic acid is not sufficient to
many enzymatic reactions, including GABA reverse the teratogenic effect.
an inhibitory neurotransmitter. 54. (C). This patient is suffering from partial–
50. (B). A large, multicenter, double-blind, ran- onset seizures and postherpetic neuralgia for
domized, controlled trial to compare the which pregabalin is FDA indicated for.
efficacy, tolerability, and neuropsychological Topiramate, lamotrigine, and levetiracetam
effects of ethosuximide, valproic acid, and have not yet been proven effective in this
lamotrigine favored ethosuximide. After setting. Carbamazepine could be helpful but
16 weeks of therapy, the has the risk of drug interactions at the level
freedom-from-failure rates for ethosuximide of hepatic metabolism.
and valproic acid were similar and higher 55. (C). Both valproate and phenytoin are highly
than the rate for lamotrigine. However, protein-bound antiepileptic drugs. When
attentional dysfunction was more common used together or added to a highly
with valproic acid than with ethosuximide. protein-bound medication (such as war-
As a result, ethosuximide became the drug farin), they can compete on protein binding,
268 Multiple Choice Questions for Part IV

increasing the free fraction of either drugs. abnormal initial routine EEG. An additional
Valproate free fraction decreases at higher 35% were identified to have abnormalities
concentrations due to protein saturation. on the second sleep-deprived EEG. An
56. (E). The use of the KD is being investigated in abnormal EEG predicts a higher recurrence
several neurologic conditions beyond epi- rate, and a normal EEG predicts a lower
lepsy and in traumatic brain injury, Alzhei- recurrence rate but does not rule out
mer’s disease, amyotrophic lateral sclerosis, epilepsy.
autism, glial tumors, diabetic nephropathy, 64. (E). Maternal use of valproate during preg-
and Parkinson’s disease. In addition, in nancy was associated with a significantly
development is 2-deoxy-(D)-glucose (2-DG), increased risk of autism spectrum disorder
an agent which is a nonmetabolizable glucose and childhood autism in the offspring.
analog that inhibits glycolysis. 65. (B). Lorazepam and clonazepam do not have
57. (B). Lamotrigine does not affect bone health active metabolites; lorazepam has the short-
while phenytoin, phenobarbital, and topira- est half-life.
mate can, because of their enzyme induction 66. (C). Phenytoin has nonlinear (saturable)
properties. kinetics; its half-life becomes longer after
58. (D). Topiramate should be avoided in elderly the saturation point which is usually within
because it has significant cognitive effects. the recommended therapeutic range.
These can significantly limit or compromise 67. (B). Carbamazepine induces its own meta-
their intellectual reserves. bolism so that its half-life becomes shorter
59. (B). Faciobrachial dystonic seizures are fre- with continued use. The process of autoin-
quent brief dystonic seizures, typically duction is completed over 2–4 weeks.
affecting the ipsilateral arm and face found 68. (C). Recurrence rate after the first seizure
in association with LGI1 antibodies. Facio- averages around 30–40% by two years. The
brachial dystonic seizures often precede risk is higher (approaching 50–60%) when
LGI1-antibody encephalitis. Recognition the EEG or brain MRI is positive.
may lead to early diagnosis and early insti- 69. (A). Primidone is converted into phenobar-
tution of immunotherapy, with improved bital and phenylethylmalonamide (PEMA),
outcome. which is also an active metabolite. Primi-
60. (A). Excluding carbamazepine, all the other done has acute toxic reactions that are dif-
four medications (amiodarone, cimetidine, ferent from phenobarbital. It can produce
fluconazole, and felbamate) may inhibit transient drowsiness, dizziness, ataxia, nau-
phenytoin metabolism and may cause sea, and vomiting that can be debilitating.
phenytoin accumulation. These reactions are present even before
61. (B). Excluding ceftriaxone, all the other phenobarbital has appeared as a metabolite.
three medications (erythromycin, fluoxetine, 70. (D). Among the listed antiepileptic drugs,
and propoxyphene) and grapefruit juice may gabapentin is mostly excreted in urine;
inhibit carbamazepine metabolism and may hence, its dose should be reduced according
cause carbamazepine accumulation. to the renal function.
62. (D). Estrogen may be proconvulsant as it 71. (D). Overall, risk factors that carry higher
may reduce inhibition at the GABAA seizure recurrence rate include focal-onset
receptor and also inhibits the synthesis of seizures, status epilepticus or cluster seizures
GABA. On the other hand, progesterone at first seizure, abnormal EEG with epilep-
may be anticonvulsant as it enhances inhi- tiform activity (sharp waves or spikes), and
bition at the GABAA receptor and increases abnormal brain MRI or neurological
GABA synthesis. examination.
63. (B). In one pooled analysis of 1766 subjects, 72. (E). Levetiracetam does not interact with
51% of patients (pooled analysis) had an oral contraceptive medications, while all the
Multiple Choice Questions for Part IV 269

other choices can lower oral contraceptive tend to offer pharmacological advantages in
medication levels. regard to lower protein binding, minimal
73. (B). Antiepileptic drugs with potential oph- drug–drug interaction, and absence or mini-
thalmological adverse effects include topi- mal liver inhibition/induction. They are,
ramate (acute open-angle glaucoma), however, more expensive than older AEDs.
vigabatrin (peripheral visual constriction), 81. (A). Ethosuximide and valproate are both
and ezogabine (photoreceptor damage). known to block T-type calcium channels,
74. (C). Zonisamide and topiramate are mild which conveys efficacy against absence
carbonic anhydrase inhibitors and are known seizures.
to decrease sweat production (causing 82. (C). HLA allele B*1502 is a marker for
anhydrosis or hypohydrosis). This may par- carbamazepine-induced Stevens–Johnson
ticularly be dangerous in patients with high syndrome and toxic epidermal necrolysis,
physical activity, without compensatory particularly in Han Chinese. The FDA rec-
hydration. ommends genotyping all Asians for the
75. (B). Risk of seizure recurrence after first allele before treatment initiation.
symptomatic seizure is about 33% in an 83. (E). St John’s wort (or Hypericum perfora-
acute (less than 7 days) stroke setting and tum) is a popular medicinal herb used for the
70% in a remote setting (more than 7 days). treatment of depression. It is known to
Risk of seizure recurrence after first symp- induce cytochrome P450 affecting the phar-
tomatic seizure is about 13% in the setting of macokinetics of several AEDs such as phe-
acute TBI and 45% in the setting of remote nobarbital, carbamazepine, and phenytoin,
TBI. resulting in adverse events.
76. (B). Vigabatrin’s visual field defect may 84. (A). Overall, 11–41% of patients will relapse
affect about one-third of patients with vari- after antiepileptic drug discontinuation. The
able severity. It primarily consists of relapse rate tends to be lower in children
peripheral visual field constriction. Risk (*20%) and higher in adults (*40).
factors include longer duration of treatment 85. (D). This is the consensus statement made
and higher dosage. by the International Ketogenic Diet Study
77. (C). All of the listed antiepileptic drugs are Group.
considered to be broad-spectrum except 86. (E). Brivaracetam binds to the synaptic
pregabalin which targets focal-onset seizures vesicle protein as well as blocks sodium
and may alternatively worsen generalized channels.
seizures especially myoclonic seizures. 87. (A). Compared to the Cochrane Review
78. (C). Several clinical trials were concluded or performed in 2003, in which no RCTs were
underway for (up to the current writing) the available, the Cochrane Review in 2012
treatment of acute repetitive seizures. This reviewed 4 RCTs, which showed that at least
includes intranasal diazepam and midazo- 38% of patients had a 50% decrease in sei-
lam, intramuscular diazepam (by autoinjec- zures at 3 months, with this positive
tor), and buccal midazolam. response maintained for a year. Hender-
79. (B). Zonisamide does not have an extended son CB et al’s meta-analysis in 2006 showed
release preparation, but has a prolonged 1/3 of patients having a >90% decrease in
half-life, obviating the need for such a seizures. Caraballo R et al’s multicenter
preparation. retrospective study in 2011 showed 22%
80. (C). Overall, when compared to older AEDs, with seizure freedom and 56% with greater
newer AEDs tend to have better safety profile than 75% decrease.
and tolerability (except for felbamate, viga- 88. (B). Zonisamide’s chemical structure
batrin) but comparable efficacy. They also includes a sulfa moiety and thus should be
270 Multiple Choice Questions for Part IV

avoided in patients with known history of exclusively against absence seizures. Car-
sulfa allergy. bamazepine and primidone are liver enzyme
89. (C). Traditionally, the patient would fast for inducers that can decrease warfarin efficacy
24–48 h. Once urine ketones appeared, the and worsen osteoporosis. Zonisamide can
ketogenic diet would then be initiated grad- precipitate kidney stones.
ually. Data support the fact that ketosis 91. (A). The Modified Atkins diet, the medium-
occurs without this initial fasting period and chain triglyceride diet, and the low glycemic
that tolerance of the diet may be higher index treatment are alternative dietary ther-
without this fasting period. apies developed for epilepsy treatment.
90. (B). In this case, levetiracetam is the most 92. (E). Pregabalin is not metabolized and is not
appropriate choice for treating partial sei- affected by phenytoin and other enzyme
zures. Ethosuximide is efficacious inducers.
Part V
Presurgical Evaluation
and Epilepsy Surgery
Neuroimaging in Epilepsy
21
Anuradha Singh, Priyanka Sabharwal
and Timothy Shephard

provide better details about bony structures and


Computed Axial Tomography
can easily detect hemorrhages, calcifications,
strokes after 24 h, and large tumors. However,
Computed tomography (CT) is widely available
CT may fail to recognize commonly encountered
and used in emergency rooms. With multiple
lesions in patients with epilepsy such as hip-
detector helical CT, high spatial resolution can be
pocampal atrophy, hippocampal sclerosis, corti-
obtained in all dimensions allowing 3D recon-
cal dysplasias, or low-grade gliomas (LGG).
structions and operator selected multiplanar
reformats in any plane. CT may be the only
option in patients who cannot obtain MRI due to
contraindications such as patients with pace-
Magnetic Resonance Imaging (MRI)
makers, defibrillators, specific metal prostheses
MRI is a noninvasive test, which does not cause
(e.g., cochlear implants), ferromagnetic aneur-
exposure to ionizing radiations and is considered
ysm clips, metallic foreign bodies within eyes,
safe across all age groups. It possesses excellent
and shrapnel or bullets located near vascular
spatial resolution down to millimeters. MRI
structures. There may be other barriers in
scanners with higher magnetic strengths (3-, 4-,
obtaining a conventional MRI in unstable or
or 7-Tesla) have proliferated over the past dec-
uncooperative patients, or patients with signifi-
ade. The typical MRI epilepsy protocol includes
cant claustrophobia or morbid obesity. CT scans
multiplanar diffusion, T2-weighted, FLAIR,
gradient echo, or susceptibility-weighted images
of the brain. This protocol is supplemented with
A. Singh (&) a 3D volumetric T1-weighted acquisition and
Department of Neurology, Bellevue Hospital Center, oblique coronal plane FLAIR and T2-weighted
462, 1st Avenue and 27th Street, NBV 7W11, New images orthogonal to the long axis of the tem-
York, NY 10016, USA poral lobes. Gadolinium administration may have
e-mail: anuradha.singh@bellevue.nychhc.org;
anuradha.singh@nyumc.org utility in certain patient populations with seizures
or when there is clinical suspicion for infectious,
P. Sabharwal
Department of Neurology, NYU Langone Medical inflammatory, or neoplastic etiologies.
Center, 223 East 34th Street, New York, NY 10016, The presence or absence of lesion(s) and their
USA location on brain MRI helps identify patients
e-mail: priyanka.sabharwal@nyumc.org
who may be good surgical candidates for
T. Shephard lesionectomy, corticectomy, topectomy, corpus
Department of Radiology, New York University callosotomy, or hemispherectomy. For example,
Langone Medical Center, 660 First Ave., 2nd Floor,
New York, NY 10016, USA patients with medically refractory seizures and
e-mail: timothy.shepherd@nyumc.org hippocampal sclerosis should be evaluated for

© Springer Science+Business Media LLC 2017 273


M.Z. Koubeissi and N.J. Azar (eds.), Epilepsy Board Review,
DOI 10.1007/978-1-4939-6774-2_21
274 A. Singh et al.

either standard anterior temporal lobe resection differentiation in the temporal lobe or decreased
or selective amygdalohippocampectomy. A le- white matter in the adjacent temporal lobe (e.g.,
sionectomy of a cavernoma with or without collateral eminence and temporal stem). There
corticectomy is a reasonable option if can be atrophy of the ipsilateral fornix and
anti-seizure medications are not effective in mammillary body (Fig. 21.4). High-resolution
controlling seizures. A more aggressive surgical 3D T1WI are also useful when performing hip-
approach, such as a functional hemispherectomy, pocampal volumetric analyses. However, para-
may be warranted in patients with Sturge Weber central lesions are more evident on axial
syndrome, Rasmussen’s encephalitis, large sequences. Figure 21.5 shows a less common
porencephalic cyst due to previous traumatic or case of temporal lobe seizures from a temporal
ischemic insult, hemimegalencephaly, or Dyke– lobe encephalocele involving the subjacent
Davidoff–Masson syndrome (congenital or sphenoid wing.
acquired). Neuroimaging may have a direct Neuronal migrational disorders: Hetero-
influence on therapeutic options that neurologists topias are neuronal migrational disorders
offer to their patients; e.g., a scan supporting a (NMDs) where gray matter gets arrested as
diagnosis of Tuberous Sclerosis (TS) in an infant neurons migrate from periventricular regions
with infantile spasms may justify the use of toward pia during embryonic stages.
vigabatrin or m-TOR inhibitors over adrenocor- High-resolution 3D T1-weighted volumetric
ticotrophic hormone (ACTH). Not all structural imaging provides superior gray–white contrast
lesions are epileptogenic; therefore, it is prudent that is critical to identify subtle cortical malfor-
to correlate incidental findings on MRI with mations in patients with epilepsy (Fig. 21.6).
clinical history, seizure semiology, and EEG Higher magnetic strengths (3- or 7-Tesla) can
data. Some of the commonly encountered lesions detect very subtle cortical dysplasias.
in patients with epilepsy are described below. Heterotopias can either be focal, nodular, or
Mesial temporal sclerosis: Temporal lobe multifocal (as in TS) or preferentially involve
epilepsy (TLE) is the most common form of focal one hemisphere as in hemimegalencephaly.
epilepsy. Mesial TLE is more prevalent than Subcortical band heterotopias (SBH) are typi-
neocortical epilepsy and often intractable to cally periventricular, bilateral nodular collections
anti-seizure medications. The most common of gray matter with relatively smooth margins,
identifiable lesion on MRI brain is mesial tem- which gives the appearance of a double cortex.
poral sclerosis (MTS; Fig. 21.1). In patients with Pachygyria is abnormal tissue in the right loca-
TLE, subtle anatomic features of the medial tion with abnormal sulcation and gyration of the
temporal lobes and pathologies such as MTS or mantle which is typically > 8 mm thick
incomplete hippocampal inversion (Fig. 21.2) (Fig. 21.7a). Polymicrogyria (PMG) is either
are best appreciated in an oblique coronal plane. two- or four-layered cortex, which is less than 5–
This orientation is orthogonal to the long axis of 7 mm (Fig. 21.7b). PMG is commonly associ-
the temporal lobe and reduces volume averaging ated with hypoxic-ischemic injury, or prenatal
problems for the thin laminar appearance of the cytomegalovirus (CMV) infection.
hippocampus. Oblique coronal temporal FCDs are classified into three categories (Type
high-resolution T2-weighted and FLAIR are the I, II, and III) and further divided into various
best sequences to diagnose MTS. This entity is subtypes (Table 21.1). In a fully myelinated brain,
characterized by (1) hippocampal atrophy, FCD type I may be characterized by subtle blur-
(2) increased T2 signal, and (3) abnormal mor- ring of the gray–white junction with typically
phology or loss of internal architecture of hip- normal cortical thickness, moderately increased
pocampus. In 10% of the cases, MTS can be white matter signal hyperintensities on T2/FLAIR
bilateral (Fig. 21.3). Secondary findings may images and decreased signal intensity on
include dilatation of the temporal horn of the T1-weighted images. FCD Type IIA cortical
lateral ventricle, loss of gray–white matter dysplasias are characterized by marked blurring of
21 Neuroimaging in Epilepsy 275

Fig. 21.1 Axial FLAIR (a) demonstrates enlargement of Companion coronal FLAIR (b) and T2-weighted MRI
the left lateral ventricle temporal horn and the left (c) demonstrate volume loss, hyperintensity, and subtle
hippocampus is relatively smaller and hyperintense com- laminar blurring. These are classic MRI findings for left
pared to the contralateral side. Note that the lateral aspect hippocampal sclerosis. If the amygdala also is involved,
of the left hippocampal body is abnormally smooth, and this can be classified as left mesial temporal sclerosis
hippocampal head digitations are reduced (arrow).

Fig. 21.2 Coronal T2-weighted images demonstrating inversion (IHI). This patient also had left hippocampal
globular left hippocampus (arrow, panel a) more vertical sclerosis, but it remains controversial whether IHI
left collateral sulcus and low-lying left body of the fornix predisposes to sclerosis or is just an incidental association
(arrow, panel b) consistent with incomplete hippocampal

the gray–white junction on T1 and T2-FLAIR (aka the “transmantle sign”) which marks the
images due to hypomyelination or dysmyelination involvement of radial glial neuronal bands. This
of the subcortical white matter with or without radiological feature differentiates FCD from
cortical thickening. Here, the increased white low-grade tumors. Type II lesions are more
matter signal changes on T2, WI, and FLAIR commonly seen outside the temporal lobe with
images frequently tapers toward the ventricles predilection for the frontal lobes. Type III FCD is
276 A. Singh et al.

Fig. 21.3 Coronal FLAIR


(a), T2-weighted (b) and
3D T1-weighted
demonstrate bilateral
hippocampal body
hyperintensity, laminar
blurring, and volume loss,
respectively. Findings are
consistent with bilateral
hippocampal sclerosis

typically associated with another principal lesion predominantly gyral abnormalities is caused by
such as hippocampal sclerosis, tumor, a vascular mutation in LIS1 gene (Fig. 21.8). Anteriorly
malformation, or other acquired pathology during predominant lissencephaly in heterozygous males
early life. and subcortical band heterotopia (SBH) in
Other important NMDs include lissencephaly, heterozygous females are caused by mutations of
which is characterized by smooth brain surface and the XLIS (double cortex gene on chromosome X).
abnormal gyration, which varies between agyria Schizencephaly is another rare form of MCD
and pachygyria. Lissencephaly with posteriorly which is characterized by the presence of a
21 Neuroimaging in Epilepsy 277

Fig. 21.4 Oblique coronal T2 demonstrates obvious gray matter heterotopia in the lateral wall of the right
volume loss and laminar blurring of the right hippocampal lateral ventricle, best seen on the coronal double-inversion
head (arrow, panel a) consistent with right hippocampal recovery image (arrow, panel c) compared to companion
sclerosis and right fornix atrophy (b). There is a small T2 and T1-weighted MRI (b and d)

Fig. 21.5 Axial and coronal T2-weighted MRI, and abnormality is not always associated with seizures, but
coronal FLAIR demonstrate a small encephalocele that should be considered suspicious. In this case, the finding
involves a focal portion of the fusiform gyrus cortex was concordant with semiology and EEG
extending into the right foramen ovale (arrows). The MRI

transcortical cleft that can extend from ventricles Brain tumors: Approximately 20–40% of the
to the pia with open or fused lips, and often adults with primary brain tumors experience one
polymicrogyria is seen on the lips of the schizen- seizure prior to the tumor diagnosis, and another
cephaly (Fig. 21.9). Hemimegalencephaly is the 20–45% will suffer from seizures during the
unilateral hamartomatous excessive growth of all course of the illness [1]. This incidence rate
or part of one cerebral hemisphere at different varies depending on the tumor type, the grade of
phases of embryologic development. MRI in these the tumor, and its location. Seizures are more
cases reveals an enlarged hemisphere with common in slow growing tumors such as
increased white matter volume, cortical thicken- meningiomas, gangliogliomas (GGs), dysemby-
ing, agyria, pachygyria, polymicrogyria or lissen- oplastic neuroepithelial tumors (DNETs), or dif-
cephaly, and blurring of the gray–white matter fuse low-grade tumors such as Grade II
junction. Often, a large, ipsilateral irregularly astrocytomas, oligodendrogliomas, and oligoas-
shaped ventricle may be seen. trocytomas (Table 21.2). Typically, the low-grade
278 A. Singh et al.

Fig. 21.6 Coronal FLAIR MRI (a) and serial axial anterior cingulate gyrus and adjacent left medial frontal
FLAIR MRI of the frontal lobes (b) demonstrating subtle gyrus (arrows) from a pathologically proven cortical
gray–white blurring and FLAIR hyperintensity in the left dysplasia

Fig. 21.7 a Coronal 3D T1-weighted (A) and axial show abnormal cortex that appears thickened extending
FLAIR MRI (B) demonstrate broad, simplified gyri with from the posterior right Sylvian fissure (arrow). The
relatively shallow sulci in the bifrontal regions compared fissure is lengthened with a vertical orientation toward the
to the temporal and parietal regions consistent with vertex (arrow, panel C). Findings consistent with unilat-
pachygyria. b Coronal and axial FLAIR (A and B), eral polymicrogyria
sagittal, and axial post-contrast 3D T1 (C and D) MRI

tumors do not enhance on Gd-administration. The Gangliogliomas typically present with temporal
most common location is temporal lobe, followed lobe epilepsy, presumably due to the temporal
by the parietal, frontal, and occipital lobes. lobes being a favored location (Fig. 21.10).
21 Neuroimaging in Epilepsy 279

Table 21.1 Classification of focal cortical dysplasias


Types Features
Type I Ia: abnormal vertical alignment of neurons
Ib: abnormal horizontal alignment
Ic: horizontal and vertical malalignment
Type II IIa: dysmorphic neurons without balloon cells
IIb: dysmorphic neurons with balloon cells
Type III IIIa: mesial temporal sclerosis
IIIb: glioneural tumors e.g., ganglioglioma, DNET
IIIc: vascular malformations (CCMs, AVMs, telangiectasias, and meningoangiomatosis)
IIId: prenatal or perinatal ischemic injury, TBI, and scars due to inflammatory or infectious lesions

Fig. 21.8 MRI brain axial T2-weighted image shows


lissencephaly agyria and smooth brain. Especially poste- Fig. 21.9 CT brain axial image shows a cleft in the left
riorly in a patient with LIS1 mutation hemisphere consistent with schizencephaly

Gangliogliomas are closely related to gangliocy- (60%) followed by temporal lobes (30%).
tomas, which contain essentially only mature Meningiomas are the most common extra-axial
neural ganglion cells, and ganglioneurocytoma, tumors of the central nervous system. They are
which in addition have small mature neoplas- nonglial neoplasms that originate from the
tic neurons. On MRI, these tumors may show arachnoid cap cells of the meninges and have
cystic changes or calcifications. characteristic imaging findings, although there
DNETs are cortically based benign neoplastic are many variants (Fig. 21.12). GBMs are
cortical malformations, which may show sub- aggressive malignant tumors that are associated
cortical extension in approximately 30% of with significant vasogenic edema and heteroge-
tumors giving them a triangular appearance neous enhancement. The overall incidence of
(Fig. 21.11). These tumors appear as seizures in Grade IV glioblastoma multiforme
well-defined lobulated, and solid tumors that are (GBM) patients, without considering the loca-
hyperintense on T2WI and may erode the over- tion, has been reported between 25 and 50% at
lying calvarial bone or show microcystic chan- presentation and another 20–30% during the
ges. The most common location is temporal course of the disease [2]. Metastatic lesions tend
280 A. Singh et al.

Table 21.2 Common epilepsy associated tumors


Tumors Radiological features
Meningioma Isointense on T1 and T2; homogeneous enhancement with Gd, extra-axial,
dural tail, and CSF cleft sign
Ganglioglioma Cyst with enhancing mural nodule/solid; calcifications in *50%
Dysembryoplastic neuroepithelial Bubbly cystic appearance with small cysts within the tumor that are
tumors (DNET) hyperintense on T2WI, wedge shaped mass which expands the affected gyri
and point toward the ventricle, swollen gyrus, may be associated with focal
cortical dysplasia
Pleiomorphic xanthoastrocytoma Supratentorial cyst with enhancing mural nodule which abuts the peripheral
(PXA) meninges, peritumoral edema, mild meningeal enhancement
Oligodendroglioma Hypointense on T1, hyperintense on T2, calcification seen as areas of
blooming, 50% enhance heterogeneously, minimal peritumoral edema
Hypothalamic hamartomas Nonenhancing non-neoplastic congenital gray matter heterotopia in the region
of tuber cinereum of the hypothalamus which can be sessile or pedunculated
Subependymal giant cell Heterogeneous mass near the Foramen of Monro, usually >1 cm; hypo or
astrocytomas (SEGA) isointense on T1 and hyperintense on T2, marked enhancement; other findings
of Tuberous Sclerosis such as cortical tubers and subependymal nodules,
“transmantle sign” in some tubers; nodular, ill-defined, cystic and band-like
lesions seen in the white matter and radial bands
Glioblastoma multiforme Hypo or isointense on T1, hyperintense on T2, vasogenic edema, susceptibility
(GBM) artifact on T2 from intratumoral lesions due to hemorrhage or rarely
calcification, “butterfly glioma” when bilateral and cross the corpus callosum,
necrosis may be present, peripheral or irregular nodular enhancement; no
diffusion restriction but lower ADC than low-grade tumors
Metastases Hypointense on T1 (except melanomas can be hyperintense), hyperintense on
T2 and FLAIR, intense enhancement (ring-enhancing, punctate or uniform),
often multiple lesions present at diagnosis, vasogenic edema out of proportion
to the size of the lesion, hemorrhage, and necrosis may be seen

Fig. 21.10 Axial FLAIR (a), T2-weighted (b), posteromedial temporal lobe with focal areas of contrast
pre-contrast (c), and post-contrast T1-weighted MRI enhancement most consistent with ganglioglioma. CT
(d) demonstrating solid and cystic mass in the left also often demonstrates focal calcification

to have a smaller risk for seizures, one exception the first pass of a bolus of contrast agent. This
being metastatic melanoma. method allows the radiologist to determine the
Perfusion-weighted imaging is a useful tool, relative cerebral blood volume (rCBV). In gen-
which involves several image acquisitions during eral, the underlying principle is the greater the
21 Neuroimaging in Epilepsy 281

rCBV, the higher the grade of tumor. Lack of malformations (AVMs), which can be
notable flow indicates a nonneoplastic etiology parenchymal, dural, or mixed. In contrast, low
with abnormal signal intensity, such as flow vascular malformations are cerebral cav-
demyelination. Of note, mixed oligoden- ernous malformations (CCMs), developmental
drogliomas can have low rCBV. Besides the venous anomaly (DVA), or mixed vascular
prognostic information it provides, malformations (Fig. 21.15). CCMs have a
perfusion-weighted imaging can increase the unique “popcorn” appearance with hemorrhages
yield of brain biopsy and help in differentiating of different ages. They may be bright on CT due
recurrent neoplasm from radiation necrosis. On to pooling of blood within the cavernoma. The
perfusion MRI, GBMs typically show increased most characteristic feature is blood products of
regional blood flow (Fig. 21.13). different ages with an area of hyperintensity
Another interesting but rare kind of focal representing methemoglobin surrounded by a
congenital tumor is hypothalamic hamartoma hypointense ring of hemosiderin on T2W MRI.
(HH). These tumors are typically associated with Gradient echo (GRE) sequences are useful to
ictal spells of laughter without mirth or gelastic detect CCMs and may show low or minimal
seizures (Fig. 21.14). HH are composed of enhancement on Gd. MRI brain is superior to CT
cytologically normal, small, and large neurons, scan to look for the nidus of an AVM which is
which are organized in poorly demarcated clus- hyperdense compared to adjacent brain
ters of variable size and density. These tumors (Fig. 21.16). It is easier to appreciate fast flow
are categorized by the Delalande classifications voids on T2WI due to fast flow and enlarged
I–IV. Type I has a horizontal orientation and may draining veins may be seen. Phase contrast MR
be lateralized on one side; Type II has a vertical angiography can help subtract the hematoma
orientation and an intraventricular location; components in patients who present with acute
Type III is a combination of types I and II; hemorrhage into an AVM. CTA can demonstrate
Type IV is a giant hamartoma. feeding arteries, nidus, and draining veins visi-
Vascular malformations: Vascular malfor- ble, which resembles a “bag of worm” appear-
mations can be either high flow or low flow. ance. The exact anatomy of feeding vessels and
High flow malformations include arteriovenous draining veins is often difficult to delineate, and

Fig. 21.11 Axial T2- and


T1-weighted MRI (a and
b) demonstrate bubbly T2
bright lesion in the left
superior frontal gyrus with
minimal mass effect that
does not enhance (contrast
not shown). In a patient
with seizures, these
findings are most consistent
with dysembyroblastic
neuroepithelial tumor
(DNET)
282 A. Singh et al.

Fig. 21.12 Axial pre-contrast (a) and post-contrast gray matter and shows homogeneous enhancement with
T1-weighted MRI (b) demonstrate an extra-axial mass dural tail consistent with meningioma
overlying the right frontal operculum that is isointense to

Fig. 21.13 MRI brain post-Gd T1-weighted image consistent with glioblastoma multiforme. Perfusion MRI
(a) showing ring enhancement in the right thalamus and (b) shows increased regional cerebral blood flow at the
deep gray structures with associated mass effect, midline tumor margins
shift, and obstructive hydrocephalus. Pathology was

thus angiography remains necessary. Digital brain, both in cortical layers and in blood vessels
subtraction angiography (DSA) remains the gold and identify hemosiderin in CCMs, old infarcts
standard in delineating the location and number or old contusions.
of feeding vessels supplying the central nidus and Infectious/Inflammatory disorders: A common
the pattern of venous drainage (superficial or etiology for seizures in all age groups, both in the
deep). The susceptibility-weighted images (SWI) developing and developed world, includes infec-
are particularly sensitive to iron content in the tions of the nervous system. A wide variety of
21 Neuroimaging in Epilepsy 283

Fig. 21.14 Coronal T2-weighted (a) and sagittal patient with gelastic seizures. Findings support diagnosis
T1-weighted MRI (b) demonstrating ectopic gray matter of hypothalamic hamartoma
along the wall of the 3rd ventricle (arrow) in adolescent

Fig. 21.15 Axial T2-weighted and axial susceptibility- (arrow). Pre- and post-contrast axial MRI (c and
weighted MRI (a and b) demonstrate a popcorn-like T2 d) demonstrate associated developmental venous anomaly
bright lesion with surrounding hypointense susceptibility (arrow) strongly supporting diagnosis of cavernoma

pathogens including viral, bacterial, fungal, para- additional information, which is discussed in rel-
sitic, and other opportunistic pathogens can cause evant sections in this chapter. Briefly, restricted
CNS disease in humans. Though gold standard of diffusion helps in differentiation of progenitor
diagnosis remains either biopsy or CSF analysis, abscesses from ring-enhancing lesions of other
neuroimaging can aid in rapid diagnosis by help- etiology. Also, the presence of lactate and
ing identify typical lesion patterns. While it is cytosolic amino acids and the absence of choline
outside the scope of his chapter to comprehen- on MRS are seen in the cases of pyogenic
sively discuss CNS infections, we have attempted abscesses. Autominnune encephalitides can also
to list typical radiology findings with common be associated with seizures. Figure 21.17 illus-
CNS infections associated with epilepsy in trates an example of progressive volume loss and
Table 21.3. More recent MRI techniques, such as denudation of the overlying cortex Rasmussen’s
DWI and MRS also aid in diagnosis by providing syndrome is one kind of autoimmune encephalitis
284 A. Singh et al.

Fig. 21.16 MRI brain


axial T1-weighted image
(a) shows right mesial
frontal arteriovenous
malformation. Post-Gd
T1-weighted (b) reveals
enlarged draining vein

associated with intractable unilateral seizures, TBI. SWI and DWI are the best sequences to
progressive hemiparesis or weakness on one side, detect DAI. Tong et al. group showed that number
and intellectual dysfunction. of hemorrhagic DAI lesions seen on SWI was six
Neurocutaneous Syndromes (Phacomatoses): times greater than that on conventional
These are a group of inherited disorders charac- T2-weighted 2D GRE imaging and the volume of
terized by hamartomas and neoplasms through- hemorrhage was approximately twofold greater.
out the body along with involvement of the
nervous system and skin. The neuroradiological
features of common phacomatoses, namely, Positron Emission Tomography (PET)
Tuberous sclerosis (TS), Neurofibromatosis
(NF1 and NF2), and Sturge Weber syndrome PET is a noninvasive, diagnostic imaging tech-
(SWS) are summarized in Table 21.4. nique for measuring the metabolic activity of
Figure 21.18 illustrates a few salient neuro- cells in the human body. PET studies character-
radiological features of TS and SWS in patients ize genotype–phenotype interactions because this
with medically refractory seizures. technique directly measures neurometabolic
Trauma: Traumatic brain injury (TBI), espe- changes and receptor binding. PET produces
cially severe closed skull injury and penetrating images of the body by detecting the radiation
dural injury have been well documented to cause emitted from radioactive substances. These sub-
post-traumatic epilepsy [3]. Patients with pro- stances are injected into the body and are usually
longed loss of consciousness, post-traumatic tagged with a radioactive atom (11C, 18Fl, 15O or
13
amnesia, or hemorrhage in the brain (subarach- N) that has short decay time (Table 21.6). The
noid hemorrhage, subdural, epidural, intra- radioactivity localizes in the appropriate areas of
parenchymal, and intraventricular) are at a higher the body and is detected by the PET scanner.
risk of developing immediate (onset within 24 h), Different colors and/or degrees of brightness
early (onset within a week), or late-onset epilepsy. on a PET image represent different levels of tis-
The findings on imaging vary from contusions, sue or organ function. For example, as healthy
with or without diffuse axonal injury (DAI), or tissue uses glucose for energy, it accumulates
hemorrhages in different locations (Table 21.5). some of the tagged glucose, which shows up on
T2WI and the FLAIR images are sensitive to PET images. However, epileptogenic tissue dur-
edema in the brain, while GRE and SWI are very ing interictal phases utilizes less glucose than
sensitive to microhemorrhages. Figure 21.19 healthy normal tissue, and thus, it appears less
shows temporal encephalomalcia as a result of bright than normal tissue on the PET images
21 Neuroimaging in Epilepsy 285

Table 21.3 Radiological features of common infections


Infections Radiological features
Brain abscess Ring of isodense or hyperdense tissue with central hypoattenuation on CT. MRI classically
shows T1 hypointense, T2/FLAIR hyperintense, Diffusion restricted lesion. The four stages
of an abscess with imaging findings are listed below.
* Early cerebritis—Poorly marginated cortical or subcortical hypodensity with mass effect
with little or no enhancement
* Late cerebritis—irregular rim enhancing lesion with hypodense center, better defined
than early cerebritis
* Early capsule—well-defined rim enhancing mass; an outer hypodense and inner
hyperdense rim (double rim sign) classically
* Late capsule—rim enhancing lesion with thickened capsule and diminished hypodense
central cavity
CNS tuberculosis Can be leptomeningeal, pachymeningeal, or intracranial (tuberculomas)
* Leptomeningeal—intense heterogenous basal enhancement.
* Pachymeningeal—intense enhancement of thickened meninges.
* Intracranial: T1 isointense with central hyperintensity (possible caseation), T2 isointense
with central hyperintensity (possible gliosis), ring-enhancing.
Herpes simplex Preferential involvement of mesial temporal lobes. If hemorrhagic, blooming on GRE/SWI
encephalitis
Japanese encephalitis Predominant involvement of deep gray matter, especially bilateral thalami (though may be
asymmetric) with sparing of cortices
Rabies encephalitis In classic cases, increased T2 signal in affected parts with predilection for the gray matter
especially basal ganglia, thalami, hypothalami, brainstem, limbic system, and spinal cord
as well as the frontal and parietal lobes
Neurocysticercosis Findings are based on location (parenchymal or in sub-arachnoid-intraventricular space;
which appears grape-like (racemose) cystic, often associated with ventriculitis) or stage of
infection (as below).
* Vesicular—T1 hyperintense scolex ±, Cyst with dot sign (parasitic cyst with eccentric
scolex), no or faint enhancement
* Colloidal vesicular—T1 hyperintense with surrounding edema, scolex with eccentric
focus of enhancement
* Granular nodular—Edema decreases, cyst retracts, enhancement persists but less marked
from prior stage
* Nodular calcified—end-stage quiescent, no edema, no enhancement
Toxoplasmosis Multiple lesions, T1 isointense or hypointense and T2 isointense to hyperintense, with
predilection for basal ganglia and corticomedullary junction that often show ring or nodular
enhancement. Increased lipid lactate peak on MRS is characteristic.
Schistosomiasis Rare, seen either as intracerebral and intracerebellar hematomas or nonspecific
granulomatous lesions (hypodense on CT and T1-hypointense and T2-isointense on MRI)
due to a response of the host to the ova
Toxocariasis Multiple cortical, sub-cortical, or white matter lesions that are hypodense on CT,
hyperintense on T2-weighted MRI images, and homogeneously enhancing
Cryptococcosis Leptomeningeal involvement, “soap bubble” appearance with pseudocysts—common in
mid brain and basal ganglia with parenchymal cryptococcomas. Also, tendency to spread
along peri-vascular spaces with dilated perivascular spaces

(Fig. 21.20). 18F FDG-PET is particularly helpful transporters (GLUTs). Most low-grade tumors
in identifying subtle FCDs. FDG is useful for have lower concentrations of GLUTs and can
tumor grading because most high-grade tumors, usually be distinguished from high-grade glio-
such as high-grade gliomas, medulloblastoma, mas by the lower FDG uptake on PET [4].
and primary central nervous system lymphoma, FDG-PET is a useful tool in distinguishing
have high concentrations and activity of glucose post-radiation necrosis from tumor progression,
286 A. Singh et al.

Fig. 21.17 Serial coronal FLAIR images of the left progressive volume loss and denudation of the overlying
temporal lobe in an adult patient with autoimmune cortex (arrow). Similar focal lobar volume loss can
encephalitis. Between 2010 and 2015, there was happen in Rasmussen encephalitis

Table 21.4 Neuroradiological findings in common phacomatoses


Phacomatoses Neuroradiological findings
NF1 Optic glioma, peripheral or cranial nerve sheath tumors, macrocephaly, sphenoid wing dysplasia,
(peripheral) scoliosis, astrocytomas
NF2 (Central) Vestibular schwannomas, meningiomas, schwannomas/neurilemmomas of the dorsal roots of the
spinal cord
TS Cortical tubers (calcifications, cysts, or fibrosis may be seen in tubers), subependymal nodules
(SENs), SENs near the foramen may enhance on Gd and transform into a SEGA ! serial MRIs
may show progressive growth, papilledema, and obstructive hydrocephalus; white matter
heterotopias, arachnoid cyst, aneurysms
SWS Leptomingeal angiomatoses, calcification of gyri in form of rail-road shape, cerebral atrophy,
venous, or arterial infarcts

both in high-grade gliomas and brain metastases. tracer injections (typically Tc-99 m) are
In general, recurrent tumor is FDG avid, and administered during an ictal event that allows
radiation necrosis is not FDG avid. computation of variations in rCBF between ictal
and inter-ictal states. Studies by multiple groups
have validated SISCOM as a valuable tool that
Subtraction Ictal SPECT offers improved localization of seizure focus by
Co-registered to MRI (SISCOM) visualization of the region of hyperperfusion
and thus higher neuronal activity (40–86%).
SISCOM is a novel neuroimaging technique Further, SISCOM findings serve as a guide for
that couples MR images with nuclear medicine further intracranial monitoring, electrode place-
SPECT (Single-Photon Emission Computed ment, and in determining the extent of surgical
Tomography) scans to identify areas of resection, which in turn offers a higher proba-
increased perfusion with respect to regional bility of post-surgical seizure freedom
cerebral flow (rCBF). It is commonly used as a [6–8]. Occasionally, incongruous findings have
tool in pre-surgical evaluation to localize the been reported between intracranial vEEG and
seizure focus in both pediatric and adult patients SISCOM in post-surgical patients [5]. These
with refractory epilepsy [5]. Technically, radio likely reflect an altered blood flow pattern in
21 Neuroimaging in Epilepsy 287

Fig. 21.18 Axial FLAIR (a, b) demonstrates focal T2 tubers in the posterior basal left temporal lobe were
hyperintense masses in the cortex and subcortical white responsible for the majority of seizures in this patient with
matter of the basal left temporal lobe and left inferior tuberous sclerosis. Axial CT slice (c) demonstrating
parietal lobule consistent with tubers. Axial pre-contrast cortical tram-track calcification throughout most of the
T1 and coronal T2-weighted MRI demonstrate multiple left cerebral hemisphere consistent with pial angiomato-
ependymal nodules along the bilateral caudo-thalamic sis. There is a volume loss, but with relative sparing of the
grooves. These lesions demonstrate subtle T1 hyperin- left frontal operculum. Findings are consistent with Sturge
tensity (arrow) attributed to calcification that can be seen Weber syndrome
on CT (not shown). Video EEG suggested the adjacent

Table 21.5 Radiographic findings in patients with TBI


Pathology Comments
Gliding contusions Due to sagittal angular acceleration with stretching and tearing of parasagittal veins
Hemorrhages Scalp hematoma, subdural, epidural, subarachnoid, intraparenchymal, ventricular with or
without mass effect
Diffuse axonal injury Areas affected are parasagittal regions of the frontal lobes, periventricular areas in temporal
(DAI) lobes, near internal and external capsules, gray–white matter junctions, dorsolateral
quadrants of the rostral brainstem, and cerebellum. GRE and SWI show small regions of
susceptibility artifacts

these patients rather than the actual seizure


onset zone. However, SISCOM still exists as a Magnetoencephalogram (MEG)
valuable tool in pre-surgical planning in both
identification of the ictal zone and guiding A separate chapter is devoted to MEG but is
surgical resection. For the clinician, though, it is mentioned briefly here. MEG measures small
essential to remember that the complete workup electrical currents arising inside the neurons of
must be tailored to each individual patient. the brain, which produce very weak magnetic
288 A. Singh et al.

Fig. 21.19 Axial FLAIR


(a) and coronal 3D
T1-weighted MRI
(b) demonstrating subtle
encephalomalacia in the
anterior right superior
temporal gyrus (arrows)
attributed to remote trauma
in a patient with seizures
and history of high-speed
motor vehicle accident

Table 21.6 Common ligands used during PET scans


Types of PET Techniques used
FDG Glucose metabolism
H2O Cerebral blood flow
11
[ C] Carfentanil Binds to mu-opiate receptors
[11C] Doxepine Binds to histamine H1 receptors
a [11C] methyl-L-Tryptophan Measures tryptophan metabolism by serotonin and kynurenine pathways
[11C] Flumazenil Measures tryptophan metabolism by serotonin and kynurenine pathways

Fig. 21.20 Simultaneous PET-MRI technology in a lateral left temporal lobe (arrow, panel b). Blended
temporal lobe epilepsy patient. Coronal FLAIR demon- images combining MRI and FDG data can also be
strates volume loss and hyperintensity in the left obtained to assist visual detection of abnormalities (here,
hippocampal head (arrow, panel a). Coronal FDG demon- FLAIR with color FDG map superimposed)
strates hypometabolism that involves both medial and

fields in the range of femto and pica Tesla. Superconducting Quantum Interference Devices
Patient wears a helmet containing an array of (SQUIDS). MEG has a high resolution in both
100+ sensitive magnetic field measurement space (2–3 mm) and time (<1 ms). The skull and
devices. The measurement devices are called the tissue surrounding the brain affect the
21 Neuroimaging in Epilepsy 289

Fig. 21.21 Advanced imaging workup in patient with functional MRI of the patient performing word generation
MRI-negative left temporal lobe epilepsy. task demonstrates left language dominance (b). A re-
Coronal MPRAGE with interictal MEG dipole cluster in gion-of-interest of BOLD signal in the planum temporale
the left middle temporal gyrus (blue arrow, panel a) was (presumably Wernicke’s area) demonstrated a strong
concordant with semiology and EEG. Task-based correlation with performance of the task (arrow, panel c)

magnetic fields measured by MEG much less diffusion coefficient (ADC) maps calculated from
than they affect the electrical impulses measured the diffusion-weighted images, the intensity of
by electroencephalogram. MEG is usually per- pixels is more directly proportional to extent dif-
formed with simultaneous EEG. For Magnet fusion, and the T2 weighting is removed from the
Source Imaging (MSI), information from MEG data. A bright signal on DWI and dark signal on
and MRI is coregistered to form magnetic source ADC supports the evidence of cytotoxic edema
localization images that provide detailed struc- seen with infarctions (Table 21.7). A focus of
tural–unctional information of the brain. MSI increased signal intensity on DWI with a normal
helps in characterization and localization of ADC signal suggests vasogenic edema and refer-
epileptiform activity and pre-operative mapping red to as “T2 shine through.”
of brain areas supporting sensory, motor, and
language function (Fig. 21.21). MSI is compli-
mentary to PET, fMRI, and EEG as it provides Diffusion Tensor Imaging (DTI)
unique information on the spatiotemporal
dynamics of brain activity. DTI is based on the basic principle that the dif-
fusion of water molecules in the brain is
restricted by intracellular and extracellular
Diffusion-Weighted Imaging (DWI) membranes, particularly myelin. The image
intensities are inversely related to the relative
DWI adds spatially encoding magnetic gradients mobility of water molecules in tissue and the
to the standard MRI sequence to make the image direction of motion. Anisotropy is a measure of
sensitive to the translational motion of water the orientation dependent water diffusion within
molecules. In the absence of T2 weighting, the an image voxel, e.g., CSF has a fractional ani-
“bright regions” represent decreased water diffu- sotropy of near zero, whereas highly coherent
sion, and “dark regions” represent increased water white matter has a fractional anisotropy near 1.
diffusion. Hence an acute stroke with cytotoxic The diffusion data can also be given color codes
edema and slow water diffusion will appear very based on principal direction of diffusion, and the
bright. However, a T2-bright tissue, such as edema intensity of color is proportional to the fractional
or gliosis from a late subacute or chronic infarct, anisotropy. The transverse axis (x-axis) is repre-
also can appear bright on a diffusion trace image sented by red color; the green color depicts
(this is called “T2 shine through”). In apparent anterior posterior (y axis), and blue is designated
290 A. Singh et al.

Table 21.7 DWI and Pathology DWI ADC Cause


ADC characteristics in
different disorders Stroke High Low Cytotoxic edema
Solid tumor Variable Variable Depends on the
cellularity
Arachnoid cyst Low High CSF signal intensity
Herpes encephalitis High Low Cytotoxic edema
Abscess High in the Low Dense pus
center
Acute diffuse axonal High Low White matter shearing
injury

to superior inferior (z-axis). DTI tractography tissue of interest. Though both are based on
then uses fractional anisotropy and the principal similar principles, MRS uses signal from protons
orientation of diffusion with the voxel to char- to estimate the concentration of metabolites,
acterize the coherent white matter bundle 3D chiefly N-acetyl aspartate (NAA), choline (Cho),
orientations with the brain and spinal cord. Thus, creatine (Cr), and lactate in brain tissue. 1H
tractography allows accurate diagnosis of even spectroscopy, most widely used, is useful for
subtle congenital and acquired lesions that might assessment of markers of neuronal loss such as
disrupt the axonal organization. NAA, products of anaerobic metabolism such as
Practical applications of DTI and tractography lactic acid, and direct measures of primary
in epilepsy include precise delineation of white excitatory and inhibitory neurotransmitters. 31P
matter tracts in the brain, especially in identifi- spectroscopy is directed toward the characteri-
cation of eloquent white matter tracts, such as the zation of the bio energetic status of the tissue of
arcuate fasciculus adjacent to neoplasms or interest phosphocreatine (PCr), adenosine
epileptogenic regions [9, 10]. Another area where triphosphate (ATP), and phosphoinositol (Pi).
tractography has significant implications is in Applications of MRS in epilepsy imaging are
mapping of optic radiations during pre-surgical widespread. In patients with hippocampal scle-
planning of anterior temporal lobe resection pro- rosis, the MRS shows evidence of neuronal dys-
cedures. Here, pre-operative tractography can function such as decreased NAA and decreased
demonstrate the anterior extent of Meyer’s loop, NAA/Cho and NAA/Cr ratios and decreased
which is variable between people and cannot be myoinositol (MI) in ipsilateral temporal lobe and
visualized on traditional MRI studies. Thus, one increased lipid and lactate soon after a seizure
can predict the extent of visual field defects that [13]. Conventionally, MRS has been used in
might happen post-surgery. DTI has relatively characterization and differentiation of masses that
poor spatial resolution and is less sensitive to appear equivocal on MRI. MRS can help differ-
injury at crossing fibers and close to gray–white entiate between dysplastic versus neoplastic
matter junction [11, 12]. masses, recurrent brain neoplasm versus radiation
injury, or between an abscess versus a tumor [14].
Further, MRS has also been used to screen for
MR Spectroscopy (MRS) inborn errors of metabolism such as Canavan’s
disease and creatine deficiency. There is a typi-
In the field of epilepsy, MRS acts as a valuable cally decreased NAA/Cr ratio in patients with
tool by complementing MRI. While conventional dysplastic cortex as in hemimegalencephaly.
MRI is very helpful in studying anatomy, MRS Interestingly, multiple studies have now validated
offers a noninvasive means to determine the MRS as a tool in identifying the seizure focus,
biochemical and metabolic characteristics of the thus making it useful in the evaluation of both
21 Neuroimaging in Epilepsy 291

focal and generalized epilepsy. Work by numer- with newly diagnosed brain tumors. Report of the
ous groups has shown specific metabolic abnor- Quality Standards Subcommittee of the American
Academy of Neurology. Neurology. 2000;54
malities that are confined to the seizure zone [15, (10):1886–93.
16]. Inter-ictal changes include increased inor- 2. Moots PL, Maciunas RJ, Eisert DR, Parker RA,
ganic phosphate (Pi), increased pH, and decreased Laporte K, Abou-Khalil B. The course of seizure
phosphomonoesters, a decreased PCr/Pi ratio disorders in patients with malignant gliomas. Arch
Neurol. 1995;52(7):717–24.
together with a decrease in NAA (reduction of 3. Annegers JF, Hauser WA, Coan SP, Rocca WA.
22% ipsilateral to seizure focus). Also, an increase A population-based study of seizures after traumatic
in lactic acid is usually seen post-ictally. brain injuries. N Engl J Med. 1998;338(1):20–4.
While MRS has been reported to have localizing 4. Horky LL, Treves ST. PET and SPECT in brain
tumors and epilepsy. Neurosurg Clin N Am 2011;22
value by numerous groups (65–96% chances of (2):169–84.
lateralization in TLE by proton MRS and 65–75% 5. So EL. Integration of EEG, MRI, and SPECT in
value in TLE by phosphorus MRS), research is localizing the seizure focus for epilepsy surgery.
still ongoing to determine the value of MRS in Epilepsia. 2000;41(Suppl 3):S48–54.
6. Bianchin MM, Wichert-Ana L, Velasco TR,
localization of the epileptogenic focus. Martins AP, Sakamoto AC. Imaging epilepsy with
SISCOM. Nat Rev Neurol. 2011;7(4):1–2.
7. Ahnlide JA, Rosén I, Lindén-Mickelsson Tech P,
Functional MRI (F-MRI) Källén K. Does SISCOM contribute to favorable
seizure outcome after epilepsy surgery? Epilepsia.
2007;48(3):579–88.
Functional MRI is a noninvasive imaging tech- 8. Van Paesschen W. Ictal SPECT. Epilepsia. 2004;45
nique that has grown in popularity over the past (Suppl 4):35–40.
two decades. Its role has been increasingly rec- 9. Yogarajah M, Duncan JS. Diffusion-based magnetic
ognized in clinical practice to lateralize language resonance imaging and tractography in epilepsy.
Epilepsia. 2008;49(2):189–200.
and motor functions. The intracarotid sodium 10. Luat AF, Chugani HT. Molecular and diffusion
amobarbital angiographic procedure (the tensor imaging of epileptic networks. Epilepsia.
“Wada”) and intraoperative cortical stimulation 2008;49(Suppl 3):15–22.
mapping procedures are still the clinical gold 11. Duncan JS. Imaging the Brain’s highways—diffusion
tensor imaging in epilepsy. Epilepsy Currents.
standards to localize the epileptogenic zone and 2008;8(4):85–9.
map the functional areas of the brain. However, 12. Gross DW. Diffusion tensor imaging in temporal
these mapping techniques have their own limita- lobe epilepsy. Epilepsia. 2011;52(Suppl 4):32–4.
tions due to afterdischarges and seizures produced 13. Caruso PA, Johnson J, Thibert R, Rapalino O,
Rincon S, Ratai EM. The use of magnetic resonance
by stimulation. The spatial resolution of fMRI is spectroscopy in the evaluation of epilepsy. Neu-
great, but the temporal resolution is suboptimal roimaging Clin N Am. 2013;23(3):407–24.
for dissecting out the different functional areas of 14. Kuzniecky R. Clinical applications of MR spec-
the brain that are related to a particular task. troscopy in epilepsy. Neuroimaging Clin N Am.
2004;14(3):507–16.
15. Laxer KD. Clinical applications of magnetic reso-
nance spectroscopy. Epilepsia. 1997;38(Suppl 4):
References S13–7.
16. Garcia PA, Laxer KD, Ng T. Application of spec-
troscopic imaging in epilepsy. Magn Reson Imaging.
1. Glantz MJ, Cole BF, Forsyth PA, et al. Practice 1995;13(8):1181–5.
parameter: anticonvulsant prophylaxis in patients
Neuropsychological Evaluation
in Epilepsy 22
Madison M. Berl and Leigh Sepeta

– Identify strengths to inform treatment


Neuropsychology Principles
planning
• Profile provides evidence for the localization
Neuropsychology is the study of learning and
of dysfunction/function
behavior in relationship to the brain. It is a
• Measure the cognitive or behavioral
framework that draws from neurology, neu-
impact/risk of rehabilitation, pharmacologi-
roanatomy, cognitive sciences, and clinical,
cal, surgical, or therapeutic interventions
social, developmental, and biological psychol-
– Establish the baseline of functioning for
ogy. It is critical that with pediatric evaluation,
systematic comparisons across time
the brain is understood in the context of devel-
• Increase patient preparedness and inform
opmental change. A neuropsychologist has
items selection/protocol adjustments on an
earned a PhD in clinical psychology and com-
individual basis for cognitive mapping pro-
pleted 2 years of specialized postdoctoral
cedures: Intracarotid Amytal Test (IAT)/
training.
WADA, functional MRI (fMRI), Electrocor-
tical Stimulation (ECS)
Purpose of a Neuropsychological • Help formulate appropriate treatment plans
Evaluation (educational/vocational and medical)
• Predict individual’s ability to achieve success
There are several appropriate reasons for referral in particular settings
to people with epilepsy to undergo neuropsy-
chological evaluation: Different from general psychological practice,
a neuropsychologist:
• Obtain profile of strengths and weaknesses
– Measure the presence and degree of • Does not necessarily diagnose psychiatric
behavioral and cognitive difficulties conditions or provide therapy/treatment
• May not assess specific vocational skills
(driving, interest inventory, etc.).

M.M. Berl (&)  L. Sepeta


Neuropsychology, Children’s National Health Components of a Neuropsychological
System, 111 Michigan Avenue NW, Washington,
DC 20010, USA Evaluation
e-mail: mberl@childrensnational.org
L. Sepeta Through history, testing, and behavioral obser-
e-mail: lsepeta@childrensnational.org vations, there is an examination of external

© Springer Science+Business Media LLC 2017 293


M.Z. Koubeissi and N.J. Azar (eds.), Epilepsy Board Review,
DOI 10.1007/978-1-4939-6774-2_22
294 M.M. Berl and L. Sepeta

behavior to make inferences about brain function • Although there are some findings that focal
and structure. The history is similar to other epilepsy is associated with focal deficits, this
providers where information is systematically is true for adults more than children likely due
gathered through interview with the patient and to the plasticity of children’s brains. There-
their caregivers/spouses, and record review. fore, children do not follow adult profiles.
Formal testing of abilities and behavioral obser- Moreover, even though focal epilepsy may
vations are conducted over the course of one or result in a deficit related to the location of the
more office visits or, less ideally, on an inpatient epilepsy, this is not the only deficit that the
basis. There are numerous neuropsychological person is contending with. For example, a
measures available. Table 22.1 is not an person with left temporal lob epilepsy may
exhaustive list; however, the measures listed are have verbal memory difficulties, but also has
commonly used both clinically and in research in inattention and slow processing speed.
epilepsy populations. Formal testing refers to a • Seizures (focal or generalized) may impact
standardized method of administration and scor- functioning across any or all domains.
ing of responses. A profile of strengths and • Cognitive difficulties may predate and/or
weaknesses is derived by comparing a person’s persist beyond onset, which may indicate
test scores to a normative population of a similar that cognitive difficulties may share a com-
age across domains. Domains of functioning mon underlying neuroanatomic substrate with
include the following: what is generating the seizures.
• Progressive deterioration of cognitive skills is
• General Cognitive Functioning/Intelligence observed in a minority of individuals. As
Quotient (IQ) such, a plateau and/or regression of skills is a
• Language strong impetus for surgery, in particular for
• Memory and Learning hemispherectomy, but any other resection as
• Attention well. Please see below within specific domain
• Executive-Regulatory Function areas for further discussion.
• Visual/Spatial/Nonverbal processing
• Motor There are multiple factors that combine to
• Academic Achievement determine neuropsychological outcome:
• Adaptive Functioning
• Social/Emotional – Age of onset
• Personality – Seizure type
– Underlying pathology
– Neuronal discharges (ictal and interictal)
– Episodes of status
Neuropsychological Findings – Antiseizure medications (ASMs)
by Domain in Epilepsy – Psychosocial
– Public attitudes/stigma
Overview – Individual attitude (e.g., self-worth; depression)

It is long recognized that persons with epilepsy The challenge has been to unravel this com-
have greater incidence of cognitive and psychi- plex picture due to a lack of quality studies and
atric comorbidities (Gowers 1881). Consistent challenges (e.g., suitable controls, complex
with the heterogeneity of seizure disorders, but interactions; sensitivity of neuropsychological
even often within a single type of epilepsy, no test; small sample sizes/lots of tests; validity).
single cognitive profile exists for epilepsy. Other With that caveat, some general cognitive out-
general considerations are as follows: comes include the following:
22 Neuropsychological Evaluation in Epilepsy 295

Table 22.1 Neuropsychological measures by domain


Domain Common measures—but not exhaustive list
Skill All (most) ages
Adult version Pediatric version
Intellectual functioning
Wechsler Abbreviated Scale of Intelligence-Second Edition (WASI-II)
Wechsler Adult Intelligence Scale-Fourth Wechsler Intelligence Scale for
Edition (WAIS-IV) Children-Fourth Edition (WISC-IV)
Language
Verbal fluency Delis–Kaplan Executive Function System (DKEFS)—Verbal Fluency; Controlled Oral Word
Association Test (COWA or COWAT); FAS
NEPSY-Second Edition (NEPSY-II): Word
Generation
Naming Boston Naming Test (BNT); Expressive One-Word Picture Vocabulary Test-Fourth Edition
(EOWPVT-4)
Receptive Peabody Picture Vocabulary Test-Fourth Edition (PPVT-IV); Receptive One-Word
vocabulary Picture Vocabulary Test-Fourth Edition (ROWPVT-4)
Comprehension Clinical Evaluation of Language Fundamentals-Fourth Edition (CELF-IV): Concepts and
Following Directions; Token Test
NEPSY-II: Comprehension of Instructions
subtest
Phonological Comprehensive Test of Phonological Comprehensive Test of Phonological
skills Processing-Second Edition (CTOPP-2) (to age Processing-Second Edition (CTOPP-2)
24)
Memory
Visual Rey–Osterrieth complex figure test—immediate, delayed, recognition trials
Wechsler Memory Scale-Fourth Edition Children’s Memory Scale (CMS): Dot
(WMS-IV): Visual Reproduction, Design Locations, Faces; NEPSY-II:Memory for
Memory; Brief Visual Spatial Memory Test— Designs
Revised (BVMT-R)
Verbal Wide Range Assessment of Memory and Learning-Second Edition (WRAML-2):Story Memory;
Rey Auditory Verbal Learning Test (RAVLT)
WMS-IV: Logical Memory, Verbal Paired CMS: Stories, Word Pairs; California Verbal
Associates; California Verbal Learning Learning Test-Children’s Version (CVLT-C)
Test-Second Edition (CVLT-II); Hopkins
Verbal Learning Test–Revised (HVLT-R)
Attention and executive functioning
Attention Test of Variables of Attention (TOVA); Conners’ Continuous Performance Tests-Second Edition
(CPT-II);
Test of Everyday Attention (TEA) Test of Everyday Attention for Children
(TEA-Ch)
Working Digit Span Backwards (Wechsler measures); Auditory Consonant Trigrams (ACT); WRAML-2
memory Finger Windows, Sentence Memory
Set-shifting Trail Making (Trail Making Test (TMT)) Parts A and B; DKEFS: Trail Making); 20 Questions;
Wisconsin Card Sorting Test (WCST)
Inhibition Stroop Color Word Test (Kaplan); DKEFS Color Word Interference
Planning Tower of London-DX-Second Edition; DKEFS Tower Test; Rey–Osterrieth Complex Figure Test
(continued)
296 M.M. Berl and L. Sepeta

Table 22.1 (continued)


Domain Common measures—but not exhaustive list
Skill All (most) ages
Adult version Pediatric version
Overall Behavior Rating Inventory of Executive Function (BRIEF)
Visual-spatial skills/visual-motor integration
Beery-Buktenica Developmental Test of Visual-Motor Integration-Sixth Edition (VMI):
Visual-Motor Integration, Visual Perception; Hooper Test of Visual Organization (VOT);
Judgment of Line Orientation Test (JLO); Rey–Osterrieth Complex Figure Test—Copy Trial
Fine motor
Grooved Pegboard Test or Purdue Pegboard Test; Beery-Buktenica Developmental Test of
Visual-Motor Integration-Sixth Edition (VMI): Motor Coordination subtest
Academic achievement
Woodcock Johnson III Tests of Achievement (WJ-III); Wechsler Individual Achievement Test–
Third Edition (WIAT–III); The Wide Range Achievement Test 4 (WRAT4)
Social/Emotional-Personality-Adaptive (Rating scales)
Vineland Adaptive Behavior Scales, Second Edition (VABS-II); Adaptive Behavior Assessment
System-Second Edition (ABAS); The Scales of Independent Behavior-Revised (SIB-R)
Achenbach Adult Self-Report (ASR); Beck Achenbach Child Behavior Checklist
Depression Inventory-Second Edition (CBCL)/Youth Self-Report (YSR)/Teacher
(BDI-II); Beck Anxiety Inventory (BAI); Report Form (TRF); Children’s Depression
ADHD Rating Scale-IV Inventory-Second Edition (CDI-2); Revised
Children’s Manifest Anxiety Scale-Second
Edition (RCMAS-2)
The Minnesota Multiphasic Personality The Minnesota Multiphasic Personality
Inventory-2 (MMPI-2); NEO Personality Inventory-Adolescent (MMPI-A)
Inventory-Revised (NEO PI-R)

– Generalized seizures are worse than focal, Nonetheless, even taking this cohort—excluding
and tonic–clonic seizures are worse than the one-third with IQ < 70)—there is a down-
absence ward shift of IQ with a Mean IQ (≈ 90) which
– Earlier onset is associated with more falls in the low average range compared to the
difficulties normal population where Mean IQ = 100.
– Interictal subclinical discharges are associated Risk factors for ID in epilepsy:
with transient cognitive impairment (TCI).
This has mixed evidence, but fewer studies. • Primary generalized epilepsy, West syndrome,
Lennox–Gastaut syndrome, localization-
related epilepsy, but seizure focus difficult to
Intelligence isolate
• Severe volumetric abnormalities
Approximately one-third of people with epilepsy • Early onset of epilepsy
have IQ scores below 70, which falls in the • Frequent seizures, more episodes of status
Intellectually Deficient (ID—formally MR) epilepticus
range. The majority (≈ 2/3) of people with epi- • Polytherapy
lepsy have average range (or higher) intelligence. • Comorbid diagnoses (e.g., autism)
22 Neuropsychological Evaluation in Epilepsy 297

As such, IQ is considered a proxy for out- associated with verbal memory problems.
come, disease severity, and extent of underlying Similarly, right TLE is associated (but not as
pathology. strongly) with visual memory problems. This
finding is the basis of the utility of presurgical
evaluation by providing evidence for location
Language of seizure focus and determining the risk of
postsurgical cognitive deficits. List learning
Along with memory, language is probably one of measures tend to be the most predictive of
the most studied domains in people with epilepsy hippocampal dysfunction.
given that focal epilepsy is most often in the • Unlike for other areas of functioning, there is
temporal, followed by frontal lobes. evidence for progressive loss with continued
seizures which is consistent with the changes
• Language representation: There is a higher seen on MRI.
incidence (25–30%) of atypical dominance • One hypothesis of why memory deficits are
(right or bilateral) than the normal not specific to TLE is that memory perfor-
right-handed population (5%). mance may also be disrupted due to other
– Atypical language dominance is more skills such as poor organization or attention
likely with large, early in development that rely on frontal lobe functions.
insults (e.g., perinatal stroke), with earlier
onset and with left-handedness.
– If language remains ipsilateral to focus,
a left hemispheric focus may have Attention/Executive Functions
impact on language functions (speeded
naming). Beyond the effects of IQ, attention problems are
• Appropriate simple, single-word knowledge, commonly observed.
untimed language skills.
• Adults with TLE frequently have • Prevalence of ADHD is 20–40%.
word-finding problems, which is found by • ADHD Inattentive Presentation is more
confrontation naming tasks (e.g., Boston common, and the boys and girls are equally
Naming Test), which may be related to the represented, which is different from devel-
hippocampal role in word retrieval. opmental ADHD with no seizures.
• Progressive language impairment is associ- • There may be higher rates of attention prob-
ated with Rasmussen’s encephalitis and Lan- lems with FLE and CAE.
dau–Kleffner syndrome, both of which have a • Associated issues such as nocturnal seizures
period of normal language development. or medication side effects may be the primary
cause of inattention.
• Myth that stimulants used for ADHD symp-
Memory toms would lower seizure threshold; however,
many studies have shown this to be untrue.
Similar to language findings, memory difficulties
are commonly associated with TLE. Executive functioning (EF), a set of skills that
is necessary for efficient and goal-directed
• The presence of mesial temporal pathology behavior is less well studied, but is a common
and degree of hippocampal atrophy is asso- difficulty. Aspects of EF that are shown to be
ciated with greater impairment. impaired in people with epilepsy include shifting,
• “Material specificity” of memory problems is cognitive flexibility, working memory, and
true more so in adults such that left TLE is organization.
298 M.M. Berl and L. Sepeta

• Parent questionnaire of EF was a significant attendance, there may be a larger gap following
predictor of performance and helpful in seizure onset.
identifying an “at-risk” group of children with
new-onset epilepsy.
Psychosocial

There are increased rates of mood disorders


Visual/Spatial (anxiety/depression) with a lifetime prevalence
risk of 35%.
Findings are less consistent for visuospatial skills
such as object recognition, drawing objects, and • Limbic/temporal seizures have greater risk,
visual closure. While some studies have found which may be an evidence of shared
these to be lower in right-hemispheric seizure neurophysiology
foci, others have noted that language dominance • Less clear evidence of increased rates of
may be an important factor. If language function aggression or psychosis
has reorganized to the right related to a left • Evidence for both environmental causes
hemispheric focus, a deficit in visuospatial pro- (stigma, missed school/work,
cessing may develop because the transfer of unpredictability/lifestyle changes) and shared
language to the right hemisphere is displacing neurophysiology (higher rates than other
visuospatial function to preserve language. This medical disorders)
is referred to as the “crowding hypothesis.”
Quality of life is markedly lower in people
with epilepsy and is not necessarily improved
Psychomotor/Reaction Time with seizure control. About 50% of persons with
epilepsy feel stigmatized.
Slow processing speed is a common finding in
people with epilepsy and may be due to neu-
roanatomic anomalies or treatment effects. Mapping Cognition

• Processing speed deficits are the most com- Mapping cognition is a role that may be under-
mon side effect of ASMs. Slower speed is taken primarily by either the neuropsychologist
associated with polytherapy (defined by load or neurologist or as a shared venture.
or toxicity as well as number) and type of
ASM (topiramate; phenobarbital; • Purpose is that the information is needed to
GABA-ergic inhibition). avoid morbidity of surgical procedure.
• Seizure type has been implicated, particularly • Techniques used to map language, memory,
FLE and benign rolandic epilepsy. and motor functions are changing with
available technology.
• Prior gold standard method was to pharma-
cologically inactivate ipsilateral anterior and
Academic Functioning middle cerebral arteries for several minutes.
This procedure is referred to many ways,
Poor academic achievement is associated with all including the intracarotid amobarbital test
epilepsy types. Outcome is moderated by psy- (IAT)/Wada or etomidate speech and memory
chosocial variables. As with other cognitive test (eSAM), and has no standard protocol.
skills, problems may predate seizure onset; The aims are to 1) lateralize function (lan-
however, should seizure control disrupts school guage and memory) and 2) demonstrate the
22 Neuropsychological Evaluation in Epilepsy 299

capacity of contralateral hemisphere to sus- standardized protocol exists; however, in


tain function. general, language responses must be brief
– Use of IAT/Wada is on the decline due to enough to occur during the stimulation time
drug availability and clinical validity of frame, so they may be limited to single
fMRI words. The patient must also be able to
– Consists of presenting language and answer questions immediately (without long
memory items during a brief window (1– pauses) at baseline to ensure accurate inter-
2 min) of drug effect; eSAM protocol pretation of pauses in responding related to
allows for continuous infusion which is a stimulation.
distinct advantage but not widely used
– Based on each hemisphere and in context
of baseline functioning, count errors or
aphasia to get an asymmetric index of References/Recommendations for
functioning for language and memory Further Reading
– Disadvantages are that it is invasive,
site-specific method, and feasibility relies 1. Baxendale S, Heaney D, Thompson PJ, Duncan JS.
on institution Cognitive consequences of childhood-onset temporal
– Advantage is that it is still the best lobe epilepsy across the adult lifespan. Neurology.
2010;75(8):705–11.
established method for memory function- 2. Gowers WR. Epilepsy and other chronic convulsive
ing (at this point) and is probably more disorders. 1st ed. London: J & A Churchill; 1881.
widely used in adult centers for that 3. Hermann BP, Jones JE, Sheth R, Koehn M, Becker T,
reason Fine J, Allen CA, Seidenberg M. Growing up with
epilepsy: a two-year investigation of cognitive devel-
• Functional MRI (fMRI) involves having the opment in children with new onset epilepsy. Epilepsia.
patient do language or motor tasks while in an 2008;49(11):1847–58.
MRI. The blood oxygen level-dependent 4. Hermann B, Seidenberg M, Jones J. The neurobe-
(BOLD) signal is extracted and analyzed. havioural comorbidities of epilepsy: can a natural
history be developed? Lancet Neurol. 2008;7(2):151–
There is increasing availability of fMRI 60.
packages on standard clinical systems. There 5. Jones-Gotman M, Smith M, Risse G, Westerveld M,
is an advantage of localization and lateral- Swanson SJ, Giovagnoli RA, et al. The contribution of
ization of function noninvasively; however, neuropsychology to diagnostic assessment in epilepsy.
Epilepsy Behav. 2010;18:3–12.
mapping of memory functioning is still 6. Loring DW, Meador KJ. Cognitive side effects of
fraught with practical and technical chal- antiepileptic drugs in children. Neurology. 2004;62
lenges. Disadvantage is that movement may (6):872–7.
render a study uninterpretable; however, 7. Seidenberg M, Pulsipher DT, Hermann B. Cognitive
Progression in Epilepsy. Neuropsychol Rev. 2007;17
technological advances may combat this in (4):445–54.
the near future. 8. Vingerhoets G. Cognitive effects of seizures. Seizure.
• Electrocortical stimulation (ECS) mapping is 2006;15(4):221–6.
either intraoperative or bedside mapping 9. Westerveld M. Neuropsychology of childhood epi-
lepsy. In: Yeates KO, Ris D, Taylor HG, Penning-
(grids) of function for motor or language ton B, editors. Pediatric neuropsychology. 2nd ed.
functions. Grids for seizure localization pur- New York: Guilford Press; 2009. p. 71–91.
poses are discussed elsewhere. Again no
Magnetoencephalography
and Magnetic Source Modeling 23
Manoj Raghavan

comparison, the electrical activity of the heart


Introduction
generates magnetic fields that are greater by
many orders of magnitude. The ambient elec-
The generators of neuromagnetic signals are
tromagnetic noise in an urban environment is
essentially the same as those of EEG signals.
even greater. The detection of weak magnetic
Summation of synchronous postsynaptic poten-
fields generated by the brain, therefore, requires
tials occurs to a greater degree when there are
not only highly sensitive instruments, but also a
regular arrays of similarly oriented cells, as for
magnetically quiet environment that is usually
instance in the pyramidal cell layer in the cortex.
provided by a magnetically shielded room
Magnetic fields generated by electrical currents
(MSR).
in the cortex are oriented perpendicular to the
direction of neuronal currents. This, together
with the folded geometry of the cortex, results in
some differences in the cortical surfaces that
MEG History
contribute to EEG and magnetoencephalography
The key enabling technology that allows the
(MEG) signals. MEG sensors are sensitive to
recording of very weak magnetic fields is the
magnetic fields that are orthogonal to the head
superconducting quantum interference device
surface. This corresponds to electrical fields that
(SQUID). This device is based on a quantum
are parallel to the scalp surface, such as those
phenomenon called the Josephson effect which
generated by cortical surfaces in the sulcal banks.
describes the current flow through a very thin
EEG, on the other hand, is preferentially sensi-
insulator that separates two superconductors.
tive to radially oriented electrical fields generated
Before the era of SQUIDs, David Cohen at MIT
at the crests of gyri.
had demonstrated in 1968 that it is possible to
Magnetic fields generated by the brain are of
record cortically generated magnetic fields in a
the order of 100 femtotesla (*10−13 T). For
magnetically shielded environment. The
recordings used coils wound around ferrite
cores and employed signal averaging based on
simultaneous recorded EEG signals. The earliest
commercially available SQUIDs were used by
David Cohen and others at MIT to record the
first magnetocardiogram—a signal that is sev-
M. Raghavan (&) eral orders of magnitude larger than magnetic
Department of Neurology, Medical College of fields generated by the brain. By 1971, the first
Wisconsin, 9200 W. Wisconsin Avenue, Froedtert
West, Milwaukee, WI 53226, USA MEG records of the alpha rhythm were
e-mail: mraghavan@mcw.edu demonstrated [1].

© Springer Science+Business Media LLC 2017 301


M.Z. Koubeissi and N.J. Azar (eds.), Epilepsy Board Review,
DOI 10.1007/978-1-4939-6774-2_23
302 M. Raghavan

MEG Equipment and Recording modeling of evoked responses to simple stimuli


(visual, auditory, or somatosensory). For the
From early devices with a single magnetometer, lateralization of language functions, a language
MEG recording technology has evolved over the task such as a word listening task or word
years to multichannel systems with several hun- reading task may be employed. Localization of
dred sensors. motor areas requires the patient to perform sim-
MEG recording systems are housed in a ple motor tasks such as tapping a finger. For
magnetically shielded room, which isolates the evoked responses to be sufficiently well defined
recording system from ambient magnetic inter- and stand out above the resting background
ference from various sources in the environment. oscillations, many trials of the task (typi-
In order to achieve sufficient attenuation of cally >100) are usually repeated. The responses
ambient magnetic interference, the walls of the recorded at each sensor are averaged across trials
MSR may have several layers of different types in order to obtain satisfactory signal-to-noise
of metal that attenuate magnetic interference in ratios prior to modeling the sources of these
different frequency bands. In addition to passive evoked responses.
shielding, some MSRs may also have active coils
in the walls which generate their own magnetic
fields to cancel ambient fields. Magnetic Source Modeling
Magnetic fields generated by the brain are
picked up by flux transformers which are In tandem with the development of the recording
inductively coupled to the SQUIDs. The flux hardware, the rapid evolution of computing
transformers can be configured as magnetometers technology has made it possible to take the
or gradiometers. Magnetometers—a simple recorded activity from the MEG sensors and
example being a conducting loop—produce model the cortical generators of the activity. This
output currents with magnitudes that are deter- step is referred to as magnetic source modeling or
mined by the magnetic flux through the magnetic source imaging (MSI).
loop. Gradiometers, on the other hand, are con- The objective of magnetic source modeling
figured by coupling two conducting loops either is to account for the topography of the mag-
side by side (in the same plane), or along an axis, netic fields measured at a given point in time
in such a fashion that the net output is propor- in the MEG sensors using a hypothetical gen-
tional to the difference in magnetic fluxes erator within the brain. The problem of deter-
through the two loops. These planar or axial mining brain sources from a set of
gradiometers detect magnetic field gradients measurements at the sensors is an example of
rather than absolute magnetic flux at a location. an inverse problem. In this case, the inverse
In typical MEG recording systems, an array of problem is highly underdetermined; i.e., there
flux transformers are arranged in the shape of a are far too many unknown variables and not
helmet at the bottom of a container called a de- enough constraints for there to be a unique
war. The dewar also houses the SQUIDs and is solution to the problem. Such inverse problems
filled with liquid helium to maintain the tem- are often referred to as “ill-posed” inverse
peratures low enough to permit superconductiv- problems. There are an infinite number of
ity. The outputs of these sensors are amplified, configurations of model sources within the
then digitized, and recorded using digital brain which could all produce the same
recording systems. observed sensor level recordings. In order to
In addition to localizing spontaneous epileptic make this problem tractable, we first need to
activity, MEG studies are often performed to model how magnetic fields associated with any
localize functional cortices. Localization of pri- given electrical generator within the head
mary sensory areas is performed by source propagate to the sensors. This is called the
23 Magnetoencephalography and Magnetic Source Modeling 303

forward model. The forward model requires an Source Modeling of Epileptic Activity
anatomical model of the head and the struc- and Evoked Responses
tures from which the electrical activity arises.
This is referred to as the head model. Once a When modeling the sources of epileptic activity,
forward model is defined, it is possible to the recorded MEG data are first reviewed visu-
generate many hypotheses about possible gen- ally by an experienced electroencephalographer
erators of the observed sensor measurements to identify epileptic spike events or ictal activity
and identify the hypothesis that best explains which can then be subjected to source modeling.
the measurements. Different source modeling Epileptic spikes seen in MEG may not always
techniques differ in the nature of the forward be seen in the simultaneous EEG recording, and
modeling and the types of generators likewise, not all EEG spikes are represented in
permitted. MEG. Since MEG sees the magnetic component
Of the various source modeling methods that of an electrical event in the cortex, it is in theory
have been developed over the years, equivalent more sensitive to electrical currents that are tan-
current dipole (ECD) modeling has found wide gential to the head surface—as for instance from
use in clinical applications. ECD modeling the banks of sulci. EEG, on the other hand, is
assumes that the electrical generators of activity more sensitive to radial sources, such as those
measured at MEG sensors are point dipoles: a generated at the crests of gyri. In most instances,
source and sink (positive and negative ends) however, epileptic spikes have generators that
separated by an infinitesimally small distance. are several square centimeters in area and have
Although real generators of electrical activity in both radial and tangential components. However,
the brain are not point sources, ECD modeling the spike may lead in one or the other modality
has proven to be clinically useful in localizing the depending on whether the tangential or radial
sources of epileptic spike activity and evoked sources dominate at the onset of the spike.
potentials. Dipolar models are defined by their Once epileptic spikes are identified in the
locations (x-, y-, and z-coordinates in a frame of MEG sensors, dipolar models are typically
reference to which the patients’ head model has employed to localize the sources in clinical
been co-registered) and orientation (defined by 2 applications. Figure 23.1 shows an example of
parameters). Additional “goodness-of-fit” interictal epileptic spike events whose sources
parameters quantify how well a model dipole localize to the right medial temporal regions. Due
accounts for the observed neuromagnetic fields. to the relatively short duration of MEG record-
Several alternative techniques for source ings compared to long-term EEG monitoring
modeling currently exist, for instance techniques studies, ictal events during MEG are uncommon.
that model the generators as a distributed field of However, when ictal activity is recorded, early
point dipolar sources. These distributed source ictal rhythms that precede any head movements
modeling approaches have predominantly been can be subjected to source modeling to localize
used in research applications thus far. Source seizure onset zones. An example of dipolar
modeling methods can also be applied to the source modeling applied to ictal rhythms is
electrical signals recorded by EEG. However, shown in Fig. 23.2.
because magnetic fields are not affected by CSF, There is ample evidence that magnetic source
meninges, skull and scalp, or skull breaches, the modeling of evoked responses can reliably
head modeling requirements for magnetic source localize primary sensory cortices (visual, audi-
modeling are much simpler. This translates into tory, and somatosensory). Figure 23.3 shows an
higher spatial resolution for an equivalent num- example of source modeling of somatosensory
ber of recording locations around the head for evoked response to median nerve stimulation
magnetic source modeling compared to electrical using dipolar modeling and dSPM [3]. Localiza-
source modeling [2]. tion of primary motor cortex using dipolar
304 M. Raghavan

Fig. 23.1 Dipolar sources of epileptic spikes. The panels topographic representation of the event is shown in the
above show an example of dipolar source models of top right panel, along with dipolar sources of a collection
epileptic spike activity. The panel on the far left shows of such events on axial and coronal planes through the
MEG traces from a subset of magnetometers with the dipole cluster in the bottom right panel
cursor marking an epileptic spike event. A sensor level

modeling of motor preparation potentials is, 87% of patients [6]. Using the same methods,
however, less reliable [4]. Alternative methods to Doss et al. [7] found language representation in
localize changes in beta band oscillatory activity the hemisphere to be treated with a concordance
in the motor cortex have been explored with rate of 86% with the Wada test in 35 patients,
greater success [5], although yet to be widely with a sensitivity of 80% and specificity of
adopted. 100%. Several smaller studies have reported
For lateralizing language, neuromagnetic MEG–Wada concordance rates between 69 and
responses to auditory language stimuli have been 100% using a variety of paradigms and analysis
found to be concordant with the Wada test in methods.
23 Magnetoencephalography and Magnetic Source Modeling 305

Fig. 23.2 Dipolar sources of ictal sharp rhythms. The onset as recorded in a subset of MEG sensors. Dipolar
panels above show an example of ictal source modeling sources of successive peaks of the ictal waveform are
using MEG. The traces on the left panel show seizure shown on planar views in the panels on the right

planning for epilepsy, tumors, or vascular


The Role of MEG in Presurgical lesions, and can also be used to determine
Evaluations hemispheric language dominance.
The spatial accuracy of MEG and magnetic
Unlike EEG, MEG is not indicated for the initial source modeling for localizing “irritative zones” is
evaluation of new-onset seizures but can provide second only to invasive EEG [8]. MEG-guided
valuable localization of epileptic pathology in review of MRI data has also been reported to
patients with medically refractory epilepsy who identify subtle abnormalities that were previously
are undergoing evaluations for epilepsy surgery. missed, especially focal cortical dysplasia [9–11].
MEG primarily localizes interictal epileptic However, MEG should not be viewed as a tool that
abnormalities which help identify “irritative replaces invasive EEG or other noninvasive tests
zones” in the brain. Source modeling of interictal such as PET or SPECT. There is now sufficient
spikes using MEG may be particularly useful in evidence that MEG can provide significant
patients with normal MR imaging, large or cystic non-duplicative information to improve surgical
lesions, lesions of indeterminate significance to outcomes or preempt expensive invasive intracra-
the patient’s epilepsy, or with multifocal or nial EEG studies [12–15]. While MEG may not
rapidly propagated spikes. eliminate the need for intracranial EEG studies, it
MEG is also clinically indicated for localizing can help generate better hypotheses about seizure
primary motor or sensory cortices (somatosen- onset zones, and thereby guide electrode placement
sory, visual, or auditory) to guide surgical for invasive EEG studies [16, 17].
306 M. Raghavan

Fig. 23.3 Dipolar and distributed source models (dSPM) A distributed source model for the same point in time is
for somatosensory evoked responses. The left upper panel shown in the bottom right panel (dSPM with a threshold
shows the evoked responses to somatosensory stimulation of p < 0.001). Both the dipole model and maxima of the
of the right median nerve in a subset of MEG sensors in dSPM activation localize to the post-central gyrus in an
the left central region. Dipolar sources at the peak of the area consistent with anatomically predicted hand
response are shown in the top right and bottom left panels. somatosensory representation

high-frequency oscillations are therefore of


Some Limitations of Current interest.
Clinical MEG Methodologies While most clinical applications of MEG
employ dipolar source modeling techniques,
Localizing “irritative zones” is often insufficient dipolar models are unsuitable for studying net-
to predict seizure onset zones, especially when work phenomena such as functional connectivity,
they are multifocal. Unfortunately, ictal MEG causal interactions, or network dynamics which
studies are not the norm since it is impractical to may have relevance to localizing and modeling
monitor patients in a MEG scanner for an epileptic networks.
extended period of time in order to capture sei- For localizing function, although source model-
zures. In about 20% of MEG studies, no epileptic ing of evoked responses provides good localization
spikes may be observed during the recording. In of primary sensory cortices, these techniques remain
these cases, MEG is unable to provide useful to be validated for mapping language networks in
localizing information about epileptic pathology. the anterior temporal or frontal neocortices to guide
Alternative interictal biomarkers of epilepsy such surgical resection boundaries.
as focal slow waves or pathological
23 Magnetoencephalography and Magnetic Source Modeling 307

Summary magnetoencephalography data. J Neurosurg.


2008;109(2):228–37.
6. Papanicolaou AC, et al. Magnetocephalography: a
MEG and magnetic source modeling provide a noninvasive alternative to the Wada procedure.
noninvasive technique for localizing spontaneous J Neurosurg. 2004;100(5):867–76.
and evoked brain activity with high spatiotem- 7. Doss RC, et al. Lateralizing language with magnetic
poral resolution. Single equivalent current source imaging: validation based on the Wada test.
Epilepsia. 2009;50(10):2242–8.
dipoles remain the most widely used source 8. Wheless JW, et al. A comparison of magnetoen-
modeling method in clinical applications, cephalography, MRI, and V-EEG in patients evalu-
although imaging methods are increasingly being ated for epilepsy surgery. Epilepsia. 1999;40(7):931–
explored. MEG and source modeling of epileptic 41.
9. Funke ME, et al. The role of magnetoencephalogra-
activity can provide significant non-redundant phy in “nonlesional” epilepsy. Epilepsia. 2011;52
information to help improve outcomes of epi- (Suppl 4):10–4.
lepsy surgeries or preempt expensive invasive 10. Moore KR, et al. Magnetoencephalographically
EEG studies. MEG also provides an alternative directed review of high-spatial-resolution
surface-coil MR images improves lesion detection
to fMRI for noninvasively localizing eloquent in patients with extratemporal epilepsy. Radiology.
cortices for neurosurgical planning. 2002;225(3):880–7.
11. Wilenius J, et al. Interictal MEG reveals focal cortical
dysplasias: special focus on patients with no visible
MRI lesions. Epilepsy Res. 2013;105(3):337–48.
References 12. Knake S, et al. The value of multichannel MEG and
EEG in the presurgical evaluation of 70 epilepsy
1. Cohen D. Magnetoencephalography: detection of the patients. Epilepsy Res. 2006;69(1):80–6.
brain’s electrical activity with a superconducting 13. Knowlton RC, et al. Functional imaging: II. Predic-
magnetometer. Science. 1972;175(4022):664–6. tion of epilepsy surgery outcome. Ann Neurol.
2. Leahy RM, et al. A study of dipole localization 2008;64(1):35–41.
accuracy for MEG and EEG using a human skull 14. Knowlton RC, et al. Functional imaging: I. Relative
phantom. Electroencephalogr Clin Neurophysiol. predictive value of intracranial electroencephalogra-
1998;107(2):159–73. phy. Ann Neurol. 2008;64(1):25–34.
3. Dale AM, et al. Dynamic statistical parametric 15. Paulini A, et al. Lobar localization information in
mapping: combining fMRI and MEG for epilepsy patients: MEG—a useful tool in routine
high-resolution imaging of cortical activity. Neuron. presurgical diagnosis. Epilepsy Res. 2007;76(2–
2000;26(1):55–67. 3):124–30.
4. Lin PT, Berger MS, Nagarajan SS. Motor field 16. Knowlton RC, et al. Effect of epilepsy magnetic
sensitivity for preoperative localization of motor source imaging on intracranial electrode placement.
cortex. J Neurosurg. 2006;105(4):588–94. Ann Neurol. 2009;65(6):716–23.
5. Nagarajan S, et al. Preoperative localization of hand 17. Sutherling WW, et al. Influence of magnetic source
motor cortex by adaptive spatial filtering of imaging for planning intracranial EEG in epilepsy.
Neurology. 2008;71(13):990–6.
Vagus Nerve Stimulation and Other
Neuromodulation 24
Gholam K. Motamedi

History of VNS Therapy in epilepsy


Approved Indications of VNS
1985 First animal studies
Therapy
1988 First human implant
1992 First randomized active control study (E03) The VNS has been approved by the food and
completed
drug administration (FDA) for:
1994 European community approval
1996 Second randomized active control study 1. Adjunctive therapy in patients  12 years
(E05) completed
with refractory (drug-resistant) partial onset
1997 US Food and Drug Administration epilepsy, and
commercial approval in patients
 12 years with refractory partial epilepsy
2. Adjunctive therapy in patients  18 years
with chronic or recurrent major depressive
2005 US Food and Drug Administration
commercial approval in patients episodes refractory to adequate response to
 18 years with chronic major depression  4 adequate antidepressants.
refractory to adequate treatment with  4
antidepressants
Feb. 50,000 + implants worldwide for both
2009 epilepsy and depression (currently Refractory Epilepsy—Definition
>65,000)
Wheless JW, Wyllie’s Treatment of Epilepsy: Principles
More than 50% of patients with epilepsy have
and Practice, 2011
http://www.accessdata.fda.gov/cdrh_docs/pdf/ partial epilepsy. The AED success rate in patients
p970003s050a.pdf with partial epilepsy is about 50% compared to
more than 80% rate in primary generalized epi-
lepsy [1–3]. The current definition of refractory
epilepsy requires failure of adequate trials of two
appropriate and tolerated AEDs at maximum
possible doses, whether as monotherapy or in
combination, for enough time (a follow-up per-
iod 3 times the longest inter-seizure interval or
1 year, whichever longer) [4]. However, only
about 20% of these patients will be eligible for
G.K. Motamedi (&)
Department of Neurology, Georgetown University
Hospital, 3800 Reservoir Rd., NW, Washington, DC
20007, USA
e-mail: motamedi@georgetown.edu

© Springer Science+Business Media LLC 2017 309


M.Z. Koubeissi and N.J. Azar (eds.), Epilepsy Board Review,
DOI 10.1007/978-1-4939-6774-2_24
310 G.K. Motamedi

resective brain surgery. Therefore, in these and/or asystole. In most patients, the main vagus
patients, VNS Therapy is a viable palliative nerve is the largest of the three nerves
treatment option. (Figure 24.1).

Clinical Indications of VNS Electrode Polarity and Pulse Stimulus

Patients with documented refractory partial epi- A bipolar lead transmits stimulation from the
lepsy who are not eligible for brain surgery, e.g., generator to the left vagus nerve. The lead consists
having multifocal epilepsy, unclear seizure focus, of a pin that connects to the generator on one end,
overlapping eloquent cortex, or those who are and the helices that contain the stimulation
opposed to are considered potential candidates electrodes and anchor tether on the other end.
for the VNS Therapy. VNS may have a limited
role in patients with previous unsuccessful
resective epilepsy surgery. In a recent series, Initial Clinical Trials—Overview
18.75% of such patients had  50% reduction in
seizure frequency with one case of worsening Adjunctive use of left VNS Therapy in patients
seizures, but it may be an option given its with refractory epilepsy was tried in five
potential antipsychotic and mood-stabilizing acute-phase landmark clinical studies in 45 cen-
effects [5]. ters (40 US, 1 Canada, 4 EU). A total of 454
patients were implanted with VNS with a total
patient exposure of 901 device-years. Individual
Vagus Nerve Anatomy and Lead mean patient exposure was 24 months (8 days–
Placement 7.4 years).
Eligible patients were implanted (baseline
The lead electrodes must be placed below where period 12 weeks) and the generator was activated
the superior and inferior cervical cardiac bran- 2 weeks later. In the two randomized, blinded,
ches separate from the vagus nerve. Stimulation active control trials (E03 and E05), patients were
of either of these two branches during the system randomized to: (1) HIGH group (higher fre-
diagnostics (lead test) may cause bradycardia quency, pulse width, higher duty cycle) and

Fig. 24.1 Cyberonics.com, Implantation procedure, 2010


24 Vagus Nerve Stimulation and Other Neuromodulation 311

(2) LOW group (active control) and were fol- in degenerative nerve damage. The ON time
lowed for a 14-week treatment period (E05: [6]). OFF time can be induced by continuous or very
frequent magnet activation (> 8 h) and therefore
should be avoided.
Clinical Trials: Efficacy and Safety

The HIGH group showed significant seizure Suggested Initial Dosing Settings
frequency reduction compared with the baseline
and the LOW group (24.5% vs. 6.1%, p = 0.01). The VNS is activated  2 weeks after implan-
In the HIGH group, 31% experienced  50% tation and when the healing process is com-
seizure reduction as opposed to 13% in the LOW pleted. The initial recommended parameters are
group (p = 0.02). The most common adverse as follows:
events were voice alteration and dyspnea. The
treatment was well tolerated and 97% patients – Output current of 0.25 mA,
(306 of 314) continued into the long-term – ON time 30 s/OFF time 5 min,
follow-up phase of the study (Fig. 24.2). – Signal frequencies of 20–30 Hz, and
– Pulse width 250–500 µs.

Stimulation Parameters and Safety It is recommended to keep all AEDs stable for
the first 3 months of VNS before any changes are
VNS Therapy is based on: attempted. The current intensity can be increased
by 0.25 mA every 2–4 weeks, to reach a mini-
– output current, mum of 1 mA over a 6–8 week period, or as
– signal frequency, tolerated. It is also recommended to give the
– pulse width, and patient enough time to adapt before leaving the
– ON/OFF time. office, and before the next increment.

Therefore, each parameter can be pro-


grammed in a variety of combinations to achieve Adverse Effects
optimal stimulation setting. However, based on
animal studies stimulation at high frequency The most common side effects associated with
(  50 Hz) + ON time  OFF time may result VNS Therapy include:

Fig. 24.2 Left median seizure reduction (%) of patients 2 years of follow-up (intent-to-treat analysis). Right
participating in E03 (n = 114) and E05 (n = 105) ran- proportion of patients with  50% seizure reduction in
domized placebo-controlled trials, and all VNS studies the same groups (Ben-Menachem, Lancet Neurol. 2002)
combined (E01–E05, N = 440) at 3 months, 1 year, and
312 G.K. Motamedi

– Hoarseness: up to 60% and may indicate population. Therefore, the recorded rates of
device malfunction, nerve constriction (ap- SUDEP did not seem to have been increased
parent within a few days), nerve fatigue with significantly by VNS Therapy.
intense stimulation parameters (turn off for – Manipulation of pulse generator and lead by
several days until hoarseness subsides), and patients through the skin (Twiddler’s syn-
persistent hoarseness not associated with drome): may damage or disconnect the lead
stimulation suggests nerve irritation (requires from the generator (Cyberonics.com, Physi-
immediate investigation). cian’s manual).
– Dysphagia and aspiration: there is higher risk
with preexisting swallowing difficulties.
– Dyspnea: higher risk with underlying COPD
or asthma. Precautions
– Obstructive sleep apnea (OSA): higher risk of
apneic events during the stimulation with Cardiac evaluation: in case of family history,
OSA (lower stimulus frequency of 2 Hz or past medical history, or EKG indications of
longer “OFF” time recommended). Since new dysfunctional cardiac conduction systems (reen-
onset cases of OSA have been reported, prior try pathway).
evaluation in high-risk patients should be Serum electrolytes: Mg2+, Ca2+ levels to be
considered. checked before implantation.
– Nerve damage with device malfunction: may Postoperative bradycardia: can occur in
cause painful stimulation (tape magnet over patients with cardiac arrhythmias; consider
the generator to stop stimulation if suspect a postimplant EKG and Holter monitoring.
malfunction; evaluate for possible surgical Bradycardia (<40 bpm) and/or asystole:
intervention). may occur during intraoperative system diag-
– Laryngeal irritation: more common in nostics (lead test). In such patients cardiac
smokers. monitoring at the time of device activation is
– Lead break: may prevent patients from recommended.
receiving efficient therapy. If diagnostics
suggest a fracture, turn the pulse to 0 mA
output current (to prevent possible dissolution Optimizing Parameters
of the conductor material hence pain, and Alleviating Side Effects
inflammation, and vocal cord dysfunction).
– Trauma to the vagus nerve: can occur during Optimizing Results
surgery and can result in permanent vagal
nerve dysfunction. This can be achieved by increasing output cur-
– Sudden unexplained death in epilepsy rent and/or modifying ON/OFF times (duty
(SUDEP): through August 1996, 10 cycle).
(definite/probable and possible) cases were
recorded among 1000 VNS patients (2017
patient-years of exposure) indicating an inci- Managing Side Effects
dence rate of 5/1000 patient-years. However,
estimates for non-VNS epilepsy patients The following steps may be taken as needed to
range from 1.3 to 3.5 in the epilepsy popu- alleviate side effects: decreasing signal frequency
lation and 9.3 in surgical candidate (from 30 to 20 Hz) or decreasing output current
24 Vagus Nerve Stimulation and Other Neuromodulation 313

(by 0.25 mA). If decreasing the output current closed-loop responsive neurostimulation
does not achieve tolerability, lowering the pulse (RNS) of intracranial structures [8]. The DBS
width (from 500 to 250 ls) may be considered. (thalamic stimulation) has not yet been approved
by the FDA, but it has been recently approved in
Europe [3]. The RNS system (NeuroPace device)
VNS Warning with MRI has been FDA approved in late 2014. There are
also early reports of potential benefits of stimu-
The VNS is MRI compatible including 1.5T and lating other extracranial sites (e.g., trigeminal
3T scanners. Head and extremity scans are nerve) ([9–11]).
allowed using a transmit and receive type of RF
coil. Before patient enters into the MR system
room, both output current and magnet current Responsive Cortical Stimulation
should be set to 0 mA since MRI-induced mag-
netic field may cause magnet-mode activation and Besides VNS, the other FDA-approved neu-
stimulation. After the MRI is done, reprogram- rostimulation intervention is the responsive neu-
ming is needed to restore the setting parameters. rostimulator (RNS), NeuroPace®. The idea of
MRI should not be performed on patients with RNS is based on studies that showed short trains
lead breaks, see product labeling for all condi- of electrical stimulation can stop afterdischarges
tions. Diathermy (shortwave, microwave, thera- [12, 13]. Eligible candidates are patients with
peutic—not diagnostic—ultrasound) should not focal epilepsy with a well-defined but
be used on VNS Therapy patients. non-resectable epileptogenic zone such as elo-
quent cortex or bilateral mesial temporal foci.
A multicenter, double-blinded, randomized trial
Mechanism(s) of Action of VNS involving 191 patients with partial epilepsy was
conducted (  3 seizures/month, 1 or 2 seizure
The precise mechanism of action of VNS foci). Responsive neurostimulator detecting
remains unknown. In animal models (maximum abnormal EEG connected to depth or subdural
electroshock, PTZ, alumina gel, strychnine, kin- leads was placed at 1 or 2 seizure foci. The pulse
dling), VNS prevented seizures or seizure spread generator connects with the electrodes and is
(except for the alumina gel model). A series of placed in the skull. One month after implanta-
facts have been considered to play a role in its tion, the subjects were randomized to receive or
function, e.g., VNS affects heart and respiratory not receive (sham) stimulation. After a 12-week
rates, vagus-initiated activity in the brain has blinded phase, all patients received unblinded
been localized through use of fos1 immunoreac- stimulation for 84 weeks. By the end of the
tivity, regional brain glucose metabolism (in blinded phase, stimulated group had a 37.9%
animals), and via PET imaging in human. The seizure reduction compared to the sham group at
newest version of VNS labeled Aspire adds the 17.3% (p = 0.012) [8].
advantage of cardiac rhythm detection of At 5 months postimplantation, 41.5% seizure
seizures. reduction was seen in the stimulation group
compared to a 9.4% reduction in the sham
group. The seizure reduction continued to
Investigational Neurostimulators improve during a subsequent open-label phase
where both arms received stimulation. A 50%
In recent years, two pivotal trials of neurostim- responder rate (those who achieved 50% or more
ulation in humans with drug-resistant epilepsy reduction of seizure frequency) was seen in 55%
have been conducted: deep brain stimulation after two years. The median percentage seizure
(DBS) via chronic programmed bilateral stimu- reduction was seen in 44% (at 1 year), and 53%
lation of anterior thalamus (SANTE trial) [7] and (at 2 years) of patients. Intracranial hemorrhages
314 G.K. Motamedi

Fig. 24.3 Unadjusted


median declines at the end
of the blinded phase were
14.5% in the control group
versus 40.4% in the
stimulated group. By
2 years, there was 56%
reduction in seizure
frequency with  50%
reduction in 54% (14
seizure-free for at
 6 months). There was no
symptomatic hemorrhage
or brain infection [7]

and infections each occurred in about 2% of population-based studies from Rochester, Minnesota.
implanted patients, but neither mood nor cogni- Mayo Clin Proc. 1996;71(6):576–86.
2. Sillanpää M, Jalava M, Kaleva O, Shinnar S.
tive function worsened, and quality of life Long-term prognosis of seizures with onset in
improved. Similar to the findings in VNS Ther- childhood. N Engl J Med. 1998;11;338(24):1715–22.
apy, seizure reduction appeared to further 3. Bergey GK. Neurostimulation in the treatment of
improve over time (p < 0.0001) and the RNS epilepsy. Exp Neurol. 2013;244:87–95.
4. Kwan P, Arzimanoglou A, Berg AT, Brodie MJ,
was well tolerated with acceptable safety [14]. et al. Definition of drug resistant epilepsy: consensus
proposal by the ad hoc Task Force of the ILAE
Commission on Therapeutic Strategies. Epilepsia.
Electrical Stimulation of the Anterior 2010;51(6):1069–77.
5. Koutroumanidis M, Binnie CD, Hennessy MJ, et al.
Nucleus of the Thalamus (SANTE Trial) VNS in patients with previous unsuccessful resective
epilepsy surgery: antiepileptic and psychotropic
A multicenter, double-blind, randomized clinical effects. Acta Neurol Scand. 2003;107(2):117–21.
trial including 110 patients with partial epilepsy 6. DeGiorgio CM, Schachter SC, Handforth A, et al.
Prospective long-term study of vagus nerve stimula-
(baseline seizure frequency 19.5/month) was tion for the treatment of refractory seizures. Epilep-
conducted. After a 3-month blinded phase half of sia. 2000;41(9):1195–200.
the patients received stimulation and the other 7. Fisher R, Salanova V, Witt T, et al. Electrical
half received no stimulation. Then, all patients stimulation of the anterior nucleus of thalamus for
treatment of refractory epilepsy. Epilepsia. 2010;51
received unblinded stimulation. In the last month (5):899–908.
of the blinded phase, the stimulated group had a 8. Morrell MJ, RNS System in Epilepsy Study
29% greater seizure reduction compared with the Group. Responsive cortical stimulation for the treat-
control group (p = 0.002) [7] (Fig. 24.3). ment of medically intractable partial epilepsy. Neu-
rology. 2011;27;77(13):1295–304.
9. DeGiorgio CM, Murray D, Markovic D, White-
hurst T. Trigeminal nerve stimulation for epilepsy:
References long-term feasibility and efficacy. Neurology.
2009;10;72(10):936–8.
10. DeGiorgio CM, Soss J, Cook IA, et al. Randomized
1. Hauser WA, Annegers JF, Rocca WA. Descriptive controlled trial of trigeminal nerve stimulation for
epidemiology of epilepsy: contributions of drug-resistant epilepsy. Neurology. 2013;26;80
(9):786–91.
24 Vagus Nerve Stimulation and Other Neuromodulation 315

11. Pop J, Murray D, Markovic D, DeGiorgio CM. 13. Motamedi GK, Lesser RP, Miglioretti DL, et al.
Acute and long-term safety of external trigeminal Optimizing parameters for terminating cortical after-
nerve stimulation for drug-resistant epilepsy. Epi- discharges with pulse stimulation. Epilepsia. 2002;43
lepsy Behav. 2011;22(3):574–6. doi: 10.1016/j. (8):836–46.
yebeh.2011.06.024. 14. Heck CN, King-Stephens D, Massey AD, et al.
12. Fernández IS, Loddenkemper T. Electrocorticogra- Two-year seizure reduction in adults with medically
phy for seizure foci mapping in epilepsy surgery. intractable partial onset epilepsy treated with respon-
J Clin Neurophysiol. 2013;30(6):554–70. sive neurostimulation: final results of the RNS
System Pivotal trial. Epilepsia. 2014;55(3):432–41.
Epilepsy Surgery Assessment
and Testing 25
Gholam K. Motamedi

Refractory (Drug Resistant, Treatment Options for Refractory


Intractable, Pharmacoresistant) Partial Epilepsy
Epilepsy
Besides trying different combinations of ASMs, a
The latest definition by the international league variety of treatment options are available,
against epilepsy (ILAE) task force defines including established methods of surgical resec-
refractory epilepsy as failure of “adequate” trials tion of the seizure focus, or surgical
of two antiseizure medications (ASMs) either as “non-resection” options such as vagus nerve
monotherapy or as combination (poly-) therapy, stimulation (VNS), responsive neurostimulation
to control seizures [1]. In order to meet this cri- (RNS), or multiple subpial transections (MST).
terion, it is critical to make sure that the ASMs Currently there are also several investigational
have been given enough chance (“adequate surgical treatments available such as deep brain
trial”)—i.e., they must have been used at the stimulation (DBS, which has been approved as a
maximum tolerated dose (with no severe side treatment in Europe), transcranial magnetic
effects)—and given enough time (determine sei- stimulation (TMS), trigeminal nerve stimulation
zure reduction after a follow-up period 3 times (TGNS), external VNS, or transcranial direct
the longest interseizure interval, or one year, current stimulation (tDCS).
whichever longer). Therefore, with proper man- In particular circumstances, other non-surgical
agement, in most cases the diagnosis of refrac- methods, e.g., ketogenic diet in young children,
tory epilepsy should be possible to make within can be of therapeutic value. Currently, research
1–2 years of the start of the seizures. for newer ASMs as well as novel potential
therapies such as gene therapy, cell transplanta-
tion, or vaccination is underway.

Presurgical Evaluation

The purpose of presurgical evaluation is to char-


acterize the seizure type and to lateralize and
localize the seizure onset focus in patients with
refractory partial epilepsy. Therefore, patients
G.K. Motamedi (&)
Department of Neurology, Georgetown University should be admitted to a properly equipped epi-
Hospital, 3800 Reservoir Rd., NW, Washington, DC lepsy monitoring unit (EMU) for continuous
20007, USA video-EEG monitoring. The ASMs are usually
e-mail: motamedi@georgetown.edu

© Springer Science+Business Media LLC 2017 317


M.Z. Koubeissi and N.J. Azar (eds.), Epilepsy Board Review,
DOI 10.1007/978-1-4939-6774-2_25
318 G.K. Motamedi

tapered down in order to record electrographic function of each temporal lobe separately to
and clinical seizures for the above purposes. determine whether the nonepileptic side would
Patients also need appropriate neuroimaging be capable of handling memory function by itself
studies including high-resolution epilepsy proto- after the affected temporal lobe is surgically
col MRI and other available imaging technologies removed. The test also can assist with seizure
as indicted or available such as MR spectroscopy onset side since there is typically concordance
(MRS), positron emission tomography (PET), between the seizure onset side and poor memory
single-photon emission computed tomography function on that side (upon contralateral
(SPECT), or magneto-encephalography (MEG). injection).
In case the scalp recording is of limited value,
patients who are considered potential candidates
for surgical treatment may need invasive Case Presentation #1
recordings using strips, grids, or depth electrodes
and possibly cortical mapping before an appro- A 28-year-old woman with past medical history
priate surgical procedure could be planned. After of seizures since age 9 years presented with the
lateralization and localization of the seizure onset episodes of staring and left-hand clenching before
focus, potential surgical candidates should convulsion (focal dyscognitive seizures with
undergo a series of tests for further evaluation to secondary generalization). These events occur
define their final candidacy for an appropriate twice a week on average. She had been on mul-
surgical treatment including neuropsychological tiple ASMs in the past without much improve-
testing and functional MRI or intracarotid amo- ment. She has given up college and her previous
barbital procedure (IAP, Wada test). jobs since she could not concentrate, drive a car,
or work. Currently, she suffers from tremor, and
memory and concentration problems. Her serum
Neuropsychology analysis shows high levels of phenytoin and val-
proate. A routine EEG and high-resolution brain
The main questions that are addressed by a neu- MRI were obtained (Figs. 25.1 and 25.2).
ropsychological test include determining parts of Her presurgical evaluation indicated that she
the brain that are impaired. In particular, higher was a good candidate for a right anterior tem-
cognitive functions such as verbal and visual poral lobectomy (Fig. 25.3). During the presur-
memory and language are studied. The test also gical evaluation, she was started on
helps establish a baseline for future comparison. levetiracetam, and her phenytoin and valproate
This helps predict potential postsurgical deficits. were tapered off. Later, prior to her surgery, she
Studies have shown that the best predictor of responded better to combination therapy with
postoperative adequacy is the preoperative cog- levetiracetam and lamotrigine; i.e., her seizures
nitive and psychosocial status; i.e., the lower the were less frequent and her side effects improved
preoperative cognitive and psychosocial status, significantly. The patient became seizure free
the lower the risk for further decline [2, 3]. after a right anterior temporal lobectomy and was
kept on a lower dose of monotherapy with one of
her ASMs (follow-up >4 years).
Intracarotid Amobarbital Procedure
(Wada Test)
Is Epilepsy Surgery Warranted?
This test “imitates” the prospective temporal
lobectomy by temporarily inhibiting unilateral Randomized, controlled trials (RCT) to assess
brain functions using a drug. Therefore, it helps the efficacy and safety of epilepsy surgery were
lateralizing language dominance and memory missing till 2001. In the first such study [4], 80
function. The Wada test evaluates memory patients with temporal lobe epilepsy (TLE) were
25 Epilepsy Surgery Assessment and Testing 319

Fig. 25.1 Right anterior temporal spikes

randomly assigned to surgery (n = 40) or treat-


ment with ASMs for one year (n = 40). The
primary outcome was seizure freedom and sec-
ondary outcome included seizure frequency and
severity, quality of life (QOL), disability, and
death.
At 1 year, the cumulative proportion of
patients who were seizure free was 58% in the
surgical group versus 8% in the medical group
(P < 0.001). Patients in the surgical group had
fewer complex-partial seizures (CPS) and sig-
nificantly better quality of life (P < 0.001 for
both comparisons) than the patients in the med-
ical group. Four patients (10%) had adverse
effects of surgery (mainly the expected mild
language and memory-related problems such as
Fig. 25.2 High-resolution MRI detects mesial temporal word finding and short-term memory difficulties)
sclerosis (MTS) in 80–90% of cases while one patient in the medical group died. This
320 G.K. Motamedi

Fig. 25.3 Standard anterior temporal lobectomy [4]

study confirmed that in TLE, surgery is superior (1) continued ASM (n = 23), or (2) anterior
to prolonged medical therapy. This RCT also mesial temporal lobectomy (AMTR) plus ASM
showed that randomized trials of surgery for treatment (n = 15) and were followed for two
epilepsy are feasible and appear to yield precise years. The primary outcome was seizure freedom
estimates of treatment effects. during the second year of follow-up, and the
secondary outcome was health-related quality of
life (QOL), cognitive function, and social
Early Randomized Surgical Epilepsy adaptation.
Trial (ERSET) Seizure freedom during the second year of
follow-up was reported in 11 of 15 patients in the
It has been well established that years of active surgical group versus none of the 23 in the
epilepsy predict cognitive impairment in children medical group (P < 0.001). Also, improvement
and adolescents [5, 6]. Therefore, in order to of QOL was higher in the surgical group
investigate the effects of early surgery, i.e., (P = 0.01). Memory decline occurred in 4
whether it would be superior to continued med- patients (36%) after surgery. Adverse events
ical management, Engel et al., conducted a included one stroke in a surgical case versus 3
multicenter, parallel-group RCT soon after the cases of status epilepticus in the medical group. It
failure of 2 ASM trials in patients with temporal was concluded that resective surgery plus ASM
lobe epilepsy [7]. Thirty-eight patients (18 M/20 in patients with new refractory MTLE results in
F; age  12 years) with MTS and refractory lower probability of seizures during second year
MTLE who were within 2 consecutive years of of follow-up than continued ASM treatment
adequate trials of 2 ASMs were randomized to alone.
25 Epilepsy Surgery Assessment and Testing 321

Surgical Methods temporal gyrus while preserving the neocortical


area. Other approaches to selective amygdalo-
There are different surgical methods depending hippocampectomy include:
on the patient and seizure type and other char-
acteristics including: – Transsylvian approach [10, 11].
– Subtemporal approach [12].
Temporal lobe surgery – Other variants of the transcortical approach
Lobectomy [13].
Resection of the epileptogenic zone
Lesionectomy
Corpus Callosotomy
Hemispherectomy Outcome Following Temporal Lobe
Multiple subpial transections (MST) Surgery

Long-term Surgery—MTLE and MTS

Temporal lobe surgery Temporal lobectomy provides continued


long-term seizure control but risk of seizure
Temporal lobectomy recurrence  2 years after surgery is present. In
one report, 50 consecutive post-temporal lobec-
There are different methods to remove the seizure tomy patients with MTS (mean follow-up
focus in the temporal lobe. Anterior temporal 5.8 years, range 2–9.2) seizure-free rates were
lobectomy is the classic and most commonly 82% at 12 months, 76% at 24 months, and 64%
used type of surgery, but it may be done using at 63 months [14]. Complete, or better, seizure
different approaches including: outcome was associated with significantly better
long-term QOL, and risk factor for seizure
– Standard (en bloc) anterior temporal lobec- recurrence was the reduction in ASM intake—or
tomy (ATL) including 3–6 cm of anterior absorption—in 5 of 17 patients (29%), including
temporal neocortex and 1–3 cm of mesial 3 of 5 with a first seizure recurrence within
structures (amygdala and hippocampus) 24 months.
– Modified (Yale group) and limited neocorti-
cal resection (3.5 cm from temporal pole)
sparing superior temporal gyrus, to address Standard Anterior Temporal
language deficits Lobectomy
– Selective amygdalohippocampectomy
– Stereotactic radiosurgery [8, 9] In another study, 116 patients with MTS, MTLE,
and post-anterior temporal lobectomy with
amygdalohippocampectomy (ATL-AH) were
studied (follow-up period: 6.7 years) [15].
Selective Amygdalohippocampectomy Complete seizure freedom was seen in 103
(SAH) patients (89%) and Engel Class I or II outcome in
109 patients (94%). The highest concordance
This method was introduced by Niemeyer in (i.e., test consistent with the side of eventual
1958 in an attempt to preserve the lateral tem- surgery) was seen with video-EEG (100%), PET
poral cortex out of concern for language deficits. (100%), MRI (99.0%), and Wada test (90.4%).
The technique includes accessing temporal horn The lowest concordance was seen with SPECT
to selectively resecting mesial temporal struc- (84.6%) and neuropsychological testing (82.5%).
tures through a small incision in the middle A strong Wada memory lateralization appeared
322 G.K. Motamedi

to be the predictor of excellent long-term seizure but no memory decline in the right SAH group
control, while less disparity in the memory score was seen while some even showed improvement
between the sides was the predictor of persistent [20].
seizures.

Neurocognitive Deficits and Risk


Temporal Lobectomy—Inferior Factors Following ATL
Temporal Approach
Cognitive impairment is very common in
Inferior temporal gyrus approach to mesial tem- epilepsy patients and may be negatively or pos-
poral lobe resection is safe and effective with low itively affected by surgery. Larger temporal lobe
morbidity and mortality. One study reviewed 483 resections are associated with better seizure
patients with AMTL resection via inferior tem- control, but at the same time resecting more
poral gyrus approach for TLE [16]. Thirteen functional tissues carries higher risk of cognitive
complications (2.7%) (3 months post-op) were outcome [21].
reported including eight delayed SDH (1.6%), Comparison of the changes in cognitive per-
two superficial wound infections (0.4%), one formance in relation to the extent of resection of
delayed ICH (0.2%), one small lacunar stroke mesial and lateral temporal structures (1–2 cm
(0.2%), and one transient frontalis nerve palsy and >2 cm for mesial, and  4 or  4 cm for
(0.2%). There were no deaths or severe neuro- neocortical) in 47 right-handed patients with left
logical impairments. Complications were more temporal lobectomy for MTLE showed no dif-
common among older patients. ference in cognitive outcome between the
groups. However, there was a negative correla-
tion with patient age at seizure onset [22].
Selective Amygdalohippocampectomy

SAH in TLE patients with MTS results in Standard Versus Selective Temporal
seizure-free outcomes comparable to procedures Lobe Surgery
with more extensive temporal neocortical resec-
tions [17]. Although this method was introduced A meta-analysis of standard anterior temporal
to minimize the neurocognitive side effects of lobectomy (ATL) versus selective SAH for sei-
temporal lobectomy, interestingly, at this point zure control in TLE included 11 studies (1203
there is more controversy regarding postopera- patients) and concluded that ATL is more likely
tive neuropsychological outcomes, rather than to achieve an Engel Class I outcome compared
seizure-free outcome, when compared to stan- with SAH (p < 0.01). Standard ATL confers
dard ATL. Some studies have suggested that better chance of achieving freedom from dis-
SAH results in better cognitive function com- abling seizures in patients with TLE [23].
pared to ATL [10], while others have shown no
evidence of a clear neurocognitive benefit and in
fact SAH might cause significant verbal memory Right Versus Left Temporal Lobectomy
deficits in dominant temporal lobe resection [18, (RTL vs. LTL)
19].
In another study, 76 adult patients with SAH Comparison of neuropsychological outcome
for MTLE via the trans-middle temporal gyrus following RTL versus LTL shows postoperative
approach reported 92% Engel Class I or II with decline in verbal memory after LTL, perfor-
very low surgical morbidity and no mortality. mance intelligence decline after LTL (depending
Postoperative neuropsychological testing showed on infero-lateral and basal region removal), and
verbal memory decline in the left SAH group, visuospatial memory outcome after RTL
25 Epilepsy Surgery Assessment and Testing 323

(depending on basal and hippocampal region foci while preserving the language and sensory
removal). More resection is associated with cortex as depicted above. He had no language
worse functioning and vice versa [24]. deficits after the resection. He has been com-
pletely seizure free since the surgery while con-
tinuing only one of his ASMs at minimal dose
Case Presentation #2 (he had one breakthrough seizure 5 years later
after stopping his ASM but has remained seizure
A 43-year-old man developed seizures two years since resuming the ASM (follow-up since
after surviving a left temporal aneurysm rupture. surgery >6 years)).
Following the surgery and aneurysm resection,
he did well until the seizures started. Multiple
ASMs were tried but he continued having partial Case Presentation #3
seizures with secondary generalization about
twice a month. His brain MRI findings were A 35-year-old woman presented with seizures
consistent with his history of prior surgery and an since age 5 years. She described her seizures as
encephalomalacia involving the posterior tem- “day dreaming, head turning, lip smacking,
poral lobe. His video-EEG monitoring localized right-hand posturing and at times convulsions.”
his seizure onset focus to the left temporal area In the past, she had been on phenobarbital since
including the posterior regions. His Wada test childhood, as well as several other ASMs. Her
lateralized language to the left side and showed a current seizure frequency is about 3 times per
significantly better memory function on the right. week. She lives with her family, has never
He was admitted for subdural grid placement in worked, and does not drive. Her mother reports
order to accurately localize the epileptogenic behavioral problems (outbursts). After the first
zone and perform language mapping prior to a visit, while starting her presurgical evaluation,
prospective left temporal resective surgery her ASM was changed to levetiracetam that
(Figs. 25.4, 25.5, and 25.6). resulted in some improvement in seizure fre-
Cortical mapping defined the language cortex quency, but later she responded better to the
next to the seizure focus. The patient underwent a combination therapy of levetiracetam and lam-
left temporal lobectomy including the seizure otrigine (i.e., her seizures decreased to 1–2 per

Fig. 25.4 Subdural grid


and strip placement to
cover the lateral and
basal-medial areas of the
left temporal lobe
324 G.K. Motamedi

Fig. 25.5 Ictal discharges mainly limited to electrodes 23 and 31 (red), before spreading and secondary generalization;
patient aware

Fig. 25.6 Results of


cortical mapping
25 Epilepsy Surgery Assessment and Testing 325

Fig. 25.7 Buildup of rhythmic ictal discharges in few left mesial–basal temporal electrodes. The patient underwent a
standard left ATL and has been completely seizure free since the surgery with no new deficits (follow-up >5 years)

month). Her high-resolution brain MRI was (complex-partial): 237 (59%), GTCS: 119 (30%),
normal. A routine EEG showed left temporal focal without alteration of awareness
spikes and video-EEG monitoring revealed left (simple-partial seizures): 26 (6%), and mixed: 17
temporal ictal onset, but there were a few sei- (4%). Of these 372 (93%) had temporal lobe, and
zures with indeterminate localization. An inva- 27 (7%) extra-temporal lobe resections. The
sive monitoring using subdural strip electrodes pathology showed MTS in 113 patients (28%),
was performed to confirm the localization gliosis in 237 (59%), and normal tissue in 49
(Fig. 25.7). Resective surgery was curative. (12%). The overall Engel Class I outcome is
given as follows:

Outcome Following Nonlesional – 81% at 6 months


Partial Epilepsy Surgery – 78% at 1 year
– 76% at 2 years
Surgical outcome following surgery in – 74% at 5 years
MRI-negative (nonlesional) patients with refrac- – 72% at 10 years
tory partial epilepsy can result in favorable out-
come. A review of 399 patients has shown Almost all seizures occurred during the first
positive long-term outcome after 0.5–15.7 year year after surgery. The positive predictive factor
(mean 6.2) follow-up period [25]. The seizure was seizure control during the first follow-up
types included focal dyscognitive seizures year. A Class I outcome at first year indicated
326 G.K. Motamedi

92% probability of seizure remission at 10 years. The negative predictors (risk factors) were
Negative risk factors included (1) extra-temporal (1) higher baseline seizure frequency and
seizure focus (p < 0.001), (2) previous surgery (2) preoperative generalized tonic–clonic sei-
(p < 0.001), (3) male gender (p = 0.035), and zures. Memory decline was reported with domi-
(4) normal tissue in pathology (p = 0.038). nant hippocampus resections [26].

Outcome Following ATL Case Presentation #4


in Nonlesional TLE Surgery
For the past 20 years, a 45-year-old man would
A normal MRI is not against surgery in patients wake up soon after falling asleep screaming and
with TLE. Sixty-four adult patients with refrac- flailing his arms and legs for 30 s before full
tory TLE but normal MRI who had underwent recovery. He is otherwise healthy. Currently he is
TLE surgery (1996–2009) were followed for 1– on 4 ASMs. He has been on most of the available
14.5 years (mean 4.1). Standard anterior tempo- ASMs, clonazepam, and SSRIs, for seizures,
ral lobectomy was done in 84% and an unre- sleep disorders, and “pseudoseizures,” respec-
markable pathology was reported in 45% of the tively. In the past, he had experienced a few
patients. Complete seizure freedom rates are similar episodes during the day as well. His EEGs
given as follows: and polysomnogram have all been normal in the
past. His current event frequency is 3–7 events per
– 1 year: 76% (Engel Class 1: 81%) night. He is unemployed, does not drive, and his
– 2 years: 66% (Engel Class 1: 76%) wife sleeps in a separate room (Figs. 25.8, 25.9,
– 7 years: 47% (Engel Class 1: 69%) 25.10, 25.11, 25.12, and 25.13).

Fig. 25.8 Left temporal interictal spike during sleep


25 Epilepsy Surgery Assessment and Testing 327

Fig. 25.9 Clinical seizure; arousal from sleep; movement artifact with no clear epileptiform discharges

Fig. 25.10 Postictal EEG; no clear epileptiform discharges or postictal suppression


328 G.K. Motamedi

Fig. 25.11 Bilateral subdural strips placed through burr holes for seizure onset focus lateralization (Phase 2a). From
left: frontal, left oblique, and left lateral views

Fig. 25.12 Left temporal rhythmic delta discharges recorded 10 s from seizure onset, indicating a far-field recording
from a remote focus (such as frontal lobe) that had spread to the temporal lobe

After lateralizing the seizure onset of the left yellow) indicating the epileptogenic zone located
hemisphere in Phase 2a through bilateral strips, on the edge of the fronto-polar electrode grid.
the patient underwent a second surgery by plac- Cortical mapping was performed on the most
ing more extensive grid electrodes to cover the anterior row of the superior posterior frontal grid
medial inter hemispheric, fronto-polar, superior and did not reveal any eloquent cortex.
posterior, and inferior posterior frontal lobes on Figure 25.14 shows the 3D reconstruction of
the left for accurate seizure localization as well as the seizure onset electrodes (red) and those to
cortical mapping as indicated. EEG recording which seizures rapidly spread (brown and yel-
(not shown) indicated the seizure onset elec- low). Since the seizure onset zone was located on
trodes shown in red along with electrodes to the edge of the grid, the resection included the
which the seizures rapidly spread (brown and colored electrodes in the top two rows of the grid
25 Epilepsy Surgery Assessment and Testing 329

Fig. 25.13 Location of the seizure onset zone and interictal discharges on the intracranial electrodes

as well as one row beyond that which was part of


the same gyrus, to minimize the possibility of
leaving part of the epileptogenic zone intact. The
patient has been completely seizure free since the
surgery with no deficits (follow-up >2 years).

Outcome Following Frontal Lobe


Epilepsy Surgery

Patients with frontal lobe epilepsy (FLE) and an


identifiable focal lesion are more likely to achieve
seizure freedom than those with poorly defined
seizure focus. A review and meta-analysis of 21
studies (total of 1199 patients) with FLE surgery
Fig. 25.14 3D reconstruction of the brain coregistered
with the postoperative CT to localize the electrodes in including studies with  10 patient and follow-up
relationship to gyral anatomy period  48 months showed seizure freedom
330 G.K. Motamedi

(Engel Class I outcome) in 45.1%. Significant indicated supplementary motor area (SMA)-type
predictors of long-term seizure freedom included seizures. Her high-resolution MRI showed a
lesional origin, abnormal MRI, and localized small area of increased signal intensity in the
frontal resection (vs. more extensive lobectomy). bottom of a gyrus in the right mesial frontal area
In patients with lesional FLE, improved outcome at the convexity, corresponding to the distal
was more likely to be achieved after gross-total lower extremity on the left. Therefore, she
resection rather than subtotal lesionectomy [27]. underwent invasive recording using subdural
In another study, 25 patients with history of grid electrodes covering the right frontal and
resective surgery after intracranial EEG record- temporal regions including motor and sensory
ing were reviewed. A seizure-free (Engel Class cortex, and a dual-sided 2  8 strip to record
1) outcome was seen in 15 patients (60%), while directly from the SMA. Her invasive recording
10 patients (40%) continued to have seizures revealed very frequent interictal spikes and sei-
(Classes II, III, and IV). Risk factors for lack of zures onset zone at the juncture of motor cortex,
seizure freedom included: pre-SMA but less in the SMA proper, with
spread anteriorly. Of note, the ictal discharges
– Left frontal lobe epilepsy surgery; consisted of very high-frequency (beta and
– Dominant hemisphere; gamma) discharges. After discussing the options,
– Patients without aura; the patient underwent an awake surgery for a
– Interictal epileptiform discharges in scalp limited resection including the pre-SMA and part
EEG; of the SMA proper preserving the motor (foot
– Intracranial EEG widespread (>2 cm) in and leg) cortices. Pathology confirmed focal
contrast to focal seizure onset; cortical dysplasia. She had a transitory left-sided
– Shorter latency to onset of seizure spread; and paresis following the surgery with excellent
– Ictal involvement beyond frontal lobe. recovery following rehabilitation. She remained
seizure free for one year after which her noctur-
Lack of seizure freedom is likely because of nal seizure resumed but at a significantly lower
widespread epileptogenicity (as indicated by frequency (90% of decrease) and was milder in
rapid spread of ictal activity). Early resection severity. She had no more diurnal seizures for
may improve seizure outcomes of FLE surgery, 3 years but has had rare seizures during the day
particularly in children [28, 29]. over the last 2 years (follow-up since sur-
gery >6 years). Given her condition and the
Case #5 presence of comorbidities (obstructive sleep
A 52-year-old woman presented with seizures apnea and asthma requiring steroids and resultant
since age seven; she attributed her seizures to an weight gain complicating her apnea manage-
accident two years earlier when an axe fell on her ment), her medications have not been tapered.
head. Her seizures presented as sudden arm and
leg extension with no clear loss of awareness that
happened up to 10 times day (mostly nocturnal). Supplementary Motor Area
She had tried all available ASMs (had a preg- (SMA) Seizures
nancy on phenytoin and phenobarbital with fetal
in utero exposure and congenital defects in the These seizures presented with tonic posturing of
baby). At the time of presentation, she was on 4 the extremities, usually bilateral, and may appear
ASMs at high doses. Her EEGs had been always as “fencer posturing.” Awareness is typically
normal, and in the past she had been diagnosed retained during these seizures. The primary
with “pseudoseizures” by her neurologist. epileptogenic zone is usually outside the SMA
Her scalp monitoring showed no interictal or with rapid spread to the SMA, hence the semi-
ictal EEG discharges while numerous typical ology. The interictal, and even ictal, EEG is often
seizures were recorded. The seizure semiology unrevealing. If the seizure onset focus is outside
25 Epilepsy Surgery Assessment and Testing 331

the SMA, resection of epileptogenic zone alone, manent deficits in about one-third of patients
leaving the SMA intact, might be enough [30]. with MST performed in eloquent cortex. There-
Synchronous interictal/ictal discharges in SMA fore, MST surrounding a lesionectomy may
and primary cortex with a time lag of 25/100 ms. minimize the excised volume and improve sei-
have been reported [31]. Resecting the EEG zure control [35].
onset zone within the SMA while sparing pri- However, a meta-analysis of data from 6
mary motor cortex may result in >90% seizure major epilepsy centers (211 patients, 53 with
reduction [32]. Following the SMA resection, MST alone) reported similar results between the
while preserving primary motor cortex, a transi- MST plus resection and MST alone procedures:
tory paresis or severe deficits without permanent The MST plus resection resulted in >95% seizure
loss of motor or speech functions may be seen reduction (GTCS 87%, CPS 68%, SPS 68%),
(typically lasting 24 h), but favorable surgical compared to >95% seizure reduction (GTCS
outcome is common [33]. 71%, CPS 62%, SPS 63%) in MST alone group.
The outcome seemed to be independent of
factors such as EEG localization, MST location,
Multiple Subpial Transection (MST) age at onset, or duration of epilepsy. These results
suggest that MST may be efficient by itself, with
This technique was introduced to spare the minimal neurologic compromise, and should be
eloquent cortex in patients in whom the epilep- investigated as a stand-alone procedure [36].
togenic zone lies in eloquent cortex. The NST is
based on the notion that epileptogenic dis-
charges require side-to-side (horizontal) interac- Overall Seizure-Free Outcome
tion of cortical neurons while the major
functional properties of cortical tissue depend The overall seizure-free rates following different
upon the vertical fibers. Therefore, severing the types of surgery in different brain regions are
tangential intracortical fibers in the seizure focus reported as follows:
using a small blade is performed, while vertical
fiber connections and blood vessels are pre- Temporal lobectomy: 55–80%
served [34]. Frontal lobe resections: 5–18%
In a report of 21 patients (18 intractable epi- Frontal lobectomy: 23–68%
lepsy and 3 Landau–Kleffner syndrome (LKS)) Parietal lobe resections: 45%
who underwent either resection plus MST (12) or Occipital resections: 46–88%
MST alone, in precentral and postcentral regions Hemispherotomy: 60%
(follow-up: *1–5 years), significant seizure
reduction was seen in 11 of 18 patients (61%)
and 3 LKS patients. The latter group who were
mute before operation showed significant speech Epilepsy Surgery—Long-term
recovery. There were no chronic neurological Outcome (  5 Years)
deficits. Other studies have reported up to 56%
seizure freedom and 95% seizure reduction in Excellent short-term results of resective epilepsy
patients with intractable epilepsy arising from surgery have been well established. Therefore,
eloquent cortex following combined resection review and meta-analysis of long-term outcomes
and MST versus no seizure freedom and >50% of largest case series of patients of any age after
seizure reduction in those treated with MST resective or non-resective epilepsy surgery have
alone. Predictor of complete seizure freedom been attempted. After a mean follow-up of
appears to be the disappearance of epileptiform  5 years, resective surgery resulted in the fol-
discharges in the post-op EEG. Subtle, but per- lowing seizure freedom rates:
332 G.K. Motamedi

Temporal lobe resections: 66% Scalp ictal EEG is rarely localizing. Parietal
Occipital and parietal resections: 46% lobe seizures have more variable scatter of
Frontal lobe resections: 27% interictal EEG discharges and less localizing ictal
Multiple subpial transections: 16% discharges compared to temporal or frontal lobe
Callosotomy (free of most disabling seizures): seizures. Overall, the semiology is of less value
35% in these patients. High-frequency oscillations
(HFOs) may be useful for localization as they
Therefore, long-term seizure-free rate follow- more concentrated in seizure focus.
ing temporal lobe resective surgery appears to be Postoperative sensory deficits such as tempo-
favorable similar to that of short-term-controlled rary partial hemisensory or Gerstmann syndrome
studies, but it is consistently lower after may be seen when corticectomy involves
extra-temporal or palliative surgeries [37]. post-central gyrus. However, resective surgery
can result in seizure freedom or significant sei-
zure reduction especially when a lesion is pre-
Failed Epilepsy Surgery sent. The most common pathologies include
and Reoperation low-grade tumors, cortical dysplasia, gliotic
scars, or cavernous vascular malformations.
Reoperation following a failed previous surgery Complete or nearly complete seizure freedom
can be an efficacious and reasonably safe has been reported in 65–67.5% of patients with
approach. Successful reoperation has been favorable outcome factor being the absence of
reported in patients with concordance between post-resection epileptiform discharges on the
their postsurgical imaging and electroclinical EEG [40–43].
findings, and no brain trauma or infection before
their seizure onset. A review of 15 case series
including 402 adult patients reoperated 2–
5.5 years later (reoperation rate: 3.8–14%; Occipital Lobe Epilepsy
follow-up of 6 months–4 years) post-reoperation
seizure freedom was reported at 36.6% and Auras are reported in 88% of these patients. The
complications rate at 13.5% [38]. auras consist of elementary visual hallucina-
It is also safe to use subdural grid electrodes tions, ictal amaurosis, eye movement sensations,
in patients with prior craniotomy with favorable early forced blinking or eyelid flutter, and con-
long-term seizure-free outcomes. In these tralateral visual field deficits. There is often
patients, ictal onset at the edge of original sur- eye/head deviation (usually contralateral to the
gical bed (more with lesional epilepsy) seems to side of seizure origin), loss of awareness, vari-
be a predictor of seizure freedom [39]. ous types of automatism, fumbling (typical for
temporal lobe seizures), and at times asymmet-
rical tonic or focal clonic motor patterns (char-
Parietal Lobe Epilepsy acteristic of frontal lobe seizures). Medial or
lobar lesions are more likely to cause visual field
Auras are commonly present in these seizure; 94% defects. The scalp EEG is rarely localizing [44,
of patients report somatosensory auras (painful 45]. Intracranial EEG recording correctly iden-
dysesthesias), vertigo, aphasia, or disturbances of tifies occipital lobe seizure origin in most, but
one’s body image. The ictal propagation to the not all of these patients. The variability in
SMA may result in hypermotor manifestations semiology depends on seizure spread patterns,
while propagation to the temporo-limbic regions i.e., medially, laterally, above/below the sylvian
may result in complex visual or auditory halluci- fissure, both ipsilateral and contralateral to the
nations and automatisms. seizure origin.
25 Epilepsy Surgery Assessment and Testing 333

After, focal resection seizure freedom is seen significant and long-term improvement in their
in 46–88% of patients. The most common cognitive processing speed, in particular those
pathologies include dysplasia, tumors, and glio- who were on no ASM [48].
sis. Following resection, about 50% of patients
will not experience any new visual deficits while
new quadrantanopia or hemianopia has been Extra-temporal Epilepsy Surgery
reported in 17%. Tailored resections (e.g., in in Children
lateral occipital lesions) may help preserve intact
vision in about 38% of patient [46]. In general, surgical outcomes for extra-temporal
lobe epilepsy (ETLE) are worse than those for
TLE. A meta-analysis of the available literature
Insular-Opercular Seizures (17 studies, 95 patients) reported that pathology
(cortical dysplasia) and seizure type (CPS) were
These seizures usually present as nocturnal the positive outcome predictors. Factors con-
complex motor seizures. The auras include vis- tributing to less favorable outcome seem to be
cerosensitive or somatosensory symptoms. Ictal diffuse nature of pathology involved in ETLE,
semiology consists of asymmetric tonic–dystonic difficulty localizing the seizure focus in young
posturing and/or hyperkinetic automatisms children, and involvement of “eloquent” cortex
(bimanual/bipedal activity and ballistic move- [49].
ments). Simultaneous insular and opercular ictal
discharges are present. Complex motor mani-
festations are seen when the seizure spreads to Ictal Onset High-Frequency
frontomesial regions (cingulum, superior frontal Oscillations
gyrus, and SMA) and/or mesial and neocortical
temporal lobe structures. Retrospective review of high-frequency oscilla-
Favorable outcome can be achieved with tions (HFOs > 80 Hz; sampling rate: 2000 Hz)
insular-opercular cortical resections. The most recorded in intracranial EEG in pediatric patients
common underlying pathology is focal cortical suggest a high prevalence of ictal HFO zones in
dysplasia [47]. 93% of patients. Complete resection of ictal
HFOs, regardless of the frequency bands, is highly
correlated with a favorable surgical outcome. In
Epilepsy Surgery in Children one series, complete resection resulted in 82%
seizure freedom versus 21% after incomplete
Epilepsy surgery is commonly performed in resection. The most common pathology in these
children. The long-term outcome (5–21 years) patients was cortical dysplasia [50].
studied in 47 children with age at surgery ranging
from 0.5 to 18.7 years (mean 8) reported 49%
(23/47) seizure freedom and >75% seizure References
reduction in 13% (6/47). All of these children
were assessed for cognitive function pre- and 1. Kwan P, Arzimanoglou A, Berg AT, Brodie MJ,
postsurgery and at follow-up. Twenty-one et al. Definition of drug resistant epilepsy: consensus
patients required a reoperation to achieve satis- proposal by the ad hoc task force of the ILAE
commission on therapeutic strategies. Epilepsia.
factory seizure outcomes with low complications 2010;51(6):1069–77.
rate and no increase in seizures. Cognitive 2. Meador KJ, Gilliam FG, Kanner AM, Pellock JM.
function was well preserved as 76% (34/47) Cognitive and behavioral effects of antiepileptic
followed their expected cognitive trajectory. drugs. Epilepsy Behav. 2001;2(4):SS1–SS17.
Patients who were seizure free showed
334 G.K. Motamedi

3. Taylor DC, Lochery M. Temporal lobe epilepsy: temporal lobe epilepsy surgery: corticalamygdalo-
origin and significance of simple and complex auras. hippocampectomy versus selective amygdalohip-
J Neurol Neurosurg Psychiatry. 1987;50(6):673–81. pocampectomy. J Neurosurg. 2008;108(3):517–24.
4. Wiebe S, Blume WT, Girvin JP, Eliasziw M. A ran- 18. Tanriverdi T, Dudley RW, Hasan A, et al. Memory
domized, controlled trial of surgery for temporal-lobe outcome after temporal lobe epilepsy surgery: corti-
epilepsy. Effectiveness and efficiency of surgery for coamygdalohippocampectomy versus selective
temporal lobe epilepsy study group. N Engl J Med. amygdalohippocampectomy. J Neurosurg. 2010;113
2001;345(5):311–318. (6):1164–75.
5. Farwell JR, Dodrill CB, Batzel LW. Neuropsycho- 19. Jones-Gotman M, Zatorre RJ, Olivier A, et al.
logical abilities of children with epilepsy. Epilepsia. Learning and retention of words and designs follow-
1985;26(5):395–400. ing excision from medial or lateral temporal-lobe
6. Bourgeois BF, Prensky AL, Palkes HS, Talent BK, structures. Neuropsychologia. 1997;35(7):963–73.
Busch SG. Intelligence in epilepsy: a prospective 20. Bandt SK, Werner N, Dines J, et al. Trans-middle
study in children. Ann Neurol. 1983;14(4):438–844. temporal gyrus selective amygdalohippocampectomy
7. Engel J Jr, McDermott MP, Wiebe S, et al. Early for medically intractable mesial temporal lobe
randomized surgical epilepsy trial (ERSET) study epilepsy in adults: seizure response rates, complica-
group. Early surgical therapy for drug-resistant tions, and neuropsychological outcomes. Epilepsy
temporal lobe epilepsy: a randomized trial. JAMA. Behav. 2013;28(1):17–21.
2012;7;307(9):922–930. 21. Helmstaedter C. Cognitive outcomes of different
8. Spencer D. Temporal lobectomy. In: Luders HO, surgical approaches in temporal lobe epilepsy.
editor. Epilepsy surgery. New York: Raven Press, Epileptic Disord. 2013;15(3):221–39.
1991. P. 77–81. 22. Wolf RL, Ivnik RJ, Hirschorn KA, Sharbrough FW,
9. Spencer DD, Spencer SS, Mattson RH, Wil- Cascino GD, Marsh WR. Neurocognitive efficiency
liamson PD, Novelly RA. Access to the posterior following left temporal lobectomy: standard versus
medial temporal lobe structures in the surgical limited resection. J Neurosurg. 1993;79(1):76–83.
treatment of temporal lobe epilepsy. Neurosurgery. 23. Josephson CB, Dykeman J, Fiest KM et al. System-
1984;15(5):667–71. atic review and meta-analysis of standard vs selective
10. Wieser HG, Yaşargil MG. Selective amygdalohip- temporal lobe epilepsy surgery. Neurology.
pocampectomy as a surgical treatment of mesiobasal 2013;30;80(18):1669–1676.
limbic epilepsy. Surg Neurol. 1982;17(6):445–57. 24. Graydon FJ, Nunn JA, Polkey CE, Morris RG.
11. Yaşargil MG, Wieser HG, Valavanis A, von Neuropsychological outcome and the extent of
Ammon K, Roth P. Surgery and results of selective resection in the unilateral temporal lobectomy.
amygdala-hippocampectomy in one hundred patients Epilepsy Behav. 2001;2(2):140–51.
with nonlesional limbic epilepsy. Neurosurg Clin N 25. Cohen-Gadol AA, Wilhelmi BG, Collignon F, et al.
Am. 1993;4(2):243–61. Long-term outcome of epilepsy surgery among 399
12. Park TS, Bourgeois BF, Silbergeld DL, Dodson WE. patients with nonlesional seizure foci including
Subtemporal transparahippocampal amygdalohip- mesial temporal lobe sclerosis. J Neurosurg.
pocampectomy for surgical treatment of mesial 2006;104(4):513–24.
temporal lobe epilepsy. Technical note. J Neurosurg. 26. Fong JS, Jehi L, Najm I, Prayson RA, Busch R,
1996;85(6):1172–6. Bingaman W. Seizure outcome and its predictors
13. Little AS, Smith KA, Kirlin K, Baxter LC, et al. after temporal lobe epilepsy surgery in patients with
Modifications to the subtemporal selective amyg- normal MRI. Epilepsia. 2011;52(8):1393–401.
dalohippocampectomy using a minimal-access tech- 27. Englot DJ, Wang DD, Rolston JD, Shih TT,
nique: seizure and neuropsychological outcomes. Chang EF. Rates and predictors of long-term seizure
J Neurosurg. 2009;111(6):1263–74. freedom after frontal lobe epilepsy surgery: a
14. Lowe AJ, David E, Kilpatrick CJ, et al. Epilepsy systematic review and meta-analysis. J Neurosurg.
surgery for pathologically proven hippocampal scle- 2012;116(5):1042–8.
rosis provides long-term seizure control and 28. Holtkamp M, Sharan A, Sperling MR. Intracra-
improved quality of life. Epilepsia. 2004;45(3):237– nial EEG in predicting surgical outcome in frontal
42. lobe epilepsy. Epilepsia. 2012;53(10):1739–45.
15. Elliott RE, Bollo RJ, Berliner JL, Silverberg A, et al. 29. Simasathien T, Vadera S, Najm I, Gupta A, Binga-
Anterior temporal lobectomy with amygdalohip- man W, Jehi L. Improved outcomes with earlier
pocampectomy for mesial temporal sclerosis: predic- surgery for intractable frontal lobe epilepsy. Ann
tors of long-term seizure control. J Neurosurg. Neurol. 2013;73(5):646–54.
2013;119(2):261–72. 30. Ikeda A, Sato T, Ohara S, Matsuhashi M et al.
16. Vale FL, Reintjes S, Garcia HG. Complications after “Supplementary motor area (SMA) seizure” rather
mesial temporal lobe surgery via inferiortemporal than “SMA epilepsy” in optimal surgical candidates:
gyrus approach. Neurosurg Focus. 2013;34(6):E2. a document of subdural mapping. J Neurol Sci.
17. Tanriverdi T, Olivier A, Poulin N, Andermann F, 2002;15;202(1–2):43–52.
Dubeau F. Long-term seizure outcome after mesial
25 Epilepsy Surgery Assessment and Testing 335

31. Baumgartner C, Flint R, Tuxhorn I, et al. Supple- 41. Salanova V. Parietal lobe epilepsy. J Clin Neuro-
mentary motor area seizures: propagation pathways physiol. 2012;29(5):392–6.
as studied with invasive recordings. Neurology. 42. Ristić AJ, Alexopoulos AV, So N, Wong C,
1996;46(2):508–14. Najm IM. Parietal lobe epilepsy: the great imitator
32. Kasasbeh AS, Yarbrough CK, Limbrick DD et al. among focal epilepsies. Epileptic Disord. 2012;14
Characterization of the supplementary motor area (1):22–31.
syndrome and seizure outcome after medial frontal 43. Binder DK, Podlogar M, Clusmann H, Bien C,
lobe resections in pediatric epilepsy surgery. Neuro- Urbach H, Schramm J, Kral T. Surgical treatment of
surgery. 2012;70(5):1152–1168; discussion 1168. parietal lobe epilepsy. J Neurosurg. 2009;110
33. Rostomily RC, Berger MS, Ojemann GA, Lettich E. (6):1170–8.
Postoperative deficits and functional recovery fol- 44. Williamson PD, Thadani VM, Darcey TM,
lowing removal of tumors involving the dominant Spencer DD, Spencer SS, Mattson RH. Occipital
hemisphere supplementary motor area. J Neurosurg. lobe epilepsy: clinical characteristics, seizure spread
1991;75(1):62–8. patterns, and results of surgery. Ann Neurol. 1992;31
34. Morrell F, Whisler WW, Bleck TP. Multiple subpial (1):3–13.
transection: a new approach to the surgical treatment 45. Caicoya AG, Macarrón J, Albísua J, Serratosa JM.
of focal epilepsy. J Neurosurg. 1989;70(2):231–9. Tailored resections in occipital lobe epilepsy surgery
35. Hufnagel A, Zentner J, Fernandez G, Wolf HK, guided by monitoring with subdural electrodes:
Schramm J, Elger CE. Multiple subpial transection characteristics and outcome. Epilepsy Res. 2007;77
for control of epileptic seizures: effectiveness and (1):1–10.
safety. Epilepsia. 1997;38(6):678–88. 46. Tandon N, Alexopoulos AV, Warbel A, Najm IM,
36. Spencer SS, Schramm J, Wyler A, et al. Multiple Bingaman WE. Occipital epilepsy: spatial catego-
subpial transection for intractable partial epilepsy: an rization and surgical management. J Neurosurg.
international meta-analysis. Epilepsia. 2002;43 2009;110(2):306–18.
(2):141–5. 47. Proserpio P, Cossu M, Francione S, et al.
37. Téllez-Zenteno JF, Dhar R, Wiebe S. Long-term Insular-opercular seizures manifesting with
seizure outcomes following epilepsy surgery: a sleep-related paroxysmal motor behaviors: a
systematic review and meta-analysis. Brain. stereo-EEG study. Epilepsia. 2011;52(10):1781–91.
2005;128(Pt 5):1188–98. 48. Hallböök T, Tideman P, Rosén I, Lundgren J,
38. Surges R, Elger CE. Reoperation after failed resec- Tideman E. Epilepsy surgery in children with
tive epilepsy surgery. Seizure. 2013;22(7):493–501. drug-resistant epilepsy, a long-term follow-up. Acta
39. Vadera S, Jehi L, Gonzalez-Martinez J, Bingaman W. Neurol Scand. 2013;128(6):414–21.
Safety and long-term seizure-free outcomes of sub- 49. Ansari SF, Maher CO, Tubbs RS, Terry CL,
dural grid placement in patients with a history of Cohen-Gadol AA. Surgery for extratemporal nonle-
prior craniotomy. Neurosurgery. 2013;73(3):395– sional epilepsy in children: a meta-analysis. Childs
400. Nerv Syst. 2010;26(7):945–51.
40. Salanova V, Andermann F, Rasmussen T, Olivier A, 50. Fujiwara H, Greiner HM, Lee KH, Holland-Bouley
Quesney LF. Parietal lobe epilepsy. Clinical mani- KD, et al. Resection of ictal high-frequency oscilla-
festations and outcome in 82 patients treated surgi- tions leads to favorable surgical outcome in pediatric
cally between 1929 and 1988. Brain. 1995;118(Pt epilepsy. Epilepsia. 2012;53(9):1607–17.
3):607–27.
Procedures and Outcomes
in Epilepsy Surgery 26
Ronald P. Lesser

Although surgery has risks, there are several nearby might react in way that could add addi-
reasons to consider it for patients with epilepsy. tional harm or danger.
First is the possibility of harm to the patient if Fourth are risks related to side effects of
surgery is not done, because of continued sei- anticonvulsants, particularly if patients have fre-
zures. In a study in 1997, Buck and coworkers quent seizures with the need for higher doses or
[1] found that of 300 patients with at least one for additional numbers of medications. Addi-
seizure in the previous year, 24% had sustained a tionally, for women of childbearing age, anti-
head injury, 16% had burned or scalded them- convulsants impose risks on a developing fetus,
selves, 14% had a seizure while bathing or and, for young children, seizures and medica-
swimming with the risk of drowning, 10% had a tions impose risks on development.
dental injury, and 6% had a fracture. Over the last several decades, there have been
Second, beyond actual injury is the important a number of new medications which can be used,
entity SUDEP (sudden unexpected death in epi- but unfortunately, many patients continue to
lepsy patients), an entity that may be more have seizures despite these. Overall, only 2/3–3/4
common in patients with more generalized con- of patients can be seizure-free on medication [2].
vulsive seizures and in patients in their third Studies have shown that patients with intractable
through fifth decades of life. The reason for death seizures undergoing surgery are significantly
in these patients is not known, but it is clear that more likely to be seizure-free after surgery than if
patients with seizures, particularly with intract- they continue on medication alone [3].
able seizures, can be found dead without a clear Before operating, however, we need to
explanation other than the fact that they have determine whether surgery is appropriate for the
epilepsy. particular patient, and, if so, which. Less invasive
Third are the effects that ongoing seizures, evaluations should be performed first, with
particularly those affecting consciousness, have questions to answer including the following:
on a person’s daily life. One cannot drive. One What do the clinical symptoms suggest about
may have a seizure in public or in an unfamiliar region of seizure onset? Is there a focal lesion
situation and be unable to care for oneself. Those that can be resected? Is there a focus that can be
found on EEG? Is the focus accessible surgi-
cally? Is the focus surgically separate from
regions controlling important functions? Would a
procedure other than focal resection be better?
R.P. Lesser (&) Because the symptoms of seizures can vary and
Department of Neurology, Johns Hopkins University
because of the possibility of seizures beginning
School of Medicine, 600 North Wolfe Street, 2-147
Meyer Building, Baltimore, MD 21287-7247, USA in one place but projecting to another, it is
e-mail: rl@jhmi.edu important to correlate the behaviors with the

© Springer Science+Business Media LLC 2017 337


M.Z. Koubeissi and N.J. Azar (eds.), Epilepsy Board Review,
DOI 10.1007/978-1-4939-6774-2_26
338 R.P. Lesser

EEG findings during the episodes, and for this eye, head, or body turning with tonic or dystonic
reason, patients should be referred for video EEG posturing. Orbital frontal seizures can include
monitoring so that actual seizures can be recor- unusual behaviors including hypermotor activity
ded and patient behaviors during the seizures such as rapid leg kicking or bicycling and can
analyzed. have autonomic findings, behavior arrest, and
automatisms of other types. These characteristi-
cally occur frequently during sleep and last a
Symptoms relatively short period of time. Seizures from the
frontal operculum can include salivation and
Temporal lobe seizures are the most commonly swallowing. Inferior frontal onset can include
evaluated for surgery. Onsets of seizures from findings referable to the face or to speech. Dor-
the temporal lobe can include epigastric, olfac- solateral or dorsomedial onset seizures can
tory, and gustatory sensations, emotional chan- include contralateral motor findings, premotor
ges, sense of familiarity or strangeness, area seizures tonic version, and supplementary
hallucinations, staring, and automatisms, among motor area seizures speech arrest, fencing pos-
others. One review [4] concluded that with tures, bilateral motor findings, and head version.
temporal lobe epilepsy, abdominal aura had a It is important to note that frontal lobe regions
52% sensitivity and 90% specificity for localiz- can produce seizures that are similar to one
ing seizures to the temporal lobe. Seizures arising another.
from temporal neocortex can have similar Seizures from the insula can include visceral,
symptoms. For example, basal but not mesial gustatory, and somatosensory symptoms,
temporal seizures can present with behavioral including laryngeal constriction or paresthesias
arrest or motor changes. Ictal theta activity was [4]. Parietal lobe seizures can begin with
found to have an 85% probability for temporal somatosensory phenomena, and occipital lobe
lobe epilepsy and was 80–94% correct with seizures can begin with visual auras and phe-
respect to the side of seizure onset. Lateralized nomena. However, both parietal and occipital
interictal spikes and possibly contralateral hand lobe seizures can be locally silent, with symp-
dystonia also were helpful. The authors thought toms related to the area of projection. For
that some of these also might help differentiate example, parietal lobe seizures can imitate
mesial from lateral temporal lobe epilepsy. superior frontal lobe seizures or can have sen-
Febrile seizures are thought to have a rela- sorimotor symptoms.
tionship with mesial temporal sclerosis, as is
found with mesial temporal lobe epilepsy; one
report [5] found that only 2/21 patients with Noninvasive Evaluations
neocortical temporal lobe epilepsy had a history
of febrile seizures. Seizure-free intervals were Neuropsychological evaluation is important both
found to be less common with neocortical tem- in assessing baseline functioning and in deter-
poral lobe epilepsy than with mesial onset tem- mining whether there are aspects of function,
poral lobe epilepsy. Despite neocortical onset, which are below expectations. At times, these
there nonetheless could be mild hippocampal functions can be localized to specific regions of
atrophy. Patients could have tumors or hetero- the brain, which in turn might be the sites of
topias. About half could have decreased memory origin of the patient’s seizures.
function on the Wada test. Independent con- The intracarotid sodium amobarbital or Wada
tralateral spikes were rare. Some patients had test is performed less frequently now than had
experiential auras or motionless stares. been the case in the past. When used, it has two
Frontal lobe epilepsy symptoms vary with the purposes. With the test, a medication, which
site of seizure onset [4]. With superior or inter- traditionally had been amobarbital, but now can
hemispheric onsets, there can be contralateral be another such as midazolam, is injected so as to
26 Procedures and Outcomes in Epilepsy Surgery 339

“anesthetize” one hemisphere for a few minutes describe the occurrence of two potentially
while the other is tested. One looks for language epileptogenic lesions regardless of type.
function during the period of “anesthesia,” to see Causes of extra temporal seizures in a series
whether speech remains while the hemisphere is of 133 consecutive cases included [7] cortical
not functioning, and one presents items for the dysplasia in 38% and tumor in 28%. They
patient to remember. One also tests recall mem- reported that 10/50 patients with cortical dys-
ory after the effects of the medication wear off, to plasia also had tumors; 11/50 had infarcts or
see whether new memories could be encoded remote ischemic lesions. They found four with
during the period of hemisphere inactivation. The arteriovenous malformations, 3 with Sturge–
idea is that if a function is intact during the Weber malformations, and 2 with Rasmussen’s
period of drug-induced inactivation, the tested encephalitis. 17% had no significant findings.
function is likely to be supported by the Positron-emission tomography (PET) scans
non-inactivated hemisphere. can point to areas of decreased metabolic func-
Imaging is increasingly important in evaluat- tion which in turn can be area of epileptogenesis.
ing patients with intractable seizures, with mag- Single-photon emission computed tomography
netic resonance imaging (MRI) being the most (SPECT) studies can point to areas of altered
important; one should always be performed if function in a similar way with similar inferences
possible. Important findings include evidence of regarding whether these might indicate where
mesial temporal sclerosis or other abnormality, as seizures are originating. Magnetic resonance
well as evidence of tumor, dysplasia, vascular spectroscopy (MRS) can point to areas with
anomaly, developmental defects, or other chan- altered chemistry. fMRI is being developed as a
ges. For patients with temporal lobe epilepsy, it possible alternative to the Wada test, using it to
is important to keep in mind that there can be localize language, which has been relatively
bilateral atrophy on MRI in some patients, per- successful, as well as memory, which has not
haps 20% [4]. Sometimes, surgery can nonethe- been as successful thus far.
less be performed on one side, if seizures only
originate on that side, but it adds a consideration
before deciding whether to operate and a con- Invasive Evaluations
sideration when counseling the patient with
respect to possible postoperative memory prob- Often noninvasive evaluation is sufficient to
lems. Neuroimaging [4] can show amygdala determine how and where to operate, but some
abnormalities in 55% of patients and changes in patients need implanted electrodes as well.
the enterorhinal cortex in 25% and in the fornix Depth and subdural electrodes are the ones
in 86%. In one study of patients with temporal principally used. Depth electrodes are thin
lobe epilepsy and tumors [6], astrocytomas were “tubes,” each usually containing several elec-
found in 46%, gangliogliomas in 21%, oligo- trodes and electrode wires, and which are
dendrogliomas in 18%, dysembryoplastic neu- directed through the skull and through the outer
roepithelial tumors in 6%, anaplastic cerebral tissues, aiming at more medial locations
astrocytomas in 6%, and meningiomas in 3%. such as mesial temporal lobes. However, there
Dual pathology can occur in 15–52% of are electrodes along the tube so that more lateral
patients with hippocampal sclerosis, with eti- locations including neocortex are recorded at the
ologies including heterotopias, cortical dysplasia, same time. Subdural electrodes are flat disks,
and tumors. Vascular lesions including cav- usually a few millimeters in diameter, imbedded
ernous malformations and arteriovenous malfor- in Silastic or other plastics and placed over and
mations can occur in about 5% of patients [4]. around areas of interest. Both depth and subdural
Although the term has often been used to electrodes are used to localize the area of seizure
describe the combination of hippocampal scle- onset. Subdural electrodes are commonly, and
rosis plus another lesion, it also is used to depth electrodes less commonly, stimulated
340 R.P. Lesser

electrically to determine the relationship of the important to emphasize that the reliability of
area of seizure onset to regions controlling results can depend on the intensity of stimulation.
important functions such as movement, sensa- If you stimulate at too low an intensity, you can
tion, and language. get a false-negative result. If you stimulate at too
Complications of depth electrodes include high an intensity, you can get afterdischarges
asymptomatic subdural bleeding gliosis, degen- which can produce false positives because of the
eration, and microabscesses along electrode tract spread of the afterdischarges and also can cause
[8–10]. The incidence of bleeding or infection is seizures. One should begin at a low intensity,
between 0.5 and 5%. There can be a 25% overall 0.5–1 mA, and increase in increments of 0.5–
rate of complications with subdural electrodes 1 mA. Keep in mind that the above is in mil-
[11], including 12% infection, 11% transient liamps, but the important parameter is charge
neurological deficits, 2.5% epidural hematoma, density, which depends on not only current but
2.5% increased intracranial pressure, 1.5% also electrode surface area.
infarction, and 0.5% death. Cerebrospinal fluid It is also important to emphasize that stimu-
leakage also was common. The authors found lation only assesses the cortex directly under the
that complications were more likely if there were stimulated electrodes. Charge density drops rel-
more than 60 electrodes and if the grid was left in atively rapidly with increased distance from the
more than 10 days. Other risks included older actual location of the electrodes. Also, 7/8 of the
patients, left-sided placement, and additional burr current is shunting through the cerebrospinal
holes. They observed that complication risk fluid [13, 14].
likely was less now with improved technique. In addition to stimulation, one can analyze the
brain function with a variety of methods based on
frequency or power analysis. In summary, one
Testing the Brain—Functional asks the patient to perform an activity which can
Localization be something simple such as moving the tongue,
making a fist, or wiggling the toes. One then
Functional localization can be performed in one records brain activity before during and after this
of two ways. One can alter the brain, for example activity and sees whether there are changes in
with cortical stimulation, and assess the behav- particular regions of the brain which might point
iors that occurred during the alteration. An to the area participating in controlling this
example would be to see whether there is hand or activity. There [15, 16] is a relatively good cor-
other movement during stimulation. One also can relation between the results of such analyses and
alter behavior and then assess the brain during the results of cortical stimulation, but the two
the behavior. An example might be asking the methods are slightly different so that the regions
patient to begin to read and then seeing whether localized with one technique or the other might
there is reading arrest during stimulation. Corti- be expected to, and in fact do, differ.
cal stimulation is generally performed with
recurrent pulses. These should be alternating in
polarity, so that the resulting stimulation is Temporal Lobe Surgical Resections
charge-balanced, to avoid complications due to
metal deposit on an electrode. We [12] have used Seizure Control
0.3-ms duration alternating polarity square wave
pulses, delivered at 50 pulses per second, with There are controversies regarding what to
stimulation duration varying but generally 1–2 s remove. For example, for anterior temporal
initially and then up to about 5 s for language lobectomy, some perform the same standard
testing. Intensities that are needed to obtain resection on each patient, while others tailor the
stimulation-induced changes vary; with the resection based upon the specific findings during
device we use, they can go up to 17.5 mA. It is evaluation, particularly evaluation with
26 Procedures and Outcomes in Epilepsy Surgery 341

implanted electrodes. Some surgeons perform an seizures. If patient has a normal MRI, outcome
amygdalohippocampectomy with lateral tempo- can be good if there are ipsilateral interictal EEG
ral structures left relatively intact. Some will do a discharges and a history of febrile seizures. Out-
hippocampectomy alone. There has been interest come can be worse in patients with tumors, cor-
in the use of radiosurgery and laser surgery as tical dysplasia, vascular disease, and a longer
noninvasive ways of resecting only mesial tem- duration of epilepsy [4].
poral structures. One review [4] concluded that after surgery
In a prospective study [17], Wiebe et al. com- for temporal lobe tumors, 65% of patients
pared 40 patients who underwent temporal became seizure-free and 82–86% of patients
lobectomy to 40 who were on a surgical waiting were free of disabling seizures. For mesial tem-
list for a year. The surgery was an en bloc resec- poral sclerosis, 75% of patients became
tion of 4–4.5 cm of the dominant and 6–6.5 cm of seizure-free with 41–79% free of disabling sei-
the non-dominant, temporal lobe, with removal of zures. Among patients with normal MRIs, 56–
amygdala and 1–3 cm hippocampus. One year 62% of patients became seizure-free. For patients
later, 58% of patients with surgical treatment but with cortical dysplasia, 38–54% became seizure-
only 8% of patients with medical treatment alone free. For patients with dual pathology, in this
were free of episodes with loss of consciousness. case mesial temporal sclerosis plus another
Quality of life also improved in patients who had lesion, 73% became free of disabling seizures if
surgery. However, simple partial seizure may both pathological areas were removed but only
continue: This group previously found [18] that 20% if only one of the areas was removed.
93% of surgical and 13% of medical patients had a Schmidt et al. [20] reviewed previous reports
90–100% reduction of seizures after 6 months but of a total of 1658 patients who had been off
only 35% of the surgical patients (and 6% of the medication for 5 years. They found that 25% of
medicine alone patients) were completely adults and 31% of children became seizure-free
seizure-free. Also, seizures can recur over time. In and remained off medication for 5 years.
one review [19] of patients seizure-free at one Surgery outcome has long been classified
year, 87–90% were seizure-free at 2 years, 74– using a system devised by Engel [21]. In this,
82% at 5 years, and 67–71% at 10 years. Among Class I is for patients “free of disabling seizures,”
patients who were seizure-free at 2 years, 95% but the subcategories include completely seizure-
were seizure-free of 5 years, 82% at 10 years, and free since surgery, non-disabling simple partial
68% at 15 years. Therefore, with time, seizure seizures only, some disabling seizures but none
control decreases, but the longer the patient was for 2 years, and generalized convulsive seizures
seizure-free, the better the outcome. This review with medication discontinuation. Class II was for
noted that more than half of the patients would patients with rare disabling seizures, with sub-
have their seizure recurrence in the first 6 months categories of initially free of disabling seizures,
and 95% in the first 5 years. They found that rare seizures now, rare disabling seizures since
incomplete resection was more likely in patient surgery, more than rare disabling seizures but
with seizure recurrence. They also noted the rare for the last two years, and nocturnal seizures
possibility of a “running-down” phenomenon only. Class III was defined as worthwhile
with initial seizure recurrence followed by seizure improvement, with subcategories of worthwhile
control. Good prognostic factors included early seizure reduction and prolonged seizure-free
onset of seizures, mesial temporal sclerosis with intervals of greater than half the follow-up per-
ipsilateral interictal EEG discharges, unilateral iod and at least two years. Class IV was for
MRI, PET, and SPECT findings with a single patients with no worthwhile improvement, with
lesion, and greater than 90% of the interictal EEG subcategories of significant seizure reduction, no
findings originating in one place. Poor prognostic change, or worsening of seizures. The classifi-
factors included a long duration of seizures and cation system was revised by a committee of the
the occurrence of generalized tonic–clonic International League Against Epilepsy [22]. In
342 R.P. Lesser

this classification, Class 1 was for patients Adverse effects [4] after temporal lobe epi-
completely seizure-free with no auras beginning lepsy surgery can occur in about 2% of patients
one month after surgery, Class 2 for patients with including quadrantanopsia and memory prob-
auras only, and class 3 for patients with 1–3 lems. There can be other field deficits as well as
seizure days per year. Class 4 was for patients motor, sensory, or speech deficits. With anterior
with seizures ranging from 4/year to 50% choroidal artery or other occlusions, there can be
decrease in days with seizures. Class 5 was for significant problems including strokes. Some
patients with a 50% reduction to 100% increase patients have had cerebellar hemorrhages. Death
in days with seizures. Class 6 was for patients has been reported to occur in 0.24% of patients.
with a greater than 100% increase. Classes 3–6 One review [25] found less than 1% morbidity
included patients with or without auras. including hemorrhage, infarction, pulmonary
embolus, and pseudogout. There was a 2.4%
incidence of hemiparesis and 50% incidence of
Other Outcomes visual field defects with 2–4% hemianopia. Less
than 2% of patients had infection and epidural
Memory often can be worse [23]: (a) after a hematoma or transient third nerve palsy, 20%
dominant hemisphere temporal lobe resection, transient anomia, 1–3% persistent dysphagia, and
(b) if the MRI does not show exclusive unilateral 2–20% transient psychosis or depression.
mesial temporal sclerosis, and (c) if preoperative Depression is commonly present prior to
immediate and delayed recall memory is intact. temporal lobectomy and is more common after-
In particular, there can be declines in object ward if present prior to surgery. Patients with a
naming and similar functions. Memory can history of depression are less likely to become
improve, however, if a nondominant resection is seizure-free after surgery. Moreover, there is a
performed. Surgery can be successful [19] if risk of postsurgery suicide, with an age- and
depth recordings show a unilateral ictal onset, if sex-adjusted mortality ratio of 13.3 compared to
there is ictal spiking but not a rhythmic fast the US population as a whole [26]. On the other
pattern, if there is no evolution to a distinct hand, a study found that 45% of a group of
contralateral seizure pattern, and if there is an patients experienced remission of psychiatric
interhemispheric propagation time greater than symptoms, no longer needing psychotropic
8 s. As expected from the last of these, a longer medication, after epilepsy surgery [27].
duration of ictal EEG activity before clinical
onset may at times point to a more successful
surgical result. Also, results of temporal lobe Other Surgical Resections
surgery are better if the onset of the seizure is not
diffuse on the recordings and does not begin in In one survey [19] of frontal lobe surgery patients,
the posterior temporal regions where resection is at one year 49.5% of patients had Class I onsets
more difficult [19]. With bilateral ictal onset, with 55.7% of these patients seizure-free. In
surgery can still be successful if greater than 50% 5 years, 47% had Class I seizure control and
of seizures originate from the resected side, and 30.1% of these were seizure-free. At 10 years,
if the Wada test shows adequate memory on the 41.9% had Class I outcome. 80% of recurrences
other side with no extra temporal focus [24]. In were in the first 6 months. If there were recur-
this study, 9/11 operated patients had no seizures rences, patients were less likely to become
and one of 11 had a 75% reduction in seizure seizure-free. The running-down phenomenon was
frequency. Explanations for the findings included less frequent. Good prognostic factors included
the possibility of a mirror focus, disconnection, MRI lesions and complete resection. With these,
or bilateral disease which was nonetheless the likelihood of good prognosis was 72% versus
responsive to unilateral surgery. 41% if these were not present.
26 Procedures and Outcomes in Epilepsy Surgery 343

The same authors concluded that with parietal or lesions such as porencephaly with seizures that
and occipital lobe seizures, 73.1% of patients have become intractable to medication [32].
were seizure-free in 6 months, 68.5% in one Often, these patients have multiple seizures per
year, and 54.8% in 6 years. Circumscribed day and have a complete or progressive hemi-
lesions conveyed a good prognosis. The authors paresis. There are widespread areas of potential
noted that side effects such as dysphagia or epileptogenesis in a single hemisphere. The entire
Gerstmann’s syndrome could occur and they hemisphere is removed in the classic procedure,
discuss the importance of sparing the calcarine but there are modifications including a functional
cortex and speech areas. hemispherectomy, in which the hemisphere is left
Patients with bilateral temporal lobe onsets in place but disconnected from the opposite
were discussed above. In addition, several hemisphere by section of pathways such as the
authors have commented on the importance of corpus callosum. To avoid postoperative compli-
considering that some patients with infantile cations, some surgeons will collapse the subdural
spasms and apparent bilateral ictal onset on EEG space by fixing the dura to the falx. Corticectomy
may show unilateral PET hypometabolism and plus disconnection has been performed as well as
then may be found to be the candidates for focal corticectomy plus lobectomy. 70–80% of patients
surgery. Similar situations may occur with become seizure-free. Because of the reduction in
patients with focal hamartomas or other unilat- seizures, intellectual function often improves.
eral MRI lesions but apparently multifocal sei- Despite removal of a hemisphere, patients can
zures. In many cases, the lesions are congenital walk or even run although they may need an ankle
or acquired early [28]. brace. The hand contralateral to the resected
When a region of seizure onset cannot be hemisphere has no fine finger and has little wrist
removed completely, multiple subpial transec- movement but can function as a “helper hand.”
tions can be considered [29, 30]. This involves Possible complications include subarachnoid
separating the superficial cortical horizontal bleeding, hemosiderosis, cerebrospinal fluid
connections within a gyrus while preserving the block, and hydrocephalus.
vertical pathways. Often, this is performed Multilobe resections can be performed as
adjacent to an area of resection. Transections are well. For example, this can be done in patients
typically performed at approximately 5-mm with Sturge–Weber syndrome, or with cortical
intervals, with the cuts extending 1–3 mm. The dysplasia, with the dysplasia removed as an
concept is that this disrupts “horizontal” epilep- addendum to temporal lobe resection. These
togenic propagation while preserving “vertical” methods have not been helpful in patients with
axonal connections. One should keep in mind Rasmussen’s syndrome; hemispherectomy is the
that this affects the gyral crown but not the sulci surgical treatment of choice. As expected, func-
because of the way this is done. Reports describe tional complications of hemispherectomy or
that 1/3–2/3 of patients become seizure-free, but multilobar resection often relate to the location of
later recurrences also are possible. Although the the area of surgical removal. In particular,
major gyral sulci are well known, it is important removal of the perirolandic area is more likely to
to realize that there are microsulci throughout the result in a permanent motor deficit. (But patients
cortex which are not readily seen from the cor- with Rasmussen’s syndrome often are already
tical surface and that fibers in the microsulci are hemiparetic when surgery is performed.)
not affected by this technique. Also, microscopy Corpus callosotomy has been particularly
shows that transections produce not only fiber helpful for atonic, “falling,” seizures as well as
separations but also microlesions [31]. for tonic seizures, with electrodecrement at sei-
Hemispherectomy is a useful for seizure zure onset, and for generalized tonic–clonic sei-
control in a small group of patient who have zures. While “falling” seizures may benefit, other
problems such as Rasmussen’s syndrome, seizure types may remain, so generally this
hemimegalencephaly, Sturge–Weber syndrome, should be thought of as a palliative rather than
344 R.P. Lesser

curative procedure. Focal seizures can become 7. Frater JL, Prayson RA, Morris IH, Bingaman WE.
more severe after section, and in experimental Surgical pathologic findings of extratemporal-based
intractable epilepsy: a study of 133 consecutive
models, kindling can occur more rapidly [33, resections. Arch Pathol Lab Med. 2000;124:545–
34]. It may be that this occurs because seizures 549. Doi:10.1043/0003-9985(2000)124<0545:
were originating in one hemisphere and the spfoeb>2.0.co;2.
homologous region in the opposite hemisphere 8. So N, et al. Depth electrode investigations in patients
with bitemporal epileptiform abnormalities. Ann
was helping to control, limit, or stop the actual Neurol. 1989;25:423–31.
seizure progression. After callosal section, the 9. Spencer SS. Depth versus subdural electrode studies
contralateral homologous region can no longer for unlocalized epilepsy. J. Epilepsy. 1989;2:123–7.
suppress the region of epileptogenesis. 10. Cossu M, et al. Stereoelectroencephalography in the
presurgical evaluation of children with drug-resistant
There can be an acute disconnection syn- focal epilepsy. J Neurosurg. 2005;103:333–43.
drome after callosal section, with akinetic mut- 11. Hamer HM, et al. Complications of invasive
ism, incontinence, apraxia, or the alien hand video-EEG monitoring with subdural grid electrodes.
syndrome. It is thought that this is more likely if Neurology. 2002;58:97–103.
12. Lesser RP, Crone NE, WRS W. Subdural electrodes.
the entire corpus callosum is sectioned initially. Clin Neurophysiol. 2010;121.
For this reason, many prefer to do an anterior 2/3 13. Nathan SS, Lesser RP, Gordon B, Thakor NV.
section first. The posterior third can be sectioned Electrical stimulation of the human cerebral cortex.
later if necessary, with a lower likelihood of Theoretical approach. [Review]. Adv Neurol.
1993;63:61–85.
adverse postoperative effects. However, in addi- 14. Nathan SS, Sinha SR, Gordon B, Lesser RP,
tion to the amount of callosal fibers resected, it is Thakor NV. Determination of current density distri-
possible that pressure on the brain or vascular butions generated by electrical stimulation of the
human cerebral cortex. Electroencephalogr Clin
compromise, for example due to retraction, might
Neurophysiol. 1993;86:183–92.
explain the acute effects of disconnection just 15. Crone NE, et al. Functional mapping of human
described. sensorimotor cortex with electrocorticographic spec-
tral analysis. I. Alpha and beta event-related desyn-
chronization. Brain. 1998;121:2271–99.
16. Crone NE, Miglioretti DL, Gordon B,
References Lesser RP. Functional mapping of human sensori-
motor cortex with electrocorticographic spectral
1. Buck D, Baker GA, Jacoby A, Smith DF, Chad- analysis. II. Event-related synchronization in the
wick DW. Patients’ experiences of injury as a result gamma band. Brain. 1998;121:2301–15.
of epilepsy. Epilepsia. 1997;38:439–44. 17. Wiebe S, Blume WT, Girvin JP, Eliasziw MA.
2. Kwan P, Brodie MJ. Early identification of refractory Randomized, controlled trial of surgery for
epilepsy. N Engl J Med. 2000;342:314–319. temporal-lobe epilepsy. N Engl J Med.
3. Wiebe S, Blume WT, Girvin JP, Eliasziw M. A ran- 2001;345:311–318.
domized, controlled trial of surgery for temporal-lobe 18. McLachlan RS, et al. Health-related quality of life
epilepsy. N Engl J Med. 2001;345:311–8. doi:10. and seizure control in temporal lobe epilepsy. Ann
1056/nejm200108023450501. Neurol. 1997;41:482–9.
4. Velasco TR, Mathern GW. In Wyllie’s treatment of 19. Jehi L, Martinez-Gonzalez J, Bingaman W. In
epilepsy: principles and practice. In: Cascino GD, Wyllie’s treatment of epilepsy: principles and prac-
Wyllie E, Gidal BE,Goodkin HP, editors Ch. 82, tice. In: Wyllie E, Cascino GD, Gidal BE, Good-
922–936 (Wolters Kluwer/Lippincott Williams & kin HP, editors) Ch. 90, 1007–1020 (Wolters
Wilkins, 2011). Kluwer/Lippincott Williams & Wilkins, 2011).
5. Pacia SV, et al. Clinical features of neocortical 20. Schmidt D, Baumgartner C, Loscher W. The chance
temporal lobe epilepsy. Ann Neurol. 1996;40:724– of cure following surgery for drug-resistant temporal
30. doi:10.1002/ana.410400508. lobe epilepsy. What do we know and do we need to
6. Zaatreh MM, Firlik KS, Spencer DD, Spencer SS. revise our expectations? Epilepsy Res. 2004;60:187–
Temporal lobe tumoral epilepsy: characteristics and 201. doi:10.1016/j.eplepsyres.2004.07.004.
predictors of surgical outcome. Neurology. 21. Engel, J Jr., Van Ness PC, Rasmussen TB, Oje-
2003;61:636–41. mann LM. In surgical treatment of the epilepsies (ed
J. Engel, Jr.) Ch. 52, 609–621 (Raven Press, 1993).
26 Procedures and Outcomes in Epilepsy Surgery 345

22. Wieser HG, et al. ILAE commission report. proposal surgical treatment. Ann Neurol. 1990;27:406–13.
for a new classification of outcome with respect to doi:10.1002/ana.410270408.
epileptic seizures following epilepsy surgery. Epilep- 29. Morrell F, Whisler WW, Bleck TP. Multiple subpial
sia. 2001;42:282–6. transection: a new approach to the surgical treatment
23. Stroup E, et al. Predicting verbal memory decline of focal epilepsy. J Neurosurg. 1989;70:231–9.
following anterior temporal lobectomy (ATL). Neu- 30. Kaufmann WE, Krauss GL, Uematsu S,
rology. 2003;60:1266–73. Lesser RP. Treatment of epilepsy with multiple
24. Hirsch LJ, Spencer SS, Spencer DD, Williamson PD, subpial transections: an acute histologic analysis in
Mattson R. Temporal lobectomy in patients with human subjects. Epilepsia. 1996;37:342–52.
bitemporal epilepsy as defined by depth electroen- 31. Kaufmann WE, Krauss GL, Uematsu S,
cephalography. Ann Neurol. 1991;30:347–56. Lesser RP. Treatment of epilepsy with multiple
25. Pilcher WH, Ojemann GA. In brain surgery: com- subpial transections: an acute histologic analysis in
plication, avoidance and management (ed M.L. human subjects. Epilepsia. 1996;37:342–52.
J. Apuzzo) (Churchill Livingston, 1993). 32. Arroyo S, Freeman JM. Epilepsy surgery in children:
26. Kanner AM. Do psychiatric comorbidities have a state of the art. Adv Pediatr. 1994;41:53–81.
negative impact on the course and treatment of 33. Spencer SS, Spencer DD, Glaser GH, Williamson PD,
seizure disorders? Curr Opin Neurol. 2013;26:208– Mattson RH. More intense focal seizure types after
13. doi:10.1097/WCO.0b013e32835ee579. callosal section: the role of inhibition. Ann Neurol.
27. Kanner AM, Balabanov AJ. In textbook of epilepsy 1984;16:686–93. doi:10.1002/ana.410160611.
surgery (Lüders HO, editor). Boca Raton: CRC 34. Wada JA, Sato M. The generalized convulsive
Press;2008. p. 1254–1261. seizure state induced by daily electrical stimulation
28. Chugani HT, et al. Infantile spasms: I. PET identifies of the amygdala in split brain cats. Epilepsia.
focal cortical dysgenesis in cryptogenic cases for 1975;16:417–30.
Quality of Life in Epilepsy
27
Patsy J. Ramey, Mohamad Z. Koubeissi and Nabil J. Azar

Restrictions: Patients at risk for seizures should Driving: Without a doubt, the loss of driving
be warned that all environments or situations that privileges has the greatest overall impact on
could cause harm to the patient or others, should those with epilepsy. Each state has regulations
a seizure occur, must be avoided. Specifically: and some leave the decision to the discretion of
the provider. Driving restriction typically ranges
• No working at “unprotected” heights, between three and 12 months. Tennessee, for
including roofs and ladders. example, requires six months of seizure freedom,
• No working around heavy machinery with whereas Kentucky requires three months. The
moving parts. loss of driving means relying on others for
• No construction equipment use. transportation whether this is to school, work,
• No use of manufacturing equipment, includ- shopping, or to go on a date! The loss of a license
ing, among others, fork lifts, heavy presses, may mean the loss of employment for those that
and conveyor belt systems. drive a truck or captain a boat for a living.
• Avoid known environmental triggers: heat, Commercial Truck Drivers: The restrictions
cold, humidity, dust, and fumes. are placed by the Federal Motor Carrier Safety
• Shower or bathe in minimal amounts of water Administration:
in order to avoid drowning in case of loss of Current recommendations are that restrictions
consciousness. should be determined on an individualized basis.
• Swim only when supervised by someone who The nature of the seizure and risk of recurrence
is aware of seizure history and is capable of should be considered when determining fitness
helping should a seizure occur. for specific job requirements.
• No cooking or working around open flames.
• NO DRIVING! • Persons with diagnosed epilepsy who are
seizure-free and off medication for 10 years
These restrictions stay in effect until released may be considered for licensure to operate a
at the discretion of the provider or the stipula- commercial vehicle.
tions of the state. • Onetime event thought to be nonepileptic and
requiring no antiseizure medication will pos-
sibly allow for return to driving after six
months of seizure freedom.
P.J. Ramey  M.Z. Koubeissi  N.J. Azar (&)
• Single unprovoked seizure with no recurrence
Department of Neurology, Vanderbilt University
Medical Center, Nashville, TN 37212, USA may be considered for reinstatement follow-
e-mail: dr.nabil.azar@gmail.com ing a five-year period off medication.
M.Z. Koubeissi A waiver to this determination may be made
e-mail: koubeissi@gmail.com

© Springer Science+Business Media LLC 2017 347


M.Z. Koubeissi and N.J. Azar (eds.), Epilepsy Board Review,
DOI 10.1007/978-1-4939-6774-2_27
348 P.J. Ramey et al.

if the individual has a normal EEG and has possible if a satisfactory explanation can be
been evaluated by a neurologist that special- established.
izes in epilepsy. Guide for Aviation Medical Examiners:
• Acute symptomatic seizures in the presence
of acute structural insults to the central ner- • A disturbance of consciousness without sat-
vous system with low risk for recurrence; isfactory medical explanation of the cause
there should be no restriction after they have must submit all pertinent medical records,
been seizure-free for two or more years off current neurological report, to include name
antiepileptic drugs. and dosage of medication(s) and side effects.
• Persons that have undergone any procedure This requires Federal Aviation Administra-
that penetrates the dura should not be con- tion (FAA) decision.
sidered eligible for commercial licensure [1]. • Rolandic seizure must submit all pertinent
medical records, current status report, to
Merchant Mariners Including Riverboat include name and dosage of medication(s) and
Captains: According to the US Department of side effect. Rolandic seizures may be eligible
Homeland Security and the United States Coast for certification if the applicant is seizure-free
Guard under COMDTMOTE 16700.4, NVIC for 4 years and has a normal EEG.
04-08, Enclosure (8) Mariners, including com- This requires FAA decision.
mercial ship captains and riverboat captains, are • Febrile seizure (single episode) must submit all
controlled by the Coast Guard. pertinent medical records and a current status
report if occurred prior to age 5, without recur-
• Those mariners that have seizure(s) deter- rence and off medications for 3 years of issue.
mined to be low risk of recurrence may be Otherwise, this requires FAA decision.
considered for a waiver to return to duty • Transient loss of nervous system function(s)
when they have been seizure-free and off without satisfactory medical explanation
medication for a minimum of one year. of the cause, e.g., transient global amnesia
• Those with seizures considered as high risk of must submit all pertinent medical records,
seizure recurrence must be seizure-free for a current status report, to include name
minimum of eight years [on or off medica- and dosage of medication(s) and side effects.
tion]. If they continue on medication their This requires FAA decision.
dose regimen must be stable for two years. If • Unexplained syncope, single seizure. An
they are off medication, they must be applicant who has a history of epilepsy, a
seizure-free for eight years from the time they disturbance of consciousness without satis-
stopped the medication [2]. factory medical explanation of the cause, or a
transient loss of control of nervous system
Aircraft Pilots: Neurological disorders: epi- function(s) without satisfactory medical
lepsy, seizures, stroke, paralysis, etc. The appli- explanation of the cause must be denied or
cant should provide history and treatment, deferred by the examiner. Consultation with
pertinent medical records, current status report, FAA is required.
and medication. The Examiner should obtain • Infrequently, the FAA has granted an autho-
details about such a history and report the results. rization under the special issuance section of
An established diagnosis of epilepsy, a transient part 67 14 CFR 67.401 when a seizure dis-
loss of control of nervous system function(s), or a order was present in childhood, but the indi-
disturbance of consciousness is a basis for denial vidual has been seizure-free for a number of
no matter how remote the history. Like all other years. Factors that would be considered in
conditions of aeromedical concern, the history determining eligibility in such cases would be
surrounding the event is crucial. Certification is age at onset, nature, and frequency of
27 Quality of Life in Epilepsy 349

seizures, precipitating causes, and duration of • Minimize excessive stress,


stability without medication. Follow-up • Limited work hours to 8–10 h/day,
evaluations are usually necessary to confirm • No third or midnight shift,
continued stability of an individual’s condi- • No working at unprotected heights,
tion if an authorization is granted under the • No working around heavy moving
special issuance section of part 67 14 CFR machinery,
67.401 [3]. • Avoid environmental situations that are
known triggers or that should a seizure occur
Common Seizure Triggers: Stress (positive could cause harm to the individual or others,
or negative), fatigue, medication compliance, and
excessive alcohol use, and sleep deprivation. • Provide assistive technology (AT) that can
Positive stress usually relates to vacation time or improve productivity of the individual coping
cheerful events causing change of daily routine with any cognitive issues.
including change in food and alcohol intake,
reduced sleep time, and changing time zones. College students with epilepsy should register
Negative stress mostly relates to bad news and with Student Health, protecting themselves
grief resulting in reduced sleep and poorer AED should a need arise either physically or educa-
adherence. tionally. Seizure activity frequently escalates
Accommodations: According to the United during the college years. Students in both high
States Department of Labor: school and college may need assistance with
A job accommodation is a reasonable adjustment
recording lectures, extra time to prepare for
to a job or work environment that makes it possible examinations, and accommodations for missed
for an individual with a disability to perform job classes secondary to seizure activity. Accom-
duties. Determining whether to provide accom- modations frequently requested include the
modations involves considering the required job
tasks, the functional limitations of the person doing
following:
the job, the level of hardship to the employer, and
other issues. Accommodations may include spe- • Leniency on attendance.
cialized equipment, facility modifications, adjust- • If seizures are active, they may have to rely
ments to work schedules or job duties, as well as a
whole range of other creative solutions.
on others for transportation.
• Seizures can result in an assignment not being
The Job Accommodation Network (JAN), a completed or a class missed.
service of the Office of Disability Employment • All night study sessions are not possible due
Policy (ODEP), provides a free consulting service
on workplace accommodations [4]. to risk of sleep deprivation.
• Avoidance of multiple examinations on the
Accommodations may be needed at work or same day [frequently during mid-term and
school. Looking at the classic triggers, recom- finals].
mendations, and restrictions, it is simple to jus- • Understanding that an assignment given one
tify the accommodations often needed for the day and expected back the next might not be
patient to function in their environment. The possible as it is imperative that the student
individual must understand that accommodations gets adequate rest. All night study sessions
are not always possible. can only produce negative outcomes.
A letter, worded carefully, should assure that • Short-term memory loss is not uncommon so
the individual’s needs are addressed without “pop quizzes” will be difficult and the student
violating HIPAA rules, or requesting accommo- may need other means of meeting the needed
dations that are unreasonable. results.
Accommodations in the work place include • Assistance with reading, taking notes, or
the following: recording lectures.
350 P.J. Ramey et al.

• Single college room—allowing for adequate • Observation of a third party, and


rest and no late night interruptions by • Description by the patient is not acceptable
roommates. for social security requirements;
History of treatment, response and any recent
Other accommodations occasionally
changes;
requested:
Consistency in therapy:
• Dogs/pets—both trained seizure dogs or • Attending regular clinic visits/
companions that may stay in their dorm room communications and
or apartment; • Details regarding seizure history and respon-
• Request for single-level housing; ses to therapy;
• Use of elevators when available rather than Major motor seizures must be occurring more
open stairwells; frequently than once a month—on medication;
• Access ride or other forms of discounted Minor motor seizures must be occurring more
public transportation; and frequently than once weekly in spite of being on
• Use/purchase of wheelchairs, manual or prescribed treatment for at least 3 months;
motorized. Establish whether the seizures are due to factors
beyond the individual’s control or to noncom-
Disability: According to the Social Security pliance with prescribed therapy:
Official Web site: [5]
SSR 87-6: TITLES II AND XVI: THE ROLE • Record of AED blood levels,
OF PRESCRIBED TREATMENT IN • Low levels must be explained,
THE EVALUATION OF EPILEPSY • Noncompliance,
• Abnormal absorption or metabolism, and
POLICY STATEMENT: As a result of a modern
treatment which is widely available, only a small
• The dosage is not optimal.
percentage of epileptics, who are under appropriate
treatment, are precluded from engaging in sub- Medications:
stantial gainful activity (SGA). Situations where Insurance plans are becoming more complicated.
the seizures are not under good control are usually
due to the individual’s noncompliance with the A secondary program such as Medco/Express
prescribed treatment rather than the ineffectiveness Scripts, CVS Caremark, Optum, Envi-
of the treatment itself. Noncompliance is usually sionRxPlus, and many more generally processes
manifested by failure to continue ongoing medical prescriptions. This secondary program determi-
care and to take medication at the prescribed
dosage and frequency. Determination of blood nes coverage and responses to appeals. Each
levels of anticonvulsive drugs may serve to indi- program has its own formulary and appeals
cate whether the prescribed medication is being process. It is important to understand their
taken. In a substantial number of cases, use of requirements for coverage as often there are ways
alcohol has been found to be a contributory basis
for the individual’s failure to properly follow pre- to successfully work around their roadblocks.
scribed treatment. In such cases, the individual’s These are only a few of the restrictions on pre-
alcohol abuse should be evaluated. (See SSR scriptions that may be encountered:
82-60, PPS No. 83, Titles II and XVI: Evaluation
of Drug Addiction and Alcoholism.)
• generics only—often no appeal process,
Documentation needed in the medical record: • generics preferred—appeal process available,
• quantity limits—appeal possible letter of
EEG—Corroborating the nature and frequency medical necessity,
of seizures; • formulary/nonformulary,
Detailed description of typical seizure pattern • tiering/levels of coverage: anywhere from 2–
including associated phenomena: 4 tiers with the expense going up with each
• Professional observation, tier,
27 Quality of Life in Epilepsy 351

• tier exception—often requires letter of medi- Medications Affordability


cal necessity requesting the medication be Medications are frequently too expensive for the
covered at a lower tier, patient to afford. It is difficult to accept that the
• percentage of cost, medication that is best for the patient is
• percentage of the total cost of the medication: unavailable to them because of cost. There are
20, 50%, programs that can assist patients with their
• co-pay plus percentage, medication costs. If the patient has no insurance
• co-pay plus the difference in cost of generic or at least no medication coverage, they might be
versus nongeneric, eligible for a pharmacy assistance program
• controlling the dose by maximum milligrams through the manufacturer. The same is true for
allowed/day—not always associated with those falling into the donut hole with Medicare.
FDA recommendations: The easiest method to determine what programs
– use a mixture of strengths so that no one are available to the patient is to access rxAssist.
“medication” equals more than the org. This Web site will advise of programs
allowed milligrams—Example, available and any necessary qualification. Dis-
– needing 1000 mg but insurance only count cards are also available on many medica-
allows 600 mg, and tions. The $4.00 programs offered by select
– Use 3–200 mg tabs plus 4–100 mg tabs. pharmacies often provide a very limited number
• limiting the number of tablets—this limitation of AEDs. A program called RxOutreach has
is the most difficult to comprehend, it may generic medications, including many controlled
require a maximum of two tablets per day, drugs at a price often cheaper than the usual
and it does not matter if they are 50 or insurance co-payment.
200 mg. It is important to understand this
concept as the dose is increased.
• This same concept is involved when
increasing doses. In order to get a larger dose,
References
it may be necessary to use a combination of
strengths merely to meet the number of
1. U.S. Department of Transportation Federal Motor
tablets restriction.
Carrier Safety Administration. Subpart E—physical
qualifications and examinations. § 391.41 Physical
The appeal process is often offered with par- qualifications for drivers. 2015. http://www.fmcsa.
ticular programs and you must be able to dot.gov/rules-regulations/administration/fmcsr/
fmcsrruletext.aspx?reg=391.41. Accessed 22 Oct
demonstrate the step therapy and titration 2015.
process. 2. Navigation and Vessel Inspection Circular NO.
Information that you should have ready any- 04-08, CH-1. Enclosure (8). 2015. http://www.uscg.
time an appeal is submitted: mil/hq/cg5/nvic/pdf/2008/NVIC%2004-08%20CH%
201%20with%20Enclosures%2020130607.
Accessed 22 Oct 2015.
• all previously tried AEDs, 3. United States Department of Transportation. Federal
• dates used, aviation administration: guide for aviation medical
• reason for termination, and examination. 2015. http://www.faa.gov/about/office_
org/headquarters_offices/avs/offices/aam/ame/guide/
• proof of previous use of the generic app_process/app_history/item18/l/. Accessed 22 Oct
formulation. 2015.
352 P.J. Ramey et al.

4. United States Department of Labor. Secretary of 5. Social Security Official Website. SSR 87-6:
Labor Thomas E. Perez. Disability resources. Job TITLES II AND XVI: THE ROLE OF
accommodations. http://www.dol.gov/dol/topic/ PRESCRIBED TREATMENT IN
disability/jobaccommodations.htm. Accessed 22 Oct THE EVALUATION OF EPILEPSY. http://www.
2015. socialsecurity.gov/OP_Home/rulings/di/01/SSR87-
06-di-01.html Accessed 22 Oct 2015.
8. Epilepsy Foundation. Stronger Together. http://www.
epilepsyfoundation.org/resources/newsroom/
pressreleases/Epilepsy-Foundation-And-Epilepsy-
Therapy-Project-Finalize-Merger-To-Create-Strong-
Websites—Further Reading Unified-Organization-To-Support-People-With-
Epilepsy.cfm.
9. Centers for Disease Control and Preventions. CDC:
6. Adult Epilepsy News—Get Adult Epilepsy Info. Life Saving Lives, Protecting People. Epilepsy. http://
between the lines. http://www.lifebetweenthelines. www.cdc.gov/epilepsy/projects.htm.
com/plan-ahead.html. 10. Epilepsy Foundation Middle & West Tennessee.
7. Epilepsy.com. An Epilepsy Therapy Project Initiative http://epilepsytn.pmhclients.com/index.php/
of the Epilepsy Foundation. http://www.epilepsy. programs/.
com/epilepsy/main_epilepsy.
Multiple Choice Questions for Part V

1. Computed tomography most reliably identi- C. Provide treatment for comorbid


fies which of the following epileptogenic depression
pathologies? D. Provide evidence for localization of
function and dysfunction
A. Hippocampal sclerosis
B. Focal cortical dysplasia 4. All of the following are true about MRI
C. Subacute stroke features of Dyke–Davidoff–Masson syn-
D. Low-grade glioma drome (DDMS), except?
E. Hippocampal atrophy A. Enlargement of the frontal sinus
B. Polymicrogyria
2. Magnetic source modeling refers to C. Falcine displacement
A. Modeling the cortical generators of D. Thickening of the skull vault
neuromagnetic events E. Capillary malformations
B. Modeling the microscopic neural cir-
cuitry that generates brain electrical 5. All of the following are associated with
signals intellectual deficiencies in individuals with
C. Modeling the various sources of inter- epilepsy, except:
ference in the MEG system A. Frequent episodes of status epilepticus
D. Modeling the anatomy of the brain from B. Focal epilepsy with a localizable focus
MRI C. Polytherapy
E. Modeling the magnetic properties of the D. Early onset of seizures
head and brain tissue E. West syndrome

3. The purpose of neuropsychological exami- 6. Selective amygdalohippocampectomy provides:


nation in epilepsy includes all of the fol-
lowing, except: A. Better neurocognitive outcome com-
pared to en bloc temporal resection
A. Assess cognitive difficulties resulting B. Worse neurocognitive outcome com-
from seizures or antiseizure medications pared to en bloc resection
B. Establish baseline of functioning for C. Its neurocognitive advantage is not well
systematic comparisons across time established
354 Multiple Choice Questions for Part V

D. Better but seizure freedom chances than C. Architectural dysplasia is the subtype of
temporal lobectomy FCD that is most often missed on MRI
E. Worse but seizure freedom chances than imaging
temporal lobectomy D. FCD is seldom seen with dual pathology
of HS
7. All of the following is true about E. In neonates, FCD is usually hyperintense
hemimegalencephaly, except? in T2-weighted images and hypointense
in T1-weighted images
A. White matter volume in enlarged hemi-
sphere is normal
12. Intracarotid amobarbital procedure (Wada
B. Hemispheric growth is hamartomatous
test) can be helpful in terms of:
C. There is agyria
D. There is polymicrogyria
A. Lateralizing seizure focus
E. There is lissencephaly
B. Lateralizing memory
C. Lateralizing language dominance
8. Where in the hippocampus are sclerotic
D. Predicting surgical outcome
changes typically found in MRI-negative
E. All of the above
(1.5T) temporal lobe epilepsy?
A. CA1 13. Which of the following is true regarding
B. Hippocampal tail diffusion MRI findings in the epileptogenic
C. CA3 zone during the ictal phase?
D. CA4
E. Subiculum A. During the ictal period, diffusion MRI
abnormalities are found equally in both
9. Neuromagnetic signals from the brain are of the gray and the white matter
the order of: B. Ictal onset is followed by an immediate
increase in the ADC signal
A. Nanoteslas
C. Following the short initial phase of ictal
B. Milliteslas
onset, the ADC signal is highest due to
C. Microteslas
vasogenic edema
D. Femtoteslas
D. With continued seizure activity, there is
E. Picoteslas
eventual cytotoxic edema and the ADC
signal increases
10. Which of the following is true about driving
E. During the postictal phase, if there is
restrictions in the USA?
irreversible injury due to seizure activity,
A. Uniform across all states the ADC signal decreases compared to
B. Minimal seizure freedom is 3 months neighboring tissue
C. Does not apply to provoked seizures
D. Excludes brief absence seizures 14. In drug-resistant epilepsy, deep brain stim-
E. None of the above ulation targets which of the following
structures?
11. Which of the following best characterizes
MRI findings in focal cortical dysplasia? A. Cingulate gyrus
B. Reticular formation of the thalamus
A. MRI findings can be detected in 90% of
C. Anterior nucleus of the thalamus
patients with FCD
D. Hippocampus
B. Transmantle sign is seen in patients with
E. Outer globus pallidus
cytoarchitectural dysplasia
Multiple Choice Questions for Part V 355

15. The antiepileptic effects of vagus nerve 18. Free consulting service on workplace
stimulation (VNS): accommodations is provided by:
A. FDA
A. Increases indefinitely over time until
B. Job accommodation network (JAN)
complete seizure freedom is achieved
C. “No epileptic left behind” association
B. Reaches its peak in 3 months
D. Accommodation for all office
C. Remains the same throughout the course
E. This service is not free
of treatment
D. Shows improvement over a  2 year
19. A patient who continues to have focal and
course as compared to the initial 3
secondary generalized seizures despite trying
months
7 different antiseizure medications
E. Is highest initially
A. Is likely to become seizure-free after
16. Which of the following is associated with trying more ASMs
poor verbal memory outcome after left B. Is unlikely to become seizure-free after
temporal lobectomy? trying more ASMs
C. Must be referred for presurgical
A. Left mesial temporal sclerosis on MRI evaluation
B. Adult seizure onset D. Is very likely to become seizure-free with
C. Left temporal hypometabolism on deep brain stimulation (DBS) therapy
FDG-PET E. 2 and 3
D. Poor preoperative scores on the Rey
Auditory Verbal Learning Test 20. Which of the following MRI techniques has
E. Poor preoperative scores on Boston the highest relative sensitivity for detection
Naming Test of mesial temporal sclerosis?

17. The typical SPECT pattern in a mesial tem- A. A.T2 relaxometry


poral lobe foci during the ictal (0–2 min B. Inversion recovery
from seizure onset) and subsequent C. Visual atrophy
peri/postictal (2–6 min from seizure onset) D. Inversion recovery + visual atrophy
period is characterized by which of the E. Hippocampal volumetry
following?
21. Which of the following percentages is the
A. During the ictal state, there is marked closest approximation of the false lateral-
ipsilateral hyperperfusion with a sur- ization rate for PET in patients with unilat-
rounding smaller region of eral temporal lobe epilepsy?
hypoperfusion
B. During the ictal state, severe ipsilateral A. 1–2%
hyperperfusion involves the entire tem- B. 5%
poral lobe C. 10%
C. During the peri-ictal/postictal state, sev- D. 20%
ere contralateral hypoperfusion involves E. 30%
the entire temporal lobe
22. Which of the following is true with PET
D. During the peri-ictal/postictal state, sev-
ere ipsilateral hyperperfusion involves imaging in temporal lobe epilepsy?
the entire temporal lobe A. Decreased monoamine oxidase-B ligand
E. During the ictal period, there may be binding in cases of pathologically proven
bilateral temporal lobe hypoperfusion temporal lobe epilepsy
356 Multiple Choice Questions for Part V

B. Ipsilateral to the seizure foci, there is 26. All of the following are common pitfalls in
increased l-opiate receptor binding, erroneously diagnosing epileptic lesions in
decreased r-opiate receptor binding, and normal hippocampi except?
decreased К-opiate receptor binding
C. Increased D2/D3 binding in the epilep- A. Vertically oriented hippocampus erro-
togenic temporal lobe neously suggests hippocampal dysplasia
D. Decreased benzodiazepine receptor B. The hippocampus is normally hyperin-
binding ipsilateral to the temporal tense on T1, misleading to a diagnosis of
epileptic foci bilateral MTS
E. Increased serotonin-1A (5-HT1A) C. Artifactual size asymmetry can easily be
receptor binding ipsilateral to the tem- caused by head rotation as the hip-
poral epileptic foci pocampus is large anteriorly and tapers
progressively posteriorly
23. All of the following are true about neu- D. Hippocampal sulcus remnant is a normal
ropsychological tests, except: variant that occurs in 10–15% of normal
hippocampi
A. Working memory can be assessed by
E. A choroid fissure cyst expands the
Digit Span Backwards
choroidal fissure and compresses the
B. Grooved Pegboard Test or Purdue Peg-
hippocampus, erroneously suggesting
board Test assesses visual memory
hippocampal atrophy
C. Set-shifting can be assessed by Trail
Making Test
27. Which of the following statements about
D. Stroop Color Word Test assesses exec-
MEG source modeling is incorrect?
utive function
E. Rey–Osterrieth Complex Figure Test
A. The problem of determining the cortical
assesses planning
generators of MEG activity is considered
an “ill-posed” inverse problem
24. Most common adverse events of vagus nerve
B. “Ill-posed” inverse problems have a
stimulation (VNS) includes:
unique solution
A. Tachycardia C. “Ill-posed” inverse problems have an
B. Bradycardia infinite number of possible solutions
C. Cough D. The “forward model” is a physical model
D. Hoarseness of neural sources in the brain and how
E. Asystole these sources generate electromagnetic
fields outside the head
25. All of the following are MRI findings of the E. The “head model” is a component of the
affected side in mesial temporal sclerosis forward model
except?
28. Which of the following is the most common
A. Loss of internal architecture of the
neoplastic lesion associated with new onset
hippocampus
seizures in older adults?
B. Collateral white matter atrophy of the
temporal lobe
A. DNET
C. Extratemporal enlargement of the
B. Ganglioglioma
caudate
C. Low-grade glioma
D. Extratemporal atrophy of the fornix
D. High-grade glioma
E. Dilation of the temporal horn
E. Cerebral metastasis
Multiple Choice Questions for Part V 357

29. Which of the following is true about efficacy D. Can be recorded with scalp EEG in the
of responsive neurostimulation (RNS)? absence of muscle artifact
E. Are seen only ictally
A. Seizure reduction in the 12 week blinded
phase of RNS is 17.3%
33. Which of the following is an appropriate use
B. Seizure reduction 5 months postimplan-
of CT in patients with epilepsy?
tation of RNS is 25%
C. 50% responder rate is 55% after two A. Acute emergency for evaluation of new
years onset seizures in patients with symp-
D. Seizure freedom reaches 75% tomatic causes.
E. None of the above B. Postoperative follow-up for tumor
recurrence.
30. All of the following are true regarding pos- C. Defining topographic relationships of
tictal diffusion changes except? epileptogenic lesions to functional cortex
and cortical white matter tracts.
A. If ictal activity subsides without causing
D. Detection of mesial temporal lobe
irreversible injury, a complete resolution
sclerosis.
with no residual diffusion change is
E. Pediatric new onset seizures without
noted.
apparent symptomatic cause.
B. During the immediate postictal period,
there is a decrease in the ADC.
34. A 45-year-old pilot experiences a brief epi-
C. ADC is increased with continued seizure
sode of “losing track of conversation” after a
activity due to neuronal cell death, acute
prolonged flight. His routine EEG reveals
inflammatory changes, and gliosis.
occasional left temporal rhythmic delta
D. During the immediate postictal period,
activity but is otherwise normal. His brain
there is decreased DWI signal in the
MRI is normal. The most appropriate coun-
epileptogenic area.
seling step is to:
E. If injury due to seizure activity pro-
gresses, the ADC increases compared to A. Have the pilot continue the regular flight
the preictal state and to the neighboring schedule
normal tissue. B. Restrict the pilot from long flights for at
least 6 months
31. Which of the following intracranial vascular C. Indefinitely stop piloting
malformations is most commonly associated D. Refer to psychiatry for possible malingering
with seizures? E. Offer few weeks of rest before resuming
regular flight schedule
A. Arteriovenous malformations
B. Cavernous hemangiomas 35. Which of the following is not true about
C. Epidural hematoma ADHD in individuals with epilepsy?
D. Venous angiomas
A. ADHD is seen in 20–40%
E. Capillary telangiectasias
B. In epilepsy, ADHD affects males and
females equally
32. Recent studies suggest that high-frequency
C. Temporal lobe epilepsy is associated
oscillations (HFOs) are:
with higher rates of attention problems
A. Most abundant in the frontal lobes than frontal lobe epilepsy
B. Are present in the epileptogenic zone D. Nocturnal seizures exacerbate inattention
C. Signify functional cortex and must not E. Stimulants do not lower the seizure
be resected threshold
358 Multiple Choice Questions for Part V

36. At present, MEG has an approved clinical C. May have an MRI after turning off the
indication for all of the following, except: magnet mode
D. May have an MRI after turning off the
A. Localizing cortical generators of interic-
normal and magnet mode
tal epileptic spikes in patients being
E. Refer for CT scan instead
evaluated for epilepsy surgery
B. Lateralizing language dominance prior to
40. For lateralizing language, what is the con-
brain surgery
cordance rate between neuromagnetic
C. Localizing somatosensory cortex prior to
responses to auditory language stimuli and
brain surgery
the Wada test?
D. Confirming a diagnosis of epilepsy in a
patient with negative EEG studies A. <10%
E. Localizing primary auditory cortex prior B. 25%
to brain surgery C. 50%
D. >70%
37. Magnetic source modeling refers to
41. Which of the following is true about side
A. Modeling the cortical generators of
effects of responsive neurostimulation
neuromagnetic events
(RNS)?
B. Modeling the microscopic neural cir-
cuitry that generates brain electrical A. Intracranial hemorrhages occurred at a
signals rate of 10%
C. Modeling the various sources of inter- B. Mood worsening was noted
ference in the MEG system C. Cognitive function worsened
D. Modeling the anatomy of the brain from D. Quality of life improved
MRI E. None of the above
E. Modeling the magnetic properties of the
head and brain tissue 42. Very frequent VNS magnet activation in the
setting of high-frequency stimulation
38. Which of the following statements represents (  50 Hz) can result in:
a limitation of MEG in the presurgical
evaluation of patients for epilepsy surgery? A. ON time  OFF time that does not
result in degenerative nerve damage in
A. MEG cannot accurately localize primary
laboratory animals and is safe
sensory cortices for functional mapping
B. ON time  OFF time is not technically
B. In patients with multifocal spikes, mag-
possible to induce
netic source modeling cannot accurately
C. ON time  OFF time that can result in
localize the events
degenerative nerve damage in laboratory
C. In some patients, no interictal spikes or
animals and therefore should be avoided
ictal events may be captured during the
D. OFF time  ON time that would be
MEG study
ineffective and therefore should be
D. Ictal rhythms cannot be localized using
avoided
magnetic source modeling methods
E. No effect
39. Which statement is correct about patients
43. A 34-year-old woman started having epi-
with vagal nerve stimulation (VNS)?
sodes of confusion and lip smacking.
A. May not have an MRI Her EEG showed left temporal spikes. She
B. May have an MRI after turning off the was initially started on levetiracetam but
normal mode failed to achieve good seizure control. In
Multiple Choice Questions for Part V 359

spite of adding lamotrigine, she continued to 47. A patient with history of prior left temporal
have occasional disabling seizures. Her hemorrhage and refractory left temporal
antiepileptic blood levels are therapeutic. lobe epilepsy may undergo temporal
The best next step is to: lobectomy:

A. Add clobazam A. After subdural grid electrode placement


B. Refer to RNS localizes seizure onset zone and lan-
C. Refer to VNS guage mapping is done
D. Admit for inpatient video EEG B. If the Wada test establishes that verbal
E. Start citalopram memory lateralizes to the right
C. Based on an MRI that confirms the
44. A 30-year-old female patient with chronic presence of encephalomalacia in the left
refractory temporal lobe epilepsy is typically lateral temporal area and ictal scalp EEG
more likely to become seizure-free after: that localizes seizures to the left temporal
region
A. Several attempt of polytherapy with D. If PET shows left temporal
combination of old and new generation hypometabolism
antiseizure medications (ASMs) E. If ictal SPECT shows seizure onset in the
B. Temporal lobectomy left temporal lobe
C. Repeated attempt of polytherapy with
synergistic generation ASMs 48. Which of the following is not a typical aura
D. Vagus nerve stimulation in insular seizures?
E. Responsive neurostimulation
A. Abnormal taste
45. Patients with intractable temporal lobe epi- B. Contralateral somatosensory symptoms
lepsy who have normal high-resolution brain C. Contralateral head version
MRIs: D. Laryngeal constriction
E. Visceral symptoms
A. Are not candidates for presurgical eval-
uation because of a possible poor 49. An adult patient with frequent nocturnal
outcome events soon after falling asleep, consisting of
B. Must undergo presurgical evaluation arousal from sleep and violent movements
C. Have better outcome than those with for 30 s with full recovery, most likely has:
abnormal brain MRI
D. Are unlikely to have hippocampal sei- A. Insular epilepsy
zure onset B. REM sleep behavior disorder
E. Are likely to have concomitant psy- C. Frontal lobe epilepsy
chogenic non-epileptic episodes D. Psychogenic non-epileptic seizures
E. Temporal lobe epilepsy
46. Which of the following is most associated
with an increased risk of suicidality in 50. Which of the following is true about corpus
patients with epilepsy? callosotomy?

A. Coexistent depression A. It does not help control atonic seizures


B. Higher absence frequency B. May worsen focal seizures
C. Male sex C. Akinetic mutism is not a complication
D. Lower quality of life D. Apraxia is not a complication
E. Generalized onset seizures E. Incontinence is not a complication
360 Multiple Choice Questions for Part V

51. Patients with extratemporal lobe epilepsy: 55. When comparing results of temporal lobec-
tomy to medical treatment in intractable
A. More likely need invasive recording
epilepsy, patients treated with surgery are
before epilepsy surgery
how many times more likely to be
B. More likely to be surgical candidates
seizure-free?
without invasive recording
C. More likely to need subdural recording A. 3 times
unless there are frontal lobe discharges B. 7 times
consistent with frontal lobe epilepsy C. 15 times
D. More likely to be surgical candidates D. 20 times
without invasive recording, only if brain E. 25 times
MRI shows a frontal lobe lesion
E. More likely to be surgical candidates 56. Which of the following is true about
without invasive recording, only if brain hemispherectomy?
MRI shows a parietal lobe lesion
A. About 25% of patients become
seizure-free after hemispherectomy, but
52. Which of the following is true regarding
intellectual function worsens
abdominal aura?
B. About 25% of patients become
A. It has a sensitivity of 90% for localizing seizure-free after hemispherectomy, and
seizures to the temporal lobe intellectual function improves
B. It has a specificity of 90% for localizing C. About 75% of patients become
seizures to the temporal lobe seizure-free after hemispherectomy, but
C. It indicates insular seizure onset intellectual function worsens
D. It is non-specific in localizing seizure D. About 75% of patients become
onset seizure-free after hemispherectomy, and
E. None of the above intellectual function improves
E. About 50% of patients become
53. Stimulation of either of superior and inferior seizure-free after hemispherectomy, with
cervical cardiac branches of the vagus nerve no change in intellectual function
during system diagnostics (lead test) may
cause: 57. When ictal theta activity is recorded, tem-
poral lobe epilepsy is present in:
A. Bradycardia and/or asystole
B. Tachypnea A. Less than 10%
C. Hiccups B. 10–25% of cases
D. Respiratory distress C. 25–50% of cases
E. Diarrhea D. 50–75% of cases
E. More than 75% of cases
54. Which antidepressant is least likely to
aggravate seizures? 58. Extrahippocampal abnormalities in patients
with temporal lobe epilepsy
A. Bupropion
B. Citalopram A. Are rare
C. Clomipramine B. Include atrophy of lateral temporal
D. Amoxapine structures
E. Venlafaxine C. Often include cortical dysplasia
Multiple Choice Questions for Part V 361

D. Correlate with favorable surgical out- achieve seizure freedom with the second
come if hippocampal atrophy is not antiepileptic drug?
present on presurgical MRI
A. 5%
E. None of the above
B. 15%
C. 25%
59. Which of the following is false about Ras-
D. 40%
mussen encephalitis?
E. 50%
A. Bilateral disease is rare
B. May present in adolescents or adults 64. Stimulation of the region anterior to the
C. Developmental delays do not precede premotor cortex will likely cause:
presentation
A. Salivation
D. Multilobar resections are a useful alter-
B. Eye deviation
native to hemispherectomy
C. Sensory derangement
E. Progressive hemiparesis will ensue
D. Behavioral change
E. Tinnitus
60. Which of the following psychiatric condi-
tions is most common in patients with
epilepsy?
A. Bipolar II Answers
B. Attention deficit disorder
C. Paranoid delusions 1. (C). CT scans provide better details about
D. Depression bony structures and can easily detect hem-
E. Post-traumatic stress disorder orrhages, calcifications, strokes after 24 h,
and large tumors. However, CT may fail to
61. Lifting driving restrictions of epilepsy recognize commonly encountered lesions in
patients operating large commercial vehicles patients with epilepsy such as hippocampal
require: atrophy, hippocampal sclerosis, cortical
dysplasias, or low-grade gliomas (LGGs).
A. Same rule as for private car
2. (A). Magnetic source modeling or magnetic
B. A minimum of two-year seizure freedom
source imaging (MSI) refers to the process of
C. A minimum of ten-year seizure freedom
generating best-fitting hypotheses about the
D. Being off antiepileptic drugs
cortical generators of recorded neuromag-
E. C and D
netic signals. Source modeling does not
attempt to model the microscopic neural
62. All of the following may exclude patients
circuitry that generates brain signals, but
from disability benefits except:
assumes elementary sources comprised of
A. Chronic alcohol consumption one or many current dipoles. Structural
B. Low antiepileptic blood levels models of the head and brain are requisites
C. Twice-weekly minor motor seizures for source modeling but not the objective.
D. Seizure freedom for more than three 3. (C). Unlike general psychological practice, a
months neuropsychologist does not necessarily
E. Nocturnal seizures diagnose psychiatric conditions or provide
treatment and may not assess specific voca-
63. In the newly diagnosed epilepsy, almost half tional skills (driving, interest inventory, etc).
patients will have seizure freedom with the 4. (B). Dyke–Davidoff–Masson syndrome
first antiepileptic drug. What percentage will (DDMS) presents with seizures, facial
362 Multiple Choice Questions for Part V

asymmetry, contralateral hemiparesis, and Driving restriction typically ranges between


mental retardation. Radiologic features 3 and 12 months.
include thickening of the skull vault, 11. (C). MRI is unrevealing in up to 34% of
enlargement of the frontal sinus and the patients, especially those with architectural
ethmoidal and mastoid air cells, elevation of dysplasia only. Transmantle sign is seen in
the petrous ridge, ipsilateral displacement of patients with Taylor-type dysplasia. FCD is
the falx, and capillary malformations. the most common dual pathology with HS.
Hemispherectomy may be recommended. The signal intensity of FCD varies with the
5. (B). Risk factors for intellectual deficiencies age of the patient. In neonates and young
in epilepsy include primary generalized epi- infants, FCD is usually hypointense in
lepsy, West syndrome, Lennox–Gastaut T2-weighted images and hyperintense in
syndrome, localization-related epilepsy but T1-weighted images. After white matter
seizure focus difficult to isolate; severe vol- myelination has reached significant levels
umetric abnormalities; early onset of epi- (24 months), the hyperintensity is easily
lepsy; frequent seizures and more episodes seen in FLAIR and T2-weighted images.
of status epilepticus; polytherapy; and 12. (E). The Wada test can help lateralize lan-
comorbid diagnoses (e.g., autism). guage and visual or verbal memory by
6. (C). Selective amygdalohippocampectomy inhibiting the function of one hemisphere at
provides similar seizure freedom chances to a time by amobarbital and assessing the
those of temporal lobectomy, but it is unclear function of the other hemisphere. The test
whether it is superior to lobectomy in terms also helps lateralize the seizure focus and
of neurocognitive outcomes; some studies predict surgical outcomes in terms of cog-
reported better outcomes and others reported nitive deficits as discussed in Chap. 25.
worse. 13. (C). During the ictal period, diffusion MRI
7. (A). Hemimegalencephaly refers to unilat- abnormalities are primarily found in the gray
eral hamartomatous excessive growth of all matter. Immediately at seizure onset, there is
or part of one cerebral hemisphere at differ- an increase in electrical activity, leading to
ent phases of embryologic development. increased cellular metabolism and subse-
MRI reveals an enlarged hemisphere with quent hyperperfusion. During this phase,
increased white matter volume, cortical there is no apparent change in the tissue
thickening, agyria, pachygyria, polymicro- microarchitecture detectable by diffusion
gyria, or lissencephaly and blurring of the MRI due to lack of significant edema. As
gray–white matter junction. Often, there is a seizure activity progresses, there is initially
large, ipsilateral irregularly shaped ventricle. vasogenic edema (peak of ADC signal
8. (D). In MR-negative MTLE, sclerotic chan- changes) followed by cytotoxic edema (de-
ges are limited to or predominately in the creased ADC activity). With irreversible
CA4 region (endfolium) of the Ammon’s neuronal injury, the ADC signal is increased
horn. compared to the preictal state and to the
9. (D). Neuromagnetic signals from the brain are neighboring normal tissue.
typically of the order of 10−15 T (femtotesla). 14. (C). Deep brain stimulation has shown effi-
MRI scanners have static field strengths of the cacy in patients with drug-resistant epilepsy
order of 1 T. The heart generates magnetic when targeted to the anterior nucleus of the
field of the order of a nanotesla. thalamus.
10. (B). Driving restrictions apply to both pro- 15. (D). Efficacy of VNS increases with time.
voked and unprovoked seizures. Each state Data showed improvement in seizure control
has specific regulations, and some leave the over a  2 year course as compared to the
decision to the discretion of the provider. initial 3 months of therapy.
Multiple Choice Questions for Part V 363

16. (B). Early age of onset increases the chances there is increased l-opiate receptor binding,
that the brain will functionally rearrange. increased r-opiate receptor binding, and
Mesial temporal lobe sclerosis on MRI and decreased К-opiate receptor binding. There
hypometabolism on PET suggest poor is decreased D2/D3 binding in the epilepto-
functioning and thus less risk of decline after genic temporal lobe. C-flumazenil PET is
surgery. Poor preoperative performance on very sensitive in MRI-negative temporal
naming and verbal memory makes suggest lobe epilepsy given reduced benzodiazepine
lower chances of further postoperative receptor binding. There is decreased
decline serotonin-1A receptor binding in patients
17. (A). During the ictal state, there is ipsilateral with TLE.
temporal hyperperfusion with surrounding 23. (B). Please refer to the table in Chap. 22.
severe hypoperfusion. In the peri-ictal state, 24. (D). The most common adverse effect of
there is a “postictal switch” during which VNS is hoarseness. Most of the VNS
there is severe ipsilateral hypoperfusion adverse effects have a negligible impact on
throughout the temporal lobe, with the the quality of life of treated patients and are
exception of persistent hyperperfusion in the reported as mild in most instances.
mesial temporal region. Bilateral temporal 25. (C). There is extratemporal atrophy of the
lobe hyperperfusion is only seen in seizures caudate.
of lateral temporal origin. 26. (B). The hippocampus is normally hyperin-
18. (B). The job accommodation network (JAN), tense on FLAIR, misleading to a diagnosis
a service of the Office of Disability of bilateral MTS. Amygdala and hippocam-
Employment Policy (ODEP), provides free pus are isointense on all other MR pulse
consulting services on workplace accom- sequences.
modations. Accommodations may be needed 27. (B). The problem of MEG or EEG source
at work or school depending on classic sei- modeling is considered an “ill-posed”
zure triggers, recommendations, and inverse problem. “Ill-posed” inverse prob-
restrictions. lems do not have unique solutions, but rather
19. (E). Medically intractable epilepsy is defined an infinite number of possible solutions.
as failure of two adequate trials of anti- 28. (E). Metastatic lesions are the most frequent
seizure medications to achieve seizure free- epileptogenic neoplasms in older adults.
dom. Patients who have failed to 29. (C). Seizure reduction in the 12-week blin-
medications must be referred for surgical ded phase of RNS was 37.9% compared with
evaluation since additional medication regi- 17.3% for sham. At 5 months postimplan-
mens will have very low chances of tation, 41.5% seizure reduction was seen in
achieving seizure freedom. the stimulation group compared to a 9.4%
20. (E). Hippocampal volumetry, sensitivity of reduction in the sham group. Two years after
97%. implantation of RNS, 50% responder rate
21. (A). Unrecognized ictal activity during FDG was reported in 55%.
injection could make temporal lobe con- 30. (D). During the immediate postictal period,
tralateral to the focus appear falsely depres- the DWI signal is increased in the epilepto-
sed. Prior depth electrode implantation could genic zone.
lead to contralateral hypometabolism. 31. (A). Seizures occur in 24–69% of arteri-
Increased interictal metabolism ipsilateral to ovenous malformations and 34–51% of
an epileptic focus has been described in large cavernous hemangiomas. The vast majority
cortical malformations. of capillary telangiectasias and venous
22. (D). Monoamine oxidase-B binding increa- angiomas are clinically silent. Epidural
ses with increasing gliosis in temporal lobe hematomas are not considered intracranial
epilepsy. Ipsilateral to the seizure focus, vascular malformations.
364 Multiple Choice Questions for Part V

32. (B). HFOs are oscillation in the gamma band 39. (D). Patients with vagal nerve stimulators
that are recorded with intracranial electrodes can undergo MRI imaging if both normal
at high digital sampling frequencies. They and magnet modes are turned off.
have been studied and found to be abundant 40. (D). For lateralizing language, neuromag-
in the epileptogenic zone ictally and interi- netic responses to auditory language stimuli
cally. Resection of the areas where HFOs are were found to be concordant with the Wada
seen correlated with better surgical outcome. test in 87% of patients. Other investigators
33. (A). MRI is indicated for all of the other found a concordance rate of 86% with the
answer choices. Wada test in 35 patients with a sensitivity of
34. (C). The clinical history and the described 80% and specificity of 100%. Several smal-
EEG abnormality (rhythmic temporal delta ler studies have reported MEG–Wada con-
activity) are both suggestive of partial (focal) cordance rates between 69 and 100% using a
epilepsy. Piloting an aircraft, either private variety of paradigms and analysis methods.
or commercial, is completely prohibited for 41. (D). In a 2-year follow-up study of RNS,
anyone with a history of seizures. 50% responder rate was reported in 55% of
35. (C). Prevalence of ADHD in epilepsy is 20– patents. Intracranial hemorrhages and infec-
40%. Inattentive presentation is more com- tions each occurred in about 2% of implan-
mon, and the boys and girls are equally ted patients, neither mood nor cognitive
represented, which is different from devel- function worsened, and quality of life
opmental ADHD with no seizures. There improved.
may be higher rates of attention problems 42. (C). When using high-frequency stimulation
with FLE and CAE. Associated issues like (>50 Hz), frequent VNS magnet activation
nocturnal seizures or medication side effects can result in nerve damage and thus should
may be the primary cause of inattention. be avoided.
Studies have shown that stimulants used for 43. (D). This patient likely has drug-resistant
ADHD symptoms do not lower seizure partial epilepsy of left temporal lobe origin.
threshold. He has failed two antiepileptic drugs at
36. (D). MEG is not clinically indicated at this therapeutic doses and appropriate drug
point for the evaluation of a patient with new levels. The next step is to consider epilepsy
onset seizures, or to confirm a diagnosis of surgery for which an inpatient video EEG
epilepsy. MEG and source modeling of study is part of the workup.
epileptic spikes or evoked responses is 44. (B). Temporal lobe resection has been
indicated as part a presurgical evaluation shown to be superior to continued medical
prior to epilepsy, tumor, or vascular surgery. therapy in refractory temporal lobe epilepsy
37. (A). Modeling the cortical generators of in a controlled trial. The rate of seizure
neuromagnetic event freedom with vagus nerve stimulation or
38. (C). MEG studies aimed at localizing inter- response for stimulation is much less than
ictal epileptic spikes may sometimes fail to that of lobectomy.
record any epileptic events for source mod- 45. (B). Although the surgical outcomes in
eling (*20% of cases) given the relatively non-lesional epilepsy are generally less
short duration of the recording. In these sit- optimal than those of lesional epilepsy, this
uations, it may not contribute to localizing must not deter the epileptologist from per-
epileptic dysfunction. Multifocality of spikes forming the surgical evaluation. In these
does not undermine the accuracy of the patients, sometimes intracranial monitoring
source modeling. Ictal MEG recordings are may help localize the seizure focus, and the
relatively infrequent, but early ictal rhythms surgical outcomes are often favorable. Hip-
can be modeled with good accuracy to pre- pocampal seizures occur in patients with
dict seizure onset zones. normal MRI, which can be confirmed with
Multiple Choice Questions for Part V 365

depth electrode recordings. In these patients, with akinetic mutism, incontinence, apraxia,
the pathology often shows neuronal loss in or the alien hand syndrome. It is thought that
the CA4 region of the Ammon’s horn. this is more likely if the entire corpus cal-
46. (A). Patients with epilepsy have higher risks losum is sectioned initially. For this reason,
of suicidality than the normal population. many prefer to do an anterior 2/3 section
This is mainly due to the coexistent depres- first.
sion. Antiepileptic adverse events have also 51. (A). In extratemporal lobe epilepsy, invasive
been linked to higher suicidal ideations in recordings are often used for localizing the
patients with epilepsy. seizure focus and mapping the brain, thus
47. (A). In such a patient, right hemispheric defining the surgical borders in a manner that
verbal memory lateralization improves sur- maximizes tissue resection and minimizes
gical candidacy in terms of memory out- functional deficits. While in frontal lobe
come, but is not enough because of the risk epilepsy the ictal EEG can be
language deficits if language is on the left. non-lateralizing, a lateralizing EEG or sei-
Ictal scalp EEG is not totally localizing. zure semiology is helpful primarily in guid-
Functional neuroimaging is important but ing the implantation of the intracranial
will not provide the details as regards the electrodes for further localization and map-
relationship of the seizure onset zone to the ping. Similarly, lesions are helpful in this
eloquent cortex as with intracranial regard, but do not obviate the need for
monitoring. monitoring as the seizure focus often is close
48. (C). Seizures from the insula can include to, though not within, the lesion, and nearby
visceral, gustatory, and somatosensory cortex can be eloquent.
symptoms, including laryngeal constriction 52. (B). Abdominal aura had a 52% sensitivity
or paresthesias. and 90% specificity for localizing seizures to
49. (C). This presentation is typical of frontal the temporal lobe (Velasco, T. R. and
lobe epilepsy. Insular and temporal lobe Mathern, G. W. in Wyllie’s Treatment of
epilepsy may have similar presentations, but Epilepsy: Principles and Practice (ed Cas-
they are much less frequent causes of this cino GD Wyllie E, Gidal BE, Goodkin HP)
seizure semiology. Although non-REM Ch. 82, 922–936 (Wolters Kluwer/
parasomnias may have similar presenta- Lippincott Williams & Wilkins, 2011).
tions, REM sleep behavior disorder presents 53. (A). The VNS lead electrodes must be placed
differently. The brief duration of the seizure, below where the superior and inferior cervi-
its stereotypical nature, nocturnal occur- cal cardiac branches separate from the vagus
rence, and the full recovery make psy- nerve. Stimulation of either of these two
chogenic seizures less likely. branches during the system diagnostics (lead
50. (B). Corpus callosotomy has been particu- test) may cause bradycardia and/or asystole.
larly helpful for atonic, “falling,” seizures as 54. (B). Selective serotonin reuptake inhibitors
well as for tonic seizures, with elec- (such as citalopram) are the preferred
trodecrement at seizure onset, and for gen- first-line antidepressant therapy for patients
eralized tonic–clonic seizures. While with epilepsy. The other listed drugs may
“falling” seizures may benefit, other seizure aggravate seizures especially at higher doses.
types may remain, so generally this should 55. (B). In the controlled trial of temporal
be thought of as a palliative rather than lobectomy versus continued medical treat-
curative procedure. Focal seizures can ment in intractable temporal lobe epilepsy,
become more severe after section, and in one year later 58% of patients with surgical
experimental models, kindling can occur treatment but only 8% of patients with
more rapidly. There can be an acute dis- medical treatment alone were free from epi-
connection syndrome after callosal section, sodes with loss of consciousness.
366 Multiple Choice Questions for Part V

56. (D). After hemispherectomy, 70–80% of bidirectional, and depression may precede
patients become seizure-free. Because of the epilepsy and is considered to be a risk factor
reduction in seizures, intellectual function for epilepsy. In addition, depression is often
often improves. associated with anxiety symptoms or
57. (E). Ictal theta range organized spike dis- full-blown anxiety disorder.
charge was found to have an 85% probabil- 61. (E). A diagnosis of epilepsy and the use of
ity for temporal lobe epilepsy and was 80– antiepileptic medications (AEDs) preclude
94% correct with respect to side of seizure the individual from driving a commercial
onset. motor vehicle. In order to even be consid-
58. (B). Quantitative analysis of MRIs in ered for reinstatement of commercial dri-
patients with temporal lobe epilepsy shows ver’s licensing (CDL), an individual with
frequent atrophy in medial as well as lateral epilepsy has to be off AED(s) and
temporal structures. A non-sclerotic hip- seizure-free for at least 10 years.
pocampus correlated with less optimal sur- 62. (C). Patients with epilepsy may be granted
gical outcome after lobectomy than short- or long-term disability if their seizures
hippocampal sclerosis. remain uncontrolled despite appropriate
59. (D). Rasmussen encephalitis (RE) is a rare, therapy, strict compliance, and absence of
progressive, chronic disease characterized by known controlled triggers such as alco-
seizures, progressive hemiparesis, and cog- holism. In the particular case of motor sei-
nitive loss. It occurs mainly in children with zures, major motor seizures must be
a peak of incidence at the age of 6–7 years. occurring more frequently than once a month
However adolescent and adult cases have and minor motor seizures must be occurring
been reported accounting for about 10% of more frequently than once weekly.
all cases of RE. RE occurs usually in healthy 63. (B). The first antiepileptic drug achieves
children, adolescents, and adults. RE usually seizure freedom in 47% of patients, and the
affects only one hemisphere of the brain; second antiepileptic drug achieves seizure
bilateral disease is very rare. freedom in only 14%.
60. (D). Depression affects at least one-third of 64. (B). During electrocorticography, stimula-
patients with epilepsy and is considered to be tion of the frontal eye fields (Brodmann area
the most common psychiatric comorbidity 8) which is situated just anterior to the pre-
that needs early identification and treatment. motor cortex, will likely cause contralateral
The relationship of seizure and epilepsy is eye deviation.
Index

Note: Page numbers followed by f and t indicate figures and tables, respectively.

A Anti-N-methyl-D-aspartate receptor (NMDA-R)


Absence epilepsy/seizures, 99–101 antibodies, 178
atypical, 100–101 Antiseizure medications (ASM), 317, 347
childhood, 155–156, 162, 194 Aphasia, 180, 299, 332
juvenile, 174 postictal, 143, 150
GLUT1 deficiency and, 187 Apnea–hypopnea index, 248
myoclonic, 101, 101f, 156 Apparent diffusion coefficient (ADC), 289
typical, 99–100, 99f, 100f Array-comparative genomic hybridization (aCGH), 171
Academic functioning, neuropsychological evaluation of, Arteriovenous malformations (AVMs), 184, 263, 281
298 Artifacts, 24, 26–32f, 33–53
Accommodations, 349–350 physiologic, 45–50, 46–52f
Action potential, 3–5 Atonic seizures, 107
Aircraft pilots, 348–349 Attention, neuropsychological evaluation of, 295t,
Alpha rhythms, 13 297–298
Alpha squeak, 33, 35f Autoimmune epilepsy, 177–178, 178t
Alpha-theta coma, 134, 134f Autosomal dominant nocturnal frontal lobe epilepsy
Alpha variant, 33, 34f (ADNFLE), 158, 174, 188
Amoxapine, and status epilepticus, 242, 243, 243 t Autosomal dominant parietal epilepsy with auditory
Animal models of epilepsy, 242–243, 243t, 253 features (ADPEAF), 158
Anterior cingulate seizures, 111
Anterior temporal lobectomy (ATL), 320f
neurocognitive deficits and risk factors following, 322 B
outcome following, in nonlesional TLE, 326 Band-pass or stop filter, 22
standard, 322 Barbiturates
Antiepileptic drug selection and alpha-theta coma, 134
factors influencing, 205 and beta coma, 134–135
studies on, 208 burst suppression pattern, 131f
Antiepileptic drugs (AEDs), 179 Baseline shifts, in EEG, 9, 10f
chronological order of, 207ff Bell’s phenomenon, 45
cross-sensitivity rates, 211t Benign EEG variants, 33–53
drug–drug interactions, 249 Benign epilepsy with centrotemporal spikes (BECTS), 84,
indication, 210t 143, 164
investigational, 241, 244 Benign epileptiform transients of sleep (BETS). See
level of evidence, 209t Benign sporadic sleep spikes (BSSS)
mechanism of action, 205, 207t Benign myoclonus of infancy, 167–168
new-generation, 225–233 Benign neonatal seizure syndromes, 65t, 67
old-generation, 213–223 Benign rolandic epilepsy. See Benign epilepsy with
P450 enzyme modulation of, 211t centrotemporal spikes (BECTS)
prophylaxis, 179 Benign sporadic sleep spikes (BSSS), 40, 40f
selection (see Antiepileptic drug selection) Benzodiazepines, 231–233
side effects of, 208t and alpha-theta coma, 134
spectrum of activity, 206t and beta coma, 134–135
stopping, 212 burst suppression pattern, 131f
Anti-NMDA receptor encephalitis, 193 for electrical status epilepticus in sleep, 157

© Springer Science+Business Media LLC 2017 367


M.Z. Koubeissi and N.J. Azar (eds.), Epilepsy Board Review,
DOI 10.1007/978-1-4939-6774-2
368 Index

metabolism, 232t Depression, spreading, 12


Beta coma, 134–135, 135f Developmental venous anomalies (DVAs), 184, 281
Bone health, seizure and, 251 Diazepam, 241
Brain abscess, 285t for status epilepticus, 119, 119t, 121t
Brain rhythms, substrates of, 12–13 Dietary therapies
Brain tumors, 179 complications of, 238
magnetic resonance imaging for, 273, 282f, 282t, 283f duration of, 238
Breach rhythm, 45, 45f future directions of, 239
Breast-feeding, seizure and, 250 history of, 239
Breath-holding spells, 168 ketogenic diet, 235
Brivaracetam, 241 low-glycemic-index treatment, 237, 239
Burst suppression pattern, 130–131, 131f mechanisms of, 238
medium chain triglyceride diet, 236
modified Atkins diet, 236–237, 239
C outcomes of, 239
Canavan disease, 78 patient selection, 237–238
Cannabidiol, 241–242 types of, 235–236
Carbamazepine, 217–218 Diffuse sweat artifact, 51f
pharmacokinetic parameters of, 226t Diffusion Tensor Imaging (DTI), 291
Catamenial epilepsy, 249 Diffusion-Weighted Imaging (DWI), 289, 290t
Cavernous malformations (CMs), 186 Digital Subtraction Angiography (DSA), 282
Cerebral cavernous malformations (CCMs), 281 Digitization artifacts, 51
Cerebral cortex, organization of, 3 Dipole, 7, 8f
Cerebral development, 55 Disability, 350
Channels, 56 Divalproex, 219–220
Charlie Foundation, 239 Do No Harm, 235
Chewing artifact, 50, 50f Doose syndrome. See Myoclonic-atonic seizures, epilepsy
Childhood absence epilepsy (CAE), 155–156, 163, 197 with
Children, epilepsy surgery in, 333 Dorsolateral frontal seizures, 111
Ciganek rhythm. See Midline theta rhythm Dravet syndrome, 72, 141, 149, 187
Clinical Laboratory Improvement Act (CLIA), 172 Driving, 347
Clinical validity, 172 Drug-resistant epilepsy, 210–211, 313, 317
CNS tuberculosis, 285t Dyschronism/dysmature, 63
Cognition, mapping, 300–301 Dysembryoplastic Neuroepithelial Tumors (DNET), 277,
Coma, 127–135 280t, 281
alpha-theta, 134, 134f
beta, 134–135, 135f
spindle, 133, 133f E
Commercial truck drivers, 347 Early infantile epileptic encephalopathy (EIEE), 150
Computed Axial Tomography (CAT), 273 Early Randomized Surgical Epilepsy Trial (ERSET), 320
Computed Tomography (CT), 273 Elderly, epilepsy in, 248–249
Continuous high voltage polymorphic delta (PDA), 129, Electrical status epilepticus during sleep (ESES), 86, 157
130f Electrocardiographic artifact, 48–50, 49f
Continuous spike and wave during sleep (CSWS), 164, Electrocerebral inactivity (ECI), 131–132, 132f
190 Electrode artifacts, 51, 53f
epileptic encephalopathy with, 157 Electroencephalogram (EEG), 289
Convulsive psychogenic seizures, 188 acquiring, filtering and displaying signals, 17–22,
Copy number variations (CNVs), 171, 172 20–22f
Corpus callosotomy, 343, 344 artifacts, 24, 26–32f, 33–53
Creutzfeldt-Jakob disease (CJD), 135, 136f baseline shifts, 9, 10f
Cryptococcosis, 285t brain rhythms, substrates of, 12–13
Ctenoids. See 14 -and 6-Hz positive bursts digital acquisition, processing and display, 22, 24f, 25f
Cyclosporine, and status epilepticus, 117 focal activity, 11, 11f
focal epilepsy, 109–112
generalized epilepsy, 93–107
D generalized tonic–clonic activity, 11, 12f
Delta brushes, 59–60, 61f generator, 5, 6, 6–8f
de Morsier syndrome, 180 grounding, 24
Depolarization, 4 low-voltage slow non-reactive, 132
Index 369

montages, 16–17, 17–19f failed surgery and reoperation, 332


neonatal (see Neonatal EEG) frontal lobe epilepsy surgery, outcome following,
normal variants and artifacts, 33–53 329–330
and patient safety, 24 insular-opercular seizures, 333
physiologic basis of, 3–13 intracarotid amobarbital procedure, 318
recordings, 15, 19f, 86 methods, 321
sampling rate, 24 multiple subpial transection, 331
sources of, 15, 16f neuropsychology, 318
wave generation, 9, 10f nonlesional partial epilepsy, outcome following,
Electromyographic artifact, 48, 49f 325–326
Environmental artifacts, 51 occipital lobe epilepsy, 332–333
Epilepsia partialis continua, 117 overall seizure-free outcome, 331
Epilepsy. See also Seizures parietal lobe epilepsy, 332
absence (see Absence epilepsy/seizures) presurgical evaluation, 317–318
animal models of, 243, 243t, 253 refractory epilepsy, 317
autoimmune, 177–178, 178t refractory partial epilepsy, treatment options for, 317
autosomal dominant parietal epilepsy with auditory supplementary motor area seizures, 330–331
features, 158 temporal lobe surgery, 322
childhood absence, 188 warranty of, 318–320, 319f, 320f
drug-resistant, 313 Epileptic encephalopathies, 171
epidemiology of, 203 Epilepticus, status, 115–124
etiology of, 204 Equivalent current dipoles (ECD), 303, 307
familial focal epilepsy with variable foci, 158 Eslicarbazepin, 232–233
focal (see Focal epilepsy) Estrogen, 249–250
frontal lobe (see Frontal lobe epilepsy (FLE)) Ethosuximide, 220–221
generalized (see Generalized epilepsy) for electrical status epilepticus in sleep, 157
with generalized tonic–clonic seizures, 157 pharmacokinetic parameters of, 214t
genetic analysis of, 171–176 Etomidate speech and memory test (eSAM), 298–299
imitators of, 167–170 Excitatory postsynaptic potential (EPSP), 4, 5f, 6, 7f, 9,
juvenile absence, 157 10f
juvenile myoclonic, 157, 191 Executive functions, neuropsychological evaluation of,
management in special populations, 249–251 305–306
with myoclonic absences, 156 Extratemporal lobe epilepsy (ETLE), 111, 333
with myoclonic-atonic seizures, 156, 199 Eye movements, 45
myoclonic epilepsy of infancy, 78 horizontal, 46f
neuroimaging in, 275–291 oblique, 45, 47f
neuropsychological evaluation in, 295–299 upward, 48f
progressive myoclonic, 188 vertical, 46f
progressive myoclonus, 197 Ezogabine, 232
quality of life in, 347–351
reflex, 158
refractory (see Refractory epilepsy) F
stroke and, 182–183 Faciobrachial dystonic seizures (FBDS), 178
sudden unexplained death in epilepsy, 94 Familial focal epilepsy with variable foci, 158
symptoms of, 337 Familial lateral temporal lobe epilepsy, 187
temporal lobe (see Temporal lobe epilepsy (TLE)) Familial mesial temporal lobe epilepsy (FMTLE), 158
temporal plus, 111 Faster rhythms, 13
Epilepsy surgery, procedures and outcomes in, 339–344 Febrile seizures, 174t
functional localization, 340 .recurrence of, 163
invasive evaluations, 339–340 Federal Motor Carrier Safety Administration, 347
noninvasive evaluations, 338–339 Felbamate, 225–226
symptoms, 338 pharmacokinetic parameters of, 226t
temporal lobe surgical resection, 340–342 Fertility, seizure and, 249
Epilepsy surgery assessment and testing, 317–333 Field potential (FP), 7f, 11
ATL in nonlesional TLE, outcome following, 326 Filter
case presentation, 318, 323–328, 323–328f band-pass or stop filter, 22
in children, 333 low-pass, 18, 20f, 23f, 24f
Early Randomized Surgical Epilepsy Trial, 320 high-pass, 18, 21–22f
370 Index

First seizures Generalized tonic–clonic (GTC) seizures, 11, 11f, 93–98,


provoked, 203 242
treatment decision after, 204 ictal EEG in, 98
unprovoked, 203–204, 204t, 205t post-ictal state, 94–98, 94–98f
Flumazenil, and status epilepticus, 117 Genetic absence epilepsy rats from Strasbours (GAERS),
Fluorescent in situ hybridization (FISH), 171 242, 243t
Flux transformers, 302 Genetic analysis of epilepsies, 171–176
Focal activity, 11, 11f clinical application of, 173–176, 174t, 175t
Focal cortical dysplasias (FCDs), 181 utility of, 172–173
classification of, 279t Genetic generalized epilepsies, 171
Focal epilepsy Genetic Information Non-Discrimination Act (GINA),
EEG and semiology in, 109–113 172
extratemporal lobe seizures, 111 Glioblastoma multiforme (GBM), 179, 280t
frontal lobe seizures, 87 Glossokinetic artefact, 50, 50f
ictal EEG in, 87, 88f, 89f Glucose transporter type I deficiency syndrome (Glut1
insular lobe seizures, 112 DS), 77, 187, 191, 192
occipital lobe seizures, 112 and absence epilepsy, 194
parietal lobe seizures, 112 Goldman–Hodgkin–Katz equation, 4
temporal lobe seizures, 109, 110t Gradiometers, 302
Focal spikes, features of, 83
Folinic acid responsive seizures, 65
Forward model, 303 H
Fosphenytoin, 217 Head model, 303
14 -and 6-Hz positive bursts, 36, 39, 39f Head trauma, 183
Frontal arousal rhythm (FAR), 42, 42f childhood, 183
Frontal intermittent rhythmic delta activity (FIRDA), 98, severity of, 182t
128 Hemimegalencephaly (HMEG), 180
Frontal lobe epilepsy (FLE), 103, 338 Hemispherectomy, 343
autosomal dominant nocturnal, 158, 188, 190 Herpes simplex encephalitis, 285t
surgery, outcome following, 325–326 High-frequency filter. See Low-pass filter
Frontal sharps, 60, 61f, 79 High-pass filter, 18, 21–22f
Functional localization, 340 Hippocampal sclerosis. See Mesial temporal sclerosis
Functional MRI (fMRI), 291 (MTS)
Hormonal therapy, 243
Hormones, 249
G Hyperekplexia, 197
Gabapentin, 227, 250 Hyperpolarization, 11
pharmacokinetic parameters of, 226t Hypothalamic hamartoma (HH), 159, 280t, 281
Galvanic artifact, 50, 51f Hypoxic ischemic encephalopathy, 79
Ganaxolone, 243 Hypsarrhythmia, 79
Ganglioglioma, 280t
Gelastic seizures, 159
Generalized epilepsy, 105–106t I
absence seizures, 99–101, 99–101f Ictal EEG, 86
atonic seizures, 103f, 107 in focal epilepsy, 87, 88f, 89f
ictal EEG in, 87, 89–90, 142 in generalized epilepsy, 87, 89–90, 149
idiopathic (see Idiopathic generalized epilepsy) in generalized tonic–clonic seizures, 98
myoclonic seizures, 101–107, 102–104f neonatal, 66–67f, 68
symptomatic (see Symptomatic generalized epilepsy) Idiopathic epilepsy syndromes, genes identified in, 174t
tonic–clonic (see Generalized tonic–clonic Idiopathic generalized epilepsy, 87, 88–89, 231
(GTC) seizures) JME, 85, 89–90
Generalized epilepsy with febrile seizures plus (GEFS+), Ifosfamide, and status epilepticus, 117
78, 149, 163, 194 Imipramine, and alpha-theta coma, 134
Generalized interictal epileptiform discharges, 94 Imitators of epilepsy, 167–170
Generalized periodic epileptiform discharges (GPEDs), Immune modulatory therapy, for electrical status
130, 131f epilepticus in sleep, 157
Index 371

Immunotherapy, 243–244 epileptic encephalopathy with, 157


Infantile gastroesophageal reflex disease, 199 Language, neuropsychological evaluation of, 297
Infantile spasms, 80 Lateralized periodic discharges (LPD), 182
Infectious disorders, magnetic resonance imaging for, Lateral rectus artefact, 48, 49f
282–284, 285t, 286f Left temporal lobectomy (LTL), 325
Inflammatory disorders, magnetic resonance imaging for, Lennox-Gestaut syndrome (LGS), 86, 156, 189
282–284, 286f ictal EEG in, 87, 90f
Inhalational agents, for refractory status epilepticus, 124 Levetiracetam, 229, 241
Inhibitory postsynaptic potential (IPSP), 4, 5f, 6, 7f for juvenile myoclonic epilepsy, 157
ripples, 13 pharmacokinetic parameters of, 226t
Instrumental artifacts, 51, 52f for refractory status epilepticus, 122
Insular lobe seizures, 112, 146 for status epilepticus, 122, 122t
Insular-opercular seizures Lissencephaly (LIS), 190, 277–279, 279f
surgery assessment and testing for, 333 Lithium, and status epilepticus, 117
Intellectual and developmental disability (IDD) Living with epilepsy, 347–351
defined, 247 Lorazepam
epilepsy in, 247–248 for refractory status epilepticus, 122
Intelligence, neuropsychological evaluation of, 294–295 for status epilepticus, 121–122, 122t, 122t
Inter-burst interval (IBI), 57–58 Low-frequency filter. See High-pass filter
Interictal epileptiform discharges (IEDs), 83–84, 84f, 143 Low-glycemic-index treatment (LGIT), 237, 239
generalized, 85–86, 86f Low-pass filter, 20f, 23f, 24f
Intermittent rhythmic delta activity (IRDA), 128 Low-voltage slow non-reactive EEG, 132
International league against epilepsy (IALE), 317 Lymphoma, primary central nervous system, 179, 285
Intracarotid amobarbital test (IAT), 298–299, 318
Intracranial EEG recording, filtering in, 22
Intractable epilepsy, 317 M
Invasive evaluations of epilepsy, 339–340 Magnetic resonance imaging (MRI), 273–284
Inverse problem, 302 for brain tumors, 277–281, 280f, 280t, 281f
Irritative zones, 305 functional, 291
Isaac syndrome, 177 for infectious disorders, 282–284, 285t, 284f
Isoniazid, and status epilepticus, 117 for inflammatory disorders, 282–284, 286f
for mesial temporal sclerosis, 274, 276–277f
for neuronal migrational disorders, 274–277, 278f,
J 279t
Japanese encephalitis, 285t for phacomatoses, 284, 288t, 287f
Jerks artifact, 50–51 for traumatic brain injury, 284, 287t, 288f
Juvenile absence epilepsy (JAE), 157 vagus nerve stimulation warning with, 313
Juvenile myoclonic epilepsy (JME), 157, 174, 191, 258 for vascular malformations, 281–282, 283f, 284f
Magnetic resonance spectroscopy (MRS), 290–291
Magnetic source imaging (MSI), 289, 302–303
K of epileptic activity and evoked responses, 303–304,
Karyotyping, 171 304–305f
K-complex rhythms, 12 Magnetic source modelling. See Magnetic source imaging
Ketamine, for refractory status epilepticus, 124 (MSI)
Ketogenic diet (KD), 235–236 Magnetically shielded room (MSR), 301
Ketosis, 235, 236 Magnetoencephalography (MEG), 6f, 287, 287f
Kindling model, 242, 243t current clinical methodologies, limitations of, 306
Krebs cycle, 238 equipment and recording, 302
history of, 301
role in presurgical evaluations, 305
L Magnetometers, 302
Lacosamide, 230, 241 Malformations of cortical development (MCDs),
for refractory status epilepticus, 122 180–182, 180f, 181f, 181t
for status epilepticus, 122 Mapping cognition, 298–299
Lambda waves, 42, 43f Maximal electroshock (MES) model, 242, 243t
Lamotrigine, 227–229, 242 Medical record, need of documentation for, 350–351
for juvenile myoclonic epilepsy, 157 Medications affordability, 351
pharmacokinetic parameters of, 226t Medium chain triglyceride (MCT) diet, 236
Landau–Kleffner syndrome (LKS), 331 Membrane potential (MP), 3–5, 4t, 5f, 7f, 11f
372 Index

Memory, 297 etiology of, 64t


Meningioma, 280t folinic acid responsive, 65
Mental retardation (MR), 247 long-term outcome of, 68
Meprobamate, and alpha-theta coma, 134 management of, 68
Merchant mariners, 348 pyridoxine-dependent, 65
Mesial temporal lobe epilepsy (MTLE), 320 risk factors of, 64
long-term surgery for, 321 Nernst equation, 3
trans-middle temporal gyrus approach, 322 Neurocardiogenic syncope, 169
Mesial temporal sclerosis (MTS), 183–184, 274 Neurocutaneous syndromes, magnetic resonance imaging
long-term surgery for, 321 for, 286, 286t, 286f
magnetic resonance imaging for, 273, 275–277f Neurocysticercosis, 285t
Metastases, 280t Neuroimaging, in epilepsy, 273–290
Methylazoxymethanol acetate (MAM) rat model, 242, computed axial tomography, 273
243t computed tomography, 273
Midazolam, 241 diffusion tensor imaging, 289
for refractory status epilepticus, 123, 123t diffusion weighted imaging, 290, 290t
for status epilepticus, 120t, 121, 121t functional MRI, 291
Midline theta rhythm, 33, 36, 36f magnetic resonance imaging (see Magnetic resonance
Migraines, 168 imaging (MRI))
Mitten pattern, 42, 44f magnetoencephalogram, 287, 288f
Modified Atkins diet (MAD), 236–237, 239 positron emission tomography, 284–287, 288f, 288t
Montages, 15–17, 17f, 18f of status epilepticus, 119, 119t, 120f
neonatal, 55–56, 56f subtraction ictal SPECT co-registered to MRI,
Multifocal epileptiform discharges, 80 286–287
Multilobe resections, 343 Neuromagnetic signals, 301
Multiple Subpial Transection (MST), 331, 343 Neuromodulation, 390–314
Multiplex ligation probe amplification (MLPA), 172 Neuronal migrational disorders (NMDs)
Mu-rhythm, 42, 43f lissencephaly (see Lissencephaly (LIS))
Mutations, 171 magnetic resonance imaging for, 274–277, 278f, 279t
Myoclonic absences, epilepsy with, 156 NeuroPace, 313
Myoclonic-atonic seizures, epilepsy with, 156, 164 Neuropsychological evaluation, 338, 293–299, 295–296t
Myoclonic epilepsy academic functioning, 298
of infancy, 72 attention/executive functions, 297–298
with ragged red fibres (MERRF), 86 components of, 293–294
Myoclonic seizures, 101–107, 102f intelligence, 296–297
absence, 101, 101f language, 297
clonic, 103 mapping cognition, 298–299
tonic, 104, 103f, 104f memory, 297
psychomotor/reaction time, 298
psychosocial, 298
N purpose of, 293
Neonatal EEG, 55–69 visuospatial skills, 298
abnormalities in voltage, 59, 60f Neuropsychology, 318
asymmetry, 62 Newborn EEG. See Neonatal EEG
developmental landmarks, 62f Non-convulsive status epilepticus (NCSE), 115–117, 156
dyschronism/dysmature, 63 prevalence of, 117t
ictal pattern, 68, 66–67f refractory, 124
montage, 55–56, 56f Nonepileptic paroxysmal disorders, 167–168
normal developmental landmarks, 56–57 Noninvasive evaluations of epilepsy, 338–339
normal graphoelements, 59–62, 61f Nonlesional partial epilepsy, outcome following, 325–326
positive sharp waves, 62 Non-physiologic artifacts, 51–53, 52–53f
sharp transients, 63, 63f, 63t Normal graphoelements, 59–62, 61f, 62f
technical information, 55 Nyquist sampling theorem, 22, 24f
temporal theta, 62
timing of, 56
trace continue, 57–59, 57–59f O
Neonatal non-epileptic events, 67 Occipital intermittent rhythmic delta activity (OIRDA),
Neonatal seizures, 64 128
clinical manifestations of, 65 Occipital lobe epilepsy, 112, 151
Index 373

surgery assessment and testing for, 331–332 inhibitory (see Inhibitory postsynaptic potential
Ohtahara syndrome. See Early infantile epileptic (IPSP))
encephalopathy (EIEE) Post-traumatic epilepsy, 181
Oligodendroglioma, 280t Pregabalin, 230
Orbitofrontal seizures, 111 Pregnancy, seizure and, 250
Oxcarbazepine, 218–219, 241 Presurgical evaluation, 317–318
pharmacokinetic parameters of, 214t Primary CNS lymphoma (PCNSL), 179
Primidone, 214–215
Progesterone, 249–250
P Progressive Myoclonic Epilepsy (PME), 157–158, 163,
Pallister–Hall syndrome, 159 175t, 188, 196
Parasomnias, 168–169 Propofol, for refractory status epilepticus, 123, 123t
Parietal lobe epilepsy, 112, 145 Psychogenic Nonepileptic Seizures (PNES), 169–170,
surgery assessment and testing for, 332 182
Paroxysmal Depolarization Shift (PDS), 11, 83 Psychogenic seizures
Pentobarbital, for refractory status epilepticus, 123–124, convulsive, 163, 191
123t nonepileptic, 169, 195
Pentylenetetrazole (PTZ) model, 123, 123t remission in, 191
Perampanel, 232, 241 Psychomotor/reaction time, neuropsychological
Periodic Lateralized Epileptiform Discharges (PLEDs), evaluation of, 298
84–85, 85f, 182 Psychosocial domain, neuropsychological evaluation of,
Periventricular nodular heterotopia (PVNH), 180–181, 298
181t Pyridoxine-dependent seizures, 65
Phacomatoses, magnetic resonance imaging for, 284,
286t, 286f
Phantom spike-and-wave bursts. See 6-Hz Q
spike-and-wave bursts Quality of life, 347–352
Pharmacodynamics, 223 accommodations, 349–350
Pharmacokinetics, 225, 249 aircraft pilots, 348–349
Pharmacoresistant epilepsy, 317, 319 commercial truck drivers, 347
Phenobarbital, 213–214 disability, 350
pharmacokinetic parameters of, 214t driving, 347
for status epilepticus, 122, 122t medical record, need of documentation for, 350–351
Phenylethylmalonamide (PEMA), 214 medications affordability, 351
Phenytoin, 246–248 merchant mariners, 348
pharmacokinetic parameters of, 214t restrictions, 347
for status epilepticus, 122–123, 122t seizure triggers, 349
Photoepileptiform discharges, 86
Photoparoxysmal response, 86
Physiologic EEG artifacts, 45–51, 46–51f R
Physiologic movements artefact, 50–51, 52f Rabies encephalitis, 285t
Pleiomorphic xanthoastrocytoma (PXA), 280t Reflex epilepsies, 158
Polymicrogyria (PMG), 180, 274 Refractory epilepsy
Polymorphic delta (PDA), continuous high voltage, 129, defined, 309–310
130f partial, treatment options for, 317
Polytherapy, 210–211 surgery assessment and testing for, 317
Porencephaly (POR), 180 Refractory Status Epilepticus (RSE)
Positive Occipital Sharp Transients (POSTs), 42, 44f evaluation of, 122
Positron Emission Tomography (PET), 284–286, 288f, pathophysiology of, 122
288t treatment for, 120–122
Postconceptional age (PCA), 55, 56 Relative cerebral blood volume (rCBV), 280
less than 30 weeks of gestation, 57 Remission, in psychogenic seizures, 192
between 30 and 37 weeks, 58–59, 58f, 59f Responsive cortical stimulation, 313–315
between 38 and 44 weeks, 62 Restrictions, 347
Postictal aphasia, 143, 150 Retigabine. See Ezogabine
Postsynaptic potential (PSP), 3–5 Rhythmic Mid-Temporal Discharges (RMTD). See
excitatory (see Excitatory postsynaptic potential Rhythmic mid-temporal theta bursts of drowsiness
(EPSP)) (RMTTBD)
374 Index

Rhythmic mid-temporal theta bursts of drowsiness Sharp waves, features of, 83


(RMTTBD), 33, 35f Shuddering attacks, 168
Right anterior temporal spikes, 319f Single Nucleotide Polymorphism (SNP) arrays, 171
Right Temporal Lobectomy (RTL), 322–323 Single-Photon Emission Computerized Tomography
Ripple, 13 (SPECT), 157, 181f, 286, 305, 341
Riverboat captains, 348 subtraction ictal, co-registered to MRI, 286–287
Rufinamide, 231 6-Hz spike-and-wave bursts, 39–40, 39f
60-Hz artifact, 51, 52f
Sleep disorders, 168–169
S Slow delta rhythms, 12
Sandifer syndrome, 167, 199 Slow oscillation rhythms, 12
SANTE Trial, 313, 314, 314f Slow spike-and-wave complexes, 142
Scalp EEG, 7 Small sharp spikes (SSS). See Benign sporadic sleep
electrodes, 15 spikes (BSSS)
focal epilepsy, 109 Solid angle rule, 8f
Schistosomiasis, 285t Spindle (sigma) rhythms, 13
Schizencephaly (SCZ), 180, 276, 279f Spindle coma, 133, 133f
Seizures. See also Epilepsy Spreading depression (SD), 12
absence, 99–101, 99–101f Startle disease. See Hyperekplexia
anterior cingulate, 111 Status epilepticus (SE), 115–124, 156
atonic, 107, 104f classification of, 115–117, 116t
control of, 342–344 clinical stages of, 118, 118t
convulsive psychogenic, 188 defined, 115
dorsolateral frontal, 111 epidemiology of, 115
extratemporal lobe, 111 etiology of, 117t
febrile, 141 management of, 119, 119t, 120f
first (see First seizures) medication-induced, 117
folinic acid responsive, 65 metabolic consequences of, 118–119
frontal lobe, 111–112 pathophysiology of, 118
gelastic, 159 prevalence of, 117t
generalized tonic–clonic, 247 refractory, 122–124
insular lobe, 112, 146 tonic–clonic, complications of, 118t
insular-opercular, 333 treatment for, 120–122, 120–122t
myoclonic, 101–103, 102f treatment outcome, 124
neonatal, 64, 64t uncommon causes of, 117
occipital lobe, 112, 151 Steroids, 243
orbitofrontal, 111 Stiff baby syndrome. See Hyperekplexia
parietal lobe, 112, 145 Stoke
psychogenic nonepileptic, 169–170 and epilepsy, 182–183
pyridoxine-dependent, 65 seizures after, 164, 189
after stroke, 164, 189 Subacute Sclerosing Panencephalitis (SSPE), 135–136
supplementary motor area, 111, 330 Subclinical Rhythmic Electrographic Discharge in Adults
symptoms of, 338 (SREDA), 36, 37–38f
temporal lobe, 109–111, 110t Subcortical Band Heterotopias (SBH), 180, 274, 276
after traumatic brain injury, 165, 169, 189, 191 Subependymal giant cell astrocytomas (SEGA), 280t
triggers, 267 Subtraction ictal SPECT co-registered to MRI (SISCOM),
tumor and, 179 286–287
Selective amygdalohippocampectomy (SAH), 321 Sudden unexplained death in epilepsy (SUDEP), 94, 337
Self-gratification syndrome, 169 Superconducting quantum interference device (SQUID),
Semiology, 93–107 287, 301
absence seizures, 99–101 Supplementary Motor Area (SMA) seizures, 330–331
atonic seizures, 107 Supplementary motor seizures, 111. See also
focal epilepsy, 109–112 Supplementary motor area (SMA) seizures
generalized tonic–clonic seizures, 93–94 Symptomatic generalized epilepsy, ictal EEG in, 90, 90f
myoclonic seizures, 101–103 Syncope, 169
Severe Myoclonic Epilepsy of Infancy (SMEI). See neurocardiogenic, 169
Dravet syndrome vasovagal, 169
Index 375

T history of, 309


Tacrolimus, and status epilepticus, 117 implantation procedure, 310f
Temporal alphoid rhythm. See Third rhythm initial clinical trials, 310–311
Temporal intermittent rhythmic delta activity (TIRDA), investigational neurostimulators, 313
85 lead placement, 310
Temporal lobectomy, 323 mechanisms of action, 315
anterior, 320f precautions of, 312
inferior temporal approach, 322 responsive cortical stimulation, 313–314
right versus left, 322–323 results optimization, 312
Temporal Lobe Epilepsy (TLE), 109–111, 274 side effects, management of, 312–315
familial lateral, 158 stimulation parameters of, 311
familial mesial, 158 warning with MRI, 313
outcome following ATL in nonlesional, 326 Valproate, 219–226
semiology of, 110t for electrical status epilepticus in sleep, 157
surgical resection, 340–342 for juvenile myoclonic epilepsy, 157
Temporal lobe surgery, 321 pharmacokinetic parameters of, 214t
outcomes following, 321–323 for refractory status epilepticus, 124
standard versus selective, 322 and status epilepticus, 117, 122, 122t
Temporal Plus Epilepsies (TPE), 111 Vascular malformations, 194, 281–290, 283f, 284f
Temporal theta, 62 Vasovagal syncope, 177
Theophylline, and status epilepticus, 117 Video-EEG monitoring, 161
Theta rhythms, 13 focal epilepsy, 109
Third rhythm, 40–41, 41f intellectual and developmental disability, 247
Tiagabine, 228–229 post-traumatic epilepsy, 190
pharmacokinetic parameters of, 226t psychogenic nonepileptic seizures, 171
and status epilepticus, 117 Vigabatrin, 231
Todd’s paresis, 94 and status epilepticus, 117
Topiramate, 228, 241 Visuospatial skills, neuropsychological evaluation of, 298
for juvenile myoclonic epilepsy, 157 Voltage, abnormalities in, 59, 60f
pharmacokinetic parameters of, 226t Voltage-gated potassium channel (VGKC) complex, 177
Toxocariasis, 285t
Toxoplasmosis, 285t
Trace continue, 57–59, 57–59f W
Transmantle sign, 275 Wada test, 318, 338
Traumatic brain injury (TBI), 284 Wave generation, in EEG, 9, 10f
magnetic resonance imaging for, 284, 287t, 288f West syndrome, 77
seizures after, 164, 169, 189, 191 Wickets, 40, 41f
Tremor artifact, 50–51, 52f Women with epilepsy, 349–351
Triphasic waves, 128–129, 128f, 129f
Tuberous sclerosis complex (TSC), 180, 181t
X
X-linked FLNA (filamin A) gene mutation, 181
U X-linked infantile spasms, 245t
Urine ketones, 236–237 X-linked inheritance, 180
Urine toxicity screen ASM levels, 119 X-linked lissencephaly, 175t
X-linked mutation, 173, 174t

V
Vagus nerve anatomy, 318 Y
Vagus nerve stimulation (VNS) therapy, 317–322 Yale Group approach, modified, 321
adverse effects of, 319–320 Yield, electrographic, 87
approved indications of, 317
clinical indications of, 318
dosing settings, 319 Z
efficacy and safety of, 319, 319f Zonisamide, 229–230
electrode polarity and pulse stimulus, 310

Vous aimerez peut-être aussi